Medicine Triple A
Medicine Triple A
Medicine Triple A
Section A
1. Hematology........................................................................................................1041
2. Cardiovascular System......................................................................................1066
3. Gastrointestinal System ....................................................................................1091
4. Respiratory System............................................................................................1102
5. Kidney and Urinary Tract....................................................................................1115
6. Liver and Biliary Tract........................................................................................1131
7. Endocrinology....................................................................................................1143
8. Connective Tissue Disorders..............................................................................1158
9. Nervous System.................................................................................................1169
10. Acid-Base Disorders..........................................................................................1188
11. Genetic Disorders..............................................................................................1191
12. Miscellaneous....................................................................................................1196
Section B
Practice Questions.......................................................................................1209
(Comprising of Questions from Recent Exams and NEET Pattern Questions)
Section A
1. HEMATOLOGY
A. Anemias
B. Anemia of Chronic Diseases
C. Thalassemias
D. Myeloproliferative Disorders
E. Acute Leukemias
F. Chronic Leukemias and Lymphomas
G. Plasma Cell Disorders
H. Bleeding and Coagulation Disorders
I. Blood Transfusion
J. Miscellaneous
1042 Jaypee’s Triple A
HEMATOLOGY (QUESTIONS)
MEDICINE
20. A nine month old boy of Sindhi parents presented to you
27. Best marker for iron deficiency is: (DNB 2012)
with complaints of progressive lethargy, irritability & pallor
since 6 months of age. Examination revealed severe pallor. a. Serum iron
Investigation showed Hb-3.8 mg%; MCV-58 fl; MCH-19.4 b. Serum ferritin
pg/cell. Blood film shows osmotic fragility is normal (target c. Total Iron Binding capacity
cells and normoblasts). X-ray skull shows expansion of d. Transferrin saturation
erythroid marrow. Which of the following is the most likely Ref: Harrison’s 18/e p846
diagnosis? (AI 2004) 28. The most suitable test to assess iron stores is: (DNB 2010)
a. Iron deficiency anemia a. Serum iron
b. Acute lymphoblastic anemia b. Serum Ferritin
c. Hemoglobin D disease c. TIBC
d. Hereditary spherocytosis Ref: Harrison’s 18/e p847 d. Transferrin saturation Ref: Harrison’s 18/e p846
Ans. 15. b. Increased serum... 16. b and c 17. a. Iron deficiency anemia 18. a. Iron Deficiency Anemia
19. b. Iron deficiency... 20. a. Iron deficiency anemia 21. c. Hemochromatosis 22. a. Chronic renal failure
23. c. Hookworm infectino 24. d. Alkaline pH of stomach 25. a. Increased TIBC... 26. b. Serum ferritin levels
27. b. Serum ferritin 28. b. Serum Ferritin
1044 Jaypee’s Triple A
29. Which is not seen in Iron deficiency anaemia: (AI 2000) a. B12 estimation
a. Hyper-segmented neutrophils b. Brush biopsy of the lesion
b. Microcytosis preceeds hypochromia c. Fluconazole treatment
c. MCHC < 50% d. Incision biopsy
d. Commonest cause of anaemia in India Ref: Harrison’s 18/e p867
Ref: Harrison’s 18/e p846, 866, 17/e p630, 631 37. Megaloblastic anemia should be treated with both folic acid
30. Which of the following is true about oral therapy for iron vitamin B12 because: (AI 2008)
deficiency anemia: (PGI Dec 05) a. Folic acid alone causes improvement of hematologic
a. In 300 mg elemental iron given 100 mg get absorbed symptoms but worsening of neurological symptoms
b. Reticulocytosis appears in 1 to 2 weeks and then peaks in b. It is a co factor
3-4 weeks c. It is enzyme
c. Hemoglobin levels are usually corrected in six months of d. None of the above
initiating therapy Ref: Harrison’s 18/e p871, 17/e p651
d. Decrease in absorption with improvement of symptoms 38. All of the following are features of hemolytic anemia
e. Stop the Rx after normalizing the Hb except: (PGI June 04)
Ref: Harrison’s 18/e p848, 17/e p632 a. Decreased Rbc life span
31. A patient presents with increased serum iron, decreased b. Decreased haptoglobin
TIBC, increased percent saturation and increased serum c. Unconjugated hyperbilirubinemia
ferritin. Most probable diagnosis is: (AIIMS Nov 2006) d. Bilirubin in urine
a. Anemia of chronic disease e. Altered erythroid and myeloid ratio
b. Sideroblastic anemia Ref: Harrison’s 18/e p873, Table 106-2
c. Iron deficiency anemia 39. Hemolytic anemia may be characterized by all of the
d. Thalassemia minor following except: (AIIMS May 05)
Ref: Harrison’s 18/e p848, Table 103-4, 17/e p632 a. Hyperbilirubinemia
32. Which of the following types of anemia is associated with a b. Reticulocytosis
Raised MCV and Normal MCHC: (AI 2012) c. Hemoglobinuria
a. Sideroblastic anemia d. Increased plasma haptoglobin level
b. Vitamin B12 and Folic acid deficiency Ref: Harrison’s 18/e p88, 873
c. Beta thalassemia 40. Features seen in hemolytic anemia are all except:
d. Iron deficiency anemia (AIIMS May 07)
Ref: Harrison’s 18/e p865 a. Tear drop and Burr cells
33. Megaloblastic anemia due to folic acid deficiency is b. ↓ Haptoglobin
commonly due to: (AI 2006) c. Reticulocytosis
a. Inadequate dietary intake d. Hemoglobinuria
b. Defective intestinal absorption Ref: Harrison’s 18/e p873
c. Absence of folic acid binding protein in serum 41. Reticulocytosis is seen in all except: (AIIMS May 07)
d. Absence of glutamic acid in the intestine a. P.N.H
Ref: Harrison’s 18/e p869, Table 105-5 b. Hemolysis
34. Thiamine deficiency is known to occur in all of the c. Nutritional anemia
following except: (AI 2003) d. Dyserythropoietic syndrome
a. Food Faddist Ref: Harrison’s 18/e p448
b. Homocystinemia 42. The following protein defects can cause hereditiary
c. Chronic alcoholic spherocytosis except: (AI 07)
MEDICINE
Ans. 29. a. Hyper-segmented... 30. d. Decrease in absorption... 31. b. Sideroblastic anemia 32. b. Vitamin B12 and...
33. a. Inadequate dietary... 34. b. Homocystinemia 35. a. Vitamin B12 36. a. B12 estimation
37. a. Folic acid alone... 38. d. Bilirubin in urine 39. d. Increased plasma... 40. a. Tear drop and Burr...
41. d. Dyserythropoietic... 42. c. Glycophorin C 43. b. Sickle cell disease
Hematology 1045
44. Aplastic anemia in hereditary spherocytosis precipitated d. 10 and 50%
by: (PGI Dec 05) Ref: Harrison’s 18/e p855, 17/e p638
a. Parvo virus 52. Sickle cell anemia leads to resistance towards:
b. HIV a. P. Falciparum (DNB Dec 2009)
c. Adenovirus b. P. Ovale
d. Influenza virus c. P. Malariae
e. Measles virus d. P. Vivax
Ref: Harrison’s 18/e p889 Ref: Harrison’s 18/e p855
45. Hemolysis in G6PD may be caused by all except: (AI 2008) 53. Autoimmune hemolytic anemia is associated with
a. Primaquine malignancy of which lineage: (AI 2007)
b. Chloroquine a. T cell
c. Pyrimethamine b. B cell
d. Quinine c. Pre B cell
Ref: Harrison’s 18/e p878 d. Pre T cell
46. All of the following statements are true about sickle cell Ref: Harrison’s 18/e p927
disease except: (AI 2004) 54. Autoimmune haemolytic anemia is seen in: (AI 2001)
a. Patient may require frequent blood transfusions a. ALL
b. Acute infection is the most common cause of mortality b. AML
before 3 years of age. c. CLL
c. There is positive correlation between con HBS and
d. CML
polymerization of HBS.
Ref: Harrison’s 18/e p927
d. Patient presents early in life before 6 months of age.
Ref: Harrison’s 18/e p855, 17/e p637, 638 55. Coomb’s +ve Hemolytic Anaemia is seen in except:
a. Alcoholic cirrhosis (AI 2000)
47. All of the following are true about sickle cell disease,
b. Chronic active hepatitis
except: (PGI June 2008)
c. Primary biliary cirrhosis
a. Mutation in α chain
d. Primary sclerosing cholangitis
b. Symptoms ameliorated by HbF
Ref: Harrison’s 18/e p881
c. Venoocclusive cries is cause of morbidity
d. Bone pain is presenting feature 56. Coombs positive hemolytic anemia is associated with:
Ref: Harrison’s 18/e p855, 17/e p637, 638 a. TTP (AI 2009)
b. PAN
48. Heterozygous sickle cell anemia gives protection against:
c. SLE
a. G6PD (AI 2010)
d. HUS
b. Malaria
Ref: Harrison’s 18/e p881
c. Thalassemia
d. Dengue fever 57. All of the following conditions are associated with coombs
Ref: Harrison’s 18/e p855 positive hemolytic anemia except: (AI 2009)
49. Which of the following is not seen on hemoglobin a. Thrombotic Thrombocytopenic purpura (TTP)
electrophoresis in sickle cell anemia: (AI 2001) b. Scleroderma
a. HbA c. SLE
b. HbA2 d. PAN
c. HbF Ref: Harrison’s 18/e p882
d. HbS 58. Which of the following conditions is associated with
Ref: Harrison’s 18/e p852, 17/e p635, 637 Coomb’s positive hemolytic anaemia: (AIIMS May 03)
50. Sickle cell trait patient do not have manifestations as that of a. Thrombotic thrombocytopenic purpura.
MEDICINE
Sickle cell disease, because: (AIIMS 2001) b. Progressive systemic sclerosis.
a. 50% HbS is required for occurrence of sickling c. Systemic lupus erythematosus.
b. HbA prevents sickling d. Polyarteritis nodosa
c. HbS is less than 50% & HbA has low affinity for HbS Ref: Harrison’s 18/e p882
d. HbA prevents polymerization of Hbs 59. A 20 years old female presenting with anemia, mild
Ref: Harrison’s 18/e p855 jaundice for 2 years, peripheral smear showing spherocytes,
51. The mother has sickle cell disease; Father is normal; the best investigation to be done is: (AIPGMEE 08)
Chances of children having sickle cell disease and sickle a. Reticulocyte count
cell trait respectively are: (AI 2001) b. Osmotic fragility test
a. 0 and 100% c. Coombs test
b. 25 and 25% d. Bone marrow aspiration
c. 50 and 50% Ref: Harrison’s 18/e p875, 17/e p634, 635, 659
Ans. 44. a. Parvo virus 45. c. Pyrimethamine 46. d. Patient presents... 47. a. Mutation in α chain
48. b. Malaria 49. a. HbA 50. c. HbS is less than 50%... 51. a. 0 and 100%
52. a. P. Falciparum 53. b. B cell 54. c. CLL 55. a. Alcoholic cirrhosis
56. c. SLE 57. a. Thrombotic... 58. c. Systemic lupus... 59. c. Coombs test
1046 Jaypee’s Triple A
60. A 23 Year old female presents with anemia and jaundice 67. Which of the following causes of Anemia is associated with
for 2 years. Peripheral smear shows spherocytes. The best a Hypoplastic marrow: (AI 2012)
investigation to be done is: (AIIMS Nov 2006) a. Fanconi’s Anemia
a. Reticulocyte Count b. Paroxysmal Nocturnal Hemoglobinuria (PNH)
b. Osmotic Fragility Testing c. Hypersplenism
c. Coombs Test d. Myelofibrosis
d. Bone Marrow Aspiration Ref: Harrison’s 18/e p887
Ref: Harrison’s 18/e p851, 852, 879, 17/e p654, 655, 659 68. Vasanti, a 25-year-old-girl, presents with complaints of
61. All of the following are causes of fragmented RBC in fever and weakness. On examination there is splenomegaly
peripheral blood except: (PGI June 05, 02) of 3 cm below the costal margin. Hb is 8 gm/dL, TLC is 3,000/
a. Microangiopathic hemolytic anemia mm3, platelet count is 80,000/mm3. Which of the following
b. D.I.C is the least likely diagnosis: (AIIMS Nov 2000)
c. Hemophilia-A a. Acute lymphocytic leukemia
d. Malignant hypertension b. Anemia of chronic disease
e. HELLP syndrome c. Aplastic anemia
Ref: Harrison’s 18/e p883, 17/e p658 d. Megaloblastic anemia
62. A 25-year-old pregnant lady presents with thrombocytopenia Ref: Harrison’s 18/e p888, 17/e p606
(Platelet count < 50,000) and fragmented RBC’s in peripheral 69. All of the following statements about Fanconi’s anemia are
smear. Which of the following is the least likely differential true, except: (AI 2010)
diagnosis? (AI 2012) a. Autosomal dominant inheritance
a. Thrombotic Thrombocytopenic Purpura (TTP) b. Hypocellular Bone Marrow
b. Disseminated Intravascular Coagulation (DIC) c. Congenital Anomalies
c. HELLP syndrome d. Usually normocytic/macrocytic cell Morphology
d. Evan’s syndrome Ref: Harrison’s 18/e p889, 17/e p665
Ref: Harrison’s 18/e p881, 17/e p658 70. True about aplastic anemia is all except: (PGI June 05)
63. The differential diagnosis of micro-angiopathic anemia a. Splenomegaly
includes all except: (PGI Dec 05) b. Reticulocytopenia
a. Sepsis c. Thrombocytopenia
b. Hemolytic uremic syndrome d. Neutropenia
c. MI Ref: Harrison’s 18/e p890
d. Eclampsia 71. A patient aged 63 years, is diagnosed to have severe aplastic
e. Scleroderma anemia. HLA compatible sibling is available. The best
Ref: Harrison’s 18/e p883, 17/e p658 option of treatment is: (AIIMS May 06)
64. Microangiopathic Hemolytic anemia is seen in all except: a. Anti-thymocyte globulin followed by cyclosporine
a. TTP (AIIMS Nov 2009) b. A conventional bone marrow transplantation from the
b. Metallic heart valve HLA identical sibling
c. Microscopic polyangitis c. A non-myeloablative bone marrow transplantation from
d. Anti-phospholipid syndrome the HLA identical sibling
Ref: Harrison’s 18/e p883 d. Cyclosporine
65. Microangiopathic hemolytic anemia is seen in all except: Ref: Harrison’s 18/e p891, 17/e p666, 667
a. HUS (PGI June 01, 00) 72. A 20 year old adult presents with severe hypoplastic anemia.
b. ITP What is most effective treatment: (AI 2002)
c. Malignant hypertension a. α-interferon
MEDICINE
Ans. 60. c. Coombs Test 61. c. Hemophilia-A 62. d. Evan’s syndrome 63. c. MI
64. b. Metallic heart valve 65. b. ITP 66. c. Hepatitis A 67. a. Fanconi’s Anemia
68. c. Aplastic anemia 69. a. Autosomal dominant... 70. a. Splenomegaly 71. a. Anti-thymocyte globulin...
72. d. Bone marrow... 73. a. Myelodysplastic...
Hematology 1047
74. In which of the following age group myelodysplastic b. Iron deficiency anaemia
syndrome (MDS) are most common: (AI 06) c. Chronic infections
a. 2-10 d. Sideroblastic anaemia
b. 15-20 Ref: Harrison’s 18/e p448-449, 17/e p633
c. 5 83. Which of the following is true about Anemia of Chronic
d. >50 Ref: Harrison’s 18/e p892 Disease: (DNB 2012)
75. Which is the most common cytogenetic abnormality in a. Increased TIBC
adult myelodysplastic syndrome b. Normal serum iron levels
(MDS): (AI 04) c. Normal or increased serum ferritin
a. Trisomy 8 d. Increased Transferrin saturation
b. 20q- Ref: Harrison’s 18/e p850, 17/e p631
c. 5q- 84. A 25-year-old lady on treatment for Rheumatoid Arthritis
d. Monosmy 7 Ref: Harrison’s 18/e p889 has the following laboratory findings. Hemoglobin of 9 gm/
76. P1ummer-Vinson syndrome is an: (Karnatka 2010) dL; MCV-50 fl; Ferritin 200µg/L; TIBC 274 g/dL; Serum iron
a. Iron-deficiency anemia 30µg/dL. What is the likely diagnosis: (AIIMS Nov 2011)
b. Anaplastic anemia a. Iron deficiency anemia
c. Vitamin deficiency b. Anemia of chronic disease
d. Increased hemoglobulin Ref: Harrison’s 18/e p269 c. Thalassemia Minor
d. Autoimmune Hemolytic Anemia
77. All the following are suggestive of iron deficiency anemia
Ref: Harrison’s 18/e p850
except: (J & K 2010)
a. Koilonychia 85. True regarding anaemia of chronic disease are all except:
b. Pica a. Decreased TIBC (AI 2000)
c. Decreased serum ferritin b. Increased macrophage iron in marrow
d. Decreased total iron binding capacity (TIBC) c. Decreased serum ferritin level
Ref: Harrison’s 18/e p846 d. Decreased serum iron level
Ref: Harrison’s 18/e p850, 17/e p633
78. Tactoids are seen in: (J & K 2010)
a. Multiple myeloma 86. All of the following are true about Anemia of chronic renal
b. Thalassemia failure except: (PGI Dec 05)
c. G6PD deficiency a. Normocytic normochromic anaemia
b. Erythropoietin improves the symptom
d. Sickle cell anemia Ref: Harrison’s 18/e p854
c. Dialysis worsens anemia of renal failure
79. Microcytic anaemia is seen in all, except: (J & K 2010) d. Anemia is proportional to the kidney disease
a. Iron deficiency anaemia Ref: Harrison’s 18/e p850
b. Thalassemia
87. Anemia in chronic renal failure (CRF) is due to:
c. Intrinsic factor deficiency
a. Decreased erythropoietin production (PGI June 01)
d. Sideroblastic anaemia Ref: Harrison’s 18/e p449-454
b. Iron deficiency
c. Hypoplastic bone marrow
B. ANEMIA OF CHRONIC DISEASE d. Decreased Vit B12
e. Decreased folate levels
80. All are true about anaemia due to renal failure except: Ref: Harrison’s 18/e p850, 17/e p633
a. Anaemia does not correlate with the degree of CRF
b. It is due to decrease in erythropoietin (Kerela PG 2008) C. THALASSEMIAS
c. Due to decreased dietary iron intake
MEDICINE
d. Increased blood loss due to capillary fragility and poor 88. Hemolysis is seen in: (Rajasthan 2009)
platelet function a. Thalassemia
Ref: Davidson 19/e p601, Harrison’s 18/e p850 b. Lymphoma
81. Eculizumab has been shown to be effective in treating: c. IDA
a. Neisseria meninigitidis infection (AP 2011) d. Polycythemia
b. Paroxysmal nocturnal hemoglobinuria Ref: Harrison’s 18/e p858, 326
c. Systemic infection 89. Molecular pathogenesis of α thalassemia involves:
d. Leukopenic a. Mutation in transcription promoter sequence
Ref: Harrison’s 18/e p883, 884 b. Gene deletion (Comed K 2011)
82. Reduced serum iron and binding capacity is seen in: c. Codon termination mutation
a. Thalassemia (DP PGMEE 2010) d. mRNA splicing defect
Ref: Harrison’s 18/e p659, 17/e p641; Davidson’s 21/e p1
Ans. 74. d. >50 75. c. 5q- 76. b. Anaplastic 77. d. Decreased total iron...
78. d. Sickle cell anemia 79. c. Intrinsic factor deficiency 80. a. Anaemia does not... 81. b. Paroxysmal...
82. c. Chronic infections 83. c. Normal or increased... 84. b. Anemia of Chronic... 85. c. Decreased serum...
86. c. Dialysis worsens... 87. a. Decreased... 88. a. Thalassemia 89. b. Gene deletion
1048 Jaypee’s Triple A
90. Thrombocytosis may be seen in all of the following except: 98. All are true about β-thalassemia trait except: (PGI June 04)
a. Haemolytic anaemia (J & K 2011) a. ↑ HbF
b. Polycythemia rubra vera b. ↑HbA2
c. HIV infection c. Microcytosis
d. Myelofibrosis Ref: Harrison’s 18/e p970 d. Severe anaemia
91. A couple, with a family history of beta thalassemia major Ref: Harrison’s 18/e p859
in a distant relative, has come for counseling. The husband 99. True about α-thalassemia trait: (PGI June 06)
has HbA2 of 4.8% and the wife has HbA2 of 2.3%. The risk a. Increased HbF
of having a child with beta thalassemia major is: (AI 2003) b. Increased HbA2
a. 50% c. Microcytosis
b. 25% d. Severe anemia
c. 5% Ref: Harrison’s 18/e p859, 17/e p641
d. 0% Ref: Harrison’s 18/e p500
100. A 32-year-old female, asymptomatic, not requiring blood
92. Which of the following conditions is associated with transfusion, presents with Hb 13.0 gm/dl. Her HbF levels
microcytic hypochromic anemia: (AIIMS Dec 94)
are 95%, Hb A2 1.5%. Which of the following is the most
a. Sickle cell Anemia
likely diagnosis (AIIMS May 04)
b. Thalassemia
a. Hereditary persistence of fetal hemoglobin
c. Fanconi’s anemia
b. Beta homozygous thalassemia
d. Hereditary spherocytosis Ref: Harrison’s 18/e p858
c. Thalassemia intermedia
93. A 30 year old female being evaluated for anemia reveals the d. Beta heterozygous thalassemia
following indices: RBC count = 4.5 x 1012/L; MCV 55fl; TLC Ref: Harrison’s 18/e p859, 17/e p642
8000. There is no history of blood transfusion. Which of the
following is the most likely diagnosis: (AIIMS Nov 2011) 101. A child aged 2 year presents with nonspecific symptoms
a. Iron deficiency anemia suggestive of anemia. On peripheral blood smear target
b. Thalassemia major cells are seen. He has hypochromic mic-zrocytic picture
c. Thalassemia minor and Hb of 6 gm. He also has ‘a positive family history’ Next
d. Megaloblastic anemia Ref: Harrison’s 18/e p859 investigation of choice is: (AI 01)
a. Hb electrophoresis
94. In Beta thalassemia, there is: (AIIMS May 01)
a. Increase in beta chain, decrease in alpha chain b. Coombs test
b. Decrease in beta chain, increase in alpha chain c. Liver function tests
c. Decrease in beta chain, decrease in alpha chain d. Osmotic fragility test
d. Increase in beta chain, increase in alpha chain Ref: Harrison’s 18/e p859
Ref: Harrison’s 18/e p858, 17/e p640, 641 102. A 5 year old girl came with history of progressively
95. A 25-year female presented with mild pallor and moderate increasing pallor since birth and hepatosplenomegaly.
hepatosplenomegaly. Her hemoglobin was 92 dl/L and fetal Which of the following is the most relevant test for
hemoglobin level was 65%. She has not received any blood achieving diagnosis: (AI 2004)
transfusion till date.She is most likely to be suffering from: a. Hb electrophoresis
a. Thalassemia major (AIIMS Nov 02) b. Peripheral smear examination
b. Thalassemia intermedia c. Osmotic fragility test
c. Hereditary persistent fetal hemoglobin, homozygous state d. Bone marrow examination
d. Hemoglobin D, homozygous state Ref: Harrison’s 18/e p859, 17/e p1630
Ref: Harrison’s 18/e p859, 17/e p1630 103. HbA2 levels are increased in all except: (PGI June 02)
96. A 23 years old asymptomatic female pilot has MCV-70, a. Alpha Thalassemia
ferritin–100g/L, Hb-10gm%, what is the cause: (AI 2009) b. Beta Thalassemia
MEDICINE
Ans. 90. a. Bone marrow... 91. d. 0% 92. b. Thalassemia 93. c. Thalassemia minor
94. b. Decrease in beta... 95. b. Thalassemia intermedia 96. a. Thalassemia trait 97. d. Patient requires...
98. d. Severe anaemia 99. c. Microcytosis 100. a. Hereditary persistence... 101. a. Hb electrophoresis
102. a. Hb electrophoresis 103. a. Alpha Thalassemia
104. d. Thalassaemia–major
Hematology 1049
a. Chromosomal evaluation
C. MYELOPROLIFERATIVE DISORDERS b. Bone marrow aspiration
105. Which among the following indicates secondary cause for c. Flow-cytometric analysis
polycythemia: (Kerala PG 10) d. Determination of red blood cell mass
a. Cyanosis Ref: Harrison’s 18/e p899, 898
b. Splenomegaly 114. Laboratory evaluation for the differential diagnosis of
c. Pruritis chronic myeloproliferative disorders includes all the
d. Easy brusing Ref: Harrison’s 18/e p456, 17/e p362 following except: (AIIMS Nov 2004)
106. The following are features of polycythemia rubra vera, a. Chromosomal evaluation
except: (Feb DP PGMEE 2009) b. Bone marrow aspiration
a. Increased red cell mass c. Flow-cytometric analysis
b. Low arterial oxygen saturation d. Determination of red blood cell mass
c. Presence of JAK2 mutation Ref: Harrison’s 18/e p898, 17/e p672-676
d. Splenectomy Ref: Harison’s 18/e p899, 456 115. All of the following are the causes of relative polycythemia
107. “Floating tooth sign” seen in: (UP 2011) except: (AI 2005)
a. Odontogenic myxoma a. Dehydration.
b. Ameloblastoma b. Dengue haemorrhagic fever.
c. Histiocytosis-X c. Gaisbock syndrome.
d. Odontogenic fibroma Ref: Harrison’s 18/e p2883 d. High altitude.
Ref: Harrison’s 18/e p456, 17/e p362, 363, 672
108. Polycythemia is seen in:
a. Bronchial asthma 116. A 59-year-old male came with Hb 18.0 gm/dl on
b. Corpulmonale three occasions. The resident doctor wants to exclude
c. Vitamin D excesses Polycythemia vera. Which of the following is the most
d. CCF Ref: Harrison’s 18/e p289, 457 relevant investigation: (AIIMS May 04)
a. Hematocrit
109. Which one of the following is not commonly seen in
b. Total leukocyte count
polycythemia vera?
c. Red cell mass
a. Thrombosis
d. Reticulocyte count
b. Hyperuricemia
Ref: Harrison’s 18/e p898, 899, 17/e p672, 673
c. Prone for acute leukemia
d. Spontaneous severe infection 117. Secondary Polycythemia may be seen in: (DNB 2012)
Ref: Harrison’s 18/e p899,-900 a. Cor pulmonale
b. Congestive cardiac failure
110. Plasmapheresis is indicated in each of the following except:
c. Acyanotic congenital heart disease
a. Hemolytic-uremic syndrome
d. All of the above Ref: Harrison’s 18/e p456
b. Good Pasteur’s syndrome
c. Polycythemia vera 118. Which of the following is not commonly seen in: (AI 2002)
d. Landry-Guillain Barre syndrome Polycythemia Vera?
Ref: Harrison’s 18/e p2353, 2342, 900, 3477 a. Thrombosis
b. Hyperuricemia
111. Which of the following is NOT a myeloproliferative
c. Prone for acute leukemia
disease: (AIIMS Nov 2000)
d. Spontaneous severe infection
a. Polycythemia rubra vera
Ref: Harrison’s 18/e p898,17/e p672, 673
b. Acute myeloid leukemia
c. Chronic myeloid leukemia 119. In Polycythemia vera, all the following are seen except:
d. Essential thrombocytosis a. Thrombocytopenia (AIIMS May 01)
MEDICINE
Ref: Harrison’s 18/e p898, 17/e p672 b. Increased GI bleed
c. Thrombosis
112. Massive Splenomegaly is least likely to be associated with
d. Transient visual loss
which of the following Myeloproliferative disorders:
Ref: Harrison’s 18/e p899, 17/e p672, 673
a. Chronic Myeloid Leukemia (CML) (AI 2012)
b. Polycythemia Vera (PV) 120. All of the following are seen in polycythemia rubra vera
c. Essential Thrombocytosis (ET) except: (AIIMS Nov 2000)
d. Primary Myeloproliferative (PMF) a. Increased Vit B12 binding capacity (>9000 micromols/dL)
Ref: Harrison’s 18/e p903 b. Decreased LAP Score
113. Laboratory evaluation for the differential diagnosis of c. Leucocytosis
chronic myeloproliferative disorders includes all the d. Increased platelets
following except: (AIIMS Nov 04) Ref: Harrison’s 18/e p899, 17/e p673
Ans. 105. a. Cyanosis 106. b. Low arterial oxygen... 107. c. Histiocytosis-X 108. b. Corpulmonale
109. d. Spontaneous.
110. c. Polycythemia vera 111. b. Acute myeloid... 112. c. Essential...
113. c. Flow-cytometric... 114. c. Flow-cytometric...
115. d. High altitude 116. c. Red cell mass
117. a. Cor pulmonale
118. d. Spontaneous severe... 119. a. Thrombocytopenia 120. b. Decreased LAP Score
1050 Jaypee’s Triple A
121. Which of the following is a major criteria for diagnosis of 129. Which of the followings combinations of cytogenetic
poycythemia vera: (AI 2010) abnormality and associated leukemia/lymphoma is
a. Presence of JAK-2 mutation incorrect? (AIIMS Nov 2004)
b. Low Erythropoetin levels a. t (8:14): Burkitts lymphoma
c. High Leucocyte Alkaline Phosphatase (↑LAP score) b. t (15:17) : AML-M3
d. Thrombocytosis c. t (9:22) : CML
Ref: Harrison’s 18/e p898, 17/e p672, 673 d. t (9:20) : ALL
122. A pt. being investigated for anemia has a dry marrow tap; Ref: Harrison’s 18/e p919 Table 110-2
peripheral smear reveals tear drop cells; likely diagnosis is: 130. A17-year-old boy presented with TLC of 138 × 109/ L with 80%
a. Leukemia (AI 2001) blasts on the peripheral smear. Chest X-ray demonsnstrated
b. Lymphoma a large mediastinal mass. Immunophenotyping of this
c. Myelofibrosis
patent’s blasts would most likely demonstrate:
d. Polycythemia rubra vera
a. No surface antigens (null phenotype) (AIIMS May 2006)
Ref: Harrison’s 18/e p901, 17/e p674, 675
b. An immature T cell phenotype (Tdt/CD34/CD7 positive)
c. Myeloid markers, such as CD13, CD33 and Cd15
d. B cell markers, such as CD19, CD20 and CD22
E. ACUTE LEUKEMIA Ref: Harrison’s 18/e p932, 17/e p697
123. Massive splenomegaly is seen in all, except: (J & K 2010) 131. All of the following are good prognostic factors for
a. Myelofibrosis childhood. ALL except: (AIIMS May 02, AI 07)
b. Chronic myeloid leukemia a. Hyperdiploidy
c. Hairy cell leukemia b. Female sex
d. Acute lymphatic leukemia c. Pre B cell ALL
Ref: Harrison’s 18/e p467 d. t (12 : 21) translocation
124. Leukocyte transfusions are indicated in patients with Ref: Harrison’s 18/e p925
Granulocyte count below: 132. Good prognostic Factors for ALL include all of the
a. 500/mm3 following, except: (DNB 2011)
b. 100/mm3 a. Hyperdiploidy
c. 1500/mm3 b. Female sex
d. 2000/mm3 c. Pre B cell ALL
Ref: Nelson’s 18/e p2058; Harrison’s 17/e p378
d. t (12 : 21) translocation Ref: Harrison’s 18/e p923, 924
125. Treatment of choice for intracranial ALL is: (Rajasthan 2009)
133. Which of the following is a poor prognostic factor for
a. Intrathecal methotrexate
childhood ALL: (AIIMS Nov 09)
b. Vincristine and prednisolone
a. Total Leukocyte count 4000-100,000
c. Intrathecal vincristin
b. Age < 2 years
d. Prednisolone
c. Testicular involvement
Ref: Harrison’s 18/e p840, 963
d. Blasts in peripheral smear
126. Increased LDH is seen in: (Rajasthan 2009) Ref: Harrison’s 18/e p924
a. AML
134. Which of the following is a good prognostic factor in ALL:
b. PAN
a. High WBC count (PGI 2008)
c. CML
d. All of the above b. Male sex
Ref: Harrison’s 18/e p907-910 c. Age < 2 years
d. Hyperdiploidy
MEDICINE
Ans. 121. a. Presence of... 122. c. Myelofibrosis 123. d. Acute lymphatic... 124. a. 500/mm3
125. a. Intrathecal...
126. a. AML 127. a and b 128. d. CD45RO
129. d. t (9:20) : ALL
130. b. An immature T cell... 131. c. Pre B cell ALL 132. c. Pre B cell ALL
133. c. Testicular...
134. d. Hyperdiploidy 135. b. All–trans retinoic...
Hematology 1051
136. A four year old boy was admitted with a history of b. AML–M2
abdominal pain and fever for two months maculo papular c. AML–M3
rash for ten days, and dry cough, dyspnea and wheezing for d. AML–M1 Ref: Harrison’s 18/e p908
three days. On examination liver and spleen were enlarged 141. All of the following are poor prognostic factors for acute
4 cm and 3 cm respectively below the costal margins. His myeloid leukemias, except: (AI 2003)
hemoglobin was 10.0 g/dl, platelet count 37×109/L and total a. Age more than 60 years
leukocyte count 70 × 109L, which included 80% eosinophils. b. Leucocytes count more than 1,00,000/µl
Bone marrow examination revealed a cellular marrow c. Secondary leukemias
comprising of 45% blasts and 34% Eosinophils and d. Presence of t (8:21)
eosinophilic precursors. The blasts stained negative for Ref: Harrison’s 18/e p910, 17/e p677, 678, 680
myeloperoxidase and non-specific esterase and were 142. Poor prognosis in AML is indicated by: (AI 2008)
positive for CD19, CD10, CD22 and CD20. Which of the a. Inversion 16
following is the most likely diagnosis? (AIIMS Nov 2004) b. Translocation 15/17 (t15; 17)
a. Biphenotypic acute leukemia (lymphoid and eosinophil c. Normal cytogenecity
lineage d. Monosomy 7
b. Acute eosinophilic leukemia Ref: Harrison’s 18/e p905, 910)
c. Acute lymphoblastic leukemia with hypereosinophilic 143. AML with worst prognosis is: (AIIMS Nov 2006)
syndrome a. 8/21 translocation
d. Acute myeloid leukemia with eosinophilia b. Inversion 16
Ref: Robbin’s 8/e p603 c. Normal Cytogenetics
137. A four year old boy was admitted with a history of d. Monosomy 7
abdominal pain and fever for two months, maculopapular Ref: Harrison’s 18/e p910, 17/e p680
rash for ten days, and dry cough, dyspnea and wheezing for 144. In PML, all of the following are seen except: (AI 2007)
three days. On examination, liver and spleen were enlarged a. Retinoic acid is used in treatment
4 cm and 3 cm respectively below the costal margins. His b. 15/17 translocation
hemoglobin was 10.0 g/dl, platelet count 37 × 109/L and c. CD 15/34 both seen in same cell
total leukocyte count 70 × 109/L, which included 80% d. Associated with DIVC
eosinophils. Bone marrow examination revealed a cellular Ref: Harrison’s 18/e p905, 911, 912, 17/e p678, 682
marrow comprising 45% blasts and 34% eosinophils 145. A 15-years-old boy presented with one day history of
and eosinophilic precursors. The blasts stained negative bleeding gum, subconjunctival bleed and purpuric rash.
for myeloperoxidase and nonspecific esterase and were Investigations revealed followed results: Hb- 6.4 gm/dl;
positive for CD19, CD10, CD22 and CD20. Which one of the TLC-26, 500/mm3 platelet 35,000 mm3; prothrombin time-
following statements in not true about this disease? 20 sec with a control of 13 sec; partial thromboplastin
a. Eosinophils are not part of the Neoplastic clone (AI 2005) time-50sec; and Fibrinogen 10mg/dL. Peripheral smear was
b. t(5:14) rearrangement may be detected in blasts suggestive of acute myeloblastic leukemia. Which of the
c. Peripheral blood eosinophilia may normalize with following is the most likely? (AIIMS May 2006)
chemotherapy a. Myeloblastic leukemia without maturation
d. Inv (16) is often detected in the blasts and the eosinophils b. Myeloblastic leukemia with maturation
138. Which of the following is a poor prognostic factor in Acute c. Promyelocytic leukemia
Myeloid Leukemia (AML): (AIIMS Nov 2010) d. Myelomonocytic leukemia
a. Monosomy Ref: Harrison’s 18/e p907, 17/e p679
b. Deletion of X or Y chromosome 146. Arsenic is used in treatment of: (AIIMS May 07)
c. t (8; 21) translocation a. Acute promyelocytic leukemia
MEDICINE
d. Nucleophosphin mutation b. A.L.L
Ref: Harrison’s 18/e p905 c. CML
139. Auer Rods are most frequently seen in which subtype of d. Transient myeloproliferative disorder
AML: (DNB 2012) Ref: Harrison’s 18/e p912
a. M0 147. In a patient with acute leukemia, immunophenotype
b. M 3 pattern is CD 19+ve, CD 10+ve, CD33+ve, CD13+vE. He
c. M 5 may probably have: (AIIMS May 04)
d. M 7 a. Biphenotypic leukemia
Ref: Harrison’s 18/e p908 b. ALL
140. Auer Rods are typically not seen in: (DNB 2012) c. AML-M2
a. AML–M0 d. AML-Mo Ref: Harrison’s 18/e p912, 17/e p682
Ans. 136. c. Acute... 137. d. Inv (16) is often... 138. a. Monosomy 139. b. M 3
140. a. AML–M0
141. d. Presence of t(8:21) 142. d. Monosomy 7 143. d. Monosomy 7
144. c. CD 15/34 both...
145. c. Promyelocytic leukemia 146. a. Acute... 147. a. Biphenotypic leukemia
1052 Jaypee’s Triple A
148. A patient, on treatment for leukemia, develops chest pain, 155. Best Rx for CM L is: (Rajasthan 2009)
pulmonary infiltrates and pleural effusion. The cause is: a. Autologous BMT
a. Daunorubicin (DP PGMEE 2009) b. Allogenic BMT
b. Hydroxyurea c. Alpha interferon
c. Cytarabine d. Hydroxyurea
d. Tretinoin Ref: Harrison’s 18/e p912, 913, 17/e p682, 16/e p636 Ref: Harrison’s 18/e p915-918
156. High serum uric acid, high serum phosphates and high
serum creatinine in a patient will be observed in:
F. CHRONIC LEUKEMIAS AND a. Chronic lymphocytic leukemia (DP PGMEE 2009)
LYMPHOMAS b. Obstructive jaundice
c. Status epilepticus
149. In chronic myelogenous leukaemia, the serum vitamin B12 d. Typhoid
levels are: (AP 2012) Ref: Harrison’s 18/e p926, 17/e p1736-1736
a. Normal 157. A pt with an Hb of 6, WBC count of 2000, has a normal
b. Elevated Different count except for having 6% blasts; platelets are
c. Slightly decreased reduced to 80,000; moderate splenomegaly is present;
d. Markedly decreased possible diagnosis is: (AI 2001)
Ref: Harrison’s 18/e p914 a. Leukemia
150. First line therapy recommended in chronic phase of CML is: b. Aplastic anemia
a. Hydroxyurea (J & K 2011) c. Hemolysis
b. Interferon d. ITP
c. Busulphan Ref: Harrison’s 18/e p903
d. Imatinib 158. The subtype of Hodking’s disease, which is histogentically
Ref: Harrison’s 18/e p914 distinct from all the other subtypes, is: (AI 05)
a. Lymphocyte predominant
151. All the following conditions predispose to acute
b. Nodular sclerosis
disseminated intravascular coagulopathy except:
c. Mixed cellularity
a. Acute hemolyti transfusion reaction (MP PG 2008)
d. Lymphocyte depleted
b. Amniotic fluid embolism
Ref: Harrison’s 18/e p935, 17/e p689
c. Gram negative septiemia
159. The lymphocytic and histiocytic variant of Reed Sternberg
d. Choronic myeloid leukemia
cell is seen in: (AIIMS Nov 05)
Ref: Harrison’s 18/e p978, 979, 17/e p729
a. Follicular center lymphoma
152. Mycosis fungoides is best described as: (Kerala PG 10) b. Lymphocyte depleted Hodkin’s disease
a. Cutaneous fungal infection c. Nodular sclerosis Hodkin’s disease
b. Exematous reaction d. Lymphocyte predominant Hodkin’s disease
c. Skin Ref: Harrison’s 18/e p935
d. Cutaneous T lymphoma 160. Which of the following types of Hodgkins lymphomas is
Ref: Harrison’s 18/e p932, 17/e p697 associated with a good prognosis: (DNB Dec 2010)
153. A patient presents with massive splenomegaly, lab a. Lymphocyte Predominance
examination reveals WBC count 4200/cmm, Platelet count b. Lymphocyte Depletion
90,000/cmm, and bone marrow aspiration reveals; a dry tap. c. Nodular sclerosis
Diagnosis is: (WB PG 08) d. Mixed cellularity
a. CML Ref: Harrison’s 18/e p934
b. CLL 161. Classical markers for Hodgkin’s disease is: (AIPGMEE 08)
MEDICINE
Ans. 148. d. Tretinoin 149. b. Elevated 150. d. Imatinib 151. d. Choronic myeloid...
152. d. Cutaneous T...
153. c. Hairy Cell... 154. d. Hepatitis C 155. b. Allogenic BMT
156. a. Chronic lymph...
157. a. Leukemia 158. a. Lymphocyte... 159. d. Lymphocyte...
160. a. Lymphocyte...
161. a. CD 15 and CD 30 162. a. CNS is the commonest...
Hematology 1053
163. The paraneoplastic syndrome associated with Hodgkin’s b. REAL classification
disease is: (AIIMS Nov 05) c. WHO classification
a. Nephrotic syndrome d. Rappaport classification
b. Retinopathy 171. Which of the following is not a B-cell neoplasm?
c. Cerebellar degenerative disease a. Hairy cell leukemia (AIIMS May 06)
d. Acanthosis nigricans b. Angiocentric lymphoma
Ref: Harrison’s 18/e p934, 17/e p699 c. Mantle cell lymphoma
164. All of the following are the good prognostic features d. Burkitt’s lymphoma
Hodgkin’s disease except: (AI 04) Ref: Harrison’s 18/e p920, 17/e p688
a. Haemoglobin>10 gm/dl
172. Plasmacytoid Lymphomas may be associated with:
b. WBC count<15000/mm3
a. IgG (AI 2010)
c. Absolute lymphocyte count<600/ul
b. IgM
d. Age < 45 yrs
c. IgA
Ref: Harrison’s 18/e p934
d. IgE
165. All of the following are poor prognostic factors for
Ref: Harrison’s 18/e p942, 17/e p706
Hodgkin’s disease, except: (PGI Dec 01)
a. Younger age 173. “International prognostic index” for lymphoma includes
b. Systemic manifestations the following prognostic factors except: (AI 2009)
c. Lymphocyte depletion a. Stage of disease
d. Mediastinal disease b. Number of extralymphatic sites involved
e. Stomach involvement c. LDH
Ref: Harrison’s 18/e p934 d. Hemoglobin and Albumin
166. A patient with Hodgkin’s lymphoma is having a Ref: Harrison’s 18/e p925, Table 110-9, 17/e p692
single cervical lymphnode.Biopsy showed lymphocyte 174. Intermediate grade of NHL are all except: (AI 2000)
predominant variant. Which of the following is the a. Diffuse small cell cleaved
treatment of choice: (AIIMS Nov 2000) b. Diffuse large cell l
a. Chemotherapy with Radiotherapy c. Mycosis fungoides
b. Chemotherapy only d. Diffuse mixed
c. Radiotherapy only Ref: Harrison’s 18/e p925, Table 110-9
d. No treatment needed 175. In Burkitts lymphoma, translocation seen is chromosome:
Ref: Harrison’s 18/e p935, 17/e p700 a. 12-14 translocation (AIIMS May 01)
b. 8-14 translocation
167. True for Hodgkin’s stage IA is: (PGI Dec 05)
c. 4-8 translocation
a. Chemotherapy is best
d. 12-18 translocation
b. Radiotherapy is best
Ref: Harrison’s 18/e p931, 17/e p696
c. Total radiation therapy is best treatment
176. Translocation t(2:8)(p12:q24) is associated with: (AI 2010)
d. Fever and wt loss is always present
a. Chronic Myeloid Leukemia (CML)
Ref: Harrison’s 18/e p934
b. Acute Myeloid Leukemia (AML)
168. Treatment of choice in Hodgkin’s Lymphoma is: c. T cell–ALL
a. CHOP (PGI Dec 06) d. Burkitt’s Lymphoma
b. MOPP Ref: Harrison’s 18/e p931, 17/e p696
c. ABVD 177. Burkitt’s Lymphoma is associated with which of the
d. MOPP and ABVD hybrid Ref: Harrison’s 18/e p934 following viruses: (DNB 2012)
169. All of the following statements about Treatment of a. EBV
MEDICINE
Hodgkin’s lymphoma are true, except: (PGI June 07) b. HTLV-1
a. ABVD is the most commonly used Regimen c. HHV- 8
b. Sterility is more frequent after ABVD than MOPP d. Adenovirus
c. Combination chemotherapy is the mainstay of treatment Ref: Harrison’s 18/e p931, 17/e p696, 690
in Advanced Hodgkin’s disease 178. Burkitt’s Lymphoma is associated with: (AI 2010)
d. WBC count > 15000/mm3 is a poor prognostic factor a. t (8:14)
Ref: Harrison’s 18/e p935, 17/e p699 b. t (11:14)
170. The classification proposed by the International Lymphoma c. t (15:17)
Study Group for non-Hodgkin’s lymphoma is known as: d. t (14:18)
a. Kiel classification (AI 2005) Ref: Harrison’s 18/e p931, 17/e p689
Ans. 163. c. Cerebellar... 164. c. Absolute lymphocyte... 165. a. Younger age 166. c. Radiotherapy only
167. b. Radiotherapy is... 168. c. ABVD
169. b. Sterility is more... 170. b. REAL classification
171. b. Angiocentric...
172. b. IgM 173. d. Hemoglobin and... 174. c. Mycosis fungoides
175. b. 8-14 translocation 176. d. Burkitt’s Lymphoma
177. a. EBV 178. a. t (8:14)
1054 Jaypee’s Triple A
179. All of the following statements about Burkitt’s lymphoma increased IgM and likely diagnosis? (AP 2012)
are true, except: (PGI June 02) a. IgM myeloma
a. B cell lymphoma b. Waldenstrom’s macroglobulinemia
b. 8, 14 translocation c. CLL
c. Can present as an abdominal mass d. POEMS syndrome Ref: Harrison’s 18/e p943
d. Radiotherapy is the treatment of choice 187. A 48 years of woman was admitted with a history of
Ref: Harrison’s 18/e p931 weakness for two months. On examination cervical lymph
180. All of the following statements about Hairy cell leukaemia nodes were found enlarged and spleen was palpable 2
are true except: (AI 2004) cm below the costal margin. Her hemoglobin was 10.5g/
a. Splenomegaly is conspicuous dl. Platelet count 237×109/L, and total leukocyte count
b. Results from an expansion of neoplastic T lymphocytes 40×109/L, which include 80% mature lymphoid cell with
c. Cells are positive for Tartrate Resistant Acid phosphatase coarse clumped chromatin. Bone marrow revealed a nodular
d. The cells express CD25 consistently lymphoid infiltrate. The peripheral blood lymphoid cells
Ref: Harrison’s 18/e p931 were positive for CD19, CD5, CD20 and were negative for
181. CD 19 positive, CD22 positive, CD103 positive monoclonal CD79B and FMC(-7). Which one of the following statements
B-cells with bright kappa positivity were found to comprise is not true about this disease? (AIIMS Nov 2004)
60% of the peripheral blood lymphoid cells on flow a. Trisomy 12 correlates an aggressive clinical course
cytometric analysis in a 55 year old man with massive b. Abnormalities of 13q 14 are associated with long term
splenomegaly and a total leucocyte count of 3.3 x 109/L. survival
Which one of the following is the most likely diagnosis?
c. Case with 11q22-23 deletions have excessive lymphad-
a. Splenic lymphoma with villous lymphocytes
enopathy
b. Mantle cell lymphoma) (AIIMS Nov 2004)
d. t (11;14) translocation is present in most of the cases
c. B-cell prolymphocytic leukemia
Ref: Harrison’s 18/e p923, 17/e p690
d. Hairy cell leukemia Ref: Harrison’s 18/e p920, 17/e p697
188. A 48 year old women was admitted with a history of
182. A 63 year old man presents with splenomgaly and
weakness for two months. On examination, cervical lymph
lymphadenopathy. Immunophenotype was positive for
nodes were found enlarged and spleen was palpable 2
CD19, CD79b and FMC7. The most likely diagnosis is:
cm below the costal margin. Her hemoglobin was 10.5 g/
a. Hairy cell leukemia (AIIMS May 01)
dl, platelet count 2.7×109/L and total leukocyte count
b. Mantle cell Lymphoma (MCL) (AI 2003)
40×109/L, which included 80% mature lymphoid cells with
c. Chronic Lymphocytic Leukemia (CLL)
coarse clumped chromatin. Bone marrow revealed nodular
d. Follicular Lymphoma Ref: Harrison’s 18/e p929
lymphoid infiltrate. The peripheral blood lymphoid cells
183. Which of the following vaccines are contra-indicated in were positive for CD 19, CD 5, CD20 and CD23 and were
HIV patients with CD4 counts <200/mm3? (J & K 2012) negative for CD79B and FMC-7. The histopathological
a. Pneumococcal vaccine examination of the lymph node in this patient will most
b. Hepatitis B vaccine likely exhibit effacement of lymph node architecture by:
c. Influenza vaccine a. A pseudofollicular pattern with proliferation centers
d. BCG vaccine Ref: Harrison’s 18/e p1545-1546,1047 b. A monomorphic lymphoid proliferation with a nodular
184. 80 year old, asymptomatic man present with a Total pattern (AI 2005)
Leucocyte Count of 1 lakh, with 80% lymphocytes and 20% c. A predominantly follicular pattern
PMC’s. What is the most probable diagnosis? d. A diffuse proliferation of medium to large lymphoid cells
a. HIV (AI 2005, AI 2007) with high mitotic rate
b. CML 189. A 48 year old woman was admitted with a history of
c. CLL weakness for two months. On examination, cervical lymph
d. TB Ref: Harrison’s 18/e p916, 920
MEDICINE
Ans. 179. d. Radiotherapy is... 180. b. Results from an... 181. d. Hairy cell leukemia 182. b. Mantle cell Lymphoma...
183. b. Hepatitis B vaccine 184. c. CLL
185. d. T lymphocyte CLL... 186. b. Waldenstrom’s ...
187. d. t (11;14)...
188. a. A pseudofollicular... 189. b. Chronic lymphocytic...
Hematology 1055
190. Peripheral smear with increased neutrophils, basophils, b. Absence of Philadelphia chromosome
eosinophils, and platelets is highly suggestive of: c. More than 20% blasts in blood or bone marrow
a. Acute myeloid leukemia (AIIMS May 06) d. Absent or minimal dysplasia in myeloid lieages.
b. Acute lymphoblastic leukemia Ref: Harrison’s 18/e p914
c. Chronic myelogenous leukemia 197. Mycosis fungoides which is not true: (AI 2007)
d. Myelodysplastic syndrome a. It is the most common form of cutaneous lymphoma
Ref: Harrison’s 18/e p915 b. Pautriers microabscess
191. A 60 year old man presented with fatigue, weight loss c. Indolent course and easily amenable to treatment
and heaviness in left hypochondrium for 6 months. The d. Erythroderma seen and spreads to peripheral Circulation
hemogram showed Hb. 10gm/dL, TLC 5 lakhs/mm3, platelet Ref: Harrison’s 18/e p932, 17/e p697
count 4 lakhs/mm3, DLC; neutrophil 55%, lymphocutes
4%, monocytes 2%, basophils 6%, metamyelocytes 10%,
myelocytes 18%, promyelocytes 2% and blast 3%. The most G. PLASMA CELL DISORDERS
likely cytogenetic abnormality in this case is:
a. t (1; 21) (AIIMS May 03) 198. All these are true in relation to multiple myeloma, except:
b. t (9; 22) a. Increased ESR (J & K 2010)
c. t (15; 17) b. Increased alkaline phosphatase
d. Trisomy 21 c. Punched out lesions in the bones
Ref: Harrison’s 18/e p915, 17/e p684 d. Increased plasma cells in the bone marrow
Ref: Harrison’s 18/e p712
192. In a patient suffering from chronic myeloid leukemia,
Hb falls from 11g% to 4g% in a short span of time, and 199. Splenomegaly is a rare feature of: (MP PG 2009)
splenomegaly occurs. The cause could be (select two a. Multiple myeloma
options): (PGI June 02) b. Polycythemia vera
a. Accelerated phase c. Myelofibrosis
b. CML in blast crisis d. Hairy cell leukemia
c. Ineffective erythropoesis Ref: Harrison’s 18/e p470-471, 17/e p374
d. Myelofibrosis 200. Bad prognosis in multiple myeloma is indicated by:
Ref: Harrison’s 18/e p915, 17/e p684 a. WBC>20000 (Feb DP PGMEE 2009)
193. Which one of the following is not a criterion for making b. Azotemia
a diagnosis of chronic myeloid leukemia in accelerated c. Hypocalcemia
phase: (AIIMS Nov 2004) d. Low or normal M component production
a. Blasts 10-19% of WBC’s in peripheral blood Ref: Harison’s 18/e p942
b. Basophils 10-19% of WBC’S in peripheral blood 201. Which of the following types of multiple myeloma is
c. Increasing spleen size unresponsive to therapy maximally and most commonly associated with increased
d. Persistent thrombocytosis (>1000 x 109/L) unresponsive risk of hyperviscosity syndrome? (MHPGM CET 2010)
to therapy a. IgM
Ref: Harrison’s 18/e p915, 17/e p684 b. IgG
194. Drug of choice for chronic myeloid Leukemia (CML) is: c. IgD
a. Hydroxyurea (AI 2008) d. IgA
b. Imatinib Ref: Harrison’s 18/e, p941, 17/e p704;
c. Inflliximab Robbin’s 7/e p680
d. IFN 202. Most important prognostic factor of multiple myeloma:
a. B2 microglobulin (WB PG 08)
MEDICINE
Ref: Harrison’s 18/e p915
195. Reticulocyte count is increased in all except: (J & K 2010) b. Hypercalcimea
a. Hemolytic anemia c. CRP
b. Iron deficiency anemia under specific treatment d. % of plasma cells in the marrow
c. Aplastic anemia after blood transfusion Ref: Harrison’s, 18/e p941, 17/e p704
d. In chronic hemorrhage with adequate hematanic stores 203. Punched out appearance in skull seen in: (UP 2011)
Ref: Harrison’s 18/e p452 a. Multiple myeloma
196. Which of the following is not compatible with a diagnosis b. Thalassemia
of chronic myelomonocytic leukemia? c. Carcinoma lung
(AIIMS Nov 03) d. Hyperparathyroidism
a. Peripheral blood monocytosis more than 1×109L Ref: Harrison’s 18/e p937, 938
Ans. 190. c. Chronic... 191. b. t (9; 22) 192. a. and b 193. d. Persistent...
194. b. Imatinib
195. c. Aplastic anemia... 196. c. More than 20%... 197. c. Indolent course and easily...
198. b. Increased alkaline 199. a. Multiple myeloma
200. b. Azotemia 201. a. IgM
202. a. B2 microglobulin 203. a. Multiple myeloma
1056 Jaypee’s Triple A
204. Beta 1- macroglobulin is a tumor marker for: 212. All of the following are minor criteria for multiple
a. Multiple myeloma (Rajasthan 2009) Myeloma, except: (AIIMS Nov 2010)
b. Lung cancer a. Plasmacytosis 20%
c. Colonic neoplasm b. Multiple lytic lesions
d. Choriocarcinoma c. Plasmacytoma on tissue biopsy
Ref: Harrison’s 18/e p652 d. Monoclonal Ig spike < 2g/dl for IgA and < 3.5 for IgG
205. Diagnostic criteria of multiple myeloma include: Ref: Harrison’s 18/e p940
a. Plasmacytosis > 30% (Rajasthan 2009) 213. True about Myeloma is all, except: (AIIMS Nov 01)
b. Lytic bone marrow a. Plasma cell clonal proliferation
c. Bence jones proteinuria b. Common after 50 yrs of age
d. All of the above c. Amyloidosis can occur
Ref: Harrison’s 18/e p937-940 d. Protein casts in urine are made up of complete Ig chains
206. Which of the following disease can be associated with short Ref: Harrison’s 18/e p940, 17/e p703, 704
QT interval on ECG?: (DP PGMEE 2009) 214. Which of the following may be seen in Multiple Myeloma:
a. Chronic myeloid leukemia a. Decreased Calcium (PGI June 05)
b. Multiple myeloma b. Sclerotic bone lesion
c. Chronic lymphocytic leukemia c. Bone deposition
d. Hodgkin’s disease d. Renal failure
Ref: Harrison’s 18/e p1832, 17/e p701-704 Ref: Harrison’s 18/e p940, 17/e p938, 939
207. The signal most powerful predictor of survival in multiple 215. The following is the least useful investigation in multiple
myeloma is: (DP PGMEE 2010) myeloma: (AI 2007)
a. ‘M’ component production a. ESR
b. Bone marrow plasmocytosis b. X-Ray
c. Serum beta 2-microglobulin level c. Bone scan
d. Serum calcium level d. Bone marrow biopsy Ref: Harrison’s 18/e p940
Ref: Harrison’s 18/e p712,714, 17/e p702-704 216. An elderly male presents with headache, recurrent
208. Pneumococcal vaccine is advocated for all except: infections and multiple punched out lytic lesions of X-Ray
a. Diabetes mellitus (DP PGMEE 2010) skull. The investigation that will best help in establishing a
b. Sickle cell anaemia diagnosis is: (AI 2009)
c. Renal failure a. Protein electrophoresis
d. Cystic fibrosis b. Serum calcium
Ref: Harrison’s 18/e p1158-1159, 17/e p871 c. Alkaline phosphatase levels
d. Acid phosphatase levels
209. Which of the following is the least common feature of
Ref: Harrison’s 18/e p940
Multiple Myeloma: (AI 2012)
a. Bone pain 217. Multiple myeloma is characterized by all except: (PGI 2000)
b. Normocytic Normochromic Anemia a. Presence of light chains
c. Susceptibility to bacterial infection b. Monoclonal gammopathy
d. Hyperviscosity syndrome c. Polyclonal gammopathy
Ref: Harrison’s 18/e p939 d. Hypergammaglobulinemia
Ref: Harrison’s 18/e p936, 940, 941
210. Which of the following is not a major criteria for diagnosis
218. True about Multiple myeloma are all except: (PGI 2000)
of multiple myeloma? (AI 2006)
a. Bence Jones protein in urine
a. Lytic bone lesions
b. Hypogammalobulinemia
MEDICINE
Ans. 204. a. Multiple myeloma 205. d. All of the above 206. b. Multiple myeloma 207. c. Serum beta...
208. d. Cystic fibrosis
209. d. Hyperviscosity... 210. a. Lytic bone lesions 211. c. Plasmacytoma...
212. c. Plasmacytoma...
213. d. Protein casts in urine... 214. d. Renal failure 215. c. Bone scan
216. a. Protein...
217. c. Polyclonal gammopathy 218. d. Plasmacytosis < 10% 219. c. Beta 2...
Hematology 1057
220. A 58 years old woman, who had backache and recurrent a. Waldenstrom’s macroglobulinemia
chest infections for 6 months, develops sudden weakness b. Multiple myeloma
of the legs and urinary retention. Her investigations c. Hyperparathyroidism
show hemoglobin of 7.3 gm/dl, serum calcium 12.6 mg/dl, d. Osteomalacia Ref: Harrison’s 18/e p940 Robbin’s 8/e p611
phosphate 2.5 mg/dl, alkaline phosphatse 100 u/l, serum 227. Which of the following statement is not true? (AI 2005)
albumin 3 gm/dl, globulin 7.1 gm/dl, and urea 178 mg/dl. a. Patients with IgD myeloma may present with no evident
What is the most likely diagnosis? (AI 06) M-spike on serum electrophoresis
a. Lung cancer b. A diagnosis of plasma cell leukemia can be made if
b. Disseminated tuberculosis circulating peripheral blood plasmablasts comprise 14% of
c. Multiple myeloma peripheral blood white cells in a patient with white blood
d. Osteoporosis Ref: Harrison’s 18/e p938,939 17/e p703, 704 cell count of 1 × 109/L and platelet count of 88 × 109/L
221. Ramlal 65 yrs old male, presents with low back pain c. In smoldering myeloma plasma cells constitute 10-30% of
especially at L3, anaemia and fatigability. His investigation total bone marrow cellularity
profile reveals–Hb = 7 gm%, TLC-9000/cmm, DLC-N-55%, d. In a patient with multiple myeloma, a monoclonal light
L-30%, M-10%, E-1%, B-2%, Serum proteins-8 gm %, chain may be detected in both serum and urine
ratio-2.9/5.9, ESR-90 and serum creatinine-3.2 mg%. Likely Ref: Harrison’s 18/e p940
diagnosis is: (AIIMS Nov 01)
a. Waldenstroms macroglobulinemia
b. Multiple myeloma
H. BLEEDING/COAGULATION DISORDER
c. TB spine 228. Prothrombin time is prolonged in: (Karnatka 2011)
d. Secondaries in spine Ref: Harrison’s 18/e p940, 17/e p704 a. Factor X deficiency
222. Which of the following drugs is not used for the management b. Factor IX deficiency
of multiple myeloma: (DNB) c. Factor H deficiency
a. Bortezomib d. Lupus anticoagulant
b. Hydroxyurea Ref: Harrison’s 18/e p973, Davidson’s, 21/e p996, box 24.3
c. Melphelan 229. Coagulation defect involving both the arterial and venous
d. Cyclophosphamide Ref: Harrison’s 18/e p941, 942 system is seen in: (MHPGM-CET 2010)
223. All of the following drugs are used in the treatment of a. Factor V Leiden
Multiple myeloma, except: (PGI 09) b. Protein C deficiency
a. Bortezomib c. Antithrombin III deficiency
b. Melphelan d. Hyperhomocystinemia
c. Hydroxyurea Ref: Harrison’s 18/e p462, 17/e p367; Table 59-3
d. Cyclophosphamide Ref: Harrison’s 18/e p941, 17/e p705 230. Most common congenital bleeding disorder?
224. True about smoldering myeloma is: (PGI June 2008) a. von Willebrand disease (Maharashtra 2011)
a. Monoclonal gammopathy b. Glanzmann’s thrombasthenia
b. Lytic bone lesion c. Bernard Soulier disease
c. Hypercalcemia d. Any of the Above Ref: Harrison’s 18/e p971, 17/e p723
d. Bone Marrow Plasma cell < 10% 231. PT is prolonged in all except: (Kerala PG 09)
Ref: Harrison’s 18/e p940, 17/e p704 a. Factor VII deficiency
225. An 80 year old asymptomatic woman was detected to have a b. Vit k deficiency
monoclonal spike on serum electrophoresis (IgG levels 1.5 c. warfarin therapy
d. Heparine therapy
MEDICINE
g/dl). Bone marrow revealed plasma cells of 8%. The most
likely diagnosis is: (AI 2004) Ref: Harrison’s 18/e p610;Table 75-5 Robbin’s 8/e p666
a. Multiple myeloma 232. Use of heparin is contraindicated in all except:
b. Indolent myeloma a. Active TB (Kerala PG 10)
c. Monoclonal gammopathy of unknown significance b. Previous hypersensitivity
d. Waldenstorm’s macroglobulinemia c. Bacterial endocarditis
Ref: Harrison’s 18/e p940, 17/e p704, 706 d. DIC Ref: Katzung Chap 34
226. Ramesh 60 years, presents with generalized bone pain. 233. Which among the following factors level is not altered in vit
On examination there is elevated ESR of 100 mm, serum K deficiency? (Kerala PG 10)
globulin 7, lytic lesions in the skull, serum creatinine of 3.5 a. Prothrombin
mg/dL and serum calcium of 11 mg/dL. What is the most b. Factor VIII
likely diagnosis: (AIIMS Nov 2000) c. Factor VII
d. Factor X Ref: Harrison’s p367, 728-730
Ans. 220. c. Multiple myeloma 221. b. Multiple myeloma 222. b. Hydroxyurea 223. c. Hydroxyurea
224. a. Monoclonal...
225. c. Monoclonal... 226. b. Multiple myeloma 227. b. A diagnosis of plasma...
228. a. Factor IX deficiency 2
29. d. Hyperhomocystinemia 230. a. von Willebrand... 231. d. Heparine therapy
232. d. DIC
233. b. Factor VIII
1058 Jaypee’s Triple A
234. Severe coagulation factor VIII deficiency is said to occur 243. Platelet function may be assessed by (select two options):
with activity less than: (MP PG 2008) a. Platelet adhesion Assays (PGI Dec 02)
1. 1% b. BT
b. 5% c. CT
c. 25% d. PTT
d, 50% Ref: Harrison’s 17/e p726 e. APTT Ref: Harrison’s 18/e p965
235. Arterial thrombosis is a feature of all except: (Kerala PG 09) 244. Which of the following is not involved in intrinsic pathway?
a. Homocystinemia a. Factor XII (AI 2009)
b. Factor v leiden mutation b. Factor XI
c. Dysfibrogenemia c. Factor IX
d. Polycythemia vera d. Factor VII Ref: Harrison’s 18/e p974
Ref: Harrison’s 17/e p367 245. Which of the following helps in bridging the fibrin in a clot
236. The investigation done to rule out coagulation factor XIII and stabilizes the clot? (AI 2009)
deficiency is: (MP PG 2008) a. Factor XIII
a. Paracoagulation test b. Factor V
b. 5 M urea soubility test c. Factor VIII
c. Englobulin blood clot lysis time d. Factor III Ref: Harrison’s 18/e p974, 17/e p364
d. Russel’s viper venom clotting time 246. A patient has normal PT and Platelet Count. The aPTT is
Ref: Nelson’s 18/e p2071 increased and factor VIII levels are observed to be 60 Iu/dL
237. Hemophilia is: (UP 2011) (60%). There is no associated history of bleeding even after
a. Autosomal dominant a surgical procedure. The most likely diagnosis is:
b. Autosomal recessive a. Factor IX deficiency (AIIMS Nov 2011)
c. X-linked dominant b. Factor VIII inhibitors
d. X-linked recessive Ref: Harrison’s 18/e p974 c. Lupus anticoagulant
238. Warfarin effect is decreased by: (MP PG 2008) d. Thalassemia
a. Quinidine Ref: Harrison’s 18/e p974, Robbin’s 8/e p214, 651
b. Cloribrate 247. A child underwent a tonsillectomy at 6 years of age with no
c. Barbiturate complications. He underwent a preoperative screening for
d. Cholestrymine bleeding at the age of 12 years before an elective laparotomy,
Ref: KDT’s 6/e p602 and was found to have a prolonged partial thromboplastin
239. Antiplatelet action of aspirin lasting how many days: time but normal Prothrombin time. There was no family
a. 2 (Rajasthan 2009) history of bleeding. The patient is likely to have:
b. 4 a. Acquired vitamin K deficiency (AIIMS Nov 2004)
c. 7 b. Acquired liver disease
d. 10 c. Factor XII deficiency
Ref: Harrison’s 18/e p989, 990-35 d. Mild hemophilia A Ref: Harrison’s 18/e p978
240. Vitamin K dependent coagulation factors include: (AI 2010) 248. A 35 year old lady presents with an isolated prolongation of
a. II and III aPTT. Prothrombin time (PT) and platelet count are normal
b. IX and X and there is no obvious bleeding tendency. Two years back,
c. III and V she was operated for cholecystectomy and had no adverse
d. VIII and XII Ref: Harrison’s 18/e p980 bleeding episode. Which of the following should be the
next step in evaluating this patient: (AIIMS Nov 2010)
241. Which is most likely to be increased in Vit K deficiency:
a. Factor VIII Assay
MEDICINE
Ans. 234. a. 1% 235. b. Factor v leiden... 236. b. 5 M urea soubility... 237. d. X-linked recessive
238. c. Barbiturate
239. c. 7 240. b. IX and X 241. b. P.T.
242. a. Prolonged BT
243. a. Platelet adhesion... 244. d. Factor VII 245. a. Factor XIII
246. c. Lupus anticoa...
247. c. Factor XII deficiency 248. c. Russell viper venom... 249. b. Deficiency is...
Hematology 1059
250. Spontaneous muscle bleeding is typically seen in: a. Wiskott Aldrich syndrome (DNB Dec 2009)
a. Hemophilia (DNB 2009) b. Chediak-Higashi syndrome
b. Afibrinogenemia c. Job’s syndrome
c. von Willebrand’s disease d. Bruton’s Agammaglobulinemia
d. Scott’s syndrome Ref: Harrison’s 18/e p460 Ref: Harrison’s 18/e p969, 17/e p2060
251. True about Haemophilia A are all except: (AI 2001) 259. All of the following statements about Wiskott- Aldrich
a. PTT increased syndrome are true, except: (AIIMS NOV 2008)
b. PT increased a Autosomal Recessive disorder
c. Clotting time is increased. b. Eczematous Rash
d. Serum levels of factor VIII are decreased. c. Impaired platelet aggregation in response to agonist
Ref: Harrison’s 18/e p974, 17/e p726 d. Thrombocytopenia
252. In a patient of Hemophilia to be taken for dental extraction Ref: Harrison’s 18/e p969, 17/e p2060
true is all, except: (AIIMS Nov 01) 260. The presence of small sized platelets on the peripheral
a. All patients should be screened for HIV smear is characteristic of: (AIIMS Nov 03)
b. Extraction should be done under general anaesthesia and a. Idiopathic thrombocytopenic purpura
skilled anaesthetic care b. Bernard soulier syndrome
c. Factor VIII or cryoprecipitate can be needed c. Disseminated intravascular coagulation
d. Dose of Lignocaine required for anaesthesia is same as d. Wiskott Aldrich syndrome
that for normal individuals Ref: Harrison’s 18/e p969, 17/e p2060
Ref: Harrison’s 18/e p975, 17/e p727
261. Which is not true regarding Bernard soulier syndrome?
253. True about Von Willebrand’s disease is all except:
a. Ristocetin aggregation is normal (AI 2007)
a. Increased bleeding time (AIIMS 2000)
b. Aggregation with collagen and ADP is normal
b. Factor VIII c levels are decreased in circulation.
c. Large platelets
c. Increased platelet aggregation in response to Ristocetin
d. Thrombocytopenia
d. APTT is increased
Ref: Harrison’s 18/e p969
Ref: Harrison’s 18/e p971 972
262. All of the following can cause megakaryocytic
254. What is the most common cause for Budd chiary syndrome:
thrombocytopenia, except: (AIIMS Nov 04)
a. Right ventricular failure (AIIMS Nov 2000)
a. Idiopathic thrombocytopenic purpura
b. PNH
b. Systemic lupus erythematosus
c. Valve in hepatic veins
c. Aplastic anemia
d. Polycythemia vera Ref: Harrison’s 17/e p2082
d. Disseminated intravascular coagulation (DIC)
255. A seven year old girl presents with repeated episodes of
Ref: Harrison’s 18/e p968
bleeding into joints. APTT is prolonged and PT is normal.
The most likely diagnosis is. (AI 2009) 263. Thrombocytopenia occurs in all except: (AI 2001)
a. Factor VIII deficiency (Hemophilia A) a. Henoch schonlein purpura
b. Factor VII deficiency b. TTP
c. von Willebrand Disease c. DIC
d. Factor XII deficiency d. Wiskott Aldrich syndrome
Ref: Harrison’s 18/e p971, 17/e p725, 726 Ref: Harrison’s 18/e p2797, 17/e p2128
256. Best assay for deficiency of von Willebrand factor is: 264. Palpable purpura could occur in the following conditions,
a. Bleeding time (PGI Dec 01) except: (AI 2005)
b. BT + APTT a. Thrombocytopenia.
MEDICINE
c. BT + APTT + vWF-ristocetin factor assay b. Small-vessel vasculitis.
d. PT c. Disseminated gonococcal infection.
Ref: Harrison’s 18/e p973, 972 d. Acute meningococcemia.
Ref: Harrison’s 18/e p421, 17/e p334
257. Wiskott Aldrich syndrome is characterized by all except:
265. Which of the following statements about Acute Immune
a. Thrombocytopenia (AIIMS Nov 2009)
Thrombocytopenic Purpura is not true: (PGI 09)
b. Autosomal recessive
a. Autoimmune Mediated
c. Failure of aggregation of platelets in response to agonists
b. Massive Splenomegaly
d. Eczema
c. Increased Megakaryocytes in marrow
Ref: Harrison’s 18/e p969, 17/e p2060
d. IV immunoglobulins may be required
258. A patient presents with Thrombocytopenia, eczema and e. Usually self limiting condition
recurrent infection. The most probable diagnosis is: Ref: Harrison’s 18/e p421
Ans. 250. a. Hemophilia 251. b. PT increased 252. b. Extraction should be... 253. c. Increased platelet...
254. d. Polycythemia vera 255. c. Von Willebrand...
256. c. BT + APTT... 257. b. Autosomal recessive
258. a. Wiskott Aldrich...
259. a. Autosomal Recessive... 260. d. Wiskott Aldrich... 261. a. Ristocetin aggregation...
262. c. Aplastic anemia
263. a. Henoch schonlein... 264. a. Thrombocytopenia 265. a. Autoimmune Mediated
1060 Jaypee’s Triple A
266. The following laboratory determinants is abnormally 274. The following is the finding seen in DIVC: (AI 2007)
prolonged in ITP: (AI 2002) a. Increased fibrinogen, increased antithrombin III,
a. APTT increased thrombin-antithrombin III complexes
b. Prothrombin time b. Increased FD Pdecreased PT, increased antithrombin III
c. Bleeding time c. Increased FD Pprolonged PT, increased thrombin-
d. Clotting time Ref: Harrison’s 18/e p460, 17/e p366 antithrombin complexes
267. All of the following feature may be seen in thrombotic d. Increased FDPprolonged PT, reduced Platlets
thrombocytopenic purpura, except: (AI 2002) Ref: Harrison’s 18/e p979, 17/e p929
a. Fever 275. The most sensitive test for DIC is: (AI 2001)
b. Haemolysis a. Serum fibrinogen levels
c. Hypertension b. Serum levels of fibrin degradation products (FDP)
d. Low platelets count c. Prolonged PT and PTT
Ref: Harrison’s 18/e p969, 970, 17/e p722, 723 d. Thrombocytopenia
Ref: Harrison’s 18/e p979, 17/e p729
268. All of the following statements about Thrombotic
thrombocytopenic purpura (TTP) are true, except: 276. Disseminated intravascular coagulation (DIC) differs from
a. Microangiopathic Hemolytic Anemia thrombotic thrombocytopenic purpura. In this reference the
b. Thrombocytopenia DIC is most likely characterized by: (AI 2004)
c. Normal complement levels a. Significant numbers of schistocytes
d. Grossly abnormal coagulation tests b. A brisk reticulocytosis
Ref: Harrison’s 18/e p969, 17/e p722, 723 c. Decreased coagulation factor levels
d. Significant thrombocytopenia
269. A person presents with fever and altered consciousness. Ref: Harrison’s 18/e p969, 17/e p722, 723, 729
Investigations reveal anemia with fragmented red blood
277. 1 All of the following conditions are associated with venous
cells, platelet count of 20,000/mm3, serum creatinine of 3.0
and arterial thrombotic events, except: (AIIMS May 2011)
mg % and normal PT and aPTT. Which of the following is
a. Paroxysmal Nocturnal Haemoglobinuria (PNH)
the most appropriate treatment for the patient: (PGI 09)
b. Disseminated Intravascular Coagulation
a. Plasma exchange therapy
c. Idiopathic Thrombocytopenic Purpura (ITP)
b. Corticosteroids and Intravenous Immunoglobulins
d. Heparin Induced Thrombocytopenia (HIT)
c. Anticoagulation with Heparin
Ref: Harrison’s 18/e p416, 17/e p367, 741
d. Platelet transfusion Ref: Harrison’s 18/e p970, 17/e p723
278. Coagulation defects associated with ↑ed coagulation are
270. All of the following are used in the treatment of Thrombotic seen in: (PGI Dec 06)
Thrombocytopenic Purpura, except: (PGI 09) a. ↑ Protein C
a. Plasmapharesis b. ↑ Protein B
b. Corticosteroids c. ↑ Anti thrombin III
c. Immunotherapy d. Dysfibrinogenemia
d. Heparin Ref: Harrison’s 18/e p978
e. Platelet transfusion Ref: Harrison’s 18/e p970
279. H
ypercoagulability due to defective factor V gene is called:
271. Causes of DIC include: (PGI Dec 04) a. Lisbon mutation (AIIMS Nov 03)
a. Leukemia b. Leiden mutation
b. Masive transfusion c. Antiphospholipid syndrome
c. Abruptio placentae d. Inducible thrombocytopenia syndrome
d. All of the above
Ref: Harrison’s 18/e p986
Ref: Harrison’s 18/e p978, Table 116-2, 17/e p34
280. Most common inherited thrombotic disorder is:
272. Causes of DIC include: (PGI Dec 05)
MEDICINE
Ans. 266. c. Bleeding time 267. c. Hypertension 268. d. Grossly abnormal... 269. a. Plasma Exchange...
270. d. Heparin 271.
d. All of the above 272. e. All of the above 273. c. Decreased PTT
274. d. Increased FDP...
275. b. Serum levels of... 276. c. Decreased... 277. c. Idiopathic...
278. d. Dysfibrinogenemia 279. b. Leiden mutation 280. c. Factor V leiden... 281. d. Homocystenemia...
Hematology 1061
282. All of the following are aquired causes of Hypercoagulability, b. May present with Recurrent Abortions
except: (AI 2009) c. May occur with minimal clinical manifestations
a. Infection d. Thrombotic spells can be followed by severe life
b. Inflammatory Bowel disease threatening haemmorrhage Ref: Internet
c. Myeloproliferative disorders 290. Which of the following is recommended in a woman
d. Prolonged surgery > 1 hour Ref: Harrison’s 18/e p986 with Antiphospholipid Antibodies and history of prior
283. Causes of Deep venous thrombosis include all of the abortions/still birth: (AI 2010)
following, except: (AI 2009) a. Aspirin only
a. Diabetes Mellitus b. Aspirin + Low molecular weight Heparin
b. Oral contraceptives c. Aspirin + Low molecular weight Heparin +Prednisolone.
c. Paroxysmal Nocturnal Hemoglobinuria (PNH) d. No Treatment
d. Prolonged surgery Ref: Harrison’s 18/e p2734, 17/e p2082
Ref: Harrison’s 18/e p986, 17/e p632, 732, 660
284. A patient is admitted with 3rd episode of deep venous
thrombosis. There is no history of any associated medical
I. BLOOD TRANSFUSION
illness. All of the following investigations are required for
291. The following is not true of platelet transfusion:
establishing the diagnosis except: (AIIMS Nov 2004)
a. Useful in ITP (Feb DP PGMEE 2009)
a. Protein C deficiency
b. Used in DIC
b. Antithrombin III deficiency
c. Effective for 9-10 days
c. Antibodies to factor VIII
d. Effect decrease with repeated usage
d. Antibodies to cardiolipin
Ref: Harison’s 18/e p953
Ref: Harrison’s 18/e p986, 17/e p367, 732, 733, 734
292. Cryoprecipitate is rich in factor: (DP PGMEE 2010)
285. Anti Phospholipid Syndrome (APS) is associated with all of
a. II
the following except: (AI 2008)
b. V
a. Pancytopenia
c. VIII
b. Recurrent abortions
d. Fibrinogen
c. Venous thrombosis
Ref: Harrison’s 18/e p952,953-954, 17/e p710
d. Pulmonary hypertension
Ref: Harrison’s 18/e p987, 17/e p732, 1579, 2082 293. Platelet can be stored at: (DP PGMEE 2010)
a. 20-24°C for 5 days
286. Antiphospholipid Antibody (APLA) syndrome is associated
b. 20-24°C for 8 days
with all of the following except: (AI 2010)
c. 4-8°C for 5 days
a. Bleeding disorders
d. 4-8°C for 8 days Ref: Harrison’s 18/e p965, 17/e p709
b. Thrombotic disorders
c. Coagulation disorders 294. Which of the following statements about Acute Hemolytic
d. Recurrent fetal loss blood transfusion reactions is true: (PGI June 04)
Ref: Harrison’s 18/e p2734, 17/e p732, 1579, 2082 a. Complement mediated Hemolysis is seen
b. Type III Hypersensitivity is responsible for most cases
287. All of the following statements about Antiphospholipid
c. rarely life threatening
Antibody Syndrome (APLAb) are true, except: (AI 2010)
d. Renal blood flow is always mainatained
a. Single titre of Anticardiolipin is diagnostic
e. No need for stopping transfusion
b. Commonly presents with recurrent fetal loss
Ref: Harrison’s 18/e p954
c. May cause pulmonary hypertension
d. Warfarin is given as treatment 295. MC blood transfusion reaction is: (AI 2008)
a. Febrile nonhemolytic transfusion reaction
MEDICINE
Ref: Harrison’s 18/e p2734, 17/e p732, 1579, 2082
b. Hemolysis
288. The following condition is not associated with an Anti-
c. Transmission of infections
phopholipid syndrome: (AI 2002)
d. Electrolyte imabalnce
a. Venous thrombosis
Ref: Harrison’s 18/e p955, 17/e p711
b. Recurrent foetal loss
c. Thrombocytosis 296. All of the following infections may be transmitted via
d. Neurological manifestations blood transfusion, except: (AI 2002)
Ref: Harrison’s 18/e p2734, 17/e p732, 1579 a. Parvo B-19
b. Dengue virus
289. All of the following statements about Lupus Anticoagulant
c. Cytomegalovirus
are true, except: (AI 2009)
d. Hepatitis G virus
a. May present with an isolated prolongation of APTT
Ref: Harrison’s 18/e p954, Table 113-3
Ans. 282. a. Infection 283. a. Diabetes Mellitus 284. c. Antibodies to factor... 285. a. Pancytopenia
286. a. Bleeding disorders 287. a. Single titre of...
288. c. Thrombocytosis 289. d. Thrombotic spells...
290. b. Aspirin + Low...
291. c. Effective for 9-10 days 292. c. VIII 293. a. 20-24°C for 5 days
294. a. Complement...
295. a. Febrile nonhemolytic... 296. b. Dengue virus
1062 Jaypee’s Triple A
297. All of the following viruses may be transmitted by blood 305. ESR is increased in all except:
transfusion except: (AIIMS May 09) a. Multiple myeloma
a. Parvovirus B-19 b. Sickle cell anemia
b. Hepatitis G c. Tuberculosis of lung
c. Epstein Bar virus d. Rheumatoid arthritis
d. Cytomegalovirus Ref: CMDT 10 451, 472, 748; Harrison’s 18/e p2076, 17/e p704-637
Ref: Harrison’s 18/e p954, 956, Table 113-3, 17/e p710, 711, 712 306. Neutoropenic FUO is defined when Neutrophil count is less
298. Which of the following is the least likely complication after than? (Maharashtra 2011)
massive blood transfusion: (AIIMS May 09) a. 1000/mL
a. Hyperkalemia b. 500/mL
b. Citrate toxicity c. 350/mL
c. Hypothermia d. 200/mL
d. Metabolic Acidosis Ref: Harrison’s 18/e p956 Ref: Harrison’s18/e p163, 17/e p130; 134
299. All of the following are major complications of massive 307. Which of the following is not a feature of DIC?
transfusion, except: (AI 2006) a. Prolonged PT and PTT (Kerela PG 2008)
a. Hypokalemia b. Increased fibrin degradation products
b. Hypothermia c. Thrombocytopenia
c. Hypomagnesemia d. Platelet function defect
d. Hypocalcemia Ref: Harrison’s 18/e p957 Ref: Harrison’s 18/e p978, p684, 17/e p729
300. Which of the following investigations should be done 308. All of the following tumor secrete erythropoietin except:
immediately to best confirm a non matched blood a. Cerebellar hemangioblastoma (Kerala PG 10)
transfusion reaction: (AI 2010) b. Adrenocortical tumor
a. Indirect Coomb’s test c. Renal Cell Ca
b. Direct Coomb’s test d. Hepatocellular Ca
c. Antibody in patient’s serum Ref: Harrison’s 18/e p899, 17/e p673 table 103-2
d. Antibody in donor serum 309. A 45 years old male with skin pigmentation, Hypogonadism,
Ref: Harrison’s 18/e p954, 17/e p710 DM; diagnosis is: (WB PG 08)
301. Blood components products are: (PGI Dec 05) a. Wilson’s disease
a. Whole blood b. Hemochromatosis
b. Platelets c. Addison disease
c. Fresh frozen plasma d. Porphyria cutanea tarda
d. Leukocyte reduced RBC Ref: Harrison’s 18/e p3163-3164, 17/e p2431
e. All of the above Ref: Harrison’s 18/e p952, 17/e p709, 710 310. Severe itching after hot bath occurs commonly with:
302. Cryoprecipitate contains all except: (AIIMS Nov 07) a. Myeloid leukemia (MP PG 2008)
a. Factor VIII b. Lymphatic leukemia
b. Factor IX c. Multiple myeloma
c. Fibrinogen d. Polycythemia vera
d. VWF Ref: Harrison’s 18/e p953 Ref: Harrison’s 18/e p899, 17/e p309
303. Arterial blood Gas analysis in a bottle containing heparin 311. The characteristic morphologic appearance of red blood
causes a decrease in value of: cells in disseminated intravascular coagulopathy is:
a. pCO2 a. Spherocyte (MP PG 2008)
b. HCO3 b. Microcyte
c. pH
MEDICINE
c. Schistocyte
d. All of the above Ref: Harrison’s 18/e p956
d. Macrocyte
Ref: Harrison’s 18/e p978, 17/e p729
J. MISCELLANEOUS 312. Following are features of Acute Adult T- cell leukemia/
Lymphoma except: (MP PG 2009)
304. Leukemia associated syndrome: (AP 2011) a. Venous thrombosis
a. Option A b. Aplastic anaemia
b. Down syndrome c. Arterial thrombosis
c. Option C d. Haemolytic anaemia
d. Option d Ref: Harrison’s 18/e p932, 17/e p697; Casciato’s 5/e p716
Ref: Harrison’s 18/e p905
Ans. 297. c. Epstein Bar virus 298. d. Metabolic Acidosis 299. a. Hypokalemia 300. b. Direct Coomb’s...
301. e. All of the above
302. b. Factor IX 303. d. All of the above 304. b. Down syndrome
305. b. Sickle cell anemia 306. b. 500/mL
307. d. Platelet function defect 308. d. Hepatocellular
309. b. Heamocromatosis 310. d. Polycythemia vera
311. c. Schistocyte 312. d. Haemolytic...
Hematology 1063
313. Following are characteristics of paraxysmal nocturnal 321. Which statement below is not correct about Monoclonal
hemoglobinuria except: (MP PG 2009) Gammopathy of undetermined significance (MGUS)?
a. Venous thrombosis a. M-Protein in the serum is less than 30gm/I
b. Aplastic anaemia b. Bone marrow plasma cells more than 10%
c. Arterial thrombosis c. No evidence of other B-cell Proliferative disorders
d. Haemolytic anemia d. No bone lesion
Ref: Harrison’s 18/e p883, 17/e p660 Ref: Harrison’s 18/e p3474, 3478
314. Most radiosensitive tissue of body is: (UP 2011) 322. All of the following are WHO classified Myelodysplastic
a. Neuron Syndromes except: (AP 2010)
b. Hepatocytes a. CML
c. Lymphocytes b. Refractory anemia with excess blasts
d. Endothelial cells c. Refractory anemia with ringed sideroblasts
Ref: Harrison’s 18/e p692, 691 d. Refractory anemia
315. Philadelphia (PH) chromosome: (UP 2011) Ref: Harrison’s 18/e p895
a. Deletion 323. Dengue shock syndrome occurs due to: (DP PGMEE 2009)
b. Break DNA a. Super-imposed bacterial infection
c. Balanced translocation b. Capillary leak
d. Balanced transcription c. Addison’s crisis
Ref: Harrison’s 18/e p666, 915 d. Myocarditis
316. Acute hydrops is seen in: (Rajasthan 2009) Ref: Harrison’s 18/e p1028, 17/e p1239
a. Sickle cell anemia 324. Warm-antibody immunohemolytic anaemia is seen in all
b. ABO incompatibility except: (DP PGMEE 2010)
c. DM a. SLE
d. Hypertension b. a-Methyladopa ingestion
Ref: Internet c. Quinidine
317. In DIC which is false: (Rajasthan 2009) d. Infectious mononucleosis
a. PT is prolonged Ref: Harrison’s 18/e p881, 17/e p660
b. APPT is normal 325. Which one of the following is an Autosomal Dominant
c. Fibrinogen is decreased disorder: (AI 2002)
d. Thrombocytopenia a. Cystic fibrosis
Ref: Harrison’s 18/e p463, 979 b. Hereditary spherocytosis
c. Sickle cell anemia
318. Cell of origin of hairy cell leukemia is: (Comed K 2011)
d. G-6PD deficiency
a. B cell
Ref: Harrison’s 18/e p875
b. T cell
c. NK cell 326. PNH is associated with all of the following conditions,
d. Dendritic reticulum cell except: (AI 2002)
Ref: Davidson’s 21/e p1 a. Aplastic anemia
b. ↑ed LAP scores
319. Kaposi’s sarcoma is seen in: (J & K 2011)
c. Venous thrombosis
a. Patients suffering from HIV
d. Iron deficiency anemia
b. Patients suffering from Miliary TB Ref: Harrison’s 18/e p883 17/e p660, 661
c. Patients suffering from Pterygium
327. All of the following can be associated with PNH except:
MEDICINE
d. Patients suffering from Syphilis
a. Cerebral thrombosis (AI 2009)
Ref: Harrison’s 18/e p417
b. Budd chiari syndrome
320. The key factor in the transport of carbon dioxide as c. Pancytopenia
bicarbonate is: (DP PGMEE 2010) d. Massive splenomegaly Ref: Harrison’s 18/e p884
a. The high solubility of CO2 in H2O
328. A stem cell disorder affecting all the three cell lines
b. The presence of Hb in blood
platelets, RBCs and leucocytes is: (AIIMS May 01)
c. The presence of carbonic anhydrase in the erythrocytes a. Hemolytic anaemia
d. The acidic nature of carbon dioxide and the alkaline b. Paroxysmal cold haemoglobinuria
nature of bicarbonate c. Paroxysmal nocturnal haemoglobinuna
Ref: Ganong 22/e p669-670 d. Blackfan Diamond syndrome
Ref: Harrison’s 18/e p884, 17/e p660, 661
Ans. 313. c. Arterial thrombosis 314. d. Endothelial cells 315. c. Balanced... 316. b. ABO incompatibility
317. b. APPT is normal
318. a. B cell 319. a. Patients suffering... 320. c. The presence of...
321. b. Bone marrow...
322. a. CML 323. b. Capillary leak 324. d. Infectious...
325. b. Hereditary...
326. b. ↑ed LAP scores 327. d. Massive... 328. c. Paroxysmal...
1064 Jaypee’s Triple A
329. All of the following are true about PNH, except: 338. Pancytopenia with hypo cellular marrow is seen in all
a. Hypocellular marrow (PGI Dec 2000) except: (AIIMS Nov 07)
b. Budd-chirai syndrome a. PNH
c. Thrombosis b. Megaloblastic anemia
d. LAP score low Ref: Harrison’s 18/e p884 c. Myelodysplastic syndrome
330. Which of the following is NOT seen in Paroxysmal d. Dyskeratosis congenita
Nocturnal Hemoglobinuria: (AIIMS Nov 2000) Ref: Harrison’s 18/e p887, 17/e p663
a. Thrombosis 339. Leukoerythroblastic picture may be seen in all of the
b. Hemosiderinuria following, except: (AI 2003)
c. Decreased LDH a. Myelofibrosis
d. Thrombocytopenia b. Metastatic carcinoma
Ref: Harrison’s 18/e p884, 17/e p660, 661 c. Gaucher’s disease
331. PNH is associated with a deficiency of: (AI 2010) d. Thalassemia
a. DAF Ref: Harrison’s 18/e p896, Internet 17/e p669, 670, 674, 675
b. MIRL 340. A 45 year old female patient presents with symptoms
c. GPI Anchored protein of easy bruisability and frequent headaches. Physical
d. All of the above examination show a moderate splenomegaly. Blood counts
Ref: Harrison’s 18/e p884, 17/e p660, 661 shows a normal leucocyte count and a platelet count of 1000
332. PNH is associated with deficiency of: (AI 2010) × 103/cu mm. The leucocyte alkaline phosphatase score is
a. DAF (Decay accelerating factor) normal. Which one of the following is the drug of choice for
b. MIRL (Membrane inhibitor of reactive lysis) the treatment of this patient? (AIIMS Nov 03)
c. GPI Anchored Proteins (Glycosyl phosphatidyl Inositol a. Hydroxyurea
anchored proteins) b. Radioactive phosphorus
d. LFA (Lymphocyte function associated antigen) c. Anagrelide
Ref: Harrison’s 18/e p884, 17/e p660, 661 d. Interferon alpha
333. HAM test is based upon: (AIIMS Nov 06) Ref: Harrison’s 18/e p904, 17/e p676, 677
a. GPI Anchor Proteins 341. A patient presents with a platelet count of 700 × 109/L
b. Complement with abnormalities in size, shape and granularity of
c. Spectrin protein platelets. WBC count is 12 x 109/L, hemoglobin is 11g/dl
d. Mannose binding proteins and Philadelphia chromosome is absent. The most likely
Ref: Harrison’s 18/e p884, 17/e p661 diagnosis would be: (AIIMS May. 2006)
334. Pancytopenia with cellular marrow is seen in: (AI 2007) a. Polycythemia vera
a. PNH b. Essential thrombocythemia
b. G6PD deficiency c. Chronic myeloid leukemia
c. Acquired aplastic anemia d. Leukemoid reaction
d. Thalassemia Ref: Harrison’s 18/e p887 Ref: Harrison’s 18/e p903
335. Pancytopenia with hypercellular marrow may be seen due 342. Low Erythropoetin levels are seen in: (DNB 2012)
to all of the following except: (DNB 2011) a. Aplastic Anemia
a. Myelodysplasia b. Renal failure
b. Paroxysmal Nocturnal Hemoglobinuria c. Obesity
c. Dyskeratosis congenita d. Hepatoma Ref: Harrison’s 18/e p850
d. Sarcoidosis Ref: Harrison’s 18/e p887 343. All of the following are Preleukemic conditions, except:
336. Pancytopenia with hypercellular bone marrow is seen in: a. Paroxysmal Nocturnal Haemoglobinuria (PNH)
MEDICINE
Ans. 329. a. Hypocellular... 330. c. Decreased LDH 331. d. All of the above 332. c. GPI Anchored...
333. b. Complement
334. a. PNH 335. c. Dyskeratosis... 336. a and b.
337. d. G6PD Deficiency
338. d. Dyskeratosis congenita 339. d. Thalassemia 340. c and d.
341. b. Essential...
342. b. Renal failure 343. b. Paroxysmal Cold... 344. d. Primary...
Hematology 1065
345. Which of the following drugs is recommended for the 347. Clot solubility in 5 M-Urea is a test for: (DP PGMEE 2010)
treatment of Heparin Induced thrombocytopenia: (AI 2010) a. Factor 13
a. Abciximab b. Factor 12
b. Lepirudin c. Platelet function
c. Warfarin d. Plasmin inhibitor
d. Alteplase Ref: Harrison’s 18/e p458, 17/e p369
Ref: Harrison’s 18/e p968 348. Screening tests for B cell defects: (DP PGMEE 2009)
346. A 22-year old man presents with history of bleeding from a. Isohemagglutinin titers
gums for the last 6 months. On investigation the Hb was b. CD 4 levels
found to be 8.2 gm% TLC 4400/mm, DLC N 64%, L 27%, E c. NItroblue tetrazolium dye test
3%, M 6% and platelet count of 20,000/cumm. Which one d. Candida albicans intradermal skin test
of the following investigation would be most useful in Ref: Harrison’s 18/e p2703, 17/e p2035
establishing the diagnosis: (DP PGMEE 2010) 349. Marker for NK cell activity is: (DP PGMEE 2009)
a. Bleeding time a. CD3
b. Prothrombin time b. CD4
c. Partial thromboplastin time c. CD56
d. Bone marrow examination d. CD19
Ref: Harrison’s 18/e p896, 17/e p666 Ref: Harrison’s 18/e p2658, 17/e p2024, 2058-2029
MEDICINE
Ans. 345. b. Lepirudin 346. d. Bone marrow... 347. a. Factor 13 348. a. Isohemagglutinin...
349. c. CD56
2. CARDIOVASCULAR SYSTEM
A. ARTERIAL PULSES AND JVP 8. Pulsus paradoxus is seen in (select three correct options):
a. Cardiac tamponade (PGI June 06)
1. Dicrotic pulse differ from pulse bisferiens in which of the b. Constrictive pericarditis
following? (MHPGM-CET 2007, 2010) c. HOCM
a. Characterized by two palpable peaks, both in systole d. AR
b. Characterized by two palpable peaks, one in systole and e. Severe asthma
other in diastole Ref Harrison’s 18/e p1824, 1975 17/e p1384
c. Seen in HOCM
9. Water hammer pulse is seen in: (AIIMS May 07)
d. Indicates high stroke output
a. Aortic stenosis
Ref: Harrison’s 18/e p1823, 17/e p1383
b. Aortic regurgitation
2. Slow Y decent seen in JVP: (Rajasthan 2009) c. Aortic stenosis and Aortic regurgitation
a. TR d. Mitral regurgitation
b. TS
Ref: Harrison’s 18/e p1943,1944, 17/e p1476
c. AS
d. MR 10. All of the following phases of the jugular venous pulse and
their causes are correctly matched, except: (AI 2002)
3. Which one of the following is the correct statement
a. ‘c’ wave - onset of atrial systole
regarding findings in JVP? (AP 2010)
a. Cannon wave: complete heart block b. ‘a-x’ descent - atrial relaxation
b. Slow ‘y’ descent: Tricuspid regurgitation c. ‘v-y’ - emptying of blood from right atrium into right
c. Giant ‘c’ wave: Tricuspid stenosis ventricle
d. Increased JVP with prominent pulsations: SVC obstruction d. ‘y-a’ ascent - filling of the right atrium from the vena cava
Ref: Harrison’s 18/e p1905 Ref: Harrison’s 18/e p1823, 17/e p1384
4. All of the following are causes of giant ‘a’ wave in JVP 11. Which of the following is the correct statement regarding
except: (Comed K 2011) findings in JVP: (AI 2002)
a. Tricuspid stenosis a. Cannon wave: Complete heart block
b. Pulmonary stenosis b. Slow vy descent: Tricuspid regurgitation
c. Pulmonary hypertension c. Giant c wave: Tricuspid stenosis
d. Aortic stenosis d. Increased JVP with prominent pulsations: SVC
Ref: Harrison’s 18/e p1822-1823 17/e p1384 Obstruction Ref: Harrison’s 18/e p1823, 17/e p1384
5. C.V.P (Central Venous Pressure) and pulmonary wedge 12. ‘C’ wave in JVP is due to: (AIIMS Nov 07)
pressure give an accurate assessment of all the following a. Atrial contraction
except: (DP PGMEE 2010) b. Tricuspid valve bulging into right atrium
a. Tissue perfusion c. Right atrial filling
b. Volume depletion
d. Rapid ventricular filling
c. Volume overload
Ref: Harrison’s 18/e p1823, 17/e p1384
d. Myocardial function Ref: Bailey 24/e p69
13. C wave in JVP indicates: (AI 2009)
6. Bisferiens pulse is seen in all except: (PGI Dec 05)
a. Atrial contraction
a. AS + AR
MEDICINE
b. AR b. Bulging of tricuspid valve
c. Hypertrophic cardiomyopathy c. Ventricle systole
d. TOF d. Rapid ventricular filling
Ref: Harrison’s 18/e p1824, 17/e p1383 Ref: Harrison’s 18/e p1823, 17/e p1384
7. Pulsus paradoxus is present in all except: (PGI Dec 05) 14. Typical JVP finding in cardiac tamponade: (PGI June 2000)
a. Emphysema a. Absent ‘Y’ descent
b. Pulmonary embolism b. Prominant ‘a’ wave
c. Hypovolemic shock c. Absent ‘a’ wave
d. Hypertrophic cardiomyopathy d. Prominent ‘Y’ wave
Ref: Harrison’s 18/e p1824,1825 17/e p1975 Ref: Harrison’s 18/e p1972, 17/e p1490
b. MR
c. Opening snap
c. PDA
d. Ejection click Ref: Harrison’s 18/e p1827, 17/e p1385
d. PS
e. LBBB Ref: Harrison’s 18/e p1826 30. Systolic thrill in left 2nd and 3rd intercostals space may be
seen in all of the following, except: (AI 2009)
22. Reverse splitting of S2 is seen in all except (select two
a. Subpulmonic VSD
options): (PGI Dec 05)
b. Pulmonic stenosis
a. LBBB
c. Ebstein’s anomaly
b. WPW type A
d. Pink TOF
c. LV pacing
d. Systemic hypertension
e. Post-stenotic dilatation in AS Ref: Harrison’s 18/e p1826
Ans. 15. b. Mitral Stenosis 16. b. 4-12 mmHg 17. b. Atrial septal... 18. c. Short PR interval
19. b. LBBB 20. a. ASD and e. PS 21. a, b and d 22. b and c
23. a, c and e 24. a and c 25. a. Occurs due... 26. c. Heard during...
27. c. Ejection click 28. a. It is a high... 29. d. Ejection click 30. d. Pink TOF
Cardiovascular System 1069
31. An early systolic murmur may be caused by all of the 39. Continuous murmur is found in (select three options):
following except: (AI 2003) a. AS combined with AR (PGI Dec 04)
a. Small ventricular septal defect b. Systemic A V fistula
b. Papillary muscle dysfunction c. PDA with reversal f shunt
c. Tricuspid regurgitation d. Aortopulmonary window
d. Aortic stenosis Ref: Harrison’s 18/e p1827-28, 17/e p1387 e. Rupture of sinus valsalva
32. All of the following murmurs may be heard in patients with Ref: Harrison’s 18/e p1829, 17/e p1388
aortic regurgitation except: (AIIMS Nov 02)
a. High-pitched decrescendo diastolic murmur.
b. Soft, low pitched mid distolic rumbling murmur.
C. DISEASES OF HEART RATE, RHYTHM
c. Mid-systolic ejection flow murmur AND CONDUCTION
d. Pansystolic murmur.
Ref: Harrison’s 18/e p1944, 17/e p1476 40. ST depression and T wave inversion in V1 to V6 and aVL
33. Mid-diastolic Murmur with presystolic accentuation is leads indicate: (Feb DP PGMEE 2009)
typically seen in: (DNB) a. Anterolateral wall AMI
a. Mitral stenosis b. Posterior wall AMI
b. Mitral Regurgitation c. Inferior AMI
c. Aortic stenosis d. Lateral wall AMI
d. MVP Ref: Harrison’s 18/e p1930, 17/e p1467 Ref: Harison’s 18/edn. Pg. 1836-37
34. Which of the following murmurs increase with Valsalva 41. Features in favour of ventricular tachycardia are all the
maneuver? (AIIMS Nov 2010, May 2009) following except: (Karnataka 2011)
a. MR a. AV dissociation
b. VSD b. Capture/fusion beats
c. AS c. Good response to carotid sinus massage
d. HOCM Ref: Harrison’s 18/e p, 17/e p d. A history of myocardial infarction
Ref: Davidson’s, 21/e p567, box 18.34
35. A young patient presents with a systolic murmur at the
apex. The murmur increases on both handgrip and valsalva 42. Prominent Y descent is caused by following except:
maneuver. Which of the following conditions is most a. Constrictive Pericarditis (Maharashtra 2011)
likely? (AI 04) b. Restrictive Cardiomyopathy
a. HOCM c. Cardiac tamponade
b. AS d. Right ventricular infraction
c. MVP Ref: Harrison’s 18/e p1823
d. VSD Ref: Harrison’s 18/e p1830, 17/e p1387 43. ECG finding suggestive of Left ventricular hypertrophy?
36. What is false in relation to Carey Coombs Murmur? a. RaVL + SV3>28 mm (Maharashtra 2011)
a. Delayed Diastolic Murmur (AIIMS Nov 06) b. SV1 + (RV5 or RV6)>35 mm
b. Seen in Rheumatic Fever c. Evidence of Left atrial hypertrophy
c. Associated with AR d. All of the above
d. Low Pitched Murmur Ref: Internet Ref: Harrison’s 18/e p1834, 1835, 17/e p1391, 1392
37. A continuous murmur is heard in all of the following 44. ECG feature of acute cor pulmonale (e.g Pulmonary
conditions except: (AIIMS May 05) embolism): (MHPGM-CET 2010)
a. Ventricular septal defect with aortic regurgitation a. S1Q3T3 pattern
b. Patent ductus arteriosus b. Slow “r” wave progression in leads V1 to V4
MEDICINE
c. Coronary arteriovenous fistula c. Atrial fibrillation
d. Venous hum d. All of the above three
Ref: Harrison’s 18/e p1829, 17/e p1388 Ref: Harrison’s 18/e p1834, 17/e p1391
38. Continuous murmur is present in (select two options): 45. All of the following are features of early atropinisation
a. PDA (PGI June 06) except: (Kerala PG 10)
b. AS with AR a. Blurring of vision
c. Shunt between pulmonary and subclavian artery b. Absence of sweat
d. VSD with AR c. Drying of mouth
Ref: Harrison’s 18/e p1829, 17/e p1388 d. Bradycardia
Ref: Goodman and Gillman, Chap-7
Ans. 31. d. Aortic stenosis 32. d. Pansystolic murmur 33. a. Mitral stenosis 34. a. and d
35. c. MVP 36. c. Associated with AR 37. a. Ventricular septal 38. c. Shunt between
39. b. Systemic... 40. a. Anterolateral wall AMI 41. c. Good response... 42. c. Cardiac tamponade
43. d. All of the above 44. d. All of the above three 45. d. Bradycardia
1070 Jaypee’s Triple A
46. All of the following are seen in Cardiac tamponade except: 55. Common causes of atrial fibrillation is/are: (J and K 2011)
a. Electrical alterans (Kerala PG 09) a. Coronary artery disease
b. Pulsus paradoxus b. Alcohol
c. Increased JVP c. Hypertension
d. Bradycardia d. All of the above
Ref: Harrison’s 18/e p1975, 17/e p1490 Ref: Harrison’s 18/e p1881
47. Increased QT interval seen in: (UP 2011) 56 Tall and peaked” T “waves in ECG are characteristic of:
a. Hypercalcemia a. Hypokalemia (J and K 2010)
b. Digitalis toxicity b. Hyperkalemia
c. Romano ward syndrome c. Hypernatremia
d. Hyperthermia Ref: Internet d. Hypocalcemia
48. Which is going to best declare the case as that of interatrial Ref: Harrison’s 18/e p1864
septal defect with other cardiac abnormalities? 57. Torsade de ponte’s arrhythmia is associated with:
a. Elevated pressure in left atrium (Rajasthan 2009) a. Short QT (J and K 2010)
b. Elevated pressure in right atrium b. Long QT
c. Elevated PO2 in pulmonary artery c. Left axis deviation
d. Systolic murmur d. Short PR
49. Osborn wave is seen in: Ref: Harrison’s 18/e p175, 1891
a. Hypokalemia 58. A QRS duration between 100 and 120 milliseconds suggests
b. Hypomagnesaemia all of the following, except: (DNB)
c. Hypothermia a. Normal
d. CVA b. Left anterior Fascicular Block
Ref: Harrison’s 18/e p1818, 1831 c. Left posterior Fascicular Block
50. QRS complex is seen in lst, avL denote: (Rajasthan 2009) d. Left Bundle Branch Block
a. Old Anterior MI Ref: Harrison’s 18/e p1835
b. Hyperacute Anteroseptal MI 59. Wide QRS complex ≥ 0.12 seconds may be seen in all of the
c. Hyperacute Anterior MI- following except: (DNB)
d. Old anteroseptal MI Ref: Harrison’s 18/e p1834 a. Hyperkalemia
51. Osborn wave is seen in: (Rajasthan 2008) b. Wolf Parkinson White Syndrome
a. Hypokalemia c. Ventricular Tachycardia
b. Hypomagnesaemia d. Left Anterior Fascicular Block
c. Hypothermia Ref: Harrison’s 18/e p1835
d. CVA Ref: Harrison’s 18/e p1831, 1834, 1818 60. The ECG of a 40 year old male was recorded using standard
52. Cardiac tamponade is characterized by the following, bipolar limb leads. The sum of voltage of the three standard
except: (AP 2012) leads was found to be 5 millivolts. This indicates:
a. Pulsus paradoxus a. A normal heart (AIIMS May 05)
b. Electrical alternans b. Right ventricular hypertrophy
c. Kussmaul’s sign c. Left ventricular hypertrophy
d. RV diastolic collapse by echo d. Increased cardiac muscle mass
Ref: Harrison’s 18/e p1972-1973 Ref: Guyton 10/e p127
53. Which of the following treatments is appropriate for tall 61. Low QRS voltage on ECG indicates: (DNB 2009)
peaked T waves on ECG? (DP PGMEE 2009) a. Pulmonary embolism
b. Pericardial effusion
MEDICINE
a. Atropine IV
b. Nitroprusside IV c. Cor pulmonale
c. Inhaled salbutamol d. Infective endocarditis Ref: Internet
d. Inhaled betamethasone 62. ST elevation is seen in all of the following conditions except:
Ref: Harrison’s 18/e p1860, 17/e p283-284 a. Myocardial infarction (AIIMS 2002)
54 ECG feature of Hyperkalaemia is: (J and K 2011) b. Coronary artery spasm
a. Flattened T wave c. Constrictive pericarditis
b. ST depression d. Ventricular aneurysm
c. Peaked T waves Ref: Harrison’s 18/e p1394, 1395
d. U waves
Ref: Harrison’s 18/e p1864
Ans. 46. d. Bradycardia 47. c. Romano ward syndrome 48. b. Elevated... 49. c. Hypothermia
50. d. Old anteroseptal MI 51. c. Hypothermia 52. c. Kussmaul’s sign 53. c. Inhaled salbutamol
54. c. Peaked T waves 55. d. All of the above 56. b. Hyperkalemia 57. b. Long QT
58. d. Left Bundle... 59. d. Left Anterior... 60. d. Increased 61. b. Pericardial
62. c. Constrictive
Cardiovascular System 1071
63. All of the following may cause ST segment elevation on d. Mitral regurgitation
EKG, except: (AI 2005) Ref: Harrison’s 18/e p1881, 17/e p1428
a. Early repolarization variant. 71. A chronic alcoholic develops palpitations suddenly after
b. Constrictive pericarditis. alcohol binge. Which of the following arrhythmia is most
c. Ventricular aneurysm. commonly associated with alcohol binge in the alcoholics?
d. Prizmetal angina. a. Ventricular fibrillations (AIIMS Nov 01)
Ref: Harrison’s 18/e p1837, 17/e p1395 b. Verticular premature contractions
64. QT prolongation is seen in all, except: (AIIMS June 2000) c. Atrial flutter
a. Hypothermia d. Atrial fibrillation Ref: Harrison’s 18/e p1881, 17/e p1428
b. Digitalis toxicity 72. A person with mitral regurgitation and atrial fibrillation
c. Hypocalcemia presents with syncope. On examination the person has a
d. Romanowand syndrome heart rate of 55. What is the most probable cause ? (AI 2007)
Ref: Harrison’s 18/e p1836, 17/e p1395 a. Digitalis toxicity
65. Hypocalcemia is characterized by all of the following b. Incomplete heart block
features except: (AI 03) c. Stroke
a. Numbness and tingling of circumoral region d. Subarachnoid Haemorrhage
b. Hyperactivity tendon reflexes Ref: Goodman & Gilman 11/e p889
c. Shortening of Q-T interval in ECG 73. In a patient with chronic atrial fibrillation with a regular
d. Carpopedal spasm beat of 60/min, the most probable cause is: (AIIMS May 01)
Ref: Harrison’s 18/e p1838, 1395 a. Sleep
66. The following ECG findings are seen in Hypokalemia: b. Digitalis toxicity
a. Increased PR interval with ST depression (AI 2007) c. Sino nodal block
b. Increased PR interval with peaked T wave d. Hypothyroidism Ref: Goodman & Gilman 11/e p889
c. Prolonged QT interval with T wave inversion 74. Drug of choice for Paroxysmal Supraventricular Tachycardia
d. Decreased QT interval with ST depression (PSVT) is: (DNB June 2010)
Ref: Harrison’s 18/e p1838, 17/e p1395 a. Metaprolol
67. All of the following are the electrocardiographic features of b. Amiadarane
Hyperkalemia, except: (AIIMS May 04) c. Adenosine
a. Prolonged PR interval d. Lidocaine
b. Prolonged QT interval Ref: Harrison’s 18/e p1883 17/e p1433
c. Sine wave patterns 75. What is the drug of choice to control supraventricular
d. Loss of P waves Ref: Harrison’s 18/e p1838 tachycardia: (AIIMS Nov 05)
68. Which of the following conditions does NOT cause right a. Adenosine
axis deviation in ECG? (Comed K 2010) b. Propranolol
a. Lying down posture c. Verapamil
b. End of deep inspiration d. Digoxin
c. Right ventricular hypertrophy Ref: Harrison’s 18/e p1883, 17/e p1433
d. Right bundle branch block 76. In a patient with wide-complex tachycardia, the presence
Ref: Harrison’s 18/e p1834, 17/e p1391; Dent Essentials, of all of the following in the ECG indicates ventricular
High yield NBDE Part 1 review, Kaplan Medical, 2007 edn. tachycardia except: (AIIMS Nov 03)
69. Which of the following ECG changes is least likely in: a a. Atrioventricula r dissociation
patient with left pneumothorax: (AIIMS Nov 2000) b. Fusion beats
MEDICINE
a. Inversion of T wave c. Typical right bundle branch block
b. Left axis deviation d. Capture beats
c. Small R wave 77. Which of the following is the most characteristic EKG
d. Electrical alternans finding of ventricular premature beats (select one single
Ref: Harrison’s 18/e p1838, 17/e p1395 best correct option): (PGI June 04)
70. Atrial fibrillation may occur in all the following conditions, a. Fusion beat
except: (AIIMS May 03) b. Narrow QRS complex
a. Mitral stenosis c. AV Dissociation
b. Hypothyroidism d. Wide QRS complex
c. Dilated cardiomyopathy e. Capture beat
Ref: Harrison’s 18/e p1890, 1435
Ans. 63. b. Constrictive... 64. b. Digitalis toxicity 65. c. Shortening... 66. a. Increased
67. b. Prolonged... 68. a. Lying down posture 69. b. Left axis deviation 70. b. Hypothyroidism
71. d. Atrial fibrillation 72. a. Digitalis toxicity 73. b. Digitalis toxicity 74. c. Adenosine
75. a. Adenosine 76. c. Typical right ... 77. d. Wide QRS complex
1072 Jaypee’s Triple A
78. All of the following may be seen in ventricular premature 86. In the treatment of severe bradycardia, all of the following
beats, except: (PGI June 04) can be the best modality of treatment except:
a. Fusion beat a. Atropine (AIIMS Nov. 05)
b. Narrow QRS complex b. Pacing
c. AV Dissociation c. Isoproterenol
d. Wide QRS complex d. Diltiazem Ref: Harrison’s 18/e p1867, 1874, 17/e p1416
e. Capture beat 87. A 30-year-old male presents with severe pain chest,
Ref: Harrison’s 18/e p1890,1891 17/e p1435 breathlessness, hypotension and ECG shows ST elevation
79. Accelerated Idioventricular Rhythm (AIVR) is the most in V3, V4, V5 and V6 leads. He will be best treated with:
common arrhythmia associated with: (AI 2010) a. Streptokinase (DP PGMEE 2009)
a. Dilated cardiomyopathy b. t-PA
b. Myocardial Reperfusion c. Heparin
c. Digitalis Intoxicatiun d. PTCA
d. Myocarditis Ref: Harrison’s 18/e p2021 17/e p1537, Braundwald 7/e p1183
Ref: Internet 88. Corrected QT (QTC) is: (Karnataka 2010)
80. Congenital long QT syndrome can lead to: a. 0.12-2 secs
a. Complete heart block (AIIMS May 2003) b. >0.S secs
b. Polymorphic ventricular tachycardia c. 2 to 3 secs
c. Acute myocardial infarction d. < 0.46 secs
d. Recurrent supraventricular tachycardia Ref: Harrison’s 18/e p1832, 17/e p1389; Marriott’s Practical ECG, 11/e
Ref: Harrison’s 18/e p1891 p63; Pediatric Cardiology for Practitioners, Myung K Park, 5/e p440;
81. WPW syndrome is caused by: (DNB 2012) Nursing Rapid Fire Drug facts, 2004 edn.
a. Bundle Branch Block
b. Right sided accessory pathway
c. Ectopic pacemaker in atrium
D. DISEASES OF THE HEART VALVES
d. Left bundle Branch block Ref: Harrison’s 18/e p1890
89. All of the following clinical findings are consistent with
82. The most common reentrant tachycardia associated with severe mitral stenosis except: (Karnataka 2010)
WPW syndrome is: (PGI 2012) a. Atrial fibrillation
a. Orthodromic AV reentry b. Pulsatile liver
b. Antidromic AV reentry c. Opening snap late after S2
c. Rapidly conducting AF d. Pulmonary vascular congestion
d. None Ref: Harrison 18/e p1889 Ref: OP Ghai, 7/e p387; Davidson’s, 20/e p619, Box 18.104
83. All of the following statements about WPW syndrome are 90. Rapid high frequency fluttering of anterior mitral valve
true, except: (PGI Dec 05) leaflets during systole on 2D ECHO is characteristically
a. More common in females seen with (MHPGM-CET 2010)
b. Rt ventricular aberrant is commonly seen a. Mitral regurgitation
c. Incidence ↓ with age b. Tricuspid regurgitation
d. Heart is structurally normal c. Pulmonary regurgitation
e. Incidence is 0.3 to 0.7%. Ref: Harrison 18/e p1890 d. Aortic regurgitation
84. The drug of choice in patients with Wolff-Parkinson-White Ref: Harrison’s 18/e p1942, 17/e p1477
syndrome with atrial fibrillation is: (AIIMS Nov 03) 91. Closed mitral valvotomy is contraindicated in: (UP 2011)
a. Digitalis a. Associated with tricuspid regurgitation (TR)
b. Procainamide
MEDICINE
Ans. 78. b. Constrictive... 79. b. Myocardial... 80. b. Polymorphic... 81. b. Right sided...
82. a. Orthodromic... 83. e. Incidence... 84. c. Verapamil 85. a, b and c
86. d. Diltiazem 87. d. PTCA 88. d. 0.46 secs 89. b. Pulsatile liver
90. d. Aortic ... 91. a. Associated... 92. a. Small
Cardiovascular System 1073
93. Y steep decent is seen in: (Rajasthan 2009) c. Ostium primum
a. Constrictive pericarditis d. Pulmonary thromboembolism
b. Cardiac tamponade Ref: Ghai 8/e p413
c. RVH 102. The following clinical signs are consistent with severe
d. TR mitral stenosis: (J and K 2010)
94. Pulmonary plethora is seen in all except: (Rajasthan 2009) a. Atrial fibrillation
a. TGA b. Opening snap later after S2
b. TOF c. Right ventricular heave
c. TAPVC d. Loud p2
d. ASD Ref: Internet Ref: Harrison’s 18/e p1929, 1930
95. The severity of mitral stenosis is assessed by: 103. Enlarged Pulsative liver with ascitis is typically seen in:
a. Left atrial enlargement (Rajasthan 2009) a. Tricuspid Regurgitation (TR) (AIIMS May 2011)
b. Loudness of S1 b. Mitral Regurgitation (MR)
c. Loudness of opening snap c. Mitral stenosis
d. A2-OS gap d. Pulmonary stenosis (PS)
Ref: Harrison’s 18/e p3606 Ref: Harrison’s 18/e p1823-1824
96. A 25 year old lady presents with arrythmias, palpitation, 104. All of the following may be seen as severity of mitral
substernal chest pain poorly related to exertion and finds stenosis increases except: (DNB Dec 2011)
mid systolic clicks on auscultation. What is the investigation a. Pulsatile liver
to be done? (AP 2010) b. Atrial fibrillation
a. Arteriography
c. Opening snap delayed from S2
b. Echocardiography
d. Length of murmur is prolonged
c. Chest X-ray
Ref: Harrison’s 17/e p1467, 1929
d. Electrophysiological tests
105. Severity of mitral stenosis is assessed by: (AIIMS May 03)
97. False about Mitral Valve Prolapse is (AP 2011)
a. Loud opening snap
a. Auscultatory finding required along with echo for
b. Length of murmur
diagnosis
c. Loud S1
b. Slight prolapse is asymptomatic
d. Splitting S2
c. Degree of sound is proportional to prolapse
Ref: Harrison’s 18/e p1930
d. Fibrosis of valve is associated with bad prognosis
Ref: Harrison’s 18/e p1937 106. Severity of mitral stenosis is assessed by (select two
98. Sudden Cardiac Death (SCD) can occur in the following options): (PGI June 02)
disease, except: (AP 2012) a. Loud S1
a. Prolonged Q-T syndrome b. S2-OS gap
b. W-P-W syndrome c. Prolonged diastolic murmur
c. Ventricular septal defect d. S3
d. Hypertrophic cardiomyopathy e. S4
Ref: Harrison’s 18/e p2239, 2241 Ref: Harrison’s 18/e p1930
99. The most common location of ventricular septal defect is: 107. The severity of mitral regurgitation is decided by all of the
a. Perimembranous (Comed K 2010) following clinical findings except: (AIIMS May 03)
b. Muscular a. Presence of mid-diastolic murmur across mitral valve.
c. Doubly — committed subarterial b. Wide split second heart sound.
d. Inlet septal Ref: OP Ghai, 7/e p40 c. Presence of left ventricular S3 gallop.
MEDICINE
100. Most common type of ASD is: (DP PGMEE 2010) d. Intensity of systolic murmur across mitral valve
a. Ostium primum Ref: Harrison’s 18/e p1935, 17/e p1470
b. Ostium secundum 108. A 59 year old man with severe myxomatous mitral
c. Sinusvenosus type regurgitation is asymptomatic, with a left ventricular
d. Endocardial cushion defect ejection fraction of 45% and an endsystolic diameter index
Ref: Harrison’s 18/e p1921, 17/e p1459-1460 of 2.9 cm/m2. The most appropriate treatment is: (AI 2005)
101. A 21-year old male presents with exertional dyspnea, raised a. Mitral valve repair or replacement
JVP and loud P2. ECG shows right axis deviation. All of the b. No treatment
following conditions are possible except:(DP PGMEE 2009) c. ACE inhibitor therapy
a. Atrial septal defect d. Digoxin and diuretic therapy
b. Mitral stenosis Ref: Harrison’s 18/e p1936, 17/e p1471
Ans. 109. a. Myxomatous... 110. b. Majority of ... 111. c. Echocardiography... 112. c. Aortic stenosis
113. d. Delayed peak...
114. c. Aortic... 115. b > d 116. d. Pansystolic
117. c. AR
118. c. William syndrome 119. b. Dilatation... 120. a. TR
121. a. Erythema...
122. d. Rheumatic fever.
Cardiovascular System 1075
123. Which of the following is a late manifestation of acute 131. Aschoff’s nodules are seen in: (AI 05)
Rheumatic fever? a. Subacute bacterial endocarditis
a. Subcutaneous nodule b. Libman-sacks endocarditis
b. Carditis c. Rheumatic carditis
c. Chorea d. Nonbacterial thrombotic endocarditis
d. Erythema marginatum Ref: Harrison’s 18/e p2754
Ref: Harrison’s 18/e p2752)
132. True about Rheumatic fever: (PGI Dec 03)
124. Infective endocarditis in drug abusers mostly affects: a. Chorea is aggravated during pregnancy
a. Mitral valve
b. Chorea and arthritis co-existing
b. Aortic valve
c. Subcutaneous nodules are tender
c. Tricuspid valve
d. Pulmonary valve d. Erythema multiforme seen
Ref: Harrison’s 18/e p1053 Ref: Harrison’s 18/e p2754, 17/e p2094
125. Typical features of Acute pericarditis includes: 133. True statement about Rheumatic fever in children (select
(J and K 2012) two options): (PGI Dec- 03)
a. Chest pain identical to that of myocardial infraction a. Polyarthritis
b. A friction rub that is best heard in the axilla in b. Caused by α hemolytic streptococci
midexpiration c. Erythem marginatum is most common manifestation
c. ST elevation on the ECG with upward concavity d. MC valve involvement is Mitral
d. Elevation of the serum creatine kinase e. Erythema marginatum is common in face
Ref: Harrison’s 18/e p1971 Ref: Harrison’s 18/e p2753-54
126. Which is a minor criteria for diagnosis of RF according to 134. True about Erythema Marginatum in Acute Rheumatic
modified Jones criteria? (AI 2007) fever is: (PGI Dec 01)
a. ASO titre a. Pruiritic
b. Past History of Rheumatic Fever b. Commonly involves face
c. Fever
c. Common manifestation of Acute Rheumatic fever
d. Subcutaneous nodules
d. Usually associated with carditis
Ref: Harrison’s 18/e p2755, 17/e p2095
Ref: Harrison’s 18/e p2754, 17/e p2094
127. Which of the following is not included in Jone’s Major
Criteria (AIIMS Nov 2010) 135. True about subcutaneous nodule in Rheumatic fever (select
a. Pancarditis two options): (PGI Dec 03)
b. Chorea a. Non tender
c. Subcutancous nodule b. Most common manifestation
d. High ESR Ref: Harrison’s 18/e p2755, 17/e p2095 c. Present in extensor surfaces
128. Major criteria of Rheumatic fever include (select three d. Associated with arthritis
options): Ref: Harrison’s 18/e p2754, 17/e p2094
a. Chorea 136. Diagnostic criterion for Infective Endocarditis include all,
b. Erythema nodosum except: (DNB June 2011)
c. Arthritis a. Positive Echocardiogram
d. Fever b. Positive Blood culture
e. Carditis Ref: Harrison’s 18/e p2755, 17/e p2095 c. Raised ESR
129. True about acute rheumatic fever (select three options): d. Positive Rheumatoid Factor Ref: Harrison’s 18/e p1055
a. Chorea (PGI Dec-02)
137. Infective endocarditis is least likely to occur in:
MEDICINE
b. Erythema nodosum
a. Atrial septal defect (AIIMS June 97, 98) (AIIMS May 01)
c. Arthritis
b. Small ventricular septal defect
d. Caused by antecedent α - hemolytic streptococcus
infection c. Mitral valve prolapse
e. Carditis Ref: Harrison’s 18/e p, 17/e p d. Tetrology of Fallot Ref: Harrison’s 18/e p1923, 17/e p1460
130. A 10-year-old boy, Pappu, died of acute rheumatic fever. All 138. Which of the following is least likely to be associated with
the following can be expected at autopsy except: (AI 02) Infective Endocarditis: (AI 2012)
a. Ashoff nodules a. Small ASD
b. Rupture of chordae tendinae b. Small VSD
c. Mc Callum patch c. Mild MR
d. Fibrinous pericarditis d. Mild MS Ref: Harrison’s 18/e p1863, 17/e p1922)
Ref: Harrison’s 18/e p2754
Ans. 123. c. Chorea 124. d.Tricuspid valve 125. c. ST elevation ... 126. c. Fever
127. d. High ESR
128. a, c and e 129. a, c and e 130. b. Rupture of ...
131. c. Rheumatic carditis 132. a. Chorea...
133. a and d 134. d. Usually associated ...
135. c. Present in...
136. c. Raised ESR... 137. a. Atrial septal defect 138. a. Small ASDl
1076 Jaypee’s Triple A
139. Mitral valve vegetations do not usually embolise to: 148. Flat vegetations in pockets of valves are due to: (AI 2000)
a. Lung (AI 2001) (AIIMS Nov 2001) a. Rheumatic heart disease
b. Liver b. Libman sacks Endocarditis
c. Spleen c. NBTE
d. Brain Ref: Harrison’s 18/e p1054 d. Infective endocarditis Ref: Internet
140. A woman has septic abortion done; vegetation on tricuspid 149. Firm warty vegetations along the line of closure of valves is
valve is likely to go to: (AI 2001) due to: (AI 2000)
a. Septic infarcts to lung a. Rheumatic heart disease
b. Liver b. Libman Sacks Endocarditis
c. Spleen infarcts c. NBTE
d. Emboli to brain Ref: Harrison’s 18/e p1055, 17/e p791 d. Infective Endocarditis Ref: Internet
141. Which of the following is least likely to cause infective
endocarditis: (AI 06)
a. Staphylococcus albus
F. CONGENITAL HEART DISEASES
b. Streptococcus faecalis
150. The following is true of tetralogy of fallot except:
c. Salmonella typhi
a. Squatting (Rajasthan 2009)
d. Pseudomonas aeruginosa Ref: Harrison’s 18/e p1053
b. Clubbing
142. Bacterial endocarditis is most commonly caused by: (AI 06) c. Cyanosis
a. α -Hemolytic Streptococci d. Increased lung vascularity
b. β -Hemolytic Streptococci Ref: Harrison’s 18/e p456, 1926
c. Staphylococcus aureus
151. 100% oxygen improves cyanosis in all except:
d. Cardiobaterium
a. Tetralogy of fallot (DP PGMEE 2010)
e. Staph epidermidis
b. Bronchial asthma
143. A patient with a prosthetic heart valve develops endocarditis c. Eosinophilic pneumonia
eight months after valve replacement. Most likely organism d. Interstitial lung disease
responsible is: (AIIMS Nov 2010) Ref: Harrison’s 18/e p456, 17/e p1592)
a. Staphylococcus Aureus
152. Anatomical closure of ductus arteriosus in full term
b. Staphylococcus Epidermidis
neonates occurs at (J and K 2012)
c. Streptococcus Viridans
a. Birth
d. HACEK group Ref: Harrison’s 18/e p1053, 17/e p790
b. 3–4 days
144. Acute Infective Endocarditis with abscess formation is most c. 10–21st days
commonly associated with: (DNB 2012) d. 30th days Ref: Ghai 8/e p417
a. Listeria
153. The commonest mode of inheritance of congenital heart
b. Staphylococcus
disease is (AI 2002)
c. Streptococcus
a. Autosomal dominant
d. Enterococcus Ref: Harrison’s 18/e p1053, 17/e p790
b. Autosomal recessive
145. Which of the following have most friable vegetation: c. Sex linked dominant
a. Infective endocarditis (AI 2010) d. Multifactorial Ref: Robbins 6th/592; 5th/57
b. Libman Sack’s endocarditis
154. The commonest mode of inheritance of congenital heart
c. Rheumatic heart disease
disease is (AI 2002)
d. SLE Ref: Harrison’s 18/e p1053
a. Autosomal dominant
146. Vegetations on undersurface of A.V. valves are found in: b. Autosomal recessive
MEDICINE
Ans. 139. a. Lung 140. a. Septic infarcts... 141. c. Salmonella typhi... 142. c. Staphylococcus aureus
143. c. Streptococcus... 144. b. Staphylococcus... 145. a. Infective... 146. b. Limban sack’s...
147. a. SLE... 148. b. Libman ... 149. a. Rheumatic heart 150. d. Increased...
151. a. Tetralogy... 152. b. 3–4 days... 153. d. Multifactorial 154. d. Multifactorial
155. c and e
Cardiovascular System 1077
156. All of the following are cyanotic heart diseases, except: 165. Fallots tetralogy manifestation (select two options):
a. TOF (PGI June 05) a. Left axis deviation (PGI June 06)
b. PDA b. Left ventrucular hypertrophy
c. Tricuspid Atresia c. VSD
d. Eisenmenger’s complex Ref: Harrison’s 18/e p1920 d. Blalock taussig shunt is between pulmonary artery and
157. The following features are true for tetralogy of Fallot, subclavian artery
except: (AI 2006) e. Morphine is contraindicated in cyanotic spells
a. Ventricular septal defect Ref: Harrison’s 18/e p1926-1927
b. Right ventricular hypertrophy 166. A young female presents with history of dyspnoea on
c. Atrial septal defect exertion. On examination, she has wide, fixed split S2 with
d. Pulmonary stenosis Ref: Harrison’s 18/e p1926 ejection systolic murmur (III/VI) in left second intercostal
158. Essential criteria for TOF includes all except: space. Her EKG shows left axis deviation. The most
a. Valvular stenosis (AIIMS Nov 07) probable diagnosis is: (AIIMS May 04) (AIIMS May 03)
b. Infundibular stenosis a. Total anomalous pulmonary venous drainage.
c. Over riding of aorta b. Tricuspid atresia.
d. RVH Ref: Ghai 6/e p406 c. Ostium primum atrial septal defect.
d. Ventricular septal defect with pulmonary arterial
159. Which of the following is a component of Pentalogy of
hypertension.
Fallot: (AI 2007)
Ref: Harrison’s 18/e p1921, 1922 17/e p1459
a. Atrial Septal Defect (ASD)
b. Patent Ductus Arteriosus (PDA) 167. All of the following are true about ASD except: (AI 2001)
c. Coarctation of Aorta (COA) a. Right atrial hypertrophy
d. Left Venticular Hypertrophy (LVH) b. Left atrial hypertrophy
Ref: Harrison’s 18/e p1926 c. Right ventricular hypertrophy
d. Pulmonary hypertension
160. All of the following statements about Tetralogy of fallot are
Ref: Harrison’s 18/e p1921, 1922 17/e p1459
true, except:
a. JVP is normal 168. Presence of a pansystolic murmur of mitral regurgitation
b. Second Heart sound is single with left axis deviation in a patient with ASD suggest:
c. Ejection systolic murmur is third left intercostal space a. TGA (DNB)
d. First Heart sound is soft Ref: Harrison’s 18/e p1926-27 b. Ostium secondum with floppy mitral valve
c. Ostium primum with floppy mitral valve
161. All of the following are true regarding Tetralogy of fallot
d. Pulmonary Hypertension
except: (AIIMS May 05)
Ref: Harrison’s 18/e p1921, 1922
a. Ejection systolic murmur in second intercostal space
b. Single second heart sound 169. In which of the following conditions left atrium is not
c. Predominantly left to right shunt enlarged: (AI 2006)
d. Normal jugular venous pressure a. Ventricular septal defect
Ref: Ghai 6/e p406-409 b. Atrial septal defect
c. Aortopulmonary window
162. In which of the following a ‘Coeur en Sabot’ shape of the
d. Patent ductus arteriosus
heart is seen: (AI 2004)
Ref: Harrison’s 18/e p1921-22
a. Tricuspid atresia
b. Ventricular septal defect 170. True statement about ductus arteriosus is: (AI 2000)
c. Transposition of great arteries a. It undergoes anatomic closure within 24 hrs of birth
d. Tetralogy of Fallot Ref: Harrison’s 18/e p1926 b. Forms the ligamentum venosum in later life
MEDICINE
c. It is induced to close by high levels of prostaglandins
163. Blalock and Taussig shunt is done between: (AI 2006)
d. May cause a machinery murmur by its patency.
a. Aorta to pulmonary artery
Ref: Harrison’s 18/e p1923-24
b. Aorta to pulmonary vein
c. Subclavian artery to pulmonary vein 171. A five year old child presents with Left ventricular
d. Subclavian vein to artery Ref: Harrison’s 18/e p1927 hypertrophy and central cyanosis. What is the most
probable diagnosis? (AIIMS Nov 2000)
164. Potts shunt is: (AI 2001)
a. Tricuspid atresia
a. Rt subclavian artery to rt pulmonary artery
b. Eisenmenger syndrome
b. Descending aorta to left pulmonary artery
c. Tetrology of Fallot
c. Left subclavian to left pulmonary artery
d. Total anomalous pulmonary venous drainage
d. Ascending aorta to right pulmonary artery
Ref: Harrison’s 18/e p1927
Ref: Harrison’s 18/e p1926-1927
Ans. 156. b. PDA 157. c. Atrial septal defect 158. a. Valvular stenosis... 159. a. Atrial Septal Defect (ASD)
160. d. First Heart...
161. c. Predominantly left... 162. d. Tetralogy of Fallot 163. a. Aorta to pulmonary artery
164. b. Descending aorta... 165. c and d
166. c. Ostium primum... 167. b. Left atrial...
168. c. Ostium primum...
169. b. Atrial septal defect 170. d. May cause a ... 171. a. Tricuspid atresia
1078 Jaypee’s Triple A
172. A child presents with LVH and pulmonary complications. a. Phaeochromocytoma (AI 2002)
ECG shows left axis deviation. Most likely diagnosis is: b. Carcinoid syndrome
a. TOF (AI 2001) c. Coarctation of the aorta
b. Tricuspid atresia d. Superior Mediastinal syndrome
c. TAPVC Ref: Harrison’s 18/e p1925
d. VSD 179. Which condition is most commonly associated with
173. A patient complains of intermittent claudication, dizziness coarctation of aorta? (AI 2008)
and headache; likely cardiac lesion is: a. PDA
a. TOF (AI 2001) (AIIMS Nov 2000) b. Bicuspid aortic valve
b. ASD c. Aortic stenosis
c. PDA d. VSD Ref: Harrison’s 18/e p1925, 17/e p1462
d. Coarctation of aorta Ref: Harrison’s 18/e p1925, 17/e p1462 180. Coarctation of Aorta is most commonly associated with:
174. A 1-month-old boy is referred for failure to thrive. On a. Bicuspid Aortic valve (AI 2011)
examination, he shows feature of congestive failure. The b. Patent Ductus Arteriosus (PDA)
femoral pulses are feeble as compared to branchial pulses. c. Ventricular Septal Defect (VSD)
The most likely clinical diagnosis is: (AI 2006) d. Atrial Septal Defect (ASD)
a. Congenital aortic stenosis Ref: Harrison’s 18/e p1925, 17/e p1462
b. Coarctation of aorta 181. Inferior rib notching is seen in all except: (AIIMS June 2000)
c. Patent ductus arteriosus a. Coarctation of aorta
d. Congenital aortoiliac disease Ref: Harrison’s 18/e p1925 b. Classical blalock tausing operation
175. A 27 year old man in noted to have blood pressure of c. SVC obstruction
170/100 mmHg. He has prominent aortic ejection click and d. Neurofibromatosis Ref: Internet
murmurs heard over the ribs on the both sides anteriorly 182. All of the following causes death in coarctation of Aorta
and over the back posterioly. In addition.the pulses in except: (PGI June 2000)
the lower extremities are feeble and he complains of mild a. Infective endocarditis
claudication with exertion. The most likely diagnosis is b. CCF
a. Atrial septal defect (AIIMS Nov 04) c. Intracranial hemorrhage
b. Aortic stenosis d. Anterior MI
c. Coarctation of the aorta Ref: Harrison’s 18/e p1925, 17/e p1926
d. Cardiomyopathy Ref: Harrison’s 18/e p1925, 17/e p1462
183. The most common type of total anomalous pulmonary
176. A 4 1/2- year-old girl always had to wear warm socks even venous connection is: (AI 2005)
is summer season. On physical examination, it was noticed a. Supracardiac.
that she had high blood pressure and her femoral pulse b. Infracardiac.
was weak as compared to radial and carotid pulse. A chest c. Mixed.
radiograph showed remarkable notching of ribs along with d. Cardiac. Ref: Internet
their lower borders. This was due to: (AIIMS Nov 02)
184. A five day old, full term male infant was severely cyanotic
a. Femoral artery thrombosis.
at birth. Prostaglandin E was administered initially and
b. Coarctation of aorta.
later ballooned atrial septosomy was done which showed
c. Raynaud’s disease.
improvement in oxygenation. The most likely diagnosis of
d. Takayasu’s arteritis. Ref: Harrison’s 18/e p1925, 17/e p1462
this infant is: (AI 2004)
177. A Ten year old boy presents to the pediatric emergency a. Tetralogy of Fallot
unit with seizures. Blood pressure in the upper extremity b. Transposition of great vessels
measured as 200/140 mm Hg. Femoral pulses were not c. Truncus arteriosus
MEDICINE
palpable. The most likely diagnosis amongst the following d. Tricuspid atresia Ref: Harrison’s 18/e p1927
is: (AI 2010)
185. A neonate has recurrent attacks of abdominal pain, restless
a. Takayasu Aortoarteritis
irritability and diaphoresis on feeding. Cardiac auscultation
b. Renal parenchymal disease
reveals a nonspecific murmur. He is believed to be at risk
c. Grandmal seiures
for M.I. Likely diagnosis here is: (AI 2001)
d. Coarctation of Aorta
a. ASD
Ref: Harrison’s 18/e p1925
b. VSD
178. A 20 year old young man presents with exertional c. TOF
dyspnoea, headache, and giddiness. On examination, there d. Anomalous coronary artery
is hypertension and LVH. X-ray picture shows notching of Ref: Harrison’s 18/e p1927
the anterior ends of the ribs. The most like diagnosis is:
Ans. 172. b. Tricuspid atresia 173. d. Coarctation of aorta 174. b. Coarctation of aorta 175. c. Coarctation of aorta
176. b. Coarctation of aorta 177. d. Coarctation of aorta
178. c. Coarctation of aorta 179. b. Bicuspid aortic valve
180. a. Bicuspid aortic...
181. d. Neurofibromatosis 182. d. Anterior MI 183. a. Supracardiac
184. b. Transposition of... 185. d. Anomalous coronary artery
Cardiovascular System 1079
186. Eisenmenger syndrome is characterized by all except: c. S3 gallop
a. Return of left ventricle and right ventricle to normal size. d. Hepatomegaly
b. Pulmonary veins not distended. (AI 2005) Ref: Internet
c. Pruning of peripheral pulmonary arteries. 195. Left ventricular Hypertropy is caused by all, except:
d. Dilatation of central pulmonary arteries. a. M.S (Mitral stenosis) (AIIMS Nov 09)
Ref: Harrison’s 18/e p1923 b. AS (Aortic Stenosis)
c. M.R (Mitral Regurgitation)
d. AR (Aortic Regurgitation)
G. CONGESTIVE HEART FAILURE Ref: Harrison’s 18/e p1931, 17/e p1467
187. Factors that may precipitate acute decompensation in 196. CCF is associated with increase in all the following except:
patients with chronic heart failure include a. Right atrial mean pressure (AI 2007)
a. Dietary indiscretion (Karnataka 2011) b. Serum Sodium
b. Infection c. Urea
c. Nonsteroidal anti-inflammatory drugs d. Nor epinephrine
d. All of the above Ref: Harrison’s 18/e p1906,1907 17/e p1447
Ref: Harrison’s 18/e p1907, 17/e p1448, table 227-3 197. Positive hepatojugular reflux is found in conditions except:
188. All of the following are seen in arrest except: a. Tricuspid regurgitation (AI 2009)
a. Electromechanical dissociation (Kerela PG 2008) b. Right heart failure
b. Atrial fibrillation c. Decreased after load
c. Ventricular fibrillation d. PS Ref: Harrison’s 18/e p1823, 17/e p1384
d. Ventricular tachycardia Ref: Davidson, 20/e p555 198. Positive Hepatojugular Reflux is found in all of the
189. Which of the following is not present in cardiac tamponade? following conditions, except: (AIIMS May 2011)
a. Pulsus paradoxus (Rajasthan 2009) a. Tricuspid Regurgitation
b. Kussmaul’s sign b. Precapillary Pulmonary Hypertension
c. Pulmonary edema c. Right Ventricular Infarction
d. Hypertension Ref: Harrison’s 18/e p1972, 1973 d. Decreased after-load Ref: Harrison’s 18/e p1823, 1384
190. CCF which is not true (AP 2011) 199 A Patient comes with sudden respiratory distress. On
a. Echo is diagnostic examination, bilateral basal crepts are present over chest
b. Myocardial infraction is cause suggestive of pulmonary edema. Alveolar wedge pressure
c. Includes 4 classes is normal. The likely cause is: (AIIMS June 2000)
d. ↑ JVP Ref: Harrison’s 18/e p1906 a. Narcotic overdose
b. Congestive heart failure
191 Asterixis is seen in all, except: (J and K 2010)
c. Myocardial infarction
a. Renal failure
d. Cardiogenic shock Ref: Harrison’s 18/e p1948
b. Hepatic failure
c. Left ventricular failure 200. Pulmonary edema associated with normal PCWP is
d. Hypercapnia Ref: Harrison’s 18/e p2250 observed. Which of these is not a cause: (AI 01)
a. High altitude
192. Cardiac output is decreased in all except: (J and K 2010)
b. Cocaine overdose
a. Dilated cardiomyopathy
c. Post cardiopulmonary bypass
b. CCF due to mitral regurgitation in rheumatic heart
d. Bilateral renal artery stenosis
diseases
c. Pericardial effusion 201. A 45-year-old woman underwent a modified radical
d. Pulmonary edema due to circulatory congestion in acute mastectomy 4 years ago. She was treated for multiple bone
MEDICINE
glomerulonephritis Ref: Harrison’s 18/e p2218 metastases with cyclophosphamide, doxorubicin, and
fluorouracil for 6 months. She is complaining of exertion
193. All the following are features of right sided heart failure,
on exercise, swelling of the legs, and swelling around eyes
except: (AI 2009)
in the morning. On examination, she has bilateral rales in
a. Increased PCWP
the lungs, S1, S2 audible, S3, S4 gallop present. Her BP is
b. Pulsatile liver
149/117 mm Hg, PR is 80/min, and RR is 18/min. What is the
c. Increased JVP
most likely cause for her cardiac condition? (AIIMS May 06)
d. Positive hepatojugular reflex
a. Systolic dysfunction CHF
194. Which of the following is not a major Framingham criteria
b. Drug induced cardiac toxicity
in CHF: (AIIMS Nov 2008)
c. Metastatic cardiac disease
a. Cardiomegaly
d. Pneumonia Ref: Harrison’s 16/e p584
b. Paroxysmal nocturnal dyspnea
Ans. 186. a. Return of left ... 187. d. All of the above 188. b. Atrial fibrillation 189. d. Hypertension
190. a. Echo is diagnostic 191. c. Left ventricular...
192. d. Pulmonary edema... 193. a. Increased PCWP
194. d. Hepatomegaly
195. a. M.S (Mitral stenosis) 196. b. Serum Sodium 197. c. Decreased after load
198. d. Decreased after... 199. a. Narcotic overdoses
200. d. Bilateral renal... 201. b. Drug induced
1080 Jaypee’s Triple A
202. All of the following are used in the initial management a. Low HDL
of acute life threatening cardiogenic pulmonary edema, b. Increased waist-hip ratio
except: (AI 2012) c. Low fibrinogen
a. Digoxin d. High LDL Ref: Harrison’s 18/e p1987-1989, 3280
b. Morphine 210. Tangier’s disease is associated with (AP 2012)
c. Furosemide a. High LDL
d. Positive Pressure Ventilation Ref: Harrison’s 18/e p2236 b. High TG
c. High HDL
203. All of the following medications may be used in congestive
d. Low HDL Ref: Harrison’s 18/e p3154
cardiac failure, Except: (AIIMS Nov 09)
211. All of the following are risk factors for atherosclerosis
a. Spironolactone
except: (AI 06)
b. Nitrates
a. Increased waist - hip ratio
c. Nesiritide
b. Hyperhomocysteinemia
d. Trimetazidine Ref: Harrison’s 18/e p1907-12 c. Decreased fibrinogen levels
204. All of the following are true about starting betablocker d. Decreased HDL levels
therapy in cases of CHF, except: (AIIMS Nov 2010) Ref: Harrison’s 18/e p1987, 17/e p1505
a. They should be initiated at the effective doses 212. Best predictor for future risk of cardiovascular
b. They should be gradually increased over weeks events,amongst the following is: (AI 2009)
c. Special precautions should be taken in NYHA class III a. hs CRP
and IV b. Lipoprotein ‘a’
d. Carvedilol and Metoprolol are the preferred Drugs c. Homocysteine
Ref: Harrison’s 18/e p1909, 17/e p1950 d. Interleukin 6 Ref: Harrison’s 18/e p1990
205. In a patient with chronic congestive cardiac failure, all of 213. Which of the following is the best marker to predict future
the following drugs prolong survival except: cardiac events (DNB June 2012)
a. hs CRP
a. Metoprolol (AIIMS May 04)
b. Homocystine
b. Carvedilol
c. Interleuken-6 Ref: Harrison’s 18/e p1990
c. Enalapril
214. The amino acid which is associated with atherosclerosis is:
d. Digoxin Ref: Harrison’s 18/e p1906, 1908 17/e p1448,1451
a. Lysine (AIIMS May 2006)
206. Which of the following statements is true about High b. Homocysteine
Attitude Pulmonary Edema (HAPE)? (AIIMS Nov. 06) c. Cysteine
a. Not exacerbated by exercise d. Alanine
b. Associated with pulmonary vasoconstriction Ref: Harrison’s 18/e p1987, 17/e p1505
c. Occurs only in unacclamatized individuals
215. Risk factors for coronary artery disease (CAD) (select two
d. Associated with low cardiac output
options): (PGI June 01)
Ref: Ganong 24/e p651
a. High HDL
b. Low LDL
c. Increased homocysteine levels
H. ATHEROSCLEROSIS, ISHEMIC HEART
d. Decreased fibrinogen levels
DISEASE e. Increased lipoproteins
Ref: Harrison’s 18/e p1987, 17/e p1505
207. Tuberoeruptive xanthomas are characteristic of familial
216. Predisposing factors for coronary artery disease include, all
forms of: (MP PG 2009) except: (PGI Dec 02)
a. Hyperchotesterolemia
MEDICINE
a. Homocysteinemia
b. Dysbetalipoproteinemia b. ↑ Lipoprotein B
c. Defective apoB- 100 c. ↑ Fibrinogen
d. Sitosterolemia Ref: Harrison’s 18/e p3149, 17/e p2419 d. ↑ plasminogen activator inhibitors 1
208. Which of the following is a lipid mediator? (Rajasthan 2008) Ref: Harrison’s 18/e p1987, 17/e p1505
a. Interleukin 1 217. Raised serum level of lipoprotein-(a) is a predictor of:
b. Interleukin 8 a. Cirrhosis of liver (AI 03)
c. Tumor necrosis factor a
b. Rheumatic arthritis
d. Leukotriene B4 Ref: Harper 28/e p200
c. Atherosclerosis
209. All of the following are risk factors for atherosclerosis, d. Cervical cancer Ref: Harrison’s 18/e p1987
except: (AP 2012)
Ans. 202. a. Digoxin 203. d. Trimetazidine 204. a. They should be... 205. d. Digoxin
206. b. Associated with... 207. b. Dysbetalipoproteinemia
208. d. Leukotriene B4 209. c. Low fibrinogen
210. d. Low HDL
211. c. Decreased 212. a. hs CRP 213. a. hs CRP
214. b. Homocysteine
215. c and e 216. b. ↑ Lipoprotein B 217. c. Atherosclerosis
Cardiovascular System 1081
218. Which of the following increases the susceptibility to
coronary artery disease: (AI 03)
I. MYOCARDIAL INFARCTION
a. Type V hyperlipoproteinaemia
225. 50 years male had acute myocardial infarction 4 days ago.
b. Von willebrands disease
Now he compains of substernal chest pain with radiation
c. Nephrotic syndrome
d. Systemic lupus erythematosus into left arm and jaw. Most likely cause of the chest pain is:
Ref: Internet a. Reinfaraction (MP PG 2010)
b. Pericarditis
219. In an old patient, the best indicator of probability of
c. Right ventricular infaraction
developing cardiovascular disease can be calculated by:
d. Rupture of the anterior wall
a. LDL/HDL ratio. (AIIMS May 02)
b. Triglycerides Ref: Harrison’s 18/e p2033, 17/e p1542
c. Total cholesterol 226. All of the following are true about Right Ventricular
d. Serum LDL Infarcts, except: (DNB 2012)
Ref: Harrison’s 18/e p1987 a. Nocturia
220. Most important predictor of coronary artery disease is: b. Hepatomegaly
a. VLDL (AIIMS May 09) c. Ascitis
b. LDL d. Normal JVP Ref: Harrison’s 18/e p2032
c. Chylomicron 227. All of the following statements about Universal Definition
d. LDL/HDL of Myocardial Infarction are true, except: (AIIMS Nov 2011)
Ref: Harrison’s 18/e p1987 a. Sudden, unexpected Cardiac death with symptoms of
221. All of the following dietary goals are recommended for Ischemia.
patients with high risk of coronary heart disease, except: b. Elevation of cardiac biomarkers with new regional wall
a. LDL cholesterol < 100 mg/dl (AI 2011) motion abnormality
b. Saturated fat < 7 % of total calories c. Three times increase in Troponin levels after Percutaneous
c. Salt restriction < 6 gm/day Coronary Intervention (PCI)
d. Avoid Alcohol d. Three times increase in Troponin levels after Coronary
Ref: Internet Artery Bypass Grafting (CABG) Ref: Harrison’s 18/e p105
222. Which of the following dietary interventions has shown to 228. All of the following arteries are common sites of occlusion
reduces mortality in patients with coronary heart disease: by a except: (AIIMS May 2005)
a. High Fibre diet (AIIMS May 07) a. Left anterior descending
b. Steral Esters
b. Right coronary artery
c. Potassium supplements
c. Circumflex coronary artery
d. Omega 3 polysaturated fatty acids Ref: Internet
d. Marginal artery Ref: Robbin’s 8/e p551
223. A patient with hypertriglycerdemia is treated with Omega
229. Which of the following ECG findings is associated with
-3- polyunsaturated fatty acids. Treatment with omega-3-
acute myocardial Infarction: (DNB 2012)
polyunsaturated fatty acids, will have the following effect
a. Elevation of S wave
on lipid profile: (AIIMS Nov 2006)
b. Prolonged QT interval
a. Increased LDL and Increased total cholesterol
c. Tall T waves with increased amplitude
b. Decreased LDL and Decreased total cholesterol d. Prolonged PR interval Ref: Harrison’s 18/e p1836
c. Increased LDL and Decreased total cholesterol
230. ECG is poor in detecting ischemia in areas supplied by
d. Decreased LDL and Increased total cholesterol
which of the following vessels: (AI 2011)
Ref: Harper 28/e p231
MEDICINE
a. Left Anterior Descending (LAD)
224 Which of the following statements about Atherosclerosis is b. Left Circumflex (LCx)
true: (AIIMS May 2011) c. Left Coronary Artery (LCA)
a. Intake of Unsaturated Fatty Acids is associated with d. Right Coronary Artery (RCA)
decreased risk 231. Which of the following ECG leads is most sensitive in
b. Extent of lesions in veins is similar as that in arteries detecting intraoperative myocardial ischemia:
c. Thoracic Aorta is more commonly involved than a. Lead I
abdominal aorta b. Lead II
d. Hypercholesterolemia alone does not increase the risk of c. Lead V1
atherosclerosis per se Ref: Internet d. Lead V2
e. Lead V5 Ref: Harrison’s 18/e p1836
Ans. 218. c. Nephrotic syndrome 219. a. LDL/HDL ratio 220. d. LDL/HDL 221. d. Avoid Alcohol
222. d. Omega 3...
223. c. Increased LDL ... 224. a. Intake of 225. a. Reinfaraction
226. d. Normal JVP
227. d. Three times 228. d. Marginal artery 229. c. Tall T...
230. b. Left Circumflex...
231. e. Lead V5
1082 Jaypee’s Triple A
232. A 60 year old man presents with chest pain with last 6 c. C-Reactive Protein
hours and is diagnosed as acute myocardial infarction. d. LDH Ref: Harrison’s 18/e p2023, 2024
Angiography showed involvement of anterior descending 240. Reperfusion is believed to restore contractile function of:
branch of left coronary artery. The most probable site of a. Stunned Myocardium (AIIMS May 2011)
infarct is: (AIIMS May 01) b. Hibernating Myocardium
a. Anterolateral wall c. Ischemic non-viable myocardium
b. Posterior wall d. Non ischemic viable myocardium
c. Inferior wall
241. The best possible intervention for acute myocardial
d. Septal Ref: Robbins illustrated 7/e p518, 6/e p557
infarction is: (AIIMS May 2005)
233. A 70 year old male patient presented to the emergency a. Streptokinase
department with pain in epigastrium and difficulty in b. Streptokinase and aspirin
breathing for 6 hours. On examination, his heart rate was c. Early primary coronary intervention
chest examination was normal. The patient has been taking d. Streptokinase and heparin
omeprazole for gastresophageal reflux disease for last 6 Ref: Harrison’s 18/e p2027, 17/e p1537
months. What should be the intial investigation:
242. A patient present with acute anterior wall infarction and
a. An ECG (AIIMS Nov 05)
hypotension. Which will be the immediate treatment
b. An upper GI endoscopy
modality for this patient: (AI 2007)
c. Urgent ultrasound of the abdomen
a. Intra aortic balloon counter pulsation
d. An x-ray chest Ref: Harrison’s 18/e p2001, 17/e p1517
b. Anticoagulation
234. In stable angina: (AIIMS Nov 03) c. Thrombolytic therapy
a. CK-MB is elevated d. Primary angioplasty
b. Troponin I is elevated Ref: Harrison’s 18/e p2027, 17/e p1537
c. Myoglobin is elevated
243. A patient presents with intense chestpain of 2 hrs duration.
d. The levels of cardiac markers remain unchanged.
ECG shows ST depression in leads I and V1 to V4. There is
Ref: Harrison’s 18/e p2001
associated T inversion and CPK-MB is elevated. All of the
235. ST elevation and hyperacute T waves in precordial leads V1 following should be included in the management of this
to V6 and in lead aVL indicates: (PGI Dec 2000) patient, except: (PGI June 2000)
a. Anterolateral wall MI a. Nitroglycerine drip
b. Posterior wall MI b. Aspirin
c. Inferior MI c. Coronary angiography
d. Lateral wall MI Ref: Harrison’s 18/e p1836, 17/e p1393 d. Streptokinase
236. In MI, which enzyme is raised in 4 to 6 hrs. and decreases in e. I.V. metoprolol Ref: Harrison’s 18/e p2027
3 to 4 days: (AIIMS Nov 2000) 244. A 40 years old male, chronic smoker comes with acute
a. SGOT epigastric discomfort, for past one hour. ECG showing ST
b. LDH segment elevation in inferior leads. What is the immediate
c. CPK intervention? (AIIMS Nov 07)
d. SGPT Ref: Harrison’s 18/e p2023 a. Aspirin
237. Troponin - T is a marker of: (AIIMS May 2004) b. Thrombolytic therapy
a. Renal disease c. IV pantoprazole
b. Muscular disease d. Beta blockers Ref: Harrison’s 18/e p2025, 2029 17/e p1535
c. Cirrhosis of liver 245. All of the following drugs are used in the management of
d. Myocardial infarction Ref: Harrison’s 18/e p2023 acute myocardial infarction, except: (AIIMS May 04)
238. Troponin-T is preferable to CPK-MB in the diagnosis of a. Tissue Plasminogen activator
MEDICINE
acute myocardial infarction (MI) in all of the following b. Intravenous beta blockers
situations except: (AI 2003) c. Acetylsalicylic acid
a. Bedside diagnosis of MI d. Calcium channel blockers
b. Postoperatively (after CABG) Ref: Harrison’s 18/e p2026, 17/e p1536
c. Reinfarction after 4 days 246. All of the following are used in the management of acute
d. Small infarcts Ref: Harrison’s 18/e p2024, 17/e p1534 myocardial infarction except: (PGI Dec 01)
239. Which of the following is the preferred marker for detecting a. Aspirin
Acute STEMI in Atheletes: (AI 2012) b. Heparin
a. CK-MB c. Alteplase
b. Troponin T/I d. Warfarin Ref: Harrison’s 18/e p2027-2030, 17/e p1539
Ans. 232. a. Anterolateral wall 233. a. An ECG 234. d. The levels of cardiac 235. a. Anterolateral wall MI
236. c. CPK
237. d. Myocardial infarction 238. c. Reinfarction... 239. b. Troponin T/I...
240. b. Hibernating...
241. c. Early primary... 242. d. Primary angioplasty 243. d. Streptokinase
244. a. Aspirin...
245. d. Calcium channel... 246. d. Warfarin...
Cardiovascular System 1083
247. Streptokinase and Urokinase are contraindicated in: 254. Which of the following tests is used to detect reversible
a. Intracranial malignancy (AI 2010) myocardial ischemia: (AIIMS May 09)
b. Pulmonary Embolism a. Angiography
c. AV fistula b. Thallium scan
d. Thrombophlebitis Ref: Harrison’s 18/e p2028, 17/e p1538 c. MUGA
248. All of the following are complications of streptokinase, d. Resting Echocardiography Ref: Harrison’s 18/e p2024
except: (PGI June 01) 255. Which of the following drugs has been linked with
a. Joint pain increased cardiac mortality: (AI 2012)
b. Inctracranial bleed a. Rofecoxib
c. Fever b. Metoprolol
d. Anaphylaxis c. Losartan
e. Hypotension Ref: Harrison’s 18/e p2028, 17/e p1538 d. Nicorandil Ref: Goodman Gilman 11/e p684, 686
249. A patient with acute inferior wall myocardial infarction has 256. Following an attack of myocardial infarction the mortality
developed shock. Which of the following is the most likely and morbidity of the patient is indicated by:
cause of shock: (AIIMS May 04) a. Ventricular extra systole (AIIMS June 2000)
a. Cardiac rupture b. Left ventricular ejection fraction
b. Interventricular septal perforation c. Duration of syncope
c. Papillary muscle rupture d. Percentage of narrowness of coronary artery
d. Right ventricular infarction Ref: Harrison’s 18/e p2233
Ref: Harrison’s 18/e p2233; Table 272-1 257. All of the following statements regarding subendocardial
infarction are true, except: (AI 06)
250. In a pt of acute inferior wall MI; best modalitiy of rx is:
a. These are multifocal in nature
a. IV fluids (AI 2001)
b. These often result from hypotension or shock
b. Digoxin
c. Epicardities is not seen
c. Diuretics
d. These may result in aneurysm Ref: Robbin’s 8/e p550
d. Vasodilators Ref: Harrison’s 18/e p2032, 17/e p1541
258. A myocardial infarct showing early granulation tissue has
251. A patient had an inferior wall myocardial infarction and
most likely occurred: (AI 02)
was in shock. The reason for the patient being in shock is:
a. 1 hours old
a. Mitral regurgitation (AIIMS May 02)
b. 24 hours old
b. Infarction causing septal defect
c. 1 week old
c. Right ventricular infarction
d. 1 month old Ref: Robbin’s 8/e p550; Table 12-5
d. Decreased ejection fraction from left ventricle
Ref: Harrison’s 18/e p2031, 17/e p1541 259. Subendocardial infarct is associated with all except:
a. May result in Anuerysm (WB PG 08)
252. In a patient with myocardial infarction the valvular lesion
b. Mulifocal in nature
commonly seen in: (AIIMS May 02)
c. Epicarditis is not seen
a. Aortic stenosis
d. Often result from hypotension or shock
b. Mitral regurgitation
Ref: Robbin’s 8/e p550
c. Aortic regurgitation
d. Septal defect Ref: Harrison’s 18/e p2022, 17/e p1533
253. Ramkumar a 70 year old hypertensive male was admitted J. HYPERTENSION
in the intensive care unit with transmural anterolateral
myocardial infraction. His condition was stable till fifth day 260. Calcium blocking agents of use in the treatment of
of admission, when he developed a pericardial friction rub hypertension include: (Feb DP PGMEE 2009)
MEDICINE
and pleuritic chest pain which persisted despite narcotic a. Prazosin
and steroid therapy. On the seventh morning, he suddenly b. Lidoflazine
developed marked hypotension. On examination there was c. Captopril
marked distension of the jugular veins, accompanied with d. Nifedipine Ref: Harison’s 18/e p2055-56
electromechanical dissociation. Most likely, the patient had 261. Episodic hypertension is classical feature of:
developed: (AIIMS Nov 02) a. Adrenal carcinoma (MHPGM-CET 2010)
a. Severe acute mitral regurgitation. b. Phechromocytoma
b. Ventricular septal rupture. c. Conn’s syndrome
c. Right ventricular infarction. d. Cushing’s disease
d. External cardiac rupture. Ref: Harrison’s 18/e p2963, 17/e p2270; Table 337-1
Ref: Harrison’s 18/e p1971, 17/e p1491
Ans. 247. a. Intracranial... 248. a. Joint pain... 249. b. Interventicular... 250. a. IV fluids
251. c. Right ventricular... 252. b. Mitral regurgitation
253. d. External cardiac... 254. b. Thallium scan
255. a. Rofecoxib
256. b. Left ventricular... 257. d. These may ... 258. c. 1 week old...
259. a. May result in...
260. d. Nifedipine 261. b. Phechromocytoma
1084 Jaypee’s Triple A
262. Aortic dissection which occurs in young individuals is due The drug of choice would be: (DNB 2009)
to: (MP PG 2009) a. Nifedipine
a. Hypertension b. Labetalol
b. Syphills c. Prazocin
c. Atherosclerosis d. Phenoxy benzamine
d. Connetive tissue disorders affecting the aorta Ref: Harrison’s 18/e p2058, 17/e p1561-62
Ref: Harrison’s 18/e p2063 17/e p1565 271 All of the following are useful intravenous therapy for
263. Which of the following is the most common location of hypertensive emergencies except: (AIIMS May 03)
hypertension haemorrhage? (UP 2011) a. Fenodopam
a. Pons b. Uradipil
b. Putamen c. Enalaprilat
c. Thalamus d. Nifedipine Ref: KDT 7/e p128, 129
d. Subcortical white matter Ref: Internet
272. A patient presents with headache and profuse sweating. On
264. Pulmonary hypertension occurs in: (DP PGMEE 2010) examination his blood pressure is recorded as 200/120 mm
a. Essential hypertension
Hg. Which of the following agents are not preferred (Select
b. Parkinsonism
three best options): (PGI 2009)
c. Cushing syndrome
a. Nifedipine
d. Stein leventhal syndrome Ref: Harrison’s 18/e p2077
b. Sodium nitroprusside
265. Which of the following drugs should not be used in a setting c. Phenoxybenzomine
of severe hypertension in elderly on empirical basis? d. Methyldopa
a. Enalapril (DP PGMEE 2009) e. Labetalol Ref: Harrison’s 18/e p268, 285, 858, 865, 853
b. Amlodipine
273. In Accelerated HTN what is metabolic defect:
c. Chlorthiazide
a. Normal non-ionic metabolic acidosis (PGI June 2000)
d. Prazosin Ref: Harrison’s 18/e p2056, 17/e p1560
b. Ionic gap met acidosis
266. Management of uncomplicated essential hypertension is: c. Hypomagnesemia
a. No need to treat (DNB 2009) d. Metabolic alkalosis Ref: Harrison’s 17/e p293
b. Diet modification and exercise
274. Which of the following is the most specific and sensitive
c. Diet modification, exercise and drugs
screening test for Renovascular Hypertension:
d. Drugs alone Ref: Harrison’s 18/e p2054
a. HRCT (AIIMS May 01)
267. First line drug choice for management of hypertension in b. CT Angiography
patients with angina: c. Captopril enhanced radionucleatide scan
a. Beta Blockers d. MRI Ref: Harrison’s 18/e p2048-2049)
b. ACE Inhibitors
275. Which of the following is the most specific screening test
c. Calcium Channel Blockers
for renovascular hypertension: (PGI 2008)
d. Hydralazine Ref: Harrison’s 18/e p2054-56
a. Magnetic Resonance Angiography (MRA)
268. A young patient presented with blood pressure of 190/120 b. Spiral Computed Tomographic Angiography (CT
mm of Hg without any clinical symptom and fundus Angiography)
examination is normal, treatment of choice: (PGI June 03) c. Captopril induced Radionucleotide Scan (Captopril
a. Oral Nitroglycerine Renogram)
b. I.V. Nitroglycerine d. Duplex Doppler Ultrasonography
c. Oral Enalapril Ref: Harrison’s 18/e p2048-49
d. IV Enalarpil
e. Sublingual short acting Nifedipine 276. A 20 year old female presents with a blood pressure of
MEDICINE
Ref: Harrison’s 18/e p2055, 17/e p 160/110 mm Hg. Clinical examination reveals a bruit in both
flanks. Which of the following statements about this patient
269. Monogenic AD cause of HTN? (PGI 2009)
is not true (select one option): (PGI 09)
a. 17- α Hydroxylase deficiency
a. Enalapril may deteriorate renal function
b. Gordon’s Syndrome
b. Most definitive diagnostic procedure is contrast enhanced
c. Pregnancy Exacerbated HTN
angiography
d. Glucocortiocoid responsive HTN
e. Glucocorticoid Remediable Aldosteronism c. Condition is nearly always bilateral
Ref: Harrison’s 18/e p2054-56, 17/e p1557 d. Surgical intervention may be used
e. Fibromuscular dysplasia is the likely cause in this patient
270. A male patient presents with headache, profuse sweating
Ref: Harrison’s 18/e p2375, 2376 17/e p1811-12
and palpitations with a blood pressure of 180/120 mm Hg.
Ans. 262. a. Hypertension 263. b. Putamen 264. a. Essential hypertension 265. d. Prazosin
266. c. Diet modification
267. a. Beta Blockers 268. c. Oral Enalapril 269. b, c and e
270. b. Labetalol
271. d. Nifedipine 272. a, d and c 273. d. Metabolic...
274. b. CT Angiography
275. a. Magnetic... 276. c. Condition
Cardiovascular System 1085
277. Renal artery stenosis may occur in all of the following 285. Cardiomyopathy may be seen in all of the following except:
except: (AI 2006) a. Duchene muscular dystrophy (AIIMS May 2006)
a. Atherosclerosis b. Friedrich’s ataxia
b. Fibromuscular dysplasia c. Type II glycogen storage disease
c. Takayasu’s arteritis d. Alkaptonuria
d. Polyarteritis nodosa Ref: Internet Ref: Harrison’s 18/e p1959, 1965 17/e p1481
278. In primary pulmonary hypertension basic abnormality in 286. Contractile Dysfunction is the dominant feature of which of
gene lies in: (AIIMS May 07) the following types of cardiomyopathies: (DNB 2012)
a. Bone morphogenetic protein receptor II a. Dilated cardiomyopathy
b. Endothelin b. Restrictive cardiomyopathy
c. Homebox gene c. Hypertrophic cardiomyopathy
d. PAX – 11 Ref: Harrison’s 18/e p2077, 17/e p1577 d. Infiltrative cardiomyopathy
279. Precapillary Pulmonary hypertension is caused by all Ref: Harrison’s 18/e p1952
except: (PGI June 03) 287. Which one of the following is the most common cause for
a. Mitral stenosis ‘Restrictive cardiomyopathy’: (AIIMS May 04)
b. Pulmonary vasculitis a. Alcohol
c. Primary pulmonary hypertension b. Hemochromatosis
d. Thromboembolism Ref: Harrison’s 18/e p2077-78 c. Amyloidosis
280. All are causes of pulmonary hypertension except: d. Sarcoidosis
a. Hyperventilation (AIIMS Nov 07) Ref: Harrison’s 18/e p1965, 17/e p1485
b. Morbid obesity 288. A 25 years old basket ball player suddenly collapsed while
c. High altitude undergoing an athletic event and died. At autopsy the
d. Fenfluramine Ref: Harrison’s 18/e p, 17/e p septum was hypertrophied. The most probable diagnosis
281. Pulmonary hypertension may occur in all of the following is: (AIPGMEE 08)
conditions except: (AIIMS Nov 06) a. HOCM
a. Toxic oil syndrome b. Right ventricular conduction Abnormality
b. Progressive systemic sclerosis c. Epilepsy
c. Sickle cell anemia d. Snake bite Ref: Harrison’s 18/e p1967, 1968
d. Argemone mexicana poisoning Ref: API textbook of medicine 8/e p575
Ref: Harrison’s 18/e p2082 289. A 26 year old man died suddenly during sporting activity.
At autopsy the heart revealed chamber and septum
Hypertrophy. The most likely diagnosis is: (AIIMS May 09)
K. CARDIOMYOPATHY a. HOCM
b. DCM
282. Selenium deficiency will cause: (Kerala PG 10)
c. Arrhythmogenic cardiac problem
a. Alopecia
d. Restrictive cardiomyopathy
b. Dermatitis
Ref: Harrison’s 18/e p1968, 1970, 17/e p1484-85
c. Loss of array of skeletal muscle fibres
d. Cardiomyopathy Ref: Harrison 18/e p604, 17/e p449 290.
All are true about Hypertrophic Obstructive
Cardiomyopathy, except: (AI 2000)
283. Digoxin is contraindicated in: (UP 2009)
a. β agonist are useful
a. Hypertrophic cardiomyopathy
MEDICINE
b. Asymmetrical hypertrophy of septum
b. Supraventricular tachycardia
c. Dynamic L.V. outflow obstruction
c. Congestive heart failure
d. Condition improves on passive leg raising
d. Atrial flutter
Ref: Harrison’s 18/e p1969, 17/e p1485
Ref: Harison’s 18/e p1968, 17/e p1485; CMDT-09 p363
291. The murmur of hypertrophic obstructive cardiomyopathy is
284. All the following drugs are to be avoided in hypertrophic
decreased in which of the following: (AIIMS Nov 2000)
cardiomyopathy except: (Comed K 2010)
a. Supine position
a. Diuretics
b. Standing position
b. Beta blockers
c. Valsalva maneuver
c. Digitalis d. Amyl nitrate inhalation
d. Nitrates Ref: Harrison’s 18/e p1869 Ref: Harrison’s 18/e p1969, 17/e p1485
Ans. 277. d. Polyarteritis 278. a. Bone morphogenetic... 279. a. Mitral stenosis 280. a. Hyperventilation
281. d. Argemone
282. d. Cardiomyopathy 283. a. Hypertrophic 284. b. Beta blockers
285. d. Alkaptonuria
286. a. Dilated 287. c. Amyloidosis 288. a. HOCM
289. a. HOCM
290. a. β agonist are useful 291. a. Supine position
1086 Jaypee’s Triple A
292. Which of the following drugs are contraindicated in: 299. A pt presents with engorged neck veins, BP 80/50 and pulse
a HOCM (DNB 2009) rate of 100 following blunt trauma to the chest: Diagnosis is:
b. Verepamil a. Pneumothorax (AI 2001)
c. Propanalol b. Right ventricular failure
d. Digoxin c. Cardiac tamponade
e. None of the above Ref: Harrison’s 18/e p1969, 1970) d. Hemothorax Ref: Harrison’s 18/e p1975, 17/e p1491
293. The 9 month old child of a diabetic mother presents with 300. A young motorist suffered injuries in a major road traffic
tachypnea and hepatomegaly. Echocardiography of the accident. He was diagnosed to have fracture of left femur
heart showed normal cardiac morphology with asymmetric and left humerus. He was also having fractures of multiple
septal hypertrophy. Which of the following you will give to ribs anteriorly on both the sides. On examination the blood
treat this child: (AIIMS Nov 2000) pressure was 80/60 mm Hg. and heart rate was 140/minute.
a. Digoxin The patient was agitated, restless, and tachypenic. Jugular
b. Frusemide veins were distended. Air entry was adequate in both the
c. Propranolol lung fields. Heart sounds were barely audible. Femoral
d. Isoptin Ref: Harrison’s 18/e p1969, 1970, 17/e p1485) pulses were weakly palpable but distally no pulsation
294. Aggravation of symptoms of angina in a patient when given could be felt. On priority basis, the immediate intervention
nitrates is seen in: (AIIMS June 2000) would be: (AIIMS Nov 02)
a. Aortic regurgitation a. Rapid blood transfusion.
b. Mitral regurgitation b. Urgent pericardial tap.
c. Single left coronary artery stenosis c. Intercostal tube drainage on both the sides.
d. Idiopathic hypertrophic subaortic stenosis d. Fixation of left femur and repair of femoral artery
Ref: Harrison’s 18/e p1968, 1969, 1970 17/e p1484-85 Ref: Harrison’s 18/e p1972, 1974 17/e p1490, 1491
295. A 35-year-old farmer consulted a local medical practitioner 301. A post-operative cardiac surgical patient developed sudden
for recurrent attacks of chest pain. His elder brother had hypotension, raised central venous pressure, pulsus
similar complaints and had died suddenly at the age of paradoxus at the 4th post operative hour. The most probable
40 years. The farmer was advised to take nitroglycerine diagnosis is: (AI 2003)
sublingually at the time of pain. However, the patient finds a. Excessive mediastinal bleeding
that the intensity of pain is increased by nitroglycerine. b. Ventricular dysfunction
Most probably, he is suffering from: (AIIMS Nov 02) c. Congestive cardiac failure
a. Subacute bacterial endocarditis involving the aortic d. Cardiac tamponade
valve. Ref: Harrison’s 18/e p1972, 17/e p1490
b. Hypertrophic obstructive cardiomyopathy. 302. Kussmaul’s sign is NOT seen in: (AI 2001)
c. Degenerative mitral regurgitation. a. Restrictive cardiomyopathy
d. Chronic Type A dissection of aorta. b. Constrictive pericarditis
Ref: Harrison’s 18/e p1968, 1970 17/e p1484 c. Cardiac tamponade
296. All of the following are seen in cardiac tamponade except: d. RV infarct
a. Pulsus paradoxus (AI 2004) Ref: Harrison’s 18/e p1975, 17/e p1490
b. Diastolic collapse of right ventricle on echocardiogram 303. A 30 year male patient presents to the emergency department
c. Electrical alternans with a history of acute breathlessness. On examination, JVP
d. Kussmaul’s sign Ref: Harrison’s 18/e p1975, 17/e p1490 is increased and an inspiratory decline in systolic blood
297. All of the following may be seen in patients of cardiac pressure of 14mm Hg is observed. Which of the following
tamponade except: (AI 06) statements about the condition is true: (PGI June 2008)
a. Kussmaul’s sign a. Kussmaul’s sign
MEDICINE
MEDICINE
Ref: KDT 6/e p499; Harrison’s 18/e p1890, 17/e p1435 d. All Ref: Internet
311. Investigation of choice for pulmonary thromboembolism 320. Most lethal aortic dissection: (Rajasthan 2009)
is: (Kerala PG 2008) a. Ascending arota
a. CT scan b. Descending arota
b. Ventilation perfusion scan c. Abdominal aorta
c. MRI d. Thoracic aorta Ref: Harrison’s 18/e p2063-2064
d. USG Ref: Harrison’s 18/e p2173, 16/e, p1562; 17/e, p1653 321. In cardiogenic shock due to left ventricular failure CVP is:
312. Which of the following is seen in pulmonary a. Decreased (Rajasthan 2009)
thromboembolism? b. Variable
a. S1 Q3 T3 (Kerala PG 2008) c. Increased
b. S2 Q3 T3 d. No change Ref: Harrison’s 18/e p2235, 2232
Ans. 322. c. 0.5 mm 323. a. Right atrium 324. c. Constrictive... 325. b. Acute...
326. b. Norepinephrine
327. c. Constrictive 328. a. Right lateral... 329. a. Sudden...
330. c. Fibrates
331. c. Biological valve 332. b. Microcirculation 333. d. Familial hyper...
334. a. O
335. c. MR 336. c. In Ist trimester... 337. b and d
Cardiovascular System 1089
338. All of the following may be used in the treatment of cardiac a. Fibrous endocardial thickening of Right ventricle,
arrest following ventricular fibrillation, except (select two Tricuspid valve and Pulmonary valve
best options): (PGI 09) b. Endocardial thickening of Tricuspid valve with severe
a. Atropine Tricuspid Stenosis
b. External cardiac pacing c. Collagen rich, elastic deposits in endocardium of right
c. Epinephrine ventricle and Pulmonary valve
d. Antiarrhythmic agents d. Calcification of Tricuspid and Pulmonary valve
e. Vasopressin Ref: Harrison’s 18/e p3062
Ref: Harrison’s 18/e p2239, 224417/e p1712, 1708 346. Carcinoid syndrome produces valvular disease primarily
339. What would be the first line of treatment is a patient who involving: (AIIMS May 04)
develops ventricular fibrillation after intravenous injection a. Venous valves
of potassium chloride: (AIIMS May 2004) b. Tricuspid valve
a. Cardiac massage c. Mitral valve
b. I.V Adrenaline d. Aortic valve
c. Defibrillation Ref: Harrison’s 18/e p3062, 3063
d. IPPV 347. Sudden cardiac death may occur in all of the following
Ref: Harrison’s 18/e p2244, 17/e p1712 except: (AI 06)
340. Which of the following statements about atrial myxomas is a. Dilated cardiomyopathy
true: (DNB 2011) b. Hypertrophic cardiomyopathy
a. Most common in Left Atrium c. Eisenmenger’s syndrome
b. More common in Males d. Ventricular septal defect Ref: Internet
c. Distant metastasis are seen 348. S.A. node acts as a pacemaker of the heart because of the
d. Most myxomas are familial fact that it: (AI 05)
Ref: Harrison’s 18/e p1495-1496 a. Is capable of generating impulses spontaneously
341. All of the following statement about atrial myxomas are b. Has rich sympathetic innervations
true, except: (DNB 2011) c. Has poor cholinergic innervations
a. Most common site is Left Atrium d. Generates impulses at the highest rate
b. Most common in young individuals Ref: Ganong 22/e p547
c. Distant metastasis are rare 349. Speed of conduction is fastest in: (PGI Dec 05)
d. Most myxomas are familial a. AV node
Ref: Harrison’s 18/e p1979, 17/e p b. SA node
342. All of the following are clinical features of myxoma, except: c. Bundle of His
a. Fever (AI 2002) d. Purkinje system
b. Clubbing e. Ventricular muscle Ref: Ganong 22/e p547
c. Hypertension 350. Which of the following is the order of activation after
d. Embolic phenomenon stimulation of Purkinji fibers is: (AIIMS Dec 95)
Ref: Harrison’s 18/e p1979, 17/e p1495-96 a. Septum –> Endocardium –> Epicardium
343. Gradient in pulmonary artery wedge pressure and left b. Endocrdium –> Septum –> Epicardium
ventricular end diastolic pressure is seen in: c. Epicardium –> Septum –> Endocardium
a. Aortic regurgitation (AIIMS June 2000) d. Septum –> Epicardium –> Endocardium
b. Constrictive pericarditis Ref: Ganong 22/e p547
c. Left atrial myxoma 351 The right coronary artery supplies all of the following parts
MEDICINE
d. Pulmonary thromboembolism of the conducting system in the heart except: (AI 2003)
Ref: Harrison’s 18/e p1979, 17/e p1495) a. SA Node
344. Carcinoid syndrome produces valvular disease primarily b. AV Node
involving: (AIIMS May 05) c. AV Bundle
a. Pulmonary valves d. Right Bundle branch Ref: Harrison’s 18/e p1835
b. Tricuspid valves 352. If circumflex artery gives the posterior intervenetricular
c. Mitral valves branch, this circulation is described as: (AIIMS Nov 07)
d. Aortic valves a. Right dominance
Ref: Harrison’s 18/e p3062, 17/e p2351 b. Left dominance
345. Characteristic pathological finding in carcinoid heart c. Codominance
disease is: (AI 2010) d. Undetermined Ref: Harrison’s 18/e p549
Ans. 338. a. and b. 339. c. Defibrillation 340. a. Most common... 341. d. Most myxomas...
342. c. Hypertension
343. c. Left... 344. a. Pulmonary valves 345. a. Fibrous
346. b. Tricuspid...
347. d. Ventricular 348. d. Generates... 349. d. Purkinje...
350. a. Septum...
351. d. Right... 352. b. Left dominance
1090 Jaypee’s Triple A
353. During the cardiac cycle the opening of the aortic valve a. Fibroblasts and collagen (AI 2009)
takes place at the: (AI 04) b. Granulation tissue
a. Beginning of systole c. Neutrophilic infiltration surrounding coagulative necrosis
b. End of isovolumetric contraction d. Granulomatous inflammation
c. End of diastole Ref: Robbin’s 8/e p560; Table 12-5
d. End of diastasis Ref: Ganong 24/e p540
362. A 60 year old male presented with acute chest pain of 4
354. At the end of isometric relaxation phase: (AI 2000)
hours duration. Electrocardiographic examination revealed
a. Atrioventricular valves open
new Q wave with ST segment depression. He suffered to
b. Atrioventricular valves close
his illness within 24 hours of admission. The heart revealed
c. Corresponds to peak of “C” wave in JVP
d. Corresponds to T wave in ECG Ref: Guyton 10/e p99 presence of a transmural hemorrhagic area over the septum
and anterior wall of the left ventricle. Light microscopic
355. Mean arterial pressure is calculated as: (AIIMS Nov 06)
examination is most likely to reveal: (AI 04)
a. (SBP + 2DBP)/3
b. (DBP + 2SBP)/3 a. Edema in between normal myofibrils
c. (SBP + 3DBP)/2 b. Necrotic myofibrils with presence of neutrophils
d. (DBP + 3SBP)/2 c. Coagulative necrosis of the myocytes with presence of
356. SI unit for measuring Blood Pressure is: (AI 02) granulation tissue
a. Torr d. Infiltration by histiocytes with hemosiderin laden
b. mmHg Macrophages Ref: Robbin’s 8/e p550; Table 12-5
c. kPa 363. A 70 year old man develops pneumonia and septicemia.
d. Barr Ref: Harrison’s 18/e p1824 Patient goes into renal failure and has a BP of 70/50 mm of
357. The blood pressure measured by a sphygmomanometer: Hg. Drug that should be used to maintain BP is: (AI 07)
a. Is lower than the intraarterial pressure (AI 08) a. Adrenaline
b. Is higher than the intraarterial pressure b. Ephedrine
c. Is same as the intraarterial pressure c. Phenylephrine
d. Is the same with different cuff sizes d. Nor epinephrine Ref: Harrison’s 18/e p1836-37
Ref: Harrison’s 18/e p1824
364. Renal failure in patients with septic shock occurs primarily
358. True about blood pressure measurement is all/except: from:
(AIIMS May 07)
a. Acute tubular necrosis
a. Cuff width should be 40% of arm circumference
b. Acute cortical necrosis
b. Diastolic blood pressure is indicated by fourth Korotkoff
sound c. Acute glomerulonephritis
c. Small cuff measures spuriously elevated Diastolic blood d. Acute papillary damage
pressure Ref: Harrison’s 18/e p2227, 17/e p1699
d. Monkenberg sclerosis causes Pseudohypertension 365. During cardiac imaging the phase of minimum motion of
Ref: Harrison’s 18/e p1824, Ganong 22/e p589-90 heart is: (AI 2010)
359. Spuriously high BP is seen in all except: (AIIMS May 01) a. Late systole
a. Auscultatory gap b. Mid systole
b. Small cuff c. Late diastole
c. Thick calcified vessels d. Mid diastole Ref: Internet
d. Obesity Ref: Internet 366. Cardiovascular complications of HIV infection include all
360. Which is true about measurement of BP with of the following, except: (AI 2009)
syphgnomanometer versus intra-arterial pressure a. Pericardial Effusion
MEDICINE
Ans. 353. b. End of... 354. a. Atrioventricular 355. a. SBP + 2DBP)/3 356. c. kPa
357. b. Is higher
358. b. Diastolic 359. a. Auscultatory... 360. b. More than ...
361. c. Neutrophilic...
362. b. Necrotic 363. d. Nor epinephrine 364. a. Acute tubular...
365. d. Mid diastole
366. d. Aortic Aneurysm 367. d. Coronary artery aneurysm
3. GASTROINTESTINAL SYSTEM
A. Dysphagia/Esophageal Conditions
B. Stomach and Duodenum
C. Malabsorption Syndrome and Disorders of Small Intestine
D. Inflammatory Bowel Disease
E. Colonic Disorders
F. Pancreatic Disorders
G. Miscellaneous
1092 Jaypee’s Triple A
a. Carcinoma esophagus
c. X-ray chest b. Scleroderma
d. C.T. scan Ref: Harrison’s 18/e p2434, 2427 c. Achalasia cardia
6. A patient present with dysphagia of 4 weeks duration. d. Diffuse esophageal spasm Ref: Harrison’s 18/e p2432
Now he is able to swallow liquid food only. Which of the 14. A male aged 60 years has foul breath; He regurgitates food
following is the one investigation to be done: that is eaten 3 days ago. A gurgling sound is often heard on
a. Barium studies are the best to be done (PGI June 2000) swallowing: Likely diagnosis is: (AI 2001)
b. Upper GI endoscopy is to be done a. Zenkers diverticulum
c. CT scan is needed b. Meckels diverticulum
d. Esophageal manometry c. Scleroderma
Ref: Harrison’s 18/e p2424, 2428 d. Achalasia cardia Ref: Harrison’s 18/e p2429
MEDICINE
21. Most common site for squamous cell carcinoma esophagus is: 29. Helicobacter pylori is not associated with: (AIIMS Nov 03)
a. Upper third (AI 01) a. Gastrointestinal lymphoma
b. Middle third b. Gastric cancer
c. Lower third c. Gastric leiomyoma
d. Gastro-esophageal junction d. Peptic ulcer Ref: Harrison’s 18/e p2442-43, 17/e p1858-59
Ref: Internet, Harrison’s 18/e p764 30. Endoscopic biopsy from a case of H.pylori related duodenal
22. The most common site of esophageal adenocarcinoma is: ulcer is most likely to reveal: (AIIMS Nov 04)
a. Upper 1/3rd (AIIMS Nov 96, June 2000) a. Antral predominant gastritis
b. Middle 1/3rd b. Multifocal atrophic gastritis
c. Lower 1/3rd c. Acute erosive gastritis
d. Upper 2/3rd Ref: Internet, Harrison’s 18/e p764 d. Gastric atrophy
Ref: Harrison’s 18/e p1262, 17/e p946-47
Ans. 15. d. Out pouching... 16. b. Intestinal metaplasia 17. d. Associated with... 18. e. None
19. a. Adenocarcinoma 20. d. Shows mucosal tear 21. b. Middle third 22. b. Middle 1/3rd
23. b. Middle 1/3rd 24. a. Barret’s esophagus 25. e. Pernicious... 26. a. Acute tubular necrosis
27. a. Cisapride 28. a. Curling’s ulcer 29. c. Gastric leiomyoma 30. a. Antral
1094 Jaypee’s Triple A
31. Which of the following is False regarding H.Pylori infection: blockers
(AIIMS June 2000) c. Omeprazole may helpulcers refractory to H2 blockers
a. With chronic infection urease breath test become negative d. Misoprostol is DOC in pregnant patients
b. H.Pylori infection remain lifelong if untreated Ref: Harrison’s 18/e p2449
c. Endoscopy is diagnostic 39. Patient presents with recurrent duodenal ulcer of 2.5 cm
d. Toxigenic strains usually causes ulcer size; procedure of choice: (AI 2001)
Ref: Harrison’s 18/e p1262-63, 2442, 17/e p946-47, 1858 a. Truncal vagotomy and antrectomy
32. A patient with H. Pylori infection is treated with drugs. b. Truncal vagotomy and gastrojejunostomy
The best method to detect presence of residual H.Pylori c. Highly selective vagotomy
infection in this person is: (AI 2007) d. Laparoscopic vagotomy and gastrojejunostomy
a. Rapid unrease test Ref: Harrison’s 18/e p2452-53
b. Urea breath test 40. The lowest recurrence of peptic ulcer is associated with:
c. Endoscopy and biopsy a. Gastric resection (AI 2002)
d. Serum anti H.Pylori titre b. Vagotomy + drainage
Ref: Harrison’s 18/e p2446, 17/e p947 c. Vagotomy + Antrectomy
33. Which of the following statements about peptic ulcer d. Highly selective vagotomy Ref: Harrison’s 18/e p2453
disease is true: (AIIMS May 03) 41. All are true regarding Early Post-cibal syndrome, except:
a. Helicobacter pylori eradication increases the likelihood a. Distension of abdomen (AI 2000)
of occurrence of complications. b. Managed conservatively
b. The incidence of complications has remained unchanged c. Hypermotility of intestine is common
c. The incidence of Helicobacter pylori reinfection in India d. Surgery is usually indicated Ref: Harrison’s 18/e p2452
is very low.
42. In gastric outlet obstruction in a peptic ulcer patient, the site
d. Helicobacter pylori eradication does not alter the
of obstruction is most likely to be: (AI 2002)
recurrence ratio.
a. Antrum
34. All of the following drugs are commonly used in regimens b. Duodenum
against H. pylori except: (AI 2008) c. Pylorus
a. Oxytetracycline d. Pyloric canal Ref: Harrison’s 18/e p240
b. Amoxicillin
43. What is the most characteristic of congenital hypertrophic
c. Bismuth Subsalicylate
pyloric stenosis: (AI 2003)
d. Omeprazole Ref: Harrison’s 18/e p2449
a. Affects the first born female child
35. The most common site of a benign (peptic) gastric ulcer is: b. The pyloric tumour is best felt during feeding
a. Upper third of lesser curvature (AIIMS June 04) c. The patient is commonly marasmic
b. Greater curvature d. Loss of appetite occurs early
c. Pyloric antrum
44. What is true regarding congenital hypertrophic pyloric
d. Lesser curvature near incisura angularis
stenosis: (AI 2001)
Ref: Harrison’s 18/e p2444-46
a. More common in girls
36. Artery to bleed in duodenal ulcer haemorrhage: b. Hypochloremic alkalosis
a. Splenic artery (AIIMS June 04) c. Hellers myotomy is the procedure of choice.
b. Gastroduidenal artery d. Most often manifests at birth
c. Left gastric artery
45. In a case of hypertrophic pyloric stenosis, the metabolic
d. Superior mesenteric artery Ref: Internet
disturbance is: (AI 2002)
37. A posteriorly perforating ulcer in the pyloric antrum of a. Respiratory alkalosis
the stomach is most likely to produce initial localized
MEDICINE
b. Metabolic acidosis
peritonitis or abscess formation in the following: (AI 2003) c. Metabolic alkalosis with paradoxical aciduria
a. Omental bursa (lesser sac) d. Metabolic alkalosis with alkaline urine Ref: Internet
b. Greater sac
46. Patient with pyloric stenosis secondary to peptic ulcer,
c. Right subphrenic space
complains of profuse vomiting and Na+ - 125meq/L, K+ -->
d. Hepato renal space (pouch of Morison)
2.3 meq/L and Cl- --> 85 meq/L, BE-8meq/L should be given:
Ref: Gray’s 40/e p1107
a. Half normal saline (AIIMS June 2000)
38. All of the following are true regarding a patient with acid b. Normal saline
peptic disease except: (AI 2001) c. K+ bolus
a. Misoprostol is the drug of choice in patients on NSAIDS d. Hypertonic saline
b. DU is preventable by the use of single night time H2 Ref: Internet
Ans. 31. a. With chronic... 32. b. Urea breath test 33. b. The incidence of... 34. b. Amoxicillin
35. d. Lesser curvature... 36. b. Gastroduidenal artery 37. a. Omental bursa 38. d. Misoprostol is
39. a. Truncal vagotomy... 40. c. Vagotomy... 41. d. Surgery is... 42. b. Duodenum
43. b. The pyloric ... 44. b. Hypochloremic... 45. c. Metabolic alkalosis... 46. b. Normal saline
Gastrointestinal System 1095
47. Risk factor for development of gastric Ca: (AI 2002) 54. Non absorbtion of fat soluble vitamins is due to: (DNB 2009)
a. Blood group O a. Steatorrhea
b. Duodenal ulcer b. Pancreatic endocrine insufficiency
c. Intestinal hyperplasia c. Both
d. Intestinal metaplasia type III d. None Ref: Harrison’s 18/e p2460
Ref: Harrison’s 18/e p765-66, 17/e p572, Robbins 7/e 823 55. Test for assessment of mucosal function of GIT: (AI 2009)
48. Risk factor for carcinoma stomach include all of the a. D-xylose test
following, except: (PGI June 2000) b. Small bowel study
a. Blood groupA c. Biopsy
b. Atrophic Gastrititis d. Schilling test Ref: Harrison’s 18/e p2467, 17/e p1878
c. Duodenal Peptic ulcer 56. A 41 year old patient presented with chronic diarrhoea for 3
d. Partial Gastrectomy Ref: Harrison’s 18/e p766, 17/e p572 months. A d-xylose absorption test was ordered to look for:
49. Which one of the following is the most significant risk (AIIMS Nov 02)
factor for development of gastric carcinoma? (AI 2006) a. Carbohydrate malabsorption due to mucosal disease
a. Paneth cell metaplasia b. Carbohydrate malabsorption due to chronic pancreatitis
b. Pyloric metaplasia c. Fat malabsorption due to mucosal disease
c. Intestinal metaplasia d. Fat malabsorption due to chronic pancreatitis
d. Ciliated metaplasia Ref: Harrison’s 18/e p766 Ref: Harrison’s 18/e p2465
50. When carcinoma of stomach develops secondarily to 57. A d-xylose test was requested on a patient with history of
pernicious anemia, it is usually situated in the: (AI 2006) long standing steatorrhea ‘5’ hour urine sample showed
a. Prepyloric region <4.0 gm excretion after giving 25 gm of d-xylose. The most
b. Pylorus likely diagnosis is: (PGI 09)
c. Body a. Chronic Pancreatitis
d. Fundus Ref: Harrison’s 18/e p766 b. Bacterial overgrowth syndrome
51. All the following indicates early gastric cancer except: c. Ileal disease
a. Involvement of mucosa (AI 2002) d. Celiac sprue
b. Involvement of mucosa and submucosa e. Intestinal Lymphangiectasia
c. Involvement of mucosa, submucosa and muscularis Ref: Harrison’s 18/e p2470-71
d. Involvement of mucosa, submucosa and adjacent lymph 58. Cause of False positive D-xylose test include all of the
nodes Ref: Harrison’s 18/e p765, 766, 17/e p571-72 following, except: (PGI 01)
52. An adult presented with hemetemesis and upper abdominal a. Bacterial overgrowth
pain. Endoscopy revealed a growth at the pyloric antrum of b. Renal failure
the stomach. CT scan showed growth involving the pyloric c. Ascitis
antrum without infiltration or invasion into surrounding d. Celiac sprue
structures and no evidence of distant metastasis. At e. Blind loopsyndrome Ref: Harrison’s 18/e p2470-71
Laparotomy neoplastic growth was observed to involve the 59. False positive D-xylose test may be seen in: (PGI June 01)
posterior wall of stomach and the pancreas extending 6cm a. Blind loopsyndrome
upto tail of pancreas. What will be the most appropriate b. Ascitis
surgical management: (AI 2010) c. Antibiotic therapy
a. Closure of the abdomen d. Pyloric stenosis
b. Antrectomy and vagotomy e. All of the above Ref: Harrison’s 18/e p2467
c. Partial Gastrectomy + Distal pancreatectomy 60. Which of the following statements about Schilling’s test are
MEDICINE
d. Partial Gastrectomy + Distal pancreatectomy + splenectomy true: (PGI 2009)
Ref: Harrison’s 18/e p767 a. Abnormal in pernicious anemia
b. Normal in bacterial overgrowth syndrome
c. Abnormal in ileal disease
C. MALABSORPTION SYNDROME AND d. Normal in chronic pancreatitis
DISORDERS OF SMALL INTESTINE Ref: Harrison’s 18/e p2467, 17/e p1878
61. A 12 year old girl has history of recurrent bulky stolls and
53. Which of the following is not a malabsorption syndrome: abdominal pain since 3 year of age. She has moderate pallor
a. Whipple’s disease (DNB June 2009) and her weight and height are below the 3rd percentile.
b. Coeliac disease Which of the following is the most appropriate investigation
c. Tropical sprue to make a specific diagnosis? (AI India/AIIMS)
d. Tangier’s disease Ref: Harrison’s 18/e p2460, 2476
Ans. 61. a. Small intestinal ... 62. a. Antiendomysial 63. b. Anti-endomysial... 64. a. Gluten free diet
65. a. Anti-endomysial... 66. b. Increase... 67. c and e 68. a. Dermatitis
69. e. All 70. a and b 71. a. Rice 72. a and d
73. a and d 74. c. Whipple’s disease 75. a. Whipple’s disease
Gastrointestinal System 1097
76. Histopathological findings in Whipple’s disease include all c. Giardiasis
of the following except: (AI 2008) d. Microscipic colitis Ref: Harrison’s 18/e p2480, 83
a. Marked increase in the number of macrophages in the 83. Pseudopolyps are typically seen in: (DNB Dec 2009)
mucosa a. Crohn’s disease
b. Marked increase in the number of intraepithelial b. Ulcerative colitis
lymphocytes c. Celiac disease
c. Dilatation of Lymphatics in the mucosa d. Tropical sprue Ref: Harrison’s 18/e p2480-81
d. Lipid deposition in the mucosa
84. A 41 year old male patient presented with recurrent episodes
Ref: Harrison’s 18/e p2471
of bloody diarrhea and mucus for 5 years. Despite regular
77. Which of the following parastitic infestation can lead to treatment with adequate doses of sulfasalazine, he has had
malabsorption syndrome? (AI 06) several exacerbations of his disease and required several
a. Amoebiasis weeks of steroids for the control for flares. What should be
b. Ascariasis the next line of treatment for him? (AIIMS Nov 03)
c. Hookworm infestation a. Methotrexate
d. Giardiasis Ref: Internet b. Azathioprine
78. The short bowel syndrome is characterized by all of the c. Cyclosporine
following except: (AI 2004) d. Cyclophosphamide
a. Diarrhea Ref: Harrison’s 18/e p2490, 17/e p1896
b. Hypogastrinemia 85. A patient gives chronic history of Diarrhoea and blood in
c. Weight loss stool presents with multiple fistulae in the perineum and
d. Steatorrhoea multiple stricture in small intestine. The diagnosis is:
Ref: Harrison’s 18/e p2472-73, 17/e p1882-83 a. Crohn’s disease (AIIMS June 2000)
79. Deficiency of which of the following vitamin is most b. Radiation enteritis
commonly seen in short bowel syndrome with ileal c. Ulcerative Colitis
resection: (AI 2012) d. Ischemic bowel disease
a. Vitamin B12 (Cyanocobalamine) Ref: Harrison’s 18/e p2483, 86, 17/e p1890,92
b. Vitamin B1 (Thiamine) 86. The presence of anti-Saccharomyces cerevisae antibody is a
c. Folic Acid surrogate marker of one of the following: (AI 2006)
d. Vitamin K Ref: Harrison’s 18/e p2472-73 a. Celiac disease
80. Which of the following vitamin deficiencies is most b. Crohn’s disease
commonly seen in short bowel syndrome: (AI 2012) c. Ulcerative colitis
a. Vitamin B12 d. Tropical sprue
b. Biotin Ref: Harrison’s 18/e p2483, 2486 17/e p1890, 92
c. Vitamin B1 87. Crohn’s disease may be caused by which one of the
d. Vitamin K Ref: Harrison’s 18/e p2472-73 following infectious agents: (AI 2008)
81. “Intestinal angina” is a symptom complex of the following: a. Clostridium difficle
(AIIMS May 06) b. Mycobacterium paratuberculosis
a. Postprandial abdominal pain, weight loss, acute c. Cytomegalo virus (CMV)
mesenteric vessel occlusion d. Mycoplasma Ref: Harrison’s 18/e p2480-82, 17/e p1887
b. Postprandial abdominal pain, weight loss, chronic 88. Extraintestinal manifestations of Inflammatory bowel
mesenteric vessel occlusion disease include all of the following, except: (PGI June 05)
c. Pre-prandial abdominal pain, weight loss, chronic a. Uveitis
MEDICINE
mesenteric vessel occlusion b. Sclerosing cholangitis
d. Pre-prandial abdominal pain, weight gain acute c. Osteoarthritis
mesenteric vessel occlusion Ref: Harrison’s 17/e p1911 d. Skin nodules
Ref: Harrison’s 18/e p2487-88, 17/e p1893-94
D. INFLAMMATORY BOWEL DISEASE 89. A 25 years women presents with bloody diarrhea and is
diagnosed as a case of Ulcerative colitis. Which of the
82. A 25 year old male presents with a history of chronic following condition is not associated: (AI 2002)
diarrhea. Pathological examination reveals cryptitis and a. Sclerosing cholengitis
crypt abscesses. The likely diagnosis is: (AI 2008) b. Iritis
a. Crohn’s disease c. Ankylosing spondylitis
b. Ulcerative colitis d. Pancreatitis Ref: Harrison’s 18/e p2487-88, 17/e p1893-94
Ans. 76. b. Marked increase... 77. d. Giardiasis 78. b. Hypogastrinemia... 79. a. Vitamin B12...
80. a. Vitamin B12... 81. b. Postprandial ... 82. a and b 83. b. Ulcerative colitis...
84. b. Azathioprine 85. a. Crohn’s disease. 86. b. Crohn’s disease 87. a. Clostridium difficle
88. c. Osteoarthritis 89. d. Pancreatitis
1098 Jaypee’s Triple A
E. COLONIC DISORDERS and leucopenia. In the course of the disease she develops
acute abdominal event and died. Which of the following is
90. Which one of the following is not a feature of irritable the likely finding on autopsy: (AI 2012)
bowel syndrome? (AIIMS May 05) a. Transverse ulcers
a. Abdominal pain b. Longitudinal ulcers
b. Constipation c. Pinpoint ulcers
c. Rectal bleeding d. Pseudopolyps Ref: Harrison’s 18/e p1276
d. Bloating 98. A patient present with lower gastrointestinal bleed.
Ref: Harrison’s 18/e p2496-97, 17/e p1899-1900 Sigmoidoscopy shows ulcers in the sigmoid. Biopsy from
91. A young girl presents with abdominal pain and a recent this area shows flask-shaped ulcers. Which of the following
change in bowel habit, with passage of mucus in stool. is the most appropriate treatment: (AIIMS Nov 05)
There is no associated blood in stool and symptoms are a. Intravenous ceftriaxone
increased with stress. The most likely diagnosis is: (AI 2010) b. Intravenous mentronidazole
a. Irritable bowel syndrome c. Intravenous steroids and sulphasalazine
b. Ulcerative Colitis d. Hydrocortscone enemas
c. Crohn’s disease Ref: Harrison’s 18/e p1686, 17/e p1278
d. Amebiasis 99. Investigation of choice for invasive amebiasis is: (AI 02)
Ref: Harrison’s 18/e p2496-97, 17/e p1899, 1900 a. Indirect heamagglutination
92. Which of the following drugs is used for Irritable Bowel b. ELISA
Syndrome of the constipating type: (AI 2012) c. Counter immune electrophoresis
a. Lubiprostone d. Microscopy Ref: Harrison’s 18/e p1685
b. Cholestyramine 100. Features of Typhoid Ulcers include all of the following
c. Alosetron except:
d. Rifaximin a. Bleeding
Ref: Harrison’s 18/e p2500-01 b. Perforation
93. Which of the following organism does not cause invasive c. Stricture and obstruction
diarrhea: (DNB June 2009) d. Longitudinal orientation
a. Bacillus cereus 101. A patient with leukemia on chemotherapy develops acute
b. Aeromonas sp right lower abdominal pain associated with anemia,
c. Rota virus thrombocytopenia and leukopenia. Which of following is
d. Shigella the clinical diagnosis? (AI 2006)
Ref: Harrison’s 18/e p311, 17/e p248 a. Appendictis
94 Secretory Diarrhea is caused by all of the following, except: b. Leukemic colitis
a. Cholera (PGI June 07) c. Perforation peritonitis
b. Laxatives d. Neutropenic colitis Ref: Harrison’s 18/e p2276
c. Excess phenolphthalein intake 102. Strong correlation with colorectal cancer is seen in: (AI 2003)
d. Clostridium difficle a. Peutz-Jegher’s polyp
Ref: Harrison’s 18/e p3111-12, 17/e p249 b. Familial polyposis coli
95. Features of secretory diarrhea include all of the following, c. Juvenile polyposis
except: (AI 2009) d. Hyperplastic polyp
a. Stool volume > 1D /day Ref: Harrison’s 18/e p768, 769, 7770 17/e p574, 575
b. Normal osmotic anion gap 103. Which of the following colonic polyps is not premalignant?
c. Reduces with fasting a. Juvenile polyps (AI 2006)
MEDICINE
Ans. 90. c. Rectal bleeding 91. a. Irritable bowel... 92. a. Lubiprostone 93. a. Bacillus cereus
94. d. Clostridium difficle 95. c. Reduces with fasting 96. a, b and c 97. b. Longitudinal ulcers
98. b. Intravenous... 99. b. ELISA 100. c. Stricture ... 101. d. Neutropenic colitis
102. b. Familial... 103. a. Juvenile polyps 104. a. Ulcerative colitis..
Gastrointestinal System 1099
105. A 25-year old male had pigmented macules over the palm, a. High fiber diet
sole and oral mucosa. He also had anemia and pain in b. Low fat diet
abdomen. The most probable diagnosis is: (AIIMS May 05) c. Low selenium diet
a. Albright’s syndrome d. Low protein diet Ref: Internet
b. Cushing’s syndrome
c. Peutz-Jegher’s syndrome
d. Incontinentia pigmenti Ref: Harrison’s 18/e p412 F. PANCREATIC DISORDERS
106. A patient presents with malena, hyperpigmentation over
113. Raised serum amylase levels are used to diagnose:
lips, oral mucosa and skin; and his sister is also having
a. Autoimmune disease (AIIMS May 04)
similar complaints. The diagnosis is: (AI 2000)
b. Degenerative diseases
a. Peutz Jegher’s Syndrome
c. Acute cholecystitis
b. Familial Adenomatous Polyposis
d. Acute pancreatitis
c. Gardner’s Syndrome
Ref: Harrison’s 18/e p2636, 17/e p2002, 2008
d. Villous Adenoma Ref: Harrison’s 18/e p769
114. Which of the following is not a prognostic factor for Acute
107. A girl presents with complaints of malena. On examination
Pancreatitis: (AIIMS Nov 06)
there are pigmented lesions involving her mouth and lips.
a. Serum Amylase
Two of her sisters also had similar complaints. Which of the
b. Serum Calcium
following is the most probable diagnosis: (AIIMS Nov 2000)
c. Serum Glucose
a. Kornkhite Canada syndrome
d. Serum AST Ref: Harrison’s 18/e p2636, 2637 17/e p2008
b. Puetz Jegher’s syndrome
c. Gardner’s syndrome 115. Which one is not the bad prognostic sign for pancreatitis:
d. Turcot’s syndrome Ref: Harrison’s 18/e p769 a. TLC > 16000 (AIIMS June 2000)
b. Calcium less than 8 mmom/L
108. Most important prognostic factor for colorectal carcinoma
c. Glucose > 200mg%
is: (AI 2009)
d. Prothrombin > 2 times the control
a. Site of lesion
Ref: Harrison’s 18/e p2637
b. Stage of lesion
c. Age of patient 116. Monu, a 30 year old male, a chronic alcoholic presents with
d. Lymph node status Ref: Harrison’s 18/e p771 sudden onset of epigastric pain that radiates to the back. All
are seen except: (AIIMS June 01)
109. After undergoing surgery, for Carcinoma of colon a patient
a. Low serum lipase
developed single liver metastatis of 2cm. What you do next:
b. Increased LDH
a. Resection (AI 2002)
c. Hypocalcemia
b. Chemo radiation
d. Increased serum amylase
c. Acetic acid injection
Ref: Harrison’s 18/e p2636, 2637
d. Radio frequency ablation
Ref: Harrison’s 18/e p774, 17/e p578 117. The most specific laboratory test to establish a diagnosis of
acute pancreatitis is: (PGI 08)
110. A 50-year old male, working as a hotel cook, has four
a. Serum amylase
dependent family members. He has been diagnosed with
b. Serum lipase
an early stage squamous cell cancer of anal canal. He has
c. Serum Alkaline phosphatase
more than 60% chances of cure. The best treatment option
d. Serum Calcium
is: (AI 2003)
e. Serum glucose
a. Abdomino-perineal resection
Ref: Harrison’s 18/e p2636-37, 17/e p2008
b. Combined surgery and radiotherapy
118. Cause of acute loss of vision in a patient of alcoholic
MEDICINE
c. Combined chemotherapy and radiotherapy
d. Chemotherapy alone Ref: Harrison’s 18/e p776 pancreatitis is: (AI 2000)
a. Purtscher’s retinopathy
111. All of the following genes may be involved in development
b. Sudden alcohol withdrawal
of carcinoma of colon except: (AI 2009)
c. Acute congestive glaucoma
a. APC
d. CRAO Ref: Harrison’s 18/e p2642
b. Beta – Catenin
c. K- ras 119. Which of the following types of pancreatitis has the best
d. Mismatch Repair Genes Ref: Robbin’s 8/e p822-823 prognosis? (AI 2004)
a. Alcoholic pancreatitis
112. Based on Epidemiological studies, which of the following
b. Gall stone pancreatitis
has been found to be most protective against Carcinoma
c. Post operative pancreatitis
Colon: (AI 2009)
d. Idiopathic pancreatitis
Ans. 105. c. Peutz-Jegher’s... 106. a. Peutz-Jegher’s... 107. b. Peutz-Jegher’s... 108. b. Stage of lesion
109. a. Resection
110. c. Combined... 111. None 112. a. High fiber diet
113. d. Acute pancreatitis 114. a. Serum Amylase...
115. d. Prothrombin... 116. a. Low serum lipase
117. b. Serum lipase
118. a. Purtscher’s retinopathy 119. b. Gall stone pancreatitis
1100 Jaypee’s Triple A
120. Medical treatment of pancreatitis includes: cystic fibrosis Sweat chloride levels have been observed
a. Cholestyramine (AIIMS May 04) between 40-60 mmol/l on two separate occasions. Which of
b. Aprotinin the following test should be performed next to support the
c. Calcium diagnosis of Cystic fibrosis: (AI 2009)
d. Glucagon Ref: Harrison’s 18/e p2640, 17/e p2009, 2010 a. Repeat Sweat chloride levels on a different day
121. The triad originally described by Zollinger Ellison b. Demonstrate an abnormal nasal potential difference
syndrome is characterized by: (AI 2002) c. Demonstrate an abnormal F508 mutation by DNA
a. Peptic ulceration, gastric hypersecretion, non beta cell analysis
tumour d. Demonstrate an abnormal 72 hour fecal fat collection
b. Peptic ulceration, gastric hypersecretion, beta cell tumour Ref: Harrison’s 18/e p2149, 17/e p1634
c. Peptic ulceration, achlorhydria, non beta cell tumour 128. Non invasive diarrhea is caused by: (AI 2009)
d. Peptic ulceration, achlorhydria, beta cell tumour a. Shigella
122. Treatment of choice for ZES: (AI 2007) b. Cereus
a. PPI c. Salmonella
b. Somatostatin analogues d. Y. enterocolitica Ref: Harrison’s 18/e p310, 312
c. Streptozocin 129. Features of secretory diarrhea include all of the following,
d. Sucralfate Ref: Harrison’s 18/e p2456, 17/e p1868 except: (AI 2009)
123. Gold standard test for diagnosis of Insulinoma is: (AI 2009) a. Stool volume > 1L /day
a. ‘72 hour’ fast test b. Normal osmotic anion gap
b. Plasma Glucose levels < 3 mmol/l c. Reduces with fasting
c. Plasma Insulin levels > 6µU/ml d. Painless Ref: Harrison’s 18/e p321, 17/e p249
d. C- peptide levels < 50 pmol/e 130. Which of the following is the commonest cause of lower GI
Ref: Harrison’s 18/e p3066, 3067 bleed: (PGI Dec 04)
124. Which of the following statements about Pancreatic a. Angiodysplasia
Carcinoma is not true: (AI 2009) b. Enteric fever
a. Mutation in P53 gene is associated in 75% of cases c. Diverticulosis
b. Hereditary Pancreatitis significantly increases the risk d. Colonic polyps
c. Median survival in locally advanced (stage III) disease is e. Hemorrhoids Ref: Harrison’s 18/e p2508, 17/e p259
3-6 months 131. An elderly patient presents with a prolonged history of
d. Five year survival after curative pancreaticoduodenectomy weakness and lethargy. On examination he is found to be
is 15 – 20% anemic and stool is positive for occult blood. Which of the
following is the investigation of choice: (AI 2010)
a. Colonoscopy
G. MISCELLANEOUS b. Barium meal
c. Barium enema
125. All of the following about Gastrointestinal Carcinoid d. CT abdomen Ref: Harrison’s 18/e p2409-10
tumors are true, except: (AI 2010)
132. Massive bleeding per rectum in a 70 yr old patient is due to:
a. Small intestine and appendix account for almost 60% of
a. Diverticulosis (AI 2000)
all gastrointestinal carcinoid
b. Carcinoma colon
b. Rectum is spared
c. Colitis
c. 5 year survival for carcinoid tumors is >60%
d. Polyps Ref: Harrison’s 18/e p321, 2502, 2503
d. Appendicial carcinoids are more commin in females than
males 133. A man aged 60 yrs has h/o IHD and atherosclerosis. He
presents with abdominal pain and maroon stools: likely
MEDICINE
Ans. 120. c. Calcium... 121. a. Peptic ulceration 122. a. PPI 123. a. ‘72 hour’ fast test
124. c. Median
125. b. Rectum is spared 126. b. Abnormality... 127. b. Demonstrate...
128. b. Cereus
129. c. Reduces 130. e. Hemorrhoids 131. a. Colonoscopy
132. a. Diverticulosis
133. b. Acute mesenteric... 134. c. Hyponatremia...
Gastrointestinal System 1101
135. Hypergastrinemia with hypochlorhydria is seen in: 137. The advantage of Bladder drainage over Enteric drainage
a. Zollinger Ellison Syndrome (AI 2002) after Pancreatic Transplantation is better monitoring of:
b. VIPoma a. HBA1C levels (AI 2009)
c. Pernicious anemia b. Amylase levels
d. Glucagonoma Ref: Harrison’s 18/e p867 c. Glucose levels
136. All of the following statements about Mucosa Associated d. Electrolyte levels
Lymphoid Tissue Lymphomas (MALT) are true, except: Ref: Harrison’s 18/e p928, 17/e p694
a. Present at extranodal sites (PGI June 05) 138. Parastomal hernia is most frequently seen with: (AI 2009)
b. Predisposed by H. Pylori infection a. End Colostomy
c. Present as stromal polyps b. Loop Colostomy
d. Are sensitive to chemotherapy c. End Iliostomy
Ref: Harrison’s 18/e p, 17/e p d. LoopIliostomy Ref: Internet
MEDICINE
Ans. 135. c. Present... 136. c. Present... 137. b. Amylase... 138. a. End Colostomy
4. RESPIRATORY SYSTEM
A. DISTURBANCE IN RESPIRATORY 7. A 10 year old boy with short stature presents with polyuria
and polydipsia. Laboratory values reveal a pH of 7.4; HCO3
FUNCTIONS AND RESPIRATORY FAILURE of 17; Na+ 140; pCO2 32; K+ 4.9; Cl-112. Likely acid base
abnormality is: (AIIMS Nov 2010)
1. False statement about type I respiratory failure is: (AI 2001) a. Metabolic Alkalosis
a. Decreased PaO2 b. Non-Anion gap Metabolic Acidosis
b. Decreased PaCO2 c. Anion gap Metabolic Acidosis
c. Normal PaCO2 d. Respiratory Acidosis Ref: Harrison’s 18/e p363, 365
d. Normal A-a gradient 8. Blood gas measurements of a patient shows the following
Ref: Harrison’s 18/e p2198, 2199 17/e p1590 values pH 7.2, pCO2 80 mm Hg, pO2 46 mm Hg. Which of the
2. In type - II respiratory failure, there is: (AIIMS Nov 02) following could be the most probable diagnosis:
a. Low pO2 and low pCO2 a. Acute asthma (AIIMS Nov 2000)
b. Low pO2 and high pCO2 b. Acute exacerbation of COPD
c. Normal pO2 and high pCO2 c. ARDS
d. Low pO2 and normal pCO2 d. Severe pneumonia Ref: Harrison’s 18/e p365, 2210
Ref: Harrison’s 18/e p2200, 17/e p1676 9. The blood gas parameters: pH 7.58, pC02 23 mm Hg pO2
300 mm Hg and oxygen saturation 60% are most consistent
3. A patient with salicylic acid poisoning has the following
with: (AI 03)
arterial blood gas analysis report: pH =7.12: pCO2 =
a. Carbon monoxide poisoning
18mmHg; HCO3 = 12mmol/L. The resulting acid base b. Ventilatory malfunction
abnormality can be best labeled as: (AIIMS Nov 03) c. Voluntary hyperventilation
a. Metabolic acidosis with compensatory respiratory d. Methyl alcohol poisoning Ref: Harrison’s 18/e p287, 2210
alkalosis
10. Cavernous Respiration is seen in: (DNB 2012)
b. Metabolic acidosis with compensatory respiratory a. Cavity
alkalosis b. Consolidation
c. Respiratory acidosis with metabolic alkalosis c. Fibrosis
d. Metabolic acidosis d. Interstitial Inflammation Ref: Internet
Ref: Harrison’s 18/e p2198, 365 17/e p1590
11. Bilateral Rhonchii may be seen in all of the following
4. In a patient PO2 is 85 mmHg, PCO2 – 50mmHg, pH is 7.2 except: (PGI June 08)
and HCO3 is 32 meq/l is suffering from: (AIIMS June 2000) a. Pulmonary Edema
a. Respiratory acidosis with compensatory metabolic b. Bronchiectasis
alkalosis c. Pulmonary Embolism
b. Respiratory acidosis with compensatory metabolic d. Emphysema Ref: Harrison’s 18/e p2172-73, 17/e p1584
acidosis
c. Metabolic acidosis B. OBSTRUCTIVE/RESTRICTIVE
d. Metabolic alkalosis
Ref: Harrison’s 18/e p363, 17/e p1590, 288, 289
PULMONARY DISEASES, PULMONARY
5. A patient presents with following parameters pH 7.5,p CO2 FUNCTION TESTS
30 mmHg, pO2 102 mmHg and HCO3 16 meq/l. Which of the
12. All of the following are characteristic feature of obstructive
following correctly describes the compensatory mechanism:
MEDICINE
pulmonary disease, except: (DNB June 2010)
a. Respiratory Alkalosis (AI 2010)
a. Normal Residual Volume
b. Metabolic Alkalosis
b. Decreased FEV1
c. Respiratory Acidosis
c. Normal Vital Capacity
d. Metabolic Acidosis Ref: Harrison’s 18/e p365
d. Decreased FCV1/FVC Ref: Harrison’s 18/e p2196
6. The acid base status of a patient reveals a pH = 7.46 and
13. Pulmonary function changes in acute bronchial asthma in
pCO2=30 mm Hg. The patient has a partially compensated
untreated patient: (PGI June 03)
primary: (AI 2011)
a. ↑ ed peak expiratory flow
a. Metabolic acidosis
b. ↑ ed TLC
b. Metabolic alkalosis
c. ↑ ed FVC
c. Respiratory alkalosis
d. ↑ ed RV
d. Respiratory acidosis Ref: Harrison’s 18/e p365, 363
e. ↑ ed FEV Ref: Harrison’s 18/e p2155, 2086
Ans. 1. d. Normal A-a gradient 2. b. Low pO2 and ... 3. a. Metabolic acidosis... 4. a. Respiratory...
5. d. Metabolic Acidosis 6. c. Respiratory alkalosis 7. b. Non-Anion gap... 8. b. Acute exacerbation
9. b. Ventilatory. 10. a. Cavity 11. c. Pulmonary Embolism 12. a. Normal Residual ...
13. e. ↑ ed FEV...
1104 Jaypee’s Triple A
14. The abnormal preoperative pulmonary function test in a 20. Bronchial Asthma is characterized by all of the following,
patient with severe kyphoscoliosis includes: (AI 05) except: (PGI June 05)
a. Reduced RV/TLC a. Inflammatory disease of airway
b. Reduced FEV1/FVC. b. Allergic disease of airway
c. Reduced FEV25-75 c. Hyporesponsiveness of airway
d. Increased FRC Ref: Harrison’s 17/e p1588 d. Hyperresponsiveness of airway
15. A 28 years old woman having limited cutaneousscleroderma e. Treatment is mostly inhaled steroid
for the last 10 years complains of shortness of breath for Ref: Harrison’s 18/e p2104,2108, 2111, 17/e p1598, 1601, 1603
last one month. Her pulmonary functions tests (PFT) are as 21. All of the following are true about Asthma, except:
follows: a. Charcot layden crystals may be seen in sputum
PFT Observed Predicted b. Reversible airflow obstruction is a characteristic feature
c. Large airways are involved
FVC 2.63 2.82 d. Small airways are not involved
FEV1% 88% 80% e. Intermittent asthma responds better to bronchodilator
therapy than persistant Asthma
DLCO 5.26 16.3 Ref: Harrison’s 18/e p2107
What is the most likely diagnosis in this case? (AI 06) 22. True about Asthma except: (PGI 05)
a. Inflammatory disease
a. Intestitial lung disease
b. Hyperresponsive airways
b. Pulmonary artery hypertension
c. Necrosis of airways
c. Congestive heart failure
d. Mucous plug formation
d. Bronchiectasis
e. Airway edema Ref: Harrison’s 18/e p2107, 17/e p1600
Ref: Harrison’s 18/e p2164
23. Feature of Acute severe Asthma include all of the following,
16. A patient presents with decreased vital capacity and total except:
lung volume. What is the most probable diagnosis? a. Tachycardia > 120/min
a. Bronchiectasis (AI 2007) b. Pulsus paradoxus
b. Sarcoidosis c. Respiratory acidosis
c. Cystic fibrosis d. PEFR < 50%
d. Asthma e. None
Ref: Harrison’s 18/e p2088, 2807-08, 17/e p1588, 1589 Ref: Harrison’s 18/e p2108, 2113
17. Carbon monoxide diffusion capacity decreases in all, 24. In a patient with bronchial asthma silent chest signifies:
except: (AI 2003) a. Good Prognosis (DNB 2010)
a. Emphysema b. Bad Prognosis
b. Primary pulmonary hypertension c. Grave Prognosis
c. Alveolar haemorrhage d. Not a Prognostic sign Ref: Harrison’s 18/e p2109
d. Infiltrative lung disease 25. Universal finding in Asthma is (PGI June 02)
Ref: Harrison’s 18/e p2092, 17/e p1591, 1592 a. Hypoxia
18. All of the following are true about intrinsic Asthma, except: b. Hypercarbia
a. Increasing Incidence (PGI Dec 04) c. Respiratory acidosis
b. Allergic Asthma is more common in young patients d. Metabolic Acidosis Ref: Harrison’s 18/e p2109
c. Idiosyncratic Asthma is more common older patients 26. Bronchial Asthma is best diagnosed by: (PGI Dec 01)
d. IgE is increased in Idiosyncratic Asthma a. Wheeze
MEDICINE
MEDICINE
34. In an emphysematous patient with bullous lesions, which b. Fibrous pleural thickening
is the best investigation to measure lung volume: c. Emphysema
a. Body Plethysmography d. Calcific pleural plaques
b. Gas dilution Ref: Harrison’s 18/e p2161-62, 17/e p1612, 1613
c. Transdiaphragmatic pressure
43. True statements about asbestosis: (PGI Dec 04)
d. DLCO Ref: Harrison’s 18/e p2139, 17/e p1587
a. Causes Lung Ca
35. Rampal, 45 yr old man presents with history of recurrent b. Pleural mesothelioma
hemoptysis and purulent sputum. His chest X-Ray is c. Peritoneal mesothelioma
normal, which of the following will be the next best d. Pulmonary fibrosis
investigation for him? (AIIMS Nov 01) e. All of the above Ref: Harrison’s 18/e p2123-24
a. HRCT
b. CT guided angiography
Ans. 28. c. Montelukast 29. b. Acute asthmatic attack 30. a. < 5 µ 31. a. Chronic Bronchitis
32. c. Restrictive... 33. b.Increased... 34. a. Body Plethysmography 35. a. HRCT
36. b. FEV1/FVC < 0.7 37. c. Decreased FEV1/FVC 38. b. Residual volume... 39. b. HRCT
40. d. Tobacco smokes 41. c. Rheumatoid 42. c. Emphysema 43. e. All of the above
1106 Jaypee’s Triple A
44. All of the following statements about silicosis are true, 51. True statements about ABPA include all of the following,
except: (PGI Dec 04) except: (PGI Dec 04)
a. Pleural plaques a. Serum IgE > 1000 ng/ml
b. Prediliction for upper lobes b. Eosinophils > 1000/mm3
c. Calcific hilar Lymphadenopathy c. Elevated IgG antibodies
d. Associated with tuberculosis d. Proximal Bronchiectasis
Ref: Harrison’s 18/e p2125, 2123 e. Lower lobe predominance Ref: Harrison’s 18/e p2120
45. Not seen idiopathic pulmonary hemosiderosis: (AI 2009) 52. Which is not true about aspergillosis:
a. Aspergillus Niger is the cause of fungal otitis externa
a. Eosinopenia
b. It is highly contagious
b. Iron deficiency anemia
c. Aspergilloma is common in preexisting TB, or cystic
c. Diffuse alveolar hemorrhage
disease
d. Hemoptysis Ref: Harrison’s 18/e p2165
d. Aspergillus fumigatus is a cause of bronchial Aspergillosis
46. All of the following are true about Idiopathic Pulmonary Ref: Harrison’s 18/e p1655, 17/e p1257, 1256
Hemosiderosis except: 53. 40 year old patient with history of prolonged exposure
a. Hypoxemia to Aspergillus presents with repeated episodes of
b. Alveolar capillary constriction breathlessness. Chest X-ray shows diffuse pulmonary
c. Hyperplasia of type II Pneumocytes infiltrates. Skin hypersensitivity test is positive for
d. Hemosiderin laden macrophages Aspergillus antigen Peripheral blood picture shows normal
Ref: Harrison’s 18/e p2119 eosinophil count and serum IgE levels are normal. The most
47. All of the following are associated with pulmonary likely diagnosis is:
eosinophillic pneumonia, except: (PGI 2009) a. Allergic bronchial Asthma
a. ABPA b. Allergic Bronchopulmonary Aspergillosis (ABPA)
b. Loeffler’s pneumonia c. Extrinsic Allergic Alveolitis
c. Churg Strauss Syndrome d. Invasive Pulmonary Aspergillosis
Ref: Harrison’s 18/e p2120
d. Tropical pulmonary eosinophilia
e. Wegener’s granulomatosis…? 54. All the following are features of Tropical pulmonary
Ref: Harrison’s 18/e p2119, 17/e p1610 Eosinophilia except:
a. Eosinophilia > 3000/mm3
48. Diagnostic features of allergic bronchopulmonary
b. Microfilaria in blood
aspergillosis (ABPA) include all of the following except: c. Paroxysmal cough and wheeze
a. Changing pulmonary infiltrates (AI 2003) d. Bilateral chest mottling and increased bronchovascular
b. Peripheral eosinophilia markings Ref: Harrison’s 18/e p2120, 1748
c. Serum precipitins against Aspergillosis fumigants
55. All the following diseases are associated with peripheral
d. Occurrence in patients with old cavitatory lesions. blood eosinophilia except: (AIIMS Nov 02)
Ref: Harrison’s 18/e p2120, 17/e p1610 a. Allergic bronchopulmonary aspergillosis (ABPA).
49. Diagnostic criteria for Allergic Bronchopulmonary b. Loffler’s syndrome.
Aspergillosis include all, except: (PGI Dec 05) c. Pulmonary eosinophilic granuloma.
a. Peripheral eosinophilia (>0.1× 109/mm3) d. Chagas-Strauss syndrome Ref: Harrison’s 18/e p2120
b. Central bronchiectasis 56. A 54 year old smoker man comes with severe hemoptysis
c. Episodic Asthma weight loss and oligoarthritis. Serial skiagram shows
d. Detection of Aspergillus in sputum fleeting opacities. What is the diagnosis? (AIIMS Nov 08)
e. ↑ IgG antibodies specific to A. Fumigatus a. Allergic bronchopulmonary aspergillosis
MEDICINE
Ans. 44. a. Pleural plaques 45. a. Eosinopenia 46. b. Alveolar 47. d. Tropical...
48. d. Occurrence... 49. a. Peripheral... 50. d. Bronchopulmonary 51. e. Lower...
52. b. It is highly... 53. b. Allergic... 54. b. Microfilaria in blood 55. c. Pulmonary...
56. a. Allergic... 57. b. COPD
Respiratory System 1107
b. Bilateral interstitial infiltrates
C. ADULT RESPIRATORY DISTRESS c. PCWP <18 mm Hg
SYNDROME d. Normal Left atrial pressure
Ref: Harrison’s 18/e p2205, 17/e p1680
58. Shock lung is better known as: (DNB 2010) 67. Acute lung injury is characterized by all, except:
a. Alveolar Proteinosis a. Alveolar infiltrates (PGI Dec 04)
b. Alveolar Haemorrhage b. Hypoxemia
c. Pulmonary edema c. Pulmonary shunting
d. ARDS Ref: Harrison’s 18/e p2205, 17/e p1680 d. PaO2/FlO2 <200mm of Hg
59. Acute lung injury in caused by all of the following except: e. None of the above Ref: Harrison’s 18/e p2207, 17/e p1681
a. Aspiration (AIIMS May 02) 68. The most common cause of preventable hospital death is:
b. Toxic gas inhalation a. Acute Pulmonary Embolism (DNB 2009)
c. Cardiopulmonary bypass with heart lung machine. b. Heart Failure
d. Lung contusion. Ref: Harrison’s 18/e p2205 c. Myocardial Infarction
60. ARDS is associated with: (PGI Dec-04) d. Cancer Ref: Harrison’s 18/e p2170
a. Acute pancreatitis 69. The most common cause of Pulmonary thromboembolism:
b. Trauma a. DIC (DNB 2009)
c. Severe Falciparum malaria b. Coagulation Disorder
d. All of the above c. DVT
Ref: Harrison’s 18/e p2205, 17/e p1680 d. Venous Hypertension
61. A patient comes with sudden respiratory distress, on Ref: Harrison’s 18/e p2170, 17/e p1651
examination, bilateral basal crepts are present over chest
70. All of the following conditions may predispose to
suggestive of pulmonary edema with normal alveolar
pulmonary embolism except: (AI 2003)
wedge pressure. The likely cause is: (AIIMS June 2000)
a. Protein S deficiency
a. Narcotic overdose
b. Malignancy
b. Congestive heart failure
c. Obesity
c. Myocardial infarction
d. Progesterone therapy Ref: Harrison’s 18/e p2170,2171
d. Cardiogenic shock
71. In acute pulmonary embolism, the most frequent ECG
62. Which of the following is the most characteristic feature of
finding is: (AIIMS May’ 06)
Adult Respiratory Distress Syndrome (ARDS): (AI 2012)
a. S1Q3T3 pattern
a. Diffuse alveolar damage
b. P. pulmonale
b. Hypoxemia and hypoxia
c. Sinus tachycardia
c. Surfactant deficiency
d. Right axis deviation
d. Hypocapnia Ref: Harrison’s 18/e p2206
Ref: Harrison’s 18/e p2172, 17/e p1653
63. All of the following features can seen in ARDS except:
a. Pulmonary shunting 72. D-Dimer values may be increased in all of the following
b. Reduced compliance except:
c. Hypoxemia a. Myocardial infarction
d. Hypercapnia Ref: Harrison’s 18/e p2206 b. Pneumonia
c. Anticoagulant therapy
64. Which of the following is not seen in ARDS: (DNB 2012)
d. Pregnancy Ref: Harrison’s 18/e p2172, 17/e p2172
a. Hypoxemia
b. Hypercapnia 73. D-dimer is the most sensitive diagnostic test for:
MEDICINE
c. Pulmonary edema a. Pulmonary embolism (AIIMS May 05)
d. Stiff lung Ref: Harrison’s 18/e p2206, 17/e p1680,1681 b. Acute pulmonary oedema
c. Cardiac tamponade
65. Feature of shock lung is: (AIIMS Nov 07)
d. Acute myocardial infarction
a. Diffuse alveolar damage
Ref: Harrison’s 18/e p2172, 17/e p1653
b. Usual interstitial penumonitis
c. Organizing pneumonia 74. D-Dimer is the most sensitive test for:
d. Bronchilolitis a. DVT
Ref: Harrison’s 18/e p2205, 17/e p1680 b. Pulmonary Embolism
c. Acute Pulmonary adema
66. Acute Lung Injury (ALI) is characterized by all except:
d. Acute myocardial infarction
a. PaO2/FiO2 < 200mm Hg
Ref: Harrison’s 18/e p2172, 17/e p1653
Ans. 58. d. ARDS 59. None 60. d. All... 61. a. Narcotic overdose
62. a. Diffuse... 63. None 64. b. Hypercapnia... 65. a. Diffuse...
66. a. PaO2... 67. d. PaO2... 68. a. Acute 69. c. DVT
70. d. Progesterone... 71. c. Sinus... 72. c. Anticoagulant therapy 73. a. Pulmonary embolism
74. b. Pulmonary embolism
1108 Jaypee’s Triple A
75. Best investigation when there is clinical suspicion of 83. A 29 year old unmarried female presents with progressive
pulmonary embolism in a patient is: dyspnea. Her X ray chest shows clear lung fields. Pulmonary
a. D-Dimmer Assay function testing reveals in FVC of 92%; FEV1/FVC of 89%;
b. Multidetector CT angiography and dlCO of 59%. On exercise testing her oxygen saturation
c. Doppler ultrasound drops from 92% to 86%. What is the likely diagnosis:
d. Catheter angiography a. Alveolar hypoventilation (AIIMS Nov 2008)
Ref: Harrison’s 18/e p2172, 2174 17/e p1652 b. Primary pulmonary hypertension
76. A 55 year old man who has been on bed rest for the past 10 c. Interstitial lung disease
days, complains of breathlessness and chest pain. The chest d. Anxiety Ref: Harrison’s 18/e p2076
X-ray is normal. The next investigation should be: (AI 2003)
84. A 29 year old anxious lady presents with a history of
a. Lung ventilation-perfusion scan (AI 2004)
progressive breathlessness and exercise intolerance since
b. Pulmonary arteriography
four months. Her FVC is 90% and FEV1 / FVC is 86%.
c. Pulmonary venous angiography
d. Echocardiography Oxygen saturation after exercise was observed to drop from
Ref: Harrison’s 18/e p2173, 17/e p1652, 1663 92% to 86%. What is the likely diagnosis:
a. Primary alveolar hypoventilation (AI 2010)
77. The most definitive method of diagnosing pulmonary
b. Primary pulmonary hypertension
embolism is: (AIIMS Nov 05)
a. Pulmonary arterigraphy c. Anxiety disorder
b. Radioisotope perfusion pulmonary scintigraphy d. Interstitial lung disease
c. EKG Ref: Harrison’s 18/e p2077, 17/e p1587-1589
d. Venography Ref: Harrison’s 18/e p2173, 17/e p1654 85. Chronic Cor pulmonale is seen in all except: (PGI Dec 04)
78. A young patient presents to the Emergency with Acute a. Pulmonary embolization
pulmonary embolism. Patient’s blood pressure is normal b. COPD
but echocardiography reveals Right ventricular hypokinesia c. Cystic fibrosis
and compromised cardiac output. The treatment of choice d. Primary pulmonary hypertension
in this patient is: (AIIMS Nov 2001) Ref: Harrison’s 18/e p1914, 17/e p1454
a. Thrombolytic therapy
86. The most common cause for chronic cor pulmonale is:
b. Anticoagulation with low molecular weight heparin
c. Anticoagulation with warfarin a. Recurrent pulmonary embolization (PGI Dec 04)
d. Inferior vena cava filters b. COPD
Ref: Harrison’s 18/e p2174, 17/e p1654,1655 c. Cystic fibrosis
d. Bronchial Asthma
79. In primary pulmonary hypertension basic abnormality in
gene lies in: (AIIMS May 07) e. Airway foreign body Ref: Harrison’s 18/e p1913
a. Bone morphogenic protein receptor II 87. The most common cause of acute cor pulmonale is:
b. Endothelin a. Pneumonia. (AIIMS Nov 02)
c. Homebox gene b. Pulmonary thromboembolism.
d. PAX – 11 Ref: Harrison’s 18/e p2077, 17/e p1577 c. Chronic obstructive pulmonary disease.
80. Precapillary pulmonary hypertension is caused by all d. Primary spontaneous pneumothorax.
except: (PGI June 03) Ref: Harrison’s 18/e p1914, 17/e p1454
a. Mitral stenosis 88. All the following are radiological features of Chronic Cor
b. Pulmonary vasculitis pulmonale except: (AIIMS Nov 02)
c. Primary pulmonary hypertension a. Kerley B lines
d. Thromboembolism Ref: Harrison’s 18/e p2077, 2079 b. Prominent lower lobe vessels
81. All are causes of pulmonary hypertension except: c. Pleural effusion
a. Hyperventilation (AIIMS Nov 07) d. Cardiomegaly Ref: Harrison’s 18/e p1913, 1915
MEDICINE
b. Morbid obesity
c. High altitude
d. Fenfluramine D. DISORDERS OF PLEURA
Ref: Harrison’s 18/e p2078, 17/e p1577,1579
82. Pulmonary hypertension may occur in all of the following 89. Which one of the following conditions may lead to
conditions except: (AIIMS Nov 06) exudative pleural effusion: (AI 2003)
a. Toxic oil syndrome a. Cirrhosis
b. Progressive systemic sclerosis b. Nephrotic syndrome
c. Sickle cell anemia c. Congestive heart failure
d. Argemone mexicana poisoning d. Bronchogenic carcinoma
Ref: Harrison’s 18/e p2082 Ref: Harrison’s 18/e p2180, (Table 263.1 17/e p1660, (Table 257.1
Ans. 75. b. Multidetector 76. a. Lung ventilation... 77. a. Pulmonary 78. a. Thrombolytic
. 79. a. Bone... 80. a. Mitral stenosis... 81. a. Hyperventilation... 82. d. Argemone
83. b. Primary... 84. b. Primary... 85. a. Pulmonary... 86. b. COPD
87. b. Pulmonary... 88. b. Prominent... 89. d. Bronchogenic...
Respiratory System 1109
90. All of the following are causes of Transudative pleural 98. A 30 year old female comes acute breathlessness, neck vein
effusion except: (PGI June 02) distention, and absent breath sounds and mediastinal shift.
a. Nephrotic syndrome Which of the following should be done immediately?
b. Rheumatoid arthritis a. HRCT is the investigation of choice (PGI June 2008)
c. Constrictive pericarditis b. ABG analysis should be done
d. Myxedema c. CXR
e. Pulmonary embolism d. Large bore needle puncture of pleura
Ref: Harrison’s 18/e p2178, 17/e p1658 Ref: Harrison’s 18/e p2181
91. Left sided pleural effusion seen in: (PGI June 02) 99. IPPV can cause all of the following except: (AI 2001)
a. Pancreatitis a. Barotrauma
b. Rheumatoid arthritis b. Pleural effusion
c. Hypoproteinemia c. Missing
d. CCF d. None of the above
e. Esophageal rupture Ref: Harrison’s 18/e p2180 Ref: Harrison’s 18/e p2213, 17/e p1688
92. All of the following are true about pleural effusion, except:
(select two options):
a. Horizontal fluid level
E. PNEUMONIAS AND TUBERCULOSIS
b. Decreased lung volume
100. The most common causative organism for lobar pneumonia
c. Decreased heart sounds
is:
d. Decreased chest movement
a. Staphylococcus aureus
e. Succussion splash Ref: Harrison’s 18/e p2178
b. Streptococcus pyogenes
93. A high amylase level in pleural fluid suggests a diagnosis of: c. Streptococcus pneumoniae
a. Tuberculosis (AIIMS May 03) d. Hemophilus influenza Ref: Harrison’s 18/e p2131
b. Malignancy
101. Predisposing factors for pneumococcal pneumonia include
c. Rheumatoid arthritis
all of the following except: (PGI June 01)
d. Pulmonary infarction
a. CRF
Ref: Harrison’s 18/e p2178 b. Lymphoma
94. Amylase increased in pleural fluid is seen in following c. Old Age
except: (PGI June 2000) d. Thalassemia
a. Rheumatoid arthritis e. Sickle cell disease Ref: Harrison’s 18/e p2132
b. Esophageal perforation 102. Cavitatory lesions in lung are seen in: (AI 2010)
c. Malignancy a. Primary pulmonary Tuberculosis
d. Gall stone pancreatitis Ref: Harrison’s 18/e p2178 b. Staphylococcal pneumonia
95. In an adult patient with plural effusion, the most appropriate c. Pneumoconiosis
site for pluricentesis done by inserting a needle is in: d. Interstitial Lung disease
a. 5th intercostals space in midclevicular line (AI 2002) Ref: Harrison’s 18/e p2132, 2136
b. 7th intercostal space in mid axillary’s line 103. Devi, a 28 year female, has diarrhea, confusion, high grade
c. 2nd intercostals space adjacent to the sternum fever with bilateral pneumonitis. The diagnosis is:
d. 10th intercostal space adjacent to the vertebral column a. Legionella (AI 2000)
Ref: Harrison’s 18/e p2179 b. Neisseria meningitis
96. While inserting a central venous catheter, a patient develops c. Streptococcus pneumoniae
respiratory distress. The most likely cause is: (AI 2002) d. H. influenza
MEDICINE
a. Hemothorax Ref: Harrison’s 18/e p1237
b. Pneumothorax 104. A 30 year old male presents with pneumonia and diarrhea
c. Pleural effusion five days after discharge from a hospital. The drug of choice
d. Hypovolumia for treating him is:
Ref: Harrison’s 18/e p2181 a. Vancomycin
97. Spontaneous pneumothorax is commonly seen in: b. Ciprofloxacin
a. Smokers (PGI June 02) c. Azithromycin
b. Young females d. Gentamycin
c. Old age e. Tetracycline
d. Short statured men Ref: Harrison’s 18/e p2181 Ref: Harrison’s 18/e p1238, 17/e p930,932
Ans. 90. b. Rheumatoid... 91. e. Esophageal rupture 92. a. Horizontal fluid... 93. b. Malignancy
94. a. Rheumatoid... 95. b. 7th intercostal... 96. b. Pneumothorax 97. a. Smokers
98. d. Large bore... 99. b. Pleural effusion 100. c. Streptococcus... 101. d. Thalassemia
102. b. Staphylococcal... 103. a. Legionella 104. c. Azithromycin
1110 Jaypee’s Triple A
105. An elderly male admitted for Pneumonia presents with 112. Indication for prophylaxis in pneumocystis carini
diarrhea and gripping abdominal pain five days after pneumonia include (choose two best option):
discharge from the hospital. Drug which is likely to benefit a. CD4 count < 200 /µl
is: (DNB June 2009) b. Tuberculosis
a. Imodium c. Viral load > 25,000 copies/ml
b. Metronidazole d. Oral candidiasis Ref: Harrison’s 18/e p1672, 17/e p1269
c. Diphenoxylate 113. All of the following statements about primary Tuberculosis
d. Levofloxacin Ref: Harrison’s 18/e p1239, 17/e p930, 932
are true, except:
106. Atypical pneumonia can be caused by the following a. Cavitatory lesion
microbial agents except? (AI 2005) b. Pleural effusion
a. Mycoplasma pneumoniae c. Fibrocasseous lesion
b. Legionella pneumonia d. Phlyctenular conjunctivitis
c. Human Corona virus Ref: Harrison’s 18/e p1353, 17/e p1008-1011
d. Klebsiella pneumoniae
Ref: Harrison’s 18/e p2131, 17/e p1620 114. Primary Tuberculosis most commonly involves: (DNB 2012)
a. Lungs
107. All of the following features are seen in the viral pneumonia
b. Liver
except: (AI 2005)
c. Brain
a. Presence of interstitial inflammation.
d. Intestine Ref: Harrison’s 18/e p1342
b. Predominance of alveolar exudates.
c. Bronchiolitis. 115. Primary complex in which of the following sites suggest
d. Multinucleate giant cells in the bronchiolar wall. congenital tuberculosis: (DNB 2011)
a. Lungs
Ref: Harrison’s 18/e p1485
b. Liver
108. A 45 year old, HIV positive patient presents with features
c. Lymph nodes
of pneumonia. Characteristic histopathological features
d. Skin Ref: Harrison’s 18/e p1349
suggesting pneumocystis carinii pneumonia is:
a. Prominent interstitial pneumonitis (PGI Dec 2000) 116. Most common site of primary infection in congenital
b. Eosinophilic alveolar exudates tuberculosis is: (DNB 2012)
c. Prominent mononuclear cells in alveolar exudates a. Lungs
d. Neutrophilic infiltration of alveolar interstitium b. Liver
e. Hypertrophy of type I pneumocytes c. Lymph nodes
Ref: Harrison’s 18/e p1672 d. Skin Ref: Harrison’s 18/e p1349
109. A truck driver presented with history of fever since four 117. Most commonly involved organ in congenital tuberculosis
weeks, and dry cough. He also gives a history of weight is: (DNB 2010)
loss of about 10 kg. X-ray shows bilateral reticulonodular a. Lungs
infiltrates. The most likely diagnosis is: (AI 2010) b. Liver
a. Tuberculosis c. Lymph nodes
b. Pneumocystis carinii pneumonia d. Skin Ref: Harrison’s 18/e p1349
c. Pneumococcal pneumonia 118. All of the following statements about Miliary Tuberculosis
d. Interstitial lung disease are true except:
Ref: Harrison’s 18/e p1671-72, 17/e p1267, 1267, 1268 a. May occur following primary infection
110. All of the following statements about Pneumocystis jiroveci b. May occur following secondary reactivation
are true except: (AI 2008) c. Sputum microscopy is usually negative
a. Usually associated with CMV infection
MEDICINE
Ans. 105. d. Levofloxacin... 106. d. Klebsiella pneumoniae... 107. b. Predominance... 108. b. Eosinophilic
109. b. Pneumocystis
110. a. Usually... 111. b. Usually 112. a and d
113. a. Cavitatory lesion
114. a. Lungs... 115. b. Liver 116. b. Liver
117. a. Lungs
118. d. Montoux... 119. e. All
Respiratory System 1111
120. A man presents with fever, wt loss and cough; Mantoux 127. All of the following statements about small cell carcinomas
tests reveals an induration of 17 × 19 mm; Sputum cytology are true, except: (PGI June 06)
is negative for AFb. Most likely diagnosis is: (AI 2001) a. Commonest Malignancy of lung
a. Pulmonary tuberculosis b. Associated with parancoplastic syndrome
b. Fungal infection c. Cause SVC obstruction
c. Viral infection d. Chemosensitive
d. Pneumonia e. Commonly metastasis to brain
Ref: Harrison’s 18/e p1350 Ref: Harrison’s 18/e p738, 17/e p551,552
121. A 25 year old man presented with fever, cough, expectoration 128. Which of the following statements about small cell
and breathlessness of 2 months duration. Contrast enhanced carcinoma is true? (AI 2009)
computed tomography of the chest showed bilateral upper a. Bone metastasis is uncommon
lobe fibrotic lesions and mediastinum had enlarged necrotic b. Peripheral in location
nodes with peripheral rim enhancement. Which one of the c. Chemosensitive tumor
following is the most probable diagnosis: (AI 03) d. Paraneoplastic syndrome with ↑PTH is common
a. Sarcoidosis Ref: Harrison’s 18/e p751, 17/e p551, 552, 553, 554
b. Tuberculosis
129. All of the following statements about Non Small Cell
c. Lymphoma
Carcinoma of Lung (NSCCL) are true, except: (AI 2012)
d. Silicosis Ref: Harrison’s 18/e p1350
a. Contralateral mediastinal nodes are a contraindication to
122. Rasmussen’s aneurysm arises from: surgical resection
a. Bronchial artery b. Single Agent Chemotherapy is preferred for patient > 70
b. Pulmonary artery years with advanced disease
c. Vertebral artery
c. Squamous Cell Carcinoma is the most common NSCCL
d. Posterior intercostal artery Ref: Harrison’s 18/e p1345
amongst Asian population
123. Multidrug Resistance Tuberculosis (MDR-TB) should be d. Gefitinib is most effective for female smokers with
considered in patients with: (PGI Dec 05) adenocarcinoma on histology Ref: Harrison’s 18/e p751
a. Contact with a known case of MDR TB
130. In a chronic smoker, a highly malignant aggressive and
b. Clinical Deterioration
metastatic lung carcinoma is: (AIIMS May 01)
c. Sputum smear positive at 5 months of treatment
a. Squamous cell Carcinoma
d. All of the above Ref: Harrison’s 18/e p1355
b. Small cell Carcinoma
124. A young man with tuberculosis presents with massive c. Adenocarcinoma
recurrent hemoptysis. For angiographic treatment which d. Large cell carcinoma
vascular structure should be evaluated first: (AIIMS May 03)
Ref: Harrison’s 18/e p738, 17/e p552,554
a. Pulmonary artery
b. Bronchial artery 131. Carcinoma lung responding best to chemotherapy:
c. Pulomary vein a. Squamous cell type
d. Superior vena cava b. Oat cell type
Ref: Harrison’s 18/e p284, 17/e p227 c. Adenocarcinoma
d. All respond equally Ref: Harrison’s 18/e p738
132. A 60 year old man presents with non productive cough and
F. BRONCHOGENIC TUMORS haemoptysis for 4 weeks; He has grade III clubbing, and a
125. A 62 years old man with carcinoma of lung presented to lesion in the apical lobe on x ray. Most likely diagnosis here
emergency department with respiratory distress. His EKG is: (AI 2001, AIIMS June 2000)
showed electrical alternans. The most likely diagnosis is: a. Small cell caricinoma
MEDICINE
a. Pneumothorax. (AIIMS May 03) b. Non small cell caricinoma
b. Pleural effusion. c. Fungal infection
c. Cardiac tamponade. d. Tuberculosis Ref: Harrison’s 18/e p742, 17/e p552-554
d. Constrictive pericarditis. 133. True statement about adenocarcinoma lung are (Choose
Ref: Harrison’s 18/e p1972, 17/e p1972,1973 two best options): (PGI June 05)
126. Most common bronchogenic carcinoma is: a. Common in females
a. Small cell carcinoma (AIIMS June 2000) b. Not associated with smoking
b. Squamous cell carcinoma c. Central cavitation is a charachteristic feature
c. Mixed cell carcinoma d. Peripheral involvement is common
d. Adenocarcinoma e. Upper lobe involvement is common
Ref: Harrison’s 18/e p738, 17/e p552 Ref: Harrison’s 18/e p738, 17/e p552
Ans. 120. a. Pulmonary... 121. b. Tuberculosis 122. b. Pulmonary artery 123. d. All
124. b. Bronchial artery
125. c. Cardiac tamponade... 126. d. Adenocarcinoma 127. a. Commonest...
128. c. Chemosensitive
129. d. Gefitinib... 130. b. Small cell ... 131. b. Oat cell type
132. b. Non small cell...
133. a. Common...
1112 Jaypee’s Triple A
134. Lung to lung metastasis is seen in: a. Radiotherapy
a. Adenocarcinoma of lung b. Chemotherapy
b. Squamous cell carcinoma c. Surgery
c. Small cell carcinoma d. Supportive treatment Ref: Harrison’s 18/e p745, 748
d. Neuroendocrine tumor of lung 142. Ramesh 40 yrs male patient presenting with polyuria, pain
Ref: Harrison’s 18/e p738, 17/e p554 abdomen, nausea, vomiting, altered sensorium was found to
135. A patient presents with secondaries to the adrenals. The have bronchogenic carcinoma. The electrolyte abnormality
most common site of primary is: (AI 2000) seen in him would be: (AIIMS May 02)
a. Lung a. Hypokalemia
b. Kidney b. Hyperkalemia
c. Breast c. Hypocalcaemia
d. Stomach d. Hypercalcemia Ref: Harrison’s 18/e p3100, 743
Ref: Harrison’s 18/e p739 143. A 60 year old male presented to the emergency with
136. Which of the following statements about lung carcinoma is breathlessness, facial swelling and dilated veins on the
true: (AI 2010) chest wall. The most common cause is: (AI 2003)
a. Squammous cell variant accounts for 70% of all lung a. Thymoma
cancers b. Lung cancer
b. Oat cell variant typically present with cavitation c. Hodgkin’s lymphoma
c. Oat cell variant is typically associated with hilar d. Superior vena caval obstruction
adenopathy Ref: Harrison’s 18/e p2266
d. Adenocarcinoma variant is typically central in Location 144. Which of the following tumor is most commonly associated
Ref: Harrison’s 18/e p738, 17/e p551, 552 with superior vena cava syndrome: (AI 2011)
137. A 60 years old chronic smoker presents with complaints of a. Lymphoma
hemoptysis. Her chest X-ray appears to be normal. What is b. Small cell carcinoma
the next best investigation: (AIIMS Nov. 2000) c. Non small cell carcinoma
a. Bronchoscopy d. Metastasis
b. High resolution CT Ref: Harrison’s 18/e p2266
c. Sputum cytology
d. Pulmonary function test
Ref: Harrison’s 18/e p286, 17/e p228 F. MISCELLANEOUS
138. A 60 yr old man is suspected of having bronchogenic ca;
145. A woman is admitted with complains of low-grade fever
TB has been ruled out in this pt. What should be the next
of 6 weeks duration. Chest radiograph reveals bihilar
investigation: (AI 2001)
adenopathy with clear lung fields. All of the following
a. CT guided FNAC
investigations will be useful in differential diagnosis
b. Bronchoscopy and biopsy
except: (AI 2004)
c. Sputum cytology
a. CD4/CD8 counts in the blood
d. X-Ray chest. Ref: Harrison’s 18/e p286, 17/e p554, 555
b. Serum ACE levels
139. A patient presents with a cavitatory lesion in right upper c. CECT of chest
lobe of lung. The best investigation is: (AI 2000) d. Gallium scan Ref: Harrison’s 18/e p2810
a. Branchoscopy, lavage and brushing
146. “Sleep apnoea”, is defined as a temporary pause in breathing
b. C.T. Scan
during sleep lasting at least: (AIIMS May 03)
c. X ray
a. 40 seconds
d. FNAC Ref: Harrison’s 18/e p2104
MEDICINE
b. 30 seconds
140. Serum ACE may be raised in all of the following except: c. 20 seconds
a. Sarcoidosis (AI 2005) d. 10 seconds
b. Silicosis Ref: Harrison’s 18/e p2186, 17/e p1665,1666
c. Berylliosis
147. Which of the following systems is least likely to be affected
d. Bronchogenic carcinoma
in cystic fibrosis: (DNB June 2010)
Ref: Harrison’s 18/e p2810, 17/e p2140
a. Respiratory
141. A 60 year old male was diagnosed as carcinoma right lung. b. Genitourinary
On CECT chest there was a tumor of 5 × 5 cm in upper lobe c. Hepatobiliary
and another 2 × 2 cm size tumor nodule in middle lobe. The d. Endocrine Ref: Harrison’s 18/e p2149
primary modality of treatment is: (AI 2004)
Ans. 134. a. Adenocarcinoma... 135. a. Lung 136. c. Oat cell ... 137. a. Bronchoscopy
138. b. Bronchoscopy 139. a. Bronchoscopy... 140. d. Bronchogenic... 141. c. Surgery...
142. d. Hypercalcemia...
143. d. Superior vena... 144. b. Small cell... 145. a. CD4/CD8 ...
146. d. 10 seconds...
147. d. Endocrine...
Respiratory System 1113
148. The most common cause of pulmonary infection in cystic showed ground glass appearance and culture negative.
fibrosis is: Apgar score 4 and 5 at 1 and 5 minutes. History of one
a. Pseudomonas aeroginosa month female sibling died before. What is the diagnosis?
b. Staphylococcus aureus a. TAPVC (AIIMS Nov 07)
c. Bukholderia cenocapacea b. Meconium aspiration
d. Nontuberculous mycobacteria c. Neonatal pulmonary alveolar proteinosis
Ref: Harrison’s 18/e p1017, 2148 d. Diffuse herpes simplex infection
149. Most common pathogen causing, pulmonary infection in Ref: Harrison’s 18/e p2168
cystic fibrosis in infants and young children is: 156. Which of the following is least likely cause of hemoptysis:
a. Staphylococcus aureus a. Pneumonia (PGI June 07)
b. Pseudomonas aeroginosa b. Empyema
c. Bukholderia cenocepacea c. Bronchiectasis
d. Nontubercular mycobacteria Ref: Harrison’s 18/e p2148 d. Mitral stenosis
150. Bronchial adenoma commonly present as: Ref: Harrison’s 18/e p285
a. Recurrent hemoptysis 157. Which of the following is not a feature of consolidation:
b. Cough a. Increased vocal resonance (DNB 2010)
c. Dysponea b. Dull Percussion note
d. Chest pain Ref: Harrison’s 18/e p753, 17/e p562 c. Bronchial Breath sounds
151. All of the following are true about Kartagener’s syndrome, d. Tracheal shift to side of consolidation
except: (PGI Dec 04) 158. A patient presents with sudden onset of breathlessness
a. Dextrocardia after a subclavian vein cannulation. On examination,
b. Infertility breath sounds are absent while the chest is hyperresonant
c. Mental retardation on percussion on one side. Most likely cause is: (AI 2012)
d. Bronchiectasis Ref: Harrison’s 18/e p2155 a. Iatrogenic pneumothorax
152. Kartagener’s syndrome includes all of the following, except: b. Subclavian vein air embolus
a. Situs inversus (PGI June 01) c. Malposition of cannula
b. Bronchiectasis d. Cardiac arrhythmia Ref: Harrison’s 18/e p3212
c. Sinusitis 159. The most common site of bleeding in Hemoptysis is:
d. Male infertility a. Tracheobronchial tree
e. Cystic fibrosis Ref: Harrison’s 18/e p2155, 17/e p1629 b. Pulmonary Parenchyma
153. Causes of pulmonary renal syndrome: (PGI June 07) c. Pleural Disease
a. Leptospirosis d. All of the above Ref: Harrison’s 18/e p285, 17/e p227
b. Hanta virus 160. The artery most frequently responsible for bleeding in
c. Paraquat poisoning massive hemoptysis is: (DNB 2009)
d. All of the above Ref: Robbin’s 8/e p710 a. Bronchial Artery
154. A male born at term after an uncomplicated pregnancy, b. Pulmonary Artery
labor and delivery develops severe respiratory distress c. Intersegmental Artery
within a few hours of birth. Results of routine culture d. Intercostal Artery Ref: Harrison’s 18/e p285, 17/e p227
were negative. The chest roentogram reveals a normal 161. All of the following statements about central cyanosis are
heart shadow and fine reticulonodular infiltrates radiating true except:
from the hilum. ECHO findings reveal no abnormality. a. Central cyanosis becomes evident when reduced
Family history reveals similar clinical course and death of haemoglobin < 5g/dl
MEDICINE
a male and female sibling at 1 month and 2 months of age b. Chronic Asthma may cause central cyanosis
respectively. The most likely diagnosis is: (AI 2008) c. Alveolar Hypoventilation causes central cyanosis
a. Neonatal Alveolar Proteinosis d. Methemoglobinemia causes central cyanosis
b. Total Anomalous Pulmonary Venous Circulation Ref: Harrison’s 18/e p288, 17/e p230
(TAPVC) 162. Cause of central cyanosis include all of the following,
c. Meconium Aspiration Syndrome except:
d. Diffuse Herpes simplex infection a. Chronic Asthma
Ref: Harrison’s 18/e p2168 b. Congenital Pulmonary stenosis
c. Congestive heart failure
155. 3.5 kg term male baby, born of uncomplicated pregnancy,
d. Alveolar hypoventilation
developed respiratory distress at birth, not responded to Ref: Harrison’s 18/e p288, 17/e p231
surfactant, ECHO finding revealed nothing abnormal, X-ray
Ans. 148. a. Pseudomonas ... 149. a. Staphylococcus... 150. c. Recurrent hemoptysis 151. c. Mental retardation
152. d. Cystic fibrosis
153. a. All 154. a. Neonatal ... 155. c. Neonatal...
156. b. Empyema
157. d. Tracheal... 158. a. Iatrogenic... 159. a. Tracheobronchial...
160. a. Bronchial Artery... 161. a. Central cyanosis ...
162. c. Congestive...
1114 Jaypee’s Triple A
163. Causes of unilateral clubbing include all of the following, 171. Bronchial hyperplasia may be caused by all except:
except: (PGI June 05) a. Smoking
a. Congenital cyanotic heart disease b. Theophylline
b. Panicoast tumor c. Prematurity
c. Aortic Aneurysm d. Allergy Ref: Robbin’s 8/e p687
d. Brachial AV fistulas Ref: Harrison’s 17/e p2982 172. Measurements of intravacular pressure by a pulmonary
164. Hypoxemia seen in: (PGI June 02) artery catheter should be done: (AI 2002)
a. Hypoventilation a. At end expiration
b. Decreased Fio2 b. At peak of inspiration
c. Myasthenia gravis c. During mid inspiration
d. Pulmonary emboli d. During mid expiration
e. All of the above Ref: Harrison’s 18/e p2199, 17/e p1591 173. The most common fetal response to acute hypoxia is:
165. Platypnea may be seen in all of the following conditions, a. Tachycardia (AI 2009)
except: (PGI June 05) b. Tachypnea
a. Bilateral Diaphragmatic palsy c. Bradycardia
b. Pleural effusion d. Ventricular Arrhythmia
c. Pulmonary embolism 174. Hypersensitivity Pneumonitis is classically described as a:
d. Cirrhosis a. Type I hypersensitivity reaction (AI 2009)
e. COPD Ref: Harrison’s 18/e p29 b. Tye II hypersensitivity reaction
166. A child develops acute respiratory distress, stridor, c. Type III (Immune complex) Hypersensitivity
hyperinflation on one side of chest with decreased breath d. Type IV (Cell mediated) Hypersensitivity
sound on that side. Most likely cause is: (AIIMS June 2000) Ref: Harrison’s 18/e p2116, 17/e p1607
a. Asthma 175. A child presented with severe respiratory distress two days
b. Aspiration pneumonia after birth. On examination he was observed to have a
c. Foreign body aspiration scaphoid abdomen and decreased breath sounds on the left
d. Pleural effusion side. He was managed by prompt Endotracheal intubation.
167. Static compliance is decreased in all of the following except: After ET tube placement the maximal cardiac impulse
a. ARDS (PGI 2000) shifted further to the right side. What should be the next
b. Pulmonary edema stepin management? (AI 2008)
c. Interstial fibrosis a. Confirm the position of endotracheal tube by chest X ray
d. Fibrosing alveolitis b. Remove tube and reattempt intubation
e. Emphysema c. Nasogastric tube insertion and decompress the bowel
168. Thickening of pulmonary alveolar capillary membrane is d. Chest X ray to confirm diagnosis
seen in: (PGI June 07) 176. A 7.5 months old child with cough, mild stridor is started on
a. Asthma oral antibiotics. The child showed initial improvement but
b. Bronchitis later developed wheeze, productive cough, and mild fever.
c. Pulmonary fibrosis X–ray shows hyperlucency and PFT shows an obstructive
d. Emphysema Ref: Harrison’s 18/e p2089 curve. The most probable diagnosis is: (AI 2008)
169. Smoking is generally not associated as a risk factor with: a. Bronchiolitis obliterans
a. Small cell carcinoma (AI 06) b. Post viral syndrome
b. Respiratory bronchiolitis c. Pulmonary alveolar microlithiasis
c. Emphysema d. Follicular bronchitis Ref: Harrison’s 18/e p1485
d. Bronchiolitis obliterans organizing pneumonia 177. Respiratory centre depression is caused by all of the
Ref: Harrison’s 18/e p1485 following, except: (AIIMS Nov 2010)
170. Which of the following statements is true about High a. Opium
Altitude Pulmonary Edema (HAPE)? (AIIMS Nov 06) b. Strychnine
a. Not exacerbated by exercise c. Barbiturates
MEDICINE
Ans. 163. a. Congenital ... 164. e. All... 165. a. Bilateral 166. c. Foreign...
167. e. Emphysema
168. c. Pulmonary 169. d. Bronchiolitis... 170. b. Associated...
171. b. Theophylline
172. a. At end expiration 173. c. Bradycardia... 174. c. and d.
175. b. Remove tube...
176. a. Bronchiolitis... 177. b. Strychnine...
5. KIDNEY AND URINARY TRACT
A. Urinalysis
B. Glomerular Diseases and Diabetic Glomerulopathy
C. Disorders of Renal Vasculature
D. Nephrolithiasis and Nephrocalcinosis
E. Renal Failure
F. Congenital Diseases of the Kidney
G. Renal Tumours
H. Miscellaneous
1116 Jaypee’s Triple A
MEDICINE
microalbuminuria is: (J & K 2012) 30. CRF is seen in which stage of GFR: (Rajasthan 2008)
a. 2.5 – 3.5 a. < 15
b. 3.5 – 15 b. < 30
c. 15 – 30 c. < 50
d. 30 – 45 d. < 75 Ref: Harrison’s 18/e p334-335, 2309
Ref: Harrison’s 18/e p2304, 2337, 2337 31. GFR was measured to be between 60-89 in a patients with
23. Criterion for prerenal failure: (WB PG 08) chronic renal failure. Which stage is he in?
a. Fractoal Excreation index > 1 a. Stage 1
b. Urine sodium concentration > 10 mmol/L b. Stage 2
c. Serum BUN : creatinine > 20:1 c. Stage 3
d. Urine osmolality < 500 d. Stage 4
Ref: CMDT 2008 p788 Ref: Harrison’s 18/e p2309-2310
Ans. 16. a. Suprapubic... 17. a. Quinine 18. a and c. 19. b. Creatinine Clearance
20. b. Creatinine... 21. c. Distal Tubular Function 22. d. 30 – 45 23. c. Serum BUN : creatinine...
24. c. Hematuria 25. b. Membranous GN 26. b. Membranous GN 27. b. Good pasture syndrome
28. a. Atherosclerosis 29. a. Liver 30. a. < 15 31. b. Stage 2
1118 Jaypee’s Triple A
32. All of the following are TRUE about nephritic syndrome, c. Decreased Metabolism of Vitamin K
except: (AP 2012) d. Increase in protein C
a. 85% experience minimal change in disease Ref: Harrison’s 18/e p346, 17/e p272
b. Elevated serum cholesterol 40. Nephrotic syndrome may be associated with: (PGI 2000)
c. Hypoalbuminemia is the cause of the hypoprotinemia a. Total cholesterol
d. Reduces sodium reabsorption by the kidney b. LDL cholesterol
Ref: Harrison’s 18/e p2437-2341 c. VLDL cholesterol
33. A 30-year old lady presents with malar rash, photosensitivity, d. Triglycerides
positive antinuclear antibody, and 24-hour proteinuria of 4 e. HDL cholesterol
gram and pedal oedema, the most likely renal pathology in Ref: Harrison’s 18/e p2345, 339, 17/e p173
this patient will be: (DP PGMEE 2009) 41. Most common cause of nephrotic range proteinuria in an
a. Minimal change disease adult is: (AI 2007)
b. Mesangioproliferative glomerulonephritis a. Diabetes Mellitus
c. Diffuse proliferative glomerulonephritis b. Amyloidosis
d. Membranous nephropathy c. Hypertensive nephropathy
Ref: Harrison’s 18/e p2339, 2345, d. Wegner’s Granulomatosis Ref: Harrison’s 18/e p339
17/e p1787, 1789, 1793
42. The Finnish type of congenital nephrotic syndrome occurs
34. Drugs associated with Acute Interstitial Nephritis is/are: due to gene mutations the following protein: (AI 2009, 08)
a. Penicillin (J & K 2011) a. Podocin
b. Allopurinol b. Alpha – actinin
c. NSAIDs c. Nephrin
d. All of the above d. CD2 activated protein
Ref: Harrison’s 18/e p2367 Ref: Harrison’s 18/e p2345, 17/e p1789
35. Following is the consequence of nephrotic syndrome: 43. A patient with nephrotic syndrome on longstanding
a. Hypoalbuminaemia (J & K 2011) corticosteroid therapy may develop all the following
b. Secondary hyperaldosteronism except: (AI 2002)
c. Hypercoagulibility a. Hyperglycemia
d. All of the above b. Hypertrophy of muscle
Ref: Harrison’s 18/e p2345 c. Neuropsychiatric symptoms
36. Typical features of Acute interstitial nephritis (AIN) include: d. Suppression of the pituitary adrenal axis
a. Skin rashes,arthralgia and fever (J & K 2012) Ref: Harrison’s 18/e p2345, 339
b. Peripheral blood eosinophillia 44. Cresentirc Glomerulonephritis may be seen in all of the
c. Renal impairment typically follows withdrawal of the following except? (AI 2008)
drug a. Post Streptococcal Glomerulonephritis (PSGN)
b. Renal biopsy evidence of an eosinophilic interstitial b. Henoch Schonlein Purpura (HSP)
nephritis c. Anti Basement Membrane Disease
Ref: Harrison’s 18/e p2367 d. Alport syndrome
37. All are true of Nephrotic syndrome, except: (AI 2000) Ref: Harrison’s 18/e p2338, 17/e p1785
a. RBC casts in urine 45. Crescent formation is characteristic of the following
b. Hypo-proteinemia glomerular disease: (AI-2002)
c. Oedema a. Minimal change disease
d. Hyperlipidemia b. Rapidly progressive glomerulonephritis
Ref: Harrison’s 18/e p17/e p c. Focal and segmental glomerulonephritis
MEDICINE
38. All of the following proteins are decreased in Nephrotic d. Rapidly non progressive glomerulonephritis
syndrome except: (PGI 2012) Ref: Harrison’s 18/e p2337, 17/e p1785
a. Transferrin 46. Which of the following conditions are assoiciated with
b. Fibrinogen pauci – immune crescenteric glomerulonephritis: (AI 2009)
c. Albumin a. Henoch – Schonlein Nephritis
d. Thyroxine Binding Globulin b. Lupus Nephritis (SLE)
Ref: Harrison’s 18/e p2345, 17/e p1790 c. Microscopic polyangitis
39. Hypercoagulation in Nephrotic syndrome is caused by: d. Nephritis in Alport’s syndrome
a. Loss of Antithrombin III (AI 2010) Ref: Harrison’s 18/e p2337
b. Decreased Fibrinogen
Ans. 32. d. Reduces sodium... 33. c. Diffuse proliferative... 34. d. All of above 35. d. All of above
36. d. Renal biops... 37. a. RBC casts in urine 38. b. Fibrinogen 39. a. Loss of Antithrombin III
40. e. ↓ HDL cholesterol 41. a. Diabetes Mellitus 42. c. Nephrin 43. b. Hypertrophy of muscle
44. d. Alport syndrome 45. b. Rapidly progressive... 46. c. Microscopic polyangitis
Kidney and Urinary Tract 1119
47. The prognosis of rapidly prolifeating glomerulonephritis 54. A 6 yr child presents with recurrent episodes of gross
(Crescentric GN) depends upon: (AIIMS Nov 01) hematuria for 2 yrs. He is likely to have: (AI 2008)
a. Number of crescents a. IgA nephropathy
b. Size of crescents b. Wilm’s tumour
c. Shape of crescents c. Henoch Schonlein Purpura (HSP)
d. Cellularity of crescents d. Neuroblastoma Ref: Harrison’s 18/e p2343
Ref: Harrison’s 18/e p2337 55. 12 years old Shyam presented with gross hematuria with
48. Non–proliferative Glomerulonephritis include all of the 80% dysmorphic RBC’s 2 days after a attack of upper
following, except: (AI 2008) respiratory tract infection. Diagnosis is: (AIIMS Nov 01)
a. Focal Segmental Glomerulonephritis (FSGS) a. Microangiopathic thrombotic anaemia
b. Mesangiocapillary Glomerulonephritis b. IgA Nephropathy
c. Membranous Glomerulonephritis c. PSGN
d. Amyloidosis d. H.S. purpura Ref: Harrison’s 18/e p2342
Ref: Harrison’s 18/e p2334-35, 17/e p1789
56. A six year old male baby presents to a hospital with
49. A 30-year-old man presents with generalized edema and recurrent gross hematuria for 2 years. There is no h/o
hypertension. Urine examination shows subnephrotic burning micturition or pyuria. Urine routine examination
proteinuria (< 2gm) and microscopic hematuria. Serum demonstrated no pus cells and urine culture was sterile.
complement levels are decreased and he is positive for anti- Serum C3 levels were normal. What is the most probable
hepatitis C antibodies. The most likely diagnosis is: diagnosis: (AIPGMEE-08)
a. Post streptococcal Glomerulonephritis (PSGN) (AI 2012) a. Wilm’s tumour
b. Cryoglobulinemia b. IgA nephropathy
c. Membranoproliferative Glomerulonephritis (MPGN C>B
c. Post –streptococcal glomerulonephritis
d. Focal Segmental Glomerular Sclerosis (FSGS)
d. Urinary tract infection
Ref: Harrison’s 18/e p2344
Ref: Harrison’s 18/e p2343
50. A 60-year-old woman presents with generalized edema,
57. A young man develops gross heamaturia 3 days after an
skin ulceration and hypertension. Urine examination
attack of URTI; likely renal pathology is: (AI 2001)
shows subnephrotic proteinuria (<2gm) and microscopic
a. Acute glomerulonephritis
haematuria. Serum complement levels are decreased and
b. Minimal change disease
she is positive for anti-hepatitis C antibodies. The likely
c. IgA nephropathy
diagnosis is: (AI 2012)
d. Membranous glomerulonephritis.
a. Post-streptococcal Glomerulonephritis
b. Essential Mixed Cryoglobulinemia Ref: Harrison’s 18/e p2342, 17/e p1786, 1788
c. Membranoproliferative Glomerulonephritis (MPGN) 58. True about Post-Streptococcal Glomerulonephritis is:
d. Focal SegmentalGlomerulosclerosis (FSGS) a. 50% of cases occur after pharyngitis (AI 2000)
Ref: Harrison’s 18/e p2344 b. Early treatment of Pharyngitis eliminates the risk of
51. Which of the following is not true about Berger’s disease? P.S.G.N.
a. The pathological changes are proliferative and usually c. Glomerulonephritis, secondary to skin infection, is more
confined to mesangial cells; usually focal and segmental common in summer
b. Hematuria may be gross or microscopic d. Recurrence is seen Ref: Harrison’s 18/e p2340
c. On immunofluorescence deposits contain both IgA and 59. Good pasture’s syndrome is characterized by all of the
IgG following, except: (AIIMS May 08)
d. Absence of associated proteinuria is Pathognomonic a. Glomerulonephritis
Ref: Harrison’s 18/e p2342 b. Leucocytoclastic Vasculitis
MEDICINE
52. IgA-nephropathy is seen in: (AIIMS June 2000) c. Diffuse alveolar haemorrhage
a. Membranous glomerulonephritis d. Presence of antibodies to basement membrane
b. Mesangioproliferative glomerulonephritis Ref: Harrison’s 18/e p2342, 2169
c. Focal glomerulonephritis 60. A 25 year old boy presents with renal failure. His uncle
d. Cresentic glomerulonephritis died of renal failure three years ago. Slit lamp examination
Ref: Harrison’s 18/e p2342-43 reveals Lenticonus / Keratoconus. The likely diagnosis is:
53. Increased IgA deposits are seen in: (AIIMS June 2000) (AIIMS Nov 2010)
a. Henoch Schonlein Purpura a. Autosomal dominant polycystic kidney disease (ADPCKD)
b. Minimal Change Glomemlonephritis b. Autosomal recessive polycystic kidney disease (ARPCKD)
c. Chronic Pyelonephritis c. Alport’s syndrome
d. Haemolytic Uremic Syndrome d. Denysh-Drash syndrome
Ref: Harrison’s 18/e p2343, 17/e p1788, 1789 Ref: Harrison’s 18/e p2351, 3213, 17/e p1794, 2469
Ans. 47. a. Number of crescents 48. b. Mesangiocapillary... 49. b and c 50. b. Essential Mixed...
51. d. Absence of... 52. b. Mesangioproliferative... 53. a. Henoch Schonlein... 54. a. IgA nephropathy
55. b. IgA Nephropathy 56. b. IgA nephropathy 57. c. IgA nephropathy 58. c. Glomerulonephritis...
59. b. Leucocytoclastic... 60. c. Alport’s syndrome
1120 Jaypee’s Triple A
61. Mutation in alpha 5 chain of collagen 4. The diagnosis: 67. Typical features of Lipoid nephrosis include: (PGI 2009)
a. Alport’s syndrome (AIIMS Nov 06) a. Normal light microscopy
b. Thin membrane disease b. FSGS
c. Nodular glomerulosclerosis c. Glomerular tuft sclerosis
d. Good pasture syndrome d. Effacement of foot processes
Ref: Harrison’s 18/e p3213, 17/e p2469 e. Tubular sclerosis Ref: Harrison’s 18/e p2345
62. A 7 year old boy presented with generalized edema. 68. Minimal change glomerulopathy may be seen in association
Urine examination revealed marked albuminuria. Serum with all of the following except: (AIIMS Nov 05)
biochemical examinations showed hypoalbuminaemia a. Hepatitis B
with hyperlipidemia. Kidney biopsy was undertaken. b. HIV
On light microscopic examination, the kidney appeared c. Drug - induced interstitial nephritis
normal. Electron microscopic examination is most likely to d. Hodgkin’s disease
reveal: (AIIMS Nov 03) Ref: Harrison’s 18/e p2345, 17/e p1790
a. Fusion of foot processes of the glomerular epithelial cells 69. Which of the following is the first clinically detectable sign
b. Rarefaction of glomerular basement membrane of diabetic nephropathy:
c. Deposition of electron dense material in the basement a. Serum creatinine
membrane b. Creatinine clearance
d. Thin basement membrane c. Microalbuminuria
Ref: Harrison’s 18/e p2345, 17/e p1790 d. Macroalbuminuria
63. A 7 year old girl is brought with complaints of generalized Ref: Harrison’s 18/e p2348, 2983, 17/e p2288
swelling of the body. Urinary examination reveals Grade 70. Preferred method for determining microalbuminuria is:
3 proteinuria and the presence of hyaline and fatty casts. a. Urinary dipsticks
She has no history of Hematuria. Which of the following b. 24 hour urinary protein collection
statements about her condition is true: (AI 2009) c. Urinary A/C Ratio in a spot voided sample
a. No IgG deposits or C3 deposition on Renal biopsy d. Urinary A/C Ratio in a 24 hour collection
b. Her C3 levels will be low Ref: Harrison’s 18/e p338, 2348
c. IgA Nephropathy is the likely diagnosis
71. The Recommended treatment of early diabetic nephropathy
d. Alport’s syndrome is the likely diagnosis
manifested by microalbuminuria is:
Ref: Harrison’s 18/e p2345
a. Strict glycemic control
64. True about light microscopy in minimal change disease is: b. Low protein Diet
a. Loss of foot process seen (AIIMS 2001) c. Strict glycemic control and Low Protein Diet
b. Anti GBM Abs seen d. Strict glycemic control, Low protein Diet and ACE
c. IgA deposits seen Inhibitors
d. No change seen Ref: Harrison’s 18/e p2348, 2983 17/e p2288
Ref: Harrison’s 18/e p2345
72. HIV associated nephropathy is a type of: (AIIMS Nov 04)
65. True about Minimal change disease is: a. Membranous glomerulonephritis
a. Appears normal under light microscopy but electron b. Immunotaetoid glomerulopathy
microscope shows loss of foot processes. c. Collapsing glomerulopathy
b. Mesangial deposits d. Fibrillary glomerulopathy Ref: Harrison’s 18/e p2353
c. Tram Track appearance
73. Collapsing glomerulopathy, features:
d. Gross haematuria
a. Tuft necrosis
Ref: Harrison’s 18/e p2345
b. Mesangiolysis
66. A child presents with hematuria and nephrotic syndrome. c. Proliferation of parietal epithelium cells
MEDICINE
A diagnosis of minimal change disease was made. Which of d. Hypertrophy of visceral epithelium cells
the following statements about the diagnosis is true: Ref: Harrison’s 18/e p2353
a. Glomerular function is lost due to loss of polyanions
74. Wire loop lesions are often characteristic for the following
around the foot processes
class of lupus nephritis: (AIIMS May 04)
b. Foot processes of podocytes in the Glomerular membrane
a. Mesangial proliferative glomerulonephritis (WHO class II)
are normal
b. Focal proliferative glomerulonephritis (WHO class III)
c. Glomerular function is lost due to deposition of IgA on
c. Diffuse proliferative glomerulonephritis (WHO class IV)
the glomerular membrane
d. Membranous glomerulonephritis (WHO class V)
d. Focal segmental changes are observed
Ref: Harrison’s 18/e p2727, 17/e p2076, 2077, 2078
Ref: Harrison’s 18/e p2345, 17/e p1790
Ans. 61. a. Alport’s syndrome 62. a. Fusion of foot processes... 63. a. No IgG deposits or C3... 64. d. No change seen
65. a. Appears normal... 66. a. Glomerular function... 67. a and d 68. a. Hepatitis B
69. c. Microalbuminuria 70. c. Urinary A/C Ratio... 71. c. Strict glycemic control 72. c. Collapsing glomerulopathy
73. d. Hypertrophy of... 74. c. Diffuse proliferative...
Kidney and Urinary Tract 1121
75. All of the following factors are associated with adverse b. Mesangio capillary glomerulonephritis
prognosis and high risk of Renal progression in Lupus c. Diarrhea-associated hemolytic uremic syndrome
Nephritis, except: (PGI Dec 03) d. Post-infections glomerulonephritis
a. High levels of Anti-ds DNA Ref: Harrison’s 18/e p2341, 2345
b. Persistant proteinuria (Nephrotic range > 3gm/day) 82. Hypocomplementemia is seen (select three options):
c. Hypocomplementenemia a. PSGN (PGI June 2008)
d. Anti LA (SSB) b. Membranous GN
Ref: Harrison’s 18/e p2727 c. Focal segmental DN
76. A patient who has been diagnosed with bronchiectasis 5 d. MPGN
years ago presents with edema on legs and proteinuria. T he e. Infective endocarditis
most likely finding in his kidney will be: (AI 2012) Ref: Harrison’s 18/e p2340, 2344, 1055
a. Minimal Change Disease 83. Non–proliferative Glomerulonephritis include all of the
b. Amyloid Nephropathy following, except: (AI 2008)
c. Rapidly Progressive Glomerulonephritis (RPGN) a. Focal Segmental glomerulonephritis (FSGS)
d. Crescenteric Glomerulonephritis b. Mesangiocapillary glomerulonephritis
Ref: Harrison’s 18/e p945 c. Membranous glomerulonephritis
77. Reflux Nephropathy with protenuria in the nephrotic range d. Amyloidosis
may be seen in patients with: (AIIMS Nov 06) Ref: Harrison’s 18/e p2345
a. Membranous glomerulonephritis 84. Proliferative glomerular deposits in kidney are found in:
b. Focal segmental Glomerulosclerosis a. Amyloidosis (AI-2000)
c. Nodular glomerulosclerosis b. Diabetes mellitus
d. Crescenteric glomerulonephritis c. IgA nephropathy
Ref: Harrison’s 18/e p2346 d. Membranous glomerulonephritis Ref: Robbin’s 8/e p931
78. A 30 year old man presents with generalized edema and 85. Which among the following is characteristic of a/c
hypertension. Urine examination shows subnephrotic glomerulonephritis: (Kerala PG 10)
proteinuria (< 2gm) and microscopic hematuria. Serum a. Muddy brown Cast
complement levels are decreased and he is positive for b. FeNa < 1%
antihepatitis C antibodies. The most likely diagnosis is: c. Haematuria Proteinuria
a. Post streptococcal glomerulonephritis (PSGN) (AI 2007) d. WBC
b. Mixed cryoglobulinemia Ref: Harrison’s 18/e p2300
c. Membranoproliferative glomerulonephritis (MPGN)
d. Focal segmental glomerular sclerosis (FSGS)
Ref: Harrison’s 18/e p2344 C. DISORDERS OF RENAL VASCULATURE
79. A 60 year old woman presents with generalized edema,
skin ulceration and hypertension. Urine examination 86. Which of the following type of renal tubular acidosis (RTA)
shows subnephrotic proteinuria (<2gm) and microscopic is associated with hyperkalemia? (Maharashtra 2011)
haematuria. Serum complement levels are decreased and a. RTA type IV
she is positive for antihepatitis C antibodies. The likely b. RTA type I & II
diagnosis is: (AI 2007) c. RTA type II
a. PSGN d. RTA type IV
b. Essential mixed cryoglobulinemia Ref: Harrison’s 18/e p2364
c. Membrano proliferative glomerulonephritis 87. Which of the following is s/o ARF: (Kerala PG 10)
a. FENa<1
MEDICINE
d. Focal segmental glomerulosclerosis
Ref: Harrison’s 18/e p2344 b. Renal Failure index <1
80. Serum C3 is persistently low in the following except: c. Blood urea nitrogen/cr ratio<20
a. Post streptococcal glomerulonephritis d. Urine osomolality >1.010
b. Membranoproliferative glomerulonephritis Ref: Harrison’s 17/e p1756 table 273-2
c. Lupus nephritis 88. In acute tubular necrosis all are true except? (Kerala PG 10)
d. Glomerulonephritis related to bacterial Endocarditis a. Specific gravity of urine<1.020
Ref: Harrison’s 18/e p2353, 2340 b. Urine Osmolatlity>500
81. All of the following are associated with low complement c. BU creatinine ratio<20
levels except: d. Urine sodium <20mmol/L
a. Lupus nephritis Ref: Harrison’s 17/e p1757 table 273.2
Ans. 75. d. Anti LA (SSB) 76. b. Amyloid Nephropathy 77. b. Focal segmental... 78. b and c
79. b and c 80. a. Post streptococcal... 81. c. Diarrhea-associated... 82. a, d and e
83. b. Mesangiocapillary... 84. c. IgA nephropathy 85. b. FeNa < 1% 86. d. RTA type IV
87. a and c. 88. d. Urine sodium <20mmol/L
1122 Jaypee’s Triple A
89. In acute tubular necrosis all are true except? (Kerala PG 10) 97. Renal artery stenosis may occur in all of the following
a. Specific gravity of urine<1.020 except: (AI 2006)
b. Urine Osmolatlity>500 a. Atherosclerosis
c. BU creatinine ratio<20 b. Fibromuscular dysplasia
d. Urine sodium <20mmol/L c. Takayasu’s arteritis
Ref: Harrison’s 17/e p1756; table 273.2 d. Polyarteritis nodosa Ref: Harrison’s 18/e p2375, 2796
90. Renal artery stenosis is caused by all except: (Kerala PG 09) 98. A 20 year old female presents with a blood pressure of
a. Atherosclerosis 160/110 mm Hg. Clinical examination reveals a bruit in
b. Fibromuscular dysplasia both flanks. Which of the following statements about this
c. Takayasu arteritis patient is not true (select one option) (PGI 09)
d. Buerger’s disease Ref: Harrison 17/e p1555 a. Enalapril may deteriorate renal function
91. An 8 yrs old boy presents to casualty with history of b. Most definitive diagnostic procedure is contrast enhanced
diarrhoea, followed by decreased urine output. Blood angiography
examination shows thrombocytes 90,000/cm3. Diagnosis is: c. Condition is nearly always bilateral
a. Hemolytic Uremic Syndrome (AI 2000) d. Surgical intervention may be used
b. Disseminated Intravascular Coagulation e. Fibromuscular dysplasia is the likely cause in this patient
c. Hemophilia Ref: Harrison’s 18/e p2375, 17/e p1811-1812
d. Idiopathic Thrombocytopenic Purpura 99. Renal vein thrombosis is most commonly associated with:
Ref: Harrison’s 18/e p970 a. Diabetic nephropathy (AI 2002)
92. Which of the following statements about Hemolytic Uremic b. Membranous glomerulopathy
Syndrome is least correct: (AI 2012) c. Minimal change disease
a. Usually follows Hemorrhagic colitis d. Mesangio-proliferative glomerulonephritis
b. Often self-limited Ref: Harrison’s 18/e p2344-45, 17/e p1790
c. Fever is usually mild or absent 100. Renal vein thrombosis (RVT) may be seen in: (PGI 2009)
d. Serotoxin has no role in pathogenesis a. Trauma
Ref: Harrison’s 18/e p970 b. Renal Cell Ca
93. Which of the following changes does not occur in malignant c. Pregnancy
hypertension: (AI 2008) d. Nephrotic Syndrome
a. Peticheal Haemorrhages on cortical surface e. Dehydration
b. Fibrinoid necrosis of arterioles Ref: Harrison’s 18/e p2382, 17/e p1815
c. Intimal concentric thickening 101. Renal vein thrombosis is most commonly associated with:
d. Hyaline arteriosclerosis a. Diabetic nephropathy (AI 2001)
Ref: Harrison’s 18/e p2047 b. Membranous glomerulopathy
94. Commonest histoloical finding in Benign Hypertension is: c. Minimal change disease
a. Proliferative endarteritis (AI 2009) d. Membrano-proliferative glomerulonephritis
b. Necrotizing arteriolitis Ref: Harrison’s 18/e p2382
c. Hyaline arteriosclerosis 102. All of the following are causes of Renal Vein Thrombosis,
d. Cystic medial necrosis Ref: Harrison’s 18/e p2046 except: (PGI Dec 01)
95. Which of the following is the most specific and sensitive a. Membranous Nephropathy
screening test for Renovascular Hypertension: b. Membranoproliferative glomerulonephritis
a. HRCT (AIIMS May 01) c. Lupus Nephritis
b. CT Angiography d. Renal Amyloidosis
c. Captopril enhanced radionucleatide scan e. Post streptococcal Glomerulonephritis (PSGN)
MEDICINE
Ans. 89. d. Urine sodium... 90. d. Buerger’s disease 91. a. Hemolytic Uremic... 92. d. Serotoxin has no role...
93. d. Hyaline... 94. c. Hyaline arteriosclerosis 95. b. CT Angiography 96. a. Magnetic Resonance...
97. d. Polyarteritis nodosa 98. c. Condition is nearly... 99. b. Membranous... 100. All of the above
101. b. Membranous...
102. e. Post streptococcal... 103. d. Renal vein thrombosis
Kidney and Urinary Tract 1123
104. Renal artery stenosis may occur in all of the following
except: (AI 2006) D. NEPHROLITHIASIS AND
a. Atherosclerosis NEPHROCALCINOSIS
b. Fibromuscular dysplasia
c. Takayasu’s arteritis 111. Stone which is resistant to lithotripsy: (AIIMS May 07)
d. Polyarteritis nodosa Ref: Harrison’s 18/e p2375, 2796 a. Calcium oxalate
105. A 20 year old female presents with a blood pressure of b. Triple phosphate stone
160/110 mm Hg. Clinical examination reveals a bruit in both c. Cystine stone
flanks. Which of the following statements about this patient d. Uric acid stone
is not true (select one option): (PGI 09) Ref: Harrison’s 18/e p2382, 2386; Bailey 26/e p1296
a. Enalapril may deteriorate renal function 112. Which of the following stones is hard to break by ESWL:
b. Most definitive diagnostic procedure is contrast enhanced a. Calcium Oxalate Monohydrate (AI 2010)
angiography b. Calcium Oxalate Dihydrate
c. Condition is nearly always bilateral c. Uric acid
d. Surgical intervention may be used d. Struvite
e. Fibromuscular dysplasia is the likely cause in this patient Ref: Harrison’s 18/e p2382, 2385; Bailey 26/e p1296
Ref: Harrison’s 18/e p2375, 17/e p1811-1812 113. All of the follwing types of Renal Stones are Radiopaque,
106. Renal vein thrombosis is most commonly associated with: except: (AIIMS June 2000)
a. Diabetic nephropathy (AI 2002) a. Oxalate
b. Membranous glomerulopathy b. Uric Acid
c. Minimal change disease c. Cystine
d. Mesangio-proliferative glomerulonephritis d. Mixed Ref: Harrison’s 18/e p2382, 2386; Bailey 26/e p1293
Ref: Harrison’s 18/e p2344-45, 17/e p1790 114. Renal Calculi associated with proteus infection: (AI 2009)
107. Renal vein thrombosis (RVT) may be seen in: (PGI 2009) a. Uric Acid
a. Trauma b. Triple Phosphate
b. Renal Cell Ca c. Calcium oxalate
c. Pregnancy d. Xanthine Ref: Harrison’s 18/e p2387; Bailey 26/e p1293
d. Nephrotic Syndrome 115. Ureteric colic due to stone is caused by: (AI 2008)
e. Dehydration a. Stretching of renal capsule due to back pressure
Ref: Harrison’s 18/e p2382, 17/e p1815 b. Increased peristalsis of ureter to overcome the obstruction
108. Renal vein thrombosis is most commonly associated with: c. Irriation of intramural ureter
a. Diabetic nephropathy (AI 2001) d. Extravasation of urine.
b. Membranous glomerulopathy Ref: Harrison’s 18/e p2382; Bailey 26/e p1271
c. Minimal change disease 116. Locate the renal stone with pain radiating to medial side of
d. Membrano-proliferative glomerulonephritis thigh and perineum due to slipping of stone in males:
Ref: Harrison’s 18/e p2382 a. At pelvic brim (AIIMS June 2000)
109. All of the following are causes of Renal Vein Thrombosis, b. Intramural opening of ureter
except: (PGI Dec 01) c. Junction of ureter and renal pelvis
a. Membranous Nephropathy d. At crossing of gonadal vessels and ureter
b. Membranoproliferative glomerulonephritis Ref: Harrison’s 18/e p2382, 2386; Bailey 26/e p1292
c. Lupus Nephritis 117. Referred pain from ureteric colic is felt in the groin due to
d. Renal Amyloidosis involvement of the following nerve: (AI 2003)
MEDICINE
e. Post streptococcal Glomerulonephritis (PSGN) a. Subcostal
Ref: Harrison’s 18/e p2382 b. Iliohypogastric
110 A 10 year old child develops hematuria after 2 days c. Ilioinguinal
of diarhoea. Blood film shows fragmented RBCs & d. Genitofemoral
thrombocytopenia. Ultrasound shows marked enlargement Ref: Harrison’s 18/e p2385; Bailey 26/e p1272
of both kidneys. The likely diagnosis is: (AIIMS June 09) 118. A 10-mm calculus in the right lower ureter associated with
a. Acute pyelonephritis proximal hydroureteronephrosis is best treated with:
b. Disseminated intravascular coagulopathy a. Extracorporeal shockwave lithotripsy (AI 2003)
c. Haemolytic uremic syndrome b. Antegrade percutaneous access
d. Renal vein thrombosis c. Open ureterolithotomy
Ref: Harrison’s 18/e p2382 d. Ureteroscopic retrieval
Ref: Bailey 26/e p1294
Ans. 104. d. Polyarteritis nodosa 105. c. Condition is nearly... 106. b. Membranous... 107. All of the above
108. b. Membranous...
109. e. Post streptococcal... 110. d. Renal vein thrombosis 111. c. Cystine stone
112. a. Calcium Oxalate... 113. b. Uric Acid
114. b. Triple Phosphate 115. b. Increased peristalsis...
116. a. At pelvic brim
117. d. Genitofemoral 118. d. Ureteroscopic retrieval
1124 Jaypee’s Triple A
119. A child presents with abdominal colic and hematuria. On 126. The following is NOT a feature of Renal tubular acidosis:
ultrasonography a stone 2.0 cm in diameter is seen in the a. High anion gap (Karnataka 2011)
renal pelvis. The next step in management of this case is: b. Hyperchioremic acidosis
a. Pyelolithotomy (AIIMS Nov 2000) c. No gastrointestinal disturbance
b. Nephroureterostomy d. Urine pH is inappropriately high (greater than 5.5)
c. Conservative Ref: Davidson’s 21/e p444
d. ESWL 127. Which of the following is not true about intrinsic renal
Ref: Harrison’s 18/e p2382; Bailey 26/e p1296 failure in case of ischemic ATN? (MHPGM-CET 2010)
120. Chandu, a 45 yrs male shows calcification on the Rt side of a. Specific gravity < 1.015
his abdomen in an AP view. In lateral view the calcification b. FENa <1
is seen to overlie the spine. Most likely diagnosis is: c. Urine sodium >20 mmol/L
a. Gallstones (AI 2001) d. Urine creatinine to plasma creatinine ratio <20
b. Calcified mesenteric nodes Ref: Harrison’s 17/e p271; Table 45-2
c. Renal stones 128. Acute and chronic renal failure can be differentiated by?
d. Calcified rib a. Anemia in CRF (Kerala PG 10)
Ref: Harrison’s 18/e p2382; Bailey 26/e p1293 b. Hyperphosphatemia in CRF
121. Nephrocalcinosis is seen in all except: (AIIMS May 07) c. Peripheral neuropathy in CRF
a. Sarcoidosis Ref: Harrison’s 17/e p1755
b. Distal RTA 129. Important clues of diagnosis chronic renal failure (CRF)
c. Milk alkali syndrome except: (UP 2009)
d. Medullary cystic kidney a. Bilateral contracted kidney
Ref: Harrison’s 18/e p2382, 2382, 17/e p1815, 1816 b. Massive proteinuria
122. Nephrocalcinosis is a feature of all except: (PGI June 2000) c. Osteodystrophy
a. Primary hyperparathyroidism d. Hypertension Ref: Harison’s 18/e p335 17/e p269
b. Medullary sponge kidney 130. Target Hb in CRF patient is: (Rajasthan 2009)
c. Vitamin D intoxication a. 11-12 gm
d. Pseudo hypoparathyroidism b. 10-11 gm
Ref: Harrison’s 18/e p360, 3096 c. 12-13 gm
123. Nephrocalcinosis is seen in (select two options): d. 13-14 gm
a. Medullary sponge disease (PGI Dec 05) 131. Most common cause of post renal acute kidney injruy is:
b. Acute pyelonephritis a. Bladder neck obstruction (AP 2011)
c. Acute glomerulonephritis b. Urinary tract obstruction
d. Chronic pyelonephritis c. Ureteric obstruction
e. Hyper parathyroidism d. Urethritis Ref: Harrison’s 18/e p2299
Ref: Harrison’s 18/e p360, 3096
132. The commonest cause of chronic renal failure is:
124. A patient is known to have calcium nephrocalcinosis for the a. Diabetes mellitus (Comed K 2011)
post 10 years. All of the following dietary recommendations b. Hypertension.
should be suggested, except: (AIIMS Nov 2010) c. Pyelonephritis.
a. Protein Restriction d. Cystic disease of kidneys
b. Calcium Restriction Ref: Davidson’s 20/e p486, Box 17.29; Harrison’s 18/e p2310, 17/e p1762
c. Salt Restriction
133. Causes of Acute Tubular necrosis include: (PGI June 07)
d. All of the above
a. Radiocontrast agents
Ref: Harrison’s 18/e p2382
MEDICINE
b. Psrsproteins
c. Amphoterecin B
E. RENAL FAILURE d. Abruptio placentae
e. All of the above Ref: Harrison’s 18/e p2294
125. Common cause of chronic renal failure is: (Karnataka 2010) 134. Which of the following values are suggestive of acute
a. Hypotension tubular necrosis: (AIIMS Nov 2000)
b. Hypertension a. Urine osmolality>500
c. Diabetes insipidus b. Urine sodium>40
d. Malaria c. Blood urea nitrogen/plasma creatinine>20
Ref: Davidson’s 20/e p486, Box 17.29; Harrison’s 18/e p2310 17/e p1762 d. Urine creatinine /plasma creatinine>40
Ref: Harrison’s 18/e p337, 2301, 17/e p271, 1758
Ans. 119. d. ESWL 120. c. Renal stones 121. d. Medullary cystic kidney 122. d. Pseudo hypoparathyroidism
123. a and e.
124. b. Calcium Restriction 125. b. Hypertension 126. a. High anion gap
127. c. Urine sodium...
128. a. Anemia in CRF 129. d. Hypertension 130. a. 11-12 gm
131. a. Bladder neck...
132. a. Diabetes mellitus 133. e. All of the above 134. b. Urine sodium>40
Kidney and Urinary Tract 1125
135. Fractional excretion of sodium <1 is seen in: 143. Non Oliguric Acute Renal Failure is typically associated
a. Prerenal azotemia (AIIMS Nov 07) with: (DNB Dec 2011)
b. Acute tubular necrosis a. Aminoglycoside toxicity
c. Acute ureteral obstruction b. Contrast Induced Nephrotoxity
d. Interstitial nephritis c. Hemolytic Uremic syndrome
Ref: Harrison’s 18/e p337, 2302, 17/e p1758 d. Glomerulonephritis
136. Pre-renal azotemia is associated with one of the following Ref: Harrison’s 18/e p2303-2306, 17/e p1754, 1759
characteristic feature: (AIIMS Nov 2000) 144. Hypophosphatemia is seen in all except: (AI 2007)
a. Urinary Na+ < 10 mmol/L a. Acute renal failure
b. Renal failure index > 1 b. Resolving phases of diabetic ketocidosis
c. Osmolality < 500 c. Respiratory alkalosis/COPD
d. Urinary creatinine / P. creatinine ratio < 20 d. Chronic alcoholism
Ref: Harrison’s 18/e p337, 2302, 17/e p271, 1758 e. Chronic Renal Failure
137. Pre-Renal Azotemia is characterized by all of the following Ref: Harrison’s 18/e p2303, 17/e p1759
except: (AI 2010) 145. Feature of CRF include all, except: (PGI June 05)
a. Fractional Excretion of Na < 1% a. Impotence
b. Urinary osmolality > 500 mosm/kg b. Restless legs
c. Urinary sodium concentration > 40 meq/l c. Isothenuria
d. Reversible with replacement fluids d. Broad cast in urine
Ref: Harrison’s 18/e p337, 2302, 17/e p271 e. All of the above
138. Plasma urea / creatinine ratio of 20:1 may be seen in: Ref: Harrison’s 18/e p2310
a. Rhabdomyolysis (AI 2010)
146. Decrease in GFR is apparent in which of the following
b. Ureteric calculi
stages of Chronic Kidney Disease:
c. Pre-renal failure
a. Stage I
d. Chronic glomerulonephritis
b. Stage II
Ref: Harrison’s 18/e p337, 2302, 17/e p271, 1758, 1758
c. Stage III
139. Which of the following is true about Acute tubular d. Stage IV Ref: Harrison’s 18/e p2319
necrosis: (PGI June 07)
147. Signs and symptoms of a Renal Failure are evident when
a. Urine specific gravity > 1.020
Renal function deteriorates by more than:
b. Urine osmolality >350 mosmol/kg
a. 40%
c. Urine Na < 10meq/L
d. Blood urea : creatinine ratio < 20) b. 50%
Ref: Harrison’s 18/e p337, 17/e p271 c. 60%
d. 80%
140. All of the following are true about Prerenal:
Ref: Harrison’s 18/e p335, 2317
Azotemia, Except: (PGI June 05)
a. Urinary Cr/Plasma Cr >40 148. Overt symptoms of Renal failure become evident when
b. FENa < 1 Renal Function Deteriorates by more than:
c. Urinary output < 400ml/day a. 40-50%
d. Plasma BUN/Creatinine ratio <20 b. 50-60%
e. Urinary sodium concentration < 20meq/l c. 70-80%
Ref: Harrison’s 18/e p337, 17/e p271 d. >90% Ref: Harrison’s 18/e p2289
141. Which differentiating prerenal azotemia with ATN features 149. Restless leg syndrome (RLS) is seen in: (AI 2009)
favouring pre-renal azotemia (select two options): a. Hypercalcemia
MEDICINE
a. Urine osmolality > 500 mosmol/kg (PGI Dec 2002) b. Hyperphosphatemia
b. Sodium <10 mmol/l c. Chronic renal failure
c. Plasma transferrin/Ig ratio d. Hyperkalemia
d. Fractional excretion of sodium >1 Ref: Harrison’s 18/e p218, 2317, 17/e p176, 1768
e. Plasma BUN/creatinine ratio < 20 150. Central nervous system manifestation in chronic renal
Ref: Harrison’s 18/e p337, 17/e p271 failure are result of all of the following, except: (AI 2003)
142. All of the following are true about Oliguric ARF: (AI 03) a. Hyperosmolarity
a. Anemia b. Hypocalcemia
b. Metabolic Acidosis c. Acidosis
c. Uremia d. Hyponatremia
d. Hypercalcemia Ref: Harrison’s 18/e p2317, 17/e p2317
Ref: Harrison’s 18/e p2303, 2306, 17/e p1754, 1759, 1760
Ans. 135. a. Prerenal azotemia 136. a. Urinary Na+ < 10 mmol/L 137. c. Urinary sodium... 138. c. Pre-renal failure
139. d. Blood urea...
140. d. Plasma BUN/Creatinine... 141. a and b 142. d. Hypercalcemia
143. a. Aminoglycoside... 144. a. Acute renal failure
145. e. All of the above 146. b. Stage II
147. c. 60%
148. c. 70-80% 149. c. Chronic renal failure 150. b. Hypocalcemia
1126 Jaypee’s Triple A
151. Chronic Renal Diseases / failure is commonly associated c. Intravenous infusion of glucose with insulin
with: d. Beta blockers
a. Metabolic acidosis Ref: Harrison’s 18/e p358, 359
b. Metabolic acidosis
c. Respiratory acidosis
d. Respiratory acidosis F. CONGENITAL DISEASES OF THE KIDNEY
Ref: Harrison’s 18/e p2312, 17/e p1764
159. In proximal renal tubular acidosis, most important feature
152. Which of the following metabolic complications is not seen
is: (DP PGMEE 2010)
in Chronic Renal Failure: (DNB Dec 2010)
a. Vitamin D resistant rickets
a. Hyperkalemia
b. Dehydration and fever
b. Hyponatremia
c. Nephrocalcinosis
c. Hypercalcemia
d. Bicarbonate loss
d. Hyperphosphatemia
Ref: Harrison’s 18/e p2364, 2385, 17/e p1805
Ref: Harrison’s 18/e p23213, 2317, 17/e p1763 Table 274.31
160. True about adult polycystic kidney disease is all except:
153. Renal osteodystropy differs from nutritional and genetic
a. Autosomal dominant inheritance (AIIMS 2001)
forms of osteomalacia in having: (AI 2002)
b. Hypertension is rare
a. Hypocalcemia
c. Can be associated with cysts in liver, lungs and pancreas
b. Hypercalcemia
d. Pyelonephritis is common
c. Hypophosphatemia
Ref: Harrison’s 18/e p2355, 2356
d. Hyperphosphatemia
Ref: Harrison’s 18/e p336, 3109 161. Which of the following is associated with adult polycystic
kidney disease? (AIIMS 2001)
154. Nail and half nail sign, seen in uremia is: (AIIMS May 07)
a. Berry Aneurysm in Circle of Willis
a. Due to melanin deposition
b. Saccular aneurysms of aorta
b. Increased capillary density at the distal half of nails
c. Fusiform aneurysms of aorta
c. Hypoproteinemia
d. Leutic aneurysms
d. Circulating toxin
Ref: Harrison’s 18/e p2356, 17/e p1798
Ref: Harrison’s 18/e p2317
162. Polycystic disease of the kidney may have cysts in all of the
155. All improves after dialysis except: (AIIMS Nov 08)
following organs except: (AI 2004)
a. Pericarditis
a. Lung
b. Peripheral neuropathy
b. Liver
c. Metabolic acidosis
c. Pancreas
d. Seizure
d. Spleen Ref: Harrison’s 18/e p2356, 17/e p1798
Ref: Harrison’s 18/e p2314
163. Which of the following is the common extrarenal
156. A girl aged 8 years has been admitted for dialysis. She has
involvement in autosomal dominant polycystic kidney
serum K of 7.5 meq/l, which is the fastest way to reduce the
disease: (AIIMS Nov 04)
hyperkalemia? (AI 2009)
a. Mitral valve prolapse
a. Kayexalate enema
b. Hepatic cysts
b. Infusion of insulin + glucose
c. Splenic cysts
c. IV calcium gluconate
d. Colonic diverticulosis Ref: Harrison’s 18/e p2355-56
d. IV NaHCO3
Ref: Harrison’s 18/e p359 164. Which one of the following statements is wrong regarding
adult polycystic kidney disease: (AIIMS May 04)
157. A patient in Chronic renal failure presents with tallpeaked T
a. Kidneys are enlarged in size
waves on ECG. Management includes: (select two options):
MEDICINE
Ans. 151. a. Metabolic Acidosis 152. c. Hypercalcemia 153. d. Hyperphosphatemia 154. b. Increased capillary density...
155. b and d
156. b. Infusion of insulin... 157. c. CaCl2, d. Sodium... 158. d. Beta blockers
159. d. Bicarbonate loss
160. b. Hypertension is rare 161. a. Berry Aneurysm in... 162. a. Lung
163. b. Hepatic cysts
164. b. The presentation is... 165. a. Autosomal Dominant
Kidney and Urinary Tract 1127
166. Which of the following is the most common renal cystic
disease in infants is? (AI 2005) G. RENAL TUMOURS
a. Polycystic kidney
172. Papillary necrosis is seen in all except: (MP PG 2010)
b. Simple renal cyst
a. Diabetic nephropathy
c. Unilateral renal dysplasia
b. Sickle cell anemia
d. Calyceal cyst Ref: OP Ghai 7/e p470
c. Renal cell carcinoma
167. A 12- year –old boy is referred for evaluation of nocturnal d. Analgesic abuse nephropathy
enuresis and short stature. The blood pressure is normal. Ref: Harrison’s 17/e p1825; Robins 8/e p946
The blood urea is 112 mg/dl, creatinine 6 mg/dl, sodium
173. The most common histological variant of renal cell
119 mEq/1, potassium 4 mEq/1, calcium 7mg/dl, phosphate
carcinoma is: (AIIMS Nov 2005)
6mg/dl and alkaline phosphatase 400 U/l.Urinalysis shows
a. Clear cell type
trace proteinuria with hyaline casts; no red and white cells
b. Chromophobe type
are seen. Ultrasound shows bilateral small kidneys and the
c. Papillary type
micturating cystourethrogram is normal. The most likely
d. Tubular type Ref: Harrison’s 18/e p793
diagnosis is: (AIIMS Nov 03)
a. Alport’s syndrome 174. Chromophobe variant of Renal cell carcinoma is associated
b. Medullary sponge kidney with: (AI 2010)
c. Chronic glomerulonephritis a. VHL gene mutations
d. Nephronophthisis b. Trisomy of 7 and 17 (+7, +17)
Ref: Harrison’s 18/e p2359 c. 3 p deletions (3p-)
d. Monosomy of 1 and Y (-1, -Y)
168. An 8 year old child suffering from recurrent attacks of
Ref: Harrison’s 18/e p793
polyuria since childhood presents to the paediatrics OPD.
On examination the child is short statured. Vitals and B.P. 175. A 40 year old man presented with painless hematuria
are normal. Serum Creatinine – 6 mg%, HCO3 – 16 meg, Bimanual examination revealed a ballotable mass over the
Na-134, K+ 4.2. On USG bilateral small kidneys are seen. right flank. Subsequently right nephrectomy was done and
Diagnosis is: (AIIMS May 01) mass was seen to be composed of cells with clear cytoplasm.
a. Reflux Nephropathy Areas of hemorrhage and necrosis were frequent. Cytogenic
b. Nephronophthisis analysis of this mass is likely to reveal an abnormality of:
c. Polycystic kidney disease a. Chromosome 1 (AI 2004)
d. Medullary cystic kidney disease b. Chromosome 3
Ref: Harrison’s 18/e p2359, 17/e p1799 c. Chromosome 11
d. Chromosome 17 Ref: Harrison’s 18/e p793
169. Medullary cystic disease of the kidney is best diagnosed
by: (AI 2002) 176. Paraneoplastic syndrome associated with RCC are all of the
a. Ultrasound following except: (AI 2009)
b. Nuclear scan a. Polycythemia
c. Urography b. Hypercalcemia
d. Biopsy c. Malignant hypertension
Ref: Robbin’s 8/e p960 d. Cushing syndrome Ref: Harrison’s 18/e p793
170. The most common cause of renal scarring in a 3 year old 177. A pt. presented with renal cell carcinoma invadving IVC
child is: (AI 2005) and the renal vein. False statement is:
a. Trauma. a. Preoperative biopsy is not necessary
b. Tuberculosis. b. IVC involvement indicates inoperability
c. Preoperative radiotherapy is not essential
MEDICINE
c. Vesicoureteral reflux induced pyelonephritis.
d. Interstitial nephritis. d. Chest X-ray should be done to rule out pulmonary
Ref: Ghai 8/e p506 metastasis
Ref: Harrison’s 18/e p793, 794
171. The neonatal kidney achieves concentrating ability
equivalent to adult’s kidney by: (AI 2004) 178. All are associated with Wilm’s tumor except one:
a. One year of age (AIIMS Feb 1997)
b. Eighteen months of age a. Anirida
c. Three to six months of age b. Male pseudohermaphrodite
d. Just before puberty c. Arthogryposis multiplex congenital
Ref: Ghai 8/e p464 d. Hemihypertrophy
Ref: Ghai 8/e p617
Ans. 166. c. Unilateral renal... 167. d. Nephronophthisis 168. b. Nephronophthisis 169. d. Biopsy
170. c. Vesicoureteral...
171. a. One year of age 172. c. Renal cell carcinoma 173. a. Clear cell type
174. d. Monosomy of...
175. b. Chromosome 3 176. d. Cushing syndrome 177. b. IVC involvement indicates...
178. c. Arthogryposis...
1128 Jaypee’s Triple A
179. The most important determinant of prognosis in Wilms 187. Not seen in Hemolytic-uremic syndrome: (AP 2011)
tumor: (AI 2006) a. Subnephrotic range proteinuria
a. Stage of disease b. Hypofibrogenimia
b. Loss of heterozygosity of chromsome 1p c. Thrombocytopenia
c. Histology d. Positive coombs test Ref: Harrison’s 18/e p970, 310
d. Age less than one year at presentation 188. Multiple cavitatory lesion in lungs, hematuria and renal
Ref: Ghai 8/e p617 insufficiency are features in a patient with:
180. Which of the following is the Post – Chemotherapy based a. Polyarteritis nodosa (DP PGMEE 2009)
staging system in Wilm’s tumor: (AI 2009) b. Churg strauss syndrome
a. National Wilm’s Tumor staging System (NWTSG) c. Wegener’s granulomatosis
b. International Society of Pediatric Oncology (SIOP) d. Temporal arteritis
c. AJCC TNM Ref: Harrison’s 18/e p2789, 17/e p2121
d. Chadwick 189. Vesicouretral reflux is diagnosed by: (J & K 2010)
Ref: Ghai 8/e p617 a. Abdomen ultra sound
b. Radionuclide scan
c. Intravenous pyelography
H. MISCELLANEOUS d. Voiding cystourethrography
181. Renal osteodystrophy differs from nutritional osteomalacia Ref: Harrison’s 18/e p2371-2372
by having ______: (MHPGM-CET 2010) 190. Within the normal kidney: (J & K 2012)
a. Increased phosphates a. 33% of the filtered sodium load is reabsorbed in the
b. Increased calcium proximal tubules
c. Decreased calcium b. Antidiuretic hormone(ADH) increase the water
d. None of the above Ref: Robbin’s 8/e p1219, 436 permeability of the distal tubules
182. HIV associated nephropathy is severe from of: c. 50% of the filtered sodium is reabsorbed in the loop of
a. Focal segmental glomerulosclerosis (Rajasthan 2009) Henle
b. Membranous nephropathy d. Absorption of sodium in the proximal convoluted tubule
c. Membrano –proliferative glomerulonephritis is mediated by aldosterone Ref: CMDT 2011
d. Focal segmental glomerulonephritis 191. Which of the statements about Renal Tubular Acidosis type
Ref: Harrison’s 18/e p1553, 3353 I (Type I RTA) is not true: (AI 2012)
183. Renal transplant is indicated in: (Rajasthan 2009) a. Failure to acidify urine to a pH < 5
a. Advanced chronic renal failure b. Associated with increased risk of urinary stones
b. Polycystic kidney diseases c. Associated with hyperkalemia
c. Pyelonephritis d. Treatment involves alkali replacement as bicarbonate
d. Nephrolithiasis Ref: Harrison’s 18/e p2310 Ref: Harrison’s 18/e p2364
184. Necrotizing papillitis is seen in: (Rajasthan 2008) 192. Positive Urinary Anion Gap helps to establish the diagnosis
a. Diabetes of: (AI 2009)
b. Hypothyroidism a. Alcoholic ketoacidosis
c. SLE b. Diabetic ketoacidosis
d. Nephrotic syndrome Ref: Robbin’s 8/e p934 c. Renal tubular Acidosis
185. Tumor lysis syndrome is associated with all of the following d. Acidosis in Diarrhea
laboratory features, except: (AP 2010) Ref: Harrison’s 18/e p368, 2364 17/e p292
a. Hyperkalemia 193. Interstitial nephritis is seen with all except:
b. Hypercalcemia a. Beta lactam inhibitors (AIIMS May 07)
MEDICINE
c. Hyperuricemia b. INH
d. Hyperphosphatemia c. Diuretics
Ref: Harrison’s 18/e p356, 362, 2274 d. Allopurinol
Ref: Harrison’s 18/e p2367
186. Renal Osteodystrophy can be differentiated from
nutritional & genetic forms of vitamin-D deficiency effect 194. Necrotizing pappillitis may be seen in all of the following
by having: (AP 2010) conditions except: (AI 2002)
a. Hypophosphatemia a. Sickle cell disease
b. Hyperphosphatemia b. Tuberculous pyelonephritis
c. Hypocalcemia c. Diabetes mellitus
d. Hypercalcemia d. Analgesic nephropathy
Ref: Harrison’s 18/e p336, 3109 Ref: Harrison’s 18/e p2367, 17/e p1807, 1825, 1826
Ans. 179. c. Histology 180. b. International Society... 181. a. Increased phosphates 182. a. Focal segmental...
183. a. Advanced chronic... 184. a. Diabetes
185. b. Hypercalcemia 186. b. Hyperphosphatemia
187. d. Positive coombs... 188. c. Wegener’s granulomatosis 189. d. Voiding...
190. b. Antidiuretic hormone...
191. c. Associated with...
192. c. Renal tubular Acidosis 193. b. INH 194. b. Tuberculous pyelonephritis
Kidney and Urinary Tract 1129
195. Necrotizing pappillitis is seen in: (AIIMS 2002) 203. All of the following are causes of Rhabdomyolysis and
a. Salicylate poisoning myoglobinuria, except: (PGI Dec 06)
b. Glomerulonephritis a. Hyperpyrexia
c. PNH b. Viper snake venom
d. Diabetes insipidus c. Hyperthyroidism
Ref: Harrison’s 18/e p2372 d. Multiple Hornet Stings
196. Necrotizing pappillitis is seen in all of the following e. All of the above Ref: Harrison’s 18/e p2298
except: (AIIMS May 02) 204. All of the following are true about Rhabdomyolysis,
a. Salicylate poisoning except: (PGI Dec 04)
b. Renal vascular thrombosis a. Hyperuricemia
c. PNH b. Hyperphosphatemia
d. Diabetes mellitus c. Hypercalcemia
Ref: Harrison’s 18/e p2372 d. Creatine kinase
197. Which of the following is associated with Renal Papillary Ref: Harrison’s 18/e p2300
Necrosis: 205. Features of Rhabdomyolysis include all of the following,
a. Alcohol except: (PGI Dec 04)
b. Heroine a. Acute muscular weakness
c. Morphine b. Myoglobinuria
d. Tramadol Ref: Robbin’s 8/e p945 c. Hemoglobinuria
198. Features of Hepatorenal syndrome are (select two options): d. Acute renal failure Ref: Harrison’s 18/e p2300
a. Urine sodium < 10 meq/l (PGI June 06) 206. A 65 year old male smoker presents with gross total painless
b. Normal renal histology hematuria. The most likely diagnosis is: (AI 2003)
c. Renal function abnormal even after liver become normal a. Carcinoma of urinary bladder
d. Proteinuria < 500 mg/day Ref: Harrison’s 18/e p2601 b. Benign prostatic hyperplasia
199. Which of the following statements is incorrect with regard c. Carcinoma prostate
to Hepatorenal syndrome in a patient with cirrhosis: d. Cystolithiasis
a. Createnine clearance < 40 ml/min (AI 2003) Ref: Bailey 26/e p791
b. Urinary sodium < 10mq/L 207. An elderly male presents with one episode of gross
c. Urine osmolality lower than plasma osmolality haematuria. All of the following investigations are
d. No sustained improvement in renal function after volume recommended for investigating this patient except:
expansion. a. Cystoscopy (AI 2007)
Ref: Harrison’s 18/e p2601 b. Urine microscopy for malignant cells
200. Which of the following statements is incorrect with regard c. Urine tumor markers
to heptorenal syndrome in a patient with cirrhosis: d. Intravenous Pyelogram
(AIIMS Nov 2004) Ref: Bailey 26/e p2337
a. The creatinine clearance is > 40 ml/min 208. A 60 yr old smoker came with a history of painless gross
b. The urinary sodium is less than 10 mmol/L hematuria for one day. Most logical investigation would
c. The urine osmolality is lower than the plasma osmolality be: (AI 2007)
d. There is poor response to volume expansion a. Urine routine
Ref: Harrison’s 18/e p2601 b. Plain X ray KUB
201. A 28 year old boy met with on accident and sustained severe c. USG KUB
crush injury. He is most likely to develop: (AIIMS Nov 09) d. Urine microscopy for malignant cytology
MEDICINE
a. Acute Renal Failure Ref: Bailey 26/e p2337, 338
b. Hypophosphatemia 209. A seven year old asymptomatic girl is found to have
c. Hypercalcemia persistant hypertension. There is no significant history and
d. Acute Myocardial Infarction urine examination is normal. Which of the following is the
Ref: Harrison’s 18/e p2300; Table 279-1 most likely cause: (AI 2010)
202. Rhabdomyolysis and Myoglobinuria may be seen in: a. Essential hypertension
a. Viperbite (PGI June 02) b. Renal parenchymal disease
b. Multiple Hornet Stings c. Polycystic kidney disease
c. Clostridium Perfringes d. Coarctation of aorta
d. Streptococcus Ref: Harrison’s 18/e p353, 2285
e. All of the above Ref: Harrison’s 18/e p2298
Ans. 195. a. Salicylate poisoning 196. c. PNH 197. b, c and d 198. a and b
199. c. Urine osmolality... 200. a and c
201. a. Acute Renal Failure 202. e. All of the above
203. c. Hyperthyroidism
204. c. Hypercalcemia 205. c. Hemoglobinuria 206. a. Carcinoma of urinary bladder
207. c. Urine tumor...
208. d. Urine microscopy 209. b. Renal Parenchymal...
1130 Jaypee’s Triple A
210. A child presented with intermittent episodes of left sided 213. Renal damage due to amphotericin B are all, except:
flank pain. Ultrasonography reveals a large hydronephrosis a. Azotemia (AIIMS Nov 01)
with dilated renal pelvis and cortical thinning with a b. Renal tubular acidosis
normal ureter. Kidney differential function was observed c. Glomerulonephritis
to be 19% which of the following is the best management: d. Hypokalemia Ref: Goodman Gilman 11/e p1228
a. Nephrectomy (AI 2010) 214. Which of the following drugs is not a part of the ‘Triple
b. Pyeloplasty Therapy’ immunosuppression for post-renal transplant
c. External Drainage patients? (AI 2006)
d. Endopylostomy a. Cyclosporine
211. All of the following are features of Bartter’s syndrome, b. Azathioprine
except: c. FK 506
a. Hypokalemia d. Prednisolone
b. Hypermagnesemia Ref: Harrison’s 18/e p2329, 2331
c. Hyperprostaglandin 215. The kidney produces all of the following substances except:
d. Hyper calciurea Ref: Harrison’s 18/e p353 a. Erythropoietin (J & K 2012)
212. Urinary K+ excretion is increased in: (AIIMS Nov 04) b. 1, 25-hydroxycholecalciferol
a. Bronchiectasis c. Renin
b. Meningitis d. Angiotensin-converting enzyme Ref: Ganong 24/e p703
c. Osteomyelitis
d. Hepatitis
MEDICINE
A. Ascites
B. Cirrhosis and its Complications
C. Disorders of Biliary Tract
D. Hemochromatosis
E. Hepatitis
F. Hepatocellular Carcinoma
G. Jaundice
H. Miscellaneous
1132 Jaypee’s Triple A
b. C
infection? (Kerala PG 09) c. B
a. HBV DNA d. E
b. HBs Ag Ref: Harrison’s 18/e p2546
c. IgM Anti HBe Ag 14. In chronic HBV infection, persistent active replication of
d. IgM Anti HBc Ag Ref: Harrison’s 18/e 2550, 17/e p1942 virus is indicated by: (J & K 2010)
7. A-40 years male presented with following serologic a. HBeAg
markers of hepatitis-B are Hbe Ag, HBV DNA, and anti b. Anti HBe
HBc. Diagnosis is: c. HBcAg
a. Acute hepatic-B d. All of the above
b. Acute hepatitis progress to chroic infection Ref: Harrison’s 18/e p2568-2569
Ans. 1. a. Colloid carcinoma... 2. d. USG can detect as little... 3. a. Periampullary carcinoma 4. c. HbeAg
5. b. HBsAg and anti... 6. d. IgM Anti HBc Ag 7. a. Acute hepatic-B 8. c. Anti HBc IgG HBe Ag
9. c. Hepatitis B virus 10. d. Blood transfusion 11. c. Anti HBC 12. c. Hepatitis E
13. d. E 14. a. HBeAg
Liver and Biliary Tract 1133
15. Micronodular cirrhosis is commonly seen in all except: 23. An 18 year old male presents with massive hematemesis; he
a. Chronic hepatitis B (AIIMS Nov 07) has history of fever for the past 14 days for which he was
b. Alcoholic liver disease managed with drugs; moderate splenomegaly is present;
c. Hemochromatosis diagnosis is: (AI 2001)
d. Chronic extrahepatic biliary obstruction a. NSAID induced duodenal ulcer
Ref: ‘Pathology: Basic and systemic’ by Woolfe 1998/587, 597 b. Drug induced gastritis
16. A patient presents with esophageal varices and a liver span c. Esophageal varices
of 10cm. All of the following are likely causes, except: d. None of the above Ref: Harrison’s 18/e p2598
a. Haemochromatosis (PGI Dec 04) 24. A young boy, Rahul presents with massive hematemesis.
b. Alcoholic liver disease He had fever for 15 days few days back which was treated
c. Veno-occlusive disease with few drugs. Clinical examination reveal moderate
d. Post necrotic cirrhosis splenomegaly. No other history is positive Probable
e. Budd-chiary syndrome Ref: Harrison’s 18/e p2587 diagnosis is: (AIIMS Nov 01)
17. Enlarged liver with Hepatocellular dysfunction may be a. Drug induced gastritis
seen in all of the following, except: (PGI June 05) b. Oesophageal tear
a. Wilson’s disease c. Bleeding duodenal ulcer
b. Budd chiari syndrome d. Oesophageal varices
c. Alcoholic hepatitis Ref: Harrison’s 18/e p2598
d. NASH 25. A man presents with history of hematemesis of about 500
e. Post necrotic syndrome Ref: Harrison’s 18/e p2587 mL of blood. On examination, spleen is palpable 5 cms
18. All of the following statements about primary sclerosing below the left costal margin. The most likely diagnosis is:
cholangitis are true, except: (PGI Dec 03) a. Portal hypertension (AI 2012)
a. Increased incidence in females b. Gastric ulcer
b. Associated with inflammatory bowel disease c. Drug induced
c. May involve both intra and extrahepatic ducts d. Mallory Weiss tear
d. ERCP is a sensitive investigation Ref: Harrison’s 18/e p320, Bailey 26/e p25981
Ref: Harrison’s 18/e p2596
26. All can be used as endoscopic sclerosants in the treatment
19. A child presents with massive hematemesis and systemic of variceal bleeding, except: (AIIMS Nov 01)
hypotension. He has no fever or other significant history. a. Polydochyl
Examination reveal massive splenomegaly but no b. Cynoacrylate
hepatomegaly. Likely diagnosis is: (AIIMS Nov 01) c. Alcohol
a. Hepatocellular carcinoma d. Acetic acid Ref: Harrison’s 18/e p2535
b. Bleeding duodenal ulcer
27. Stigmata of chronic liver disease include all of the following,
c. Oesophageal varices
except: (DNB 2010)
d. Non-cirrhotic portal fibrosis
a. Parmar erythema
Ref: Harrison’s 18/e p2598
b. Spinder naevi
20. Ingestion of arsenic causes: (AIIMS May 01) c. Testicular atrophy
a. Hepatic carcinoma d. Subcutaneous nodules Ref: Internet
b. Hepatic adenoma 28. Skin stigmata of Liver disease include all of the following,
c. Non cirrhotic portal fibrosis except: (DNB 2011)
d. Hepatic cirrhosis Ref: API 6/e p591; API 17/e p621 a. Parmar erythema
21. Which of the following is the most common presenting b. Paper money skin
MEDICINE
symptom of non-cirrhotic portal hypertension? (AI 2006) c. Drumstick fingers
a. Chronic liver failure d. Subcutaneous nodules
b. Ascites Ref: Hepatology: Textbook and Atlas by Kunte Springer 2008/84
c. Upper gastrointestinal bleeding 29. Which of the following is not a precipitating factor for
d. Encephalopathy Ref: Harrison’s 18/e p2598 hepatic encephalopathy in patients with chronic liver
22. In patients with cirrhosis of the liver the site of obstruction disease? (AIIMS May 05)
in the portal system is in the: (AI 2004) a. Hypokalemia
a. Hepatic vein b. Hyponatremia
b. Post sinusoidal c. Hypoxia
c. Extrahepatic portal vein d. Metabolic acidosis
d. Sinusoids Ref: Harrison’s 18/e p2597 Ref: Harrison’s 18/e p2601; 17/e p1979
Ans. 15. a. Chronic hepatitis B 16. d. Post necrotic... 17. e. Post necrotic syndrome 18. a. Increased incidence in...
19. d. Non-cirrhotic... 20. c. Non cirrhotic portal... 21. c. Upper gastrointestinal... 22. d. Sinusoids
23. c. Esophageal varices 24. d. Oesophageal... 25. a. Portal Hypertension 26. d. Acetic acid
27. d. Subcutaneous... 28. d. Subcutaneous nodules 29. d. Metabolic acidosis
1134 Jaypee’s Triple A
30. Hepatic encephalopathy is predisposed by all, except: 38. Which of the following statements is incorrect with regard
a. Hyperkalemia (PGI June 03) to hepatorenal syndrome in a patient with cirrhosis?
b. Dehydration a. Creatinine clearance < 40 mL/min (AI 2003)
c. Constipation b. Urinary sodium < 10mq/L
d. G1 Bleeding c. Urine osmolality lower than plasma osmolality
e. SBP d. No sustained improvement in renal function after
Ref: Harrison’s 18/e p2601 Harrison’s 17/e p2601 volume expansion
31. Features of acute fulminant hepatic failure include all of the Ref: Harrison’s 18/e p2601
following except: (PGI June 01) 39. Which of the following statements is incorrect with regard
a. Hyperglycemia to heptorenal syndrome in a patient with cirrhosis:
b. Hypomagnesemia (AIIMS Nov 2004)
c. Hepatorenal syndrome a. The creatinine clearance is > 40 mL/min
d. Intracranial hemorrhage b. The urinary sodium is less than 10 mmol/L
e. Coagulopathy c. The urine osmolality is lower than the plasma osmolality
32. In a child with acute liver failure, the most important d. There is poor response to volume expansion
prognostic factor for death is: (AIIMS 06) Ref: Harrison’s 18/e p2601
a. Increasing transaminase
b. Increasing bilirubin
c. Increasing prothrombin time
C. DISORDERS OF BILIARY TRACT
d. Gram-negative sepsis
40. 5-nucleotidase activity is increased in: (AI 2005)
Ref: Harrison’s 18/e p2530 17/e p1926
a. Bone diseases
33. In patients with acute liver failure, the best prognostic b. Prostate cancer
indicator is: (AIIMS May 01) c. Chronic renal failure
a. Serum albumin d. Cholestatic disorders
b. Serum alpha-fetoprotein Ref: Harrison’s 18/e p2529, 17/e p1925
c. Serum bilirubin
41. The most common site of intestinal obstruction in gallstone
d. Factor V estimation
ileus is: (AIIMS May 05, AI 04)
Ref: API 17/e p602; CMDT 2003/ 637; 09/592
a. Duodenum
34. A symmetric high-voltage, triphasic slow wave pattern is b. Jejunum
seen on EEG in the following: (AIIMS May 06) c. Ileum
a. Hepatic encephalopathy d. Sigmoid colon
b. Uremic encephalopathy Ref: Harrison’s 18/e p2618
c. Hypoxic encephalopathy
42. Which of the following is not an indication for
d. Hypercarbic encephalopathy Ref: Harrison’s 16/e p1868
cholecystectomy: (AIIMS May 05)
35. Flapping tremors may be associated with all of the a. 70 year old male with symptomatic gall stone
following, except: (DNB 2010) b. 20 years old male with sickle cell anemia and symptomatic
a. Hepatic encephalopathy gallstones
b. Uremia c. 65 year old female with a large gallbladder polyp
c. CO2 narcosis d. 55 year old with an asymptomatic gallstone
d. Thyrotoxicosis Ref: Internet Ref: Harrison’s 18/e p2620
36. Following liver transplantation, recurrence of primary 43. A 69 year old male patient having coronary artery disease
disease in the liver most likely occurs in: (AIIMS Nov 05) was found to have gall bladder stones while undergoing a
a. Wilson disease
MEDICINE
Ans. 30. a. Hyperkalemia 31. a. Hyperglycemia 32. c. Increasing prothrombin... 33. d. Factor V estimation
34. a. Hepatic... 35. d. Thyrotoxicosis 36. b. Autoimmune hepatitis 37. a and b
38. c. Urine osmo-*/lality... 39. a and c 40. d. Cholestatic disorders 41. c. Ileum
42. d. 55 year old with... 43. c. No surgery for...
Liver and Biliary Tract 1135
51. Which of the following statements about hemochromatosis
D. HEMOCHROMATOSIS is not true: (DNB June 2011)
a. Hypogonadism may be seen
44. Best investigation in alcoholic liver disease is: b. Arthropathy may occur
a. GGT (Kerala PG 09) c. Diabetes mellitus may develop
b. SGPT d. Desferrioxamine is treatment of choice
c. SGOT Ref: Harrison’s 18/e p2312, 17/e p1764, 2433
d. Alkaline phosphatase
52. All of the following statements about hereditary
Ref: Harrison’s 18/e p2590, 17/e p1970
hemochromatosis are true except: (AI 2008)
45. Antimitochondrial antibodies are found in: (AP 2012) a. Arthropathy involving small joints of hands may be seen
a. Drug induced hepatitis b. Skin pigmentation is a frequent presentation
b. Chronic active hepatitis c. Desferroxamine is the treatment of choice
c. Primary biliary cirrhosis d. Hypogonadism may be seen
d. All of the above Ref: Harrison’s 18/e p3165, 3166, 17/e p2433
Ref: Harrison’s 18/e p2595 53. All are seen in hemochromatosis except:
46. A 55 year old man with a 10 years history of alcohol a. Hypogonadism (AI 2008)
consumption developed fatty liver. Which of the following b. Arthropathy
best explains the pathogenesis of the liver disease: c. Bronze diabetes
a. Increased synthesis of glycerol 3- phosphate d. Desferrioxamine is the treatment of choice.
b. Decreased hydrolysis of fat in adipose tissue. Ref: Harrison’s 18/e p3166, 17/e p2433
c. Increased synthesis of VLDL (MP PG 2010) 54. Which of the following statements about Hemochromatosis
d. Increases beta – oxidation of fatty acids is true: (AI 2009)
Ref: Harpaer’s textbook of biochemistry 28/e p218 a. Shows complete penetrance
47. Which one of the following is least expected to precipaitate b. Inherited as an autosomal recessive disorder
hepatic encephalopathy in a liver cirrhosis patient? c. Phlebotomy is curative
a. Peritoneal tap (AP 2010) d. More common in females
b. Antibodies treatment Ref: Harrison’s 18/e p3163, 17/e p2430, 2431, 2432
c. Variceal bleed
d. Hypokalemia
Ref: Harrison’s 18/e p2601 E. HEPATITIS
48. A patient presents with Arthritis, hyperpigmentation of
55. A 50 year old lady presented with history of pain upper
skin and hypogonadism, likely diagnosis is: (AI 2001)
abdomen, nausea, and decreased appetite for 5 days. She
a. Hemochromatosis
had undergone cholecystectomy 2 years back. Her bilirubin
b. Ectopic ACTH secreting tumour of the lung
c. Wilson’s disease was 10 mg/dL, SGPT 900 IU/L SGOT 700 IU/L and serum
d. Rheumatoid arthritis alkaline phosphatase was 280 IU/L. What is the most likely
Ref: Harrison’s 18/e p3164, 17/e p2433 diagnosis:
a. Acute pancreatitis (AIIMS Nov 05)
49. Earliest phenotypic manifestation of idiopathic hereditary
b. Acute cholangitis
hemochromatosis is: (AIIMS May 07)
c. Acute viral hepatitis
a. Postprandial increase in serum iron concentration
d. Posterior penetration of peptic ulcer
b. Elevated serum ferritin level
Ref: Harrison’s 18/e p2549
c. Slate grey pigmentation of skin
d. Increased transferrin saturation 56. Councilman bodies are seen in: (AIIMS Nov 07)
MEDICINE
Ref: Harrison’s 18/e p3165, 17/e p2431, 2432 a. Wilson disease
b. Alcoholic hepatitis
50. Lallo, aged 54 years, who is a known diabetic patient develops
c. Acute viral hepatitis
cirrhosis. There is associated skin hyperpigmentation and
d. Auto immune hepatitis
restrictive cardiomyopathy which of the following is the
Ref: Robbins 7/e p899; ‘Pathology’ by wolf (1998)/572
best initial test to diagnose this case: (AIIMS Nov 2000)
a. Iron binding capacity 57. The commonest hepatotropic virus progressing to chronicity
b. Serum ferritin is:
c. Serum copper a. HEV (AIIMS May 01)
d. Serum ceruloplasmin b. HAV
Ref: Harrison’s 18/e p3165, 17/e p2433 c. HBV
d. HCV Ref: Harrison’s 18/e p2546
Ans. 44. c. SGOT 45. c. Primary biliary cirrhosis 46. c. Increased synthesis of... 47. a. Peritoneal tap
48. a. Hemochromatosis 49. d. Increased transferrin... 50. a. Iron binding capacity 51. d. Desferrioxamine is...
52. c. Desferroxamine is... 53. d. Desferrioxamine is the... 54. b. Inherited as an... 55. c. Acute viral hepatitis
56. c. Acute viral hepatitis 57. d. HCV
1136 Jaypee’s Triple A
58. Chronic hepatitis is caused by (select two options): d. IgE HBsAg antibody
a. Hepatitis A (PGI June 03) Ref: Harrison’s 18/e p2539, 2540, 2544; 17/e p1938, 1934, 1943
b. Hepatitis B 67. Which of the following markers in the blood is the most
c. Hepatitis C reliable indicator of recent hepatitis B infection?
d. Hepatitis E a. HBsAg (AIIMS May 03)
e. Hepatitis G Ref: Harrison’s 18/e p2567 b. IgG anti – HBs
59. Which of the following hepatitis viruses have significant c. IgM anti – HBc
perinatal transmission: (AI 2003) d. IgM anti – Hbe Ref: Harrison’s 18/e p2539
a. Hepatitis E virus 68. Which of the following is a marker for Active Hepatitis B?
b. Hepatitis C virus a. HBe Ag (DNB 2012)
c. Hepatitis B virus b. IgM Anti HBs Ag
d. Hepatitis A virus Ref: Harrison’s 18/e p2546 c. HBs Ag
60. Non-parenteral hepatitis is: (AI 2000) d. Ig G Anti HB1 Ag Ref: Harrison’s 18/e p2540
a. Hep E 69. All of the following are markers of active replicative of
b. Hep B chronic hepatitis B, except: (AIIMS Nov 08)
c. Hep C a. HBV DNA
d. Hep D Ref: Harrison’s 18/e p2546 b. HBV DNA Polymerase
61. All of the following are correctly matched, except: c. HBeAg
a. LKM1 - Autoimmune Hepatitis (AIIMS Nov 2010) d. AST and ALT
b. LKM2 - Drug Induced Hepatitis Ref: Harrison’s 18/e p2540, 2550
c. LKM1 - Chronic Hepatitis C 70. Which of the following is the most sensitive marker of
d. LKM2 - Chronic Hepatitis D active Hepatitis B virus replication: (PGI 08)
Ref: Harrison’s 18/e p2567 a. HBV DNA
62. Chronic Active Hepatitis can be best differentiated from b. HBV DNA polymerase
Chronic Persistant Hepatitis by: (DNB 2011) c. HBeAg
a. HBs Ag d. Transaminases Ref: Harrison’s 18/e p2540, 2550
b. Antibody to HBs Ag 71. Acute hepatitis B is diagnosed by: (PGI Dec 04)
c. Histopathology a. HBsAg
d. None of the above b. HBe Ag
63. Hepatitis B can be transmitted through all of the following, c. Anti- HBs AG
except: (DNB 2011) d. IgM anti – HBc
a. Semen e. HBV DNA Ref: Harrison’s 18/e p2550
b. Blood 72. A 35-year-old male patient presented with history of
c. Breast milk jaundice for 15 days. The onset was preceded by a prodromal
d. Fecal-oral (stool) illness. His serum tested positive for HBsAg. A clinical
Ref: Harrison’s 18/e p2546 diagnosis of acute Hepatitis B was made. What should be
64. Chances of vertical transmission of Hepatitis B may be as the next best confirmatory investigation: (AIIMS May 04)
high as: (DNB 2011) a. Anti-HBeAg antibody
a. 25% b. HBe antigen
b. 40% c. Anti-HBe IgM antibody
c. 60% d. HBV DNA by PCR Ref: Harrison’s 18/e p2550
d. 90% 73. A thirty-year man presented with nausea, fever and
Ref: Harrison’s 18/e p2547
MEDICINE
Ans. 58. b and c 59. c. Hepatitis B virus 60. a. Hep E 61. d. LKM2 - Chronic...
62. c. Histopathology 63. d. Fecal-oral (stool) 64. d. 90% 65. b. Presence of IgM anti...
66. a. IgM HBcAg... 67. a. HBsAg 68. a. HBe Ag 69. d. AST and ALT
70. a. HBV DNA 71. a and d 72. None 73. b. Acute hepatitis B infection...
Liver and Biliary Tract 1137
74. A male patient is observed to be HBs Ag antigen positive c. Anti HBS Ag
HBe Ag antigen negative and anti-HBe antibody positive. d. HBe Ag Ref: Harrison’s 18/e p2550
HBV DNA copies are observed to be 100,000/ml while 82. All of the following should be included during preliminary
SGOT and SGPT are elevated to 6 times the upper limit of evaluation of a case of suspected acute viral hepatitis
normal value. What is the likely diagnosis: (AI 2010) except: (AIIMS June 2000)
a. HBV surface mutant a. Hbs Ag
b. HBV precore mutant b. IgM anti HBc
c. Wild HBs Ag c. Anti-HCV
d. Inactive HBV carrier Ref: Harrison’s 18/e p2541 d. IgM anti HBe
75. A patient is found to be positive for HBs Ag on routine Ref: Harrison’s 18/e p2550
laboratory evaluation. Other serological tests for hepatitis 83. A 30-year-old patient presented with history of jaundice for 30
are unremarkable. He is clinically asymptomatic and days. His liver function tests showed bilirubin of 100mg/dL,
liver enzymes are within the normal range. Which of the SGOT/SGPT-1100/1450, serum alkaline phosphatase-240 IU.
following best describes his diagnosis: (AI 2010) He was positive for Hbs Ag. What should be the confirmatory
a. Inactive HBV carrier test to establish Acute hepatitis B infection?
b. Acute hepatitis B a. IgM Anti-HBc antibody (AIIMS May 06)
c. Chronic hepatitis B b. HbeAg
d. Active HBV carrier Ref: Harrison’s 18/e p2550 c. HBV DNA by PCR
76. The following is a marker of acute hepatitis B infection: d. Anti-HBc antibody Ref: Harrison’s 18/e p2550
a. DNA polymerase (AIIMS Nov 07) 84. HBV Replication is indicated by all of the following,
b. Hepatitis core antigen except: (PGI Dec 06)
c. Anti HBs a. HBV DNA
d. IgG to core antigen b. DNA polymerase
Ref: Harrison’s 18/e p2550 c. HBe Ag
77. All of the following are seen in active chronic hepatitis B d. HBs Ag Ref: Harrison’s 18/e p2550
except: (AIIMS Nov 07) 85. Hera lal 35 years old man was found +ve for HBsAg and
a. IgM against core antigen HBeAg, accidentally during screening of blood donation.
b. Total core antibody On lab examination SGOT and SGPT are normal. What
c. HbeAg should you do next: (AI 2002)
d. HbsAg Ref: Harrison’s 18/e p2550 a. Liver biopsy
78. In a patient only Anti HBsAg is positive in serum, all other b. Interferon therapy
viral markers are negative. This indicates: c. Observation
a. Acute hepatitis (AIIMS June 2000) d. HBV-DNA estimation
b. Chronic active hepatitis Ref: Harrison’s 18/e p2550
c. Persistent carrier 86. A person is screened for blood donation. Which of the
d. Immunized person with hepatitis B vaccine following serology is safe for blood donation: (PGI June 03)
Ref: Harrison’s 18/e p2550 a. Anti HBsAg positive
79. A patient is found to be positive only for Anti HBsAg. All b. HBsAg positive
other viral markers are negative.The likely diagnosis is: c. Anti HBC positive (IgM)
a. Vaccination (PGI 2009) d. HBcAg positive
b. Chronic hepatitis B Ref: Harrison’s 18/e p2550
c. Acute hepatitis B 87. A blood donor is not considered for safe transfusion, if he
MEDICINE
d. Fulminant hepatitis B Ref: Harrison’s 18/e p2550 has: (AI 2000)
80. The marker for determining efficacy of hepatitis B a. Anti HBs Ag +ve
vaccination is: (DNB 2010) b. Anti HBs Ag and HBc Ag +ve
a. HBs Ag c. Hbs Ag +ve, and IgM anti HBc +ve
b. IgM Anti HBc d. Anti HBe +ve Ref: Harrison’s 18/e p2550
c. IgG Anti HBc 88. Reverse transcriptase of hepatitis B virus is coded on the
d. Anti – HBs Ag Ref: Harrison’s 18/e p2550 following gene: (AI 2000)
81. The only serological marker present in the ‘window period’ a. C gene
of Hepatitis B is: (DNB 2011) b. S gene
a. Anti HBc c. P gene
b. HBs Ag d. X gene Ref: Harrison’s 18/e p2540
Ans. 74. b. HBV precore... 75. a. Inactive HBV carrier 76. a. DNA polymerase 77. a. IgM against core antigen
78. d. Immunized person... 79. a. Vaccination 80. d. Anti – HBs Ag 81. a. Anti HBc
82. d. IgM anti HBe 83. a. IgM Anti-HBc antibody 84. d. HBs Ag 85. d. HBV-DNA estimation
86. a. Anti HBsAg positive 87. c. Hbs Ag +ve, and IgM... 88. c. P gene
1138 Jaypee’s Triple A
89. A young male was found to be HBsAg positivity and HBe 96. Interferon treatment is recommended in chronic hepatitis B
Ag negative. His liver enzymes were normal. The next step in patients with: (PGI June 07)
in management of this young male should be: (AI 2008) a. ↑ HBV DNA and normal ALT
a. Lamivudine therapy b. ↑ HBV DNA and ↑ ALT
b. Lamivudine plus IFN therapy c. ↑ HBV DNA and compensated cirrhosis
c. Start IFN therapy d. ↑ HBV DNA and decompensated cirrhosis
d. Serial monitoring Ref: Harrison’s 18/e p2575 Ref: Harrison’s 18/e p2575, 2576
90. A 26 year old man was observed to be positive for HBs Ag 97. Agents recommended for treatment of chronic Hepatitis B
but negative for HBe Ag. The AST / ALT levls were observed include all of the following, except: (PGI June 05)
to be within normal limits. The next step in management of a. Interferon
this patient should be: (DNB 2011) b. Lamuvudine
a. Serial monitoring c. Adefovir
b. Antiviral treatment with Lamivudine monotherapy d. Entecevir
c. Antiviral treatment with Lamivudine and pulsed IFN e. Famcyclovir Ref: Harrison’s 18/e p2569
d. Antiviral treatment with IFN a alone 98. Chronic liver disease is most commonly caused by:
Ref: Harrison’s 18/e p2576 a. Hepatitis B (AI 2000)
91. Lamivudine is recommended for treatment of chronic b. Hepatitis A
hepatitis B when: (DNB 2012) c. Hepatitis C
a. HBe Ag positive d. Hepatitis E Ref: Harrison’s 18/e p2546
b. HBe Ag Negative 99. Hepatitis C virus is associated with: (AI 2000)
c. ALT > 2 xULN a. Anti LKM antibody
d. Viral DNA > 102 copies b. Scleroderma
Ref: Harrison’s 18/e p2576 c. Cryoglobulinemia
92. Which one of the following pairs regarding Hepatitis B is d. Polyarteritis nodosa Ref: Harrison’s 18/e p2550
not correctly matched: (AIIMS Nov 2010) 100. Extrahepatic manifestations of Hepatitis C include all of the
a. Acute Viral Hepatitis B - Supportive care following except: (PGI June 04)
b. Acute Viral Hepatitis B - Antiviral therapy a. Lichen planus
c. Chronic Viral Hepatitis B - Supportive care b. Celiac Disease
d. Chronic Viral Hepatitis B - Antiviral therapy c. Glomerulonephritis
Ref: Harrison’s 18/e p2554 d. Cryoglobulinemia
93. All of the following statements about Hepatitis B are true, e. Polyarthritis Ref: Harrison’s 18/e p2549,2552
except: (AIIMS Nov 2010) 101. Hepatic C is associated with all except: (PGI June 08)
a. Vertical transmission is more common than horizontal a. PAN
transmission in non-endemic areas b. Dermatomyositis-like syndrome
b. Age of onset determines prognosis c. Lichen Planus
c. Period of communicability lasts several months d. Psoriasis Ref: Harrison’s 18/e p2579
d. Virus can be detected in blood one month before jaundice
Ref: Harrison’s 18/e p2547 102. A 30 year old patient with H/O antibodies to HCV for 6 months
duration and his AST/ALT is normal. There is no symptom or
94. Extrahepatic manifestations of hepatitis B include all of the stigmata of liver disease. The most appropriate approach:
following except: (PGI June 01) a. Reassure the patient (PGI June 04)
a. Aplastic anemia b. Repeat titre every three years
b. Guillain Barre syndrome c. Repeat enzymes every year
c. Pancreatitis d. Do liver biopsy and start antiviral drugs accordingly.
MEDICINE
Ans. 89. d. Serial monitoring 90. a. Serial monitoring 91. c. ALT > 2 xULN 92. b. Acute Viral...
93. a. Vertical... 94. d. Gall stone disease 95. d. Focal Segmental... 96. b. ↑ HBV DNA and...
97. e. Famcyclovir 98. c. Hepatitis C 99. a. Anti LKM antibody 100. b. Celiac Disease
101. b. Dermatomyositis... 102. d. Do liver biopsy and...
103. e. Determine HCV...
Liver and Biliary Tract 1139
104. A 55 year old male patient was diagnosed to have chronic 112. All of the following statements about non alcoholic fatty
hepatitis C. He responded to treatment with interferon. liver disease are true, except: (PGI June 01)
However, after one year of follow up he showed a relapse a. Common in diabetics
of disease. Which of the following would be the next most b. Clofibrate provides effective treatment
appropriate choice? (AIIMS Nov 03) c. Commonest cause of cryptogenic cirihosis
a. Ribavarin and interferon d. Associated with elevated transminases
b. Lamivudine and interferon Ref: Harrison’s 18/e p2605
c. Nevirapine and lamivudine
d. Indinavir and ribavarin.
Ref: Harrison’s 18/e p2583 F. HEPATOCELLULAR CARCINOMA
105. Sustained response to antiviral therapy (IFN + Ribavarin)
113. Which of the following is seen in gallstone disease?
in hepatitis C is indicated by: (PGI Dec 04)
a. Decreased bile and cholesterol ratio (Kerela PG 2008)
a. High HCV-RNA
b. Increased bile and cholesterol ratio
b. Cirrhosis
c. Equal bile and cholesterol ratio
c. Age > 40 years
d. Cholesterol only
d. Genotype I
Ref: Harrison’s 16/e p1181; 1882; 18/e 2616, 2617, 17/e p1992
e. Female Sex
Ref: Harrison’s 18/e p2583 114. Which of the following disease is not a cause of indirect
hyperbilirubinemia? (DP PGMEE 2009)
106. During an epidemic of hepatitis E, fatality is maximum in:
a. Rotor’s syndrome
a. Pregnant women (AI 2000)
b. Criggler najjar syndrome
b. Infants
c. Gilbert syndrome
c. Malnourished male
d. Hereditary spherocytosis
d. Adolescents Ref: Harrison’s 18/e p2552
Ref: Harrison’s 18/e p2536, 17/e p1927
107. Most common cause of acute sporadic hepatitis in India is:
115. 38 year old man Babbu, a chronic alcoholic, presents with
a. Hepatitis E (DNB 2012)
pain in abdomen. On examination his liver is enlarged and
b. Hepatitis B
serum α fetoprotein is elevated. The most likely diagnosis
c. Hepatitis C
is: (AI 2000)
d. Hepatitis D Ref: Harrison’s 18/e p2548
a. Hepatocellular carcinoma
108. Characteristic auto antibodies of autoimmune hepatitis b. Liver cell hyperplasia
include all of the following, except: (PGI Dec 06) c. Hepatic adenoma
a. Antinuclear antibodies (ANA) d. Hepatitis
b. Anti SLA Ref: Harrison’s 18/e p779, 17/e p582
c. Anti LKM1
116. Increase in alpha-fetoprotein is seen in: (AIIMS June 2000)
d. ANCA Ref: Harrison’s 18/e p2586
a. Hepatoblastoma
109. Ratio of AST/ALT > 1 is present in: (AIIMS May 07) b. Neuroblastoma
a. Non alcoholic steatohepatitis c. Thymoma
b. Alcoholic hepatitis d. Angiosarcoma Ref: Robbin’s 8/e p327; Table 7-12
c. Wilson’s disease
117. All of the following are true about fibrolamellar carcinoma
d. All of the above
of the liver except: (AI 2001)
Ref: Harrison’s 18/e p2529, 2590
a. More common in females
110. Which is useful to decrease mortality and renal failure in b. Better prognosis than HCC
acute liver disease due to alcoholism: (AIIMS May 07) c. AFP levels always greater than 1000
MEDICINE
a. Pentoxyfylline d. Occur in younger individuals
b. Orlistat Ref: Harrison’s 18/e p784, 17/e p585
c. S-Adenosyl methionine
d. Syrlamysin
Ref: Harrison’s 18/e p2591 G. JAUNDICE
111. Non alcoholic steatohepatitis is seen in all, except:
a. Diabetes (PGI June 05) 118. Conjugated hyperbilirubinemia is seen in:
b. Obesity a. Gilbert’s syndrome (AIIMS May 2006)
c. Hypertriglyceridemia b. Criggler Najjar syndrome
d. Total parentral nutrition c. Breast milk jaundice
e. Gall stone disease Ref: Harrison’s 18/e p2604 d. Dubin Johnson syndrome Ref: Harrison’s 18/e p2535
Ans. 104. a. Ribavarin and... 105. e. Female Sex 106. a. Pregnant women 107. a. Hepatitis E
108. d. ANCA
109. b. Alcoholic hepatitis 110. a. Pentoxyfylline 111. e. Gall stone disease
112. b. Clofibrate provides... 113. a. Decreased bile and...
114. a. Rotor’s syndrome 115. a. Hepatocellular carcinoma
116. a. Hepatoblastoma
117. c. AFP levels always... 118. d. Dubin Johnson..
1140 Jaypee’s Triple A
119. A patient presents with unconjugated hyperbilirubinemia 127. True about Crigler Najjar type II syndrome is: (PGI Dec 97)
and presence of urobilinogen in urine. Which amongst the a. Diglucuronide deficiency
following is the least likely diagnosis: (AI 2010) b. Recessive trait
a. Hemolytic jaundice c. Kernicterus is seen
b. Crigler Najjar syndrome d. Phenobarbitone is not useful Ref: Harrison’s 18/e p2533
c. Gilbert’s syndrome
d. Dubin Johnson syndrome Ref: Harrison’s 18/e p2535
120. A patient presents with unconjugated hyperbilirubinemia
H. MISCELLANEOUS
and elevated urobilinogen levels in urine. The most likely
128. All of the following are true of Reye’s syndrome, except:
diagnosis is: (AI 2010)
a. If frequently complicated viral infections
a. Hemolytic Jaundice
b. Prothrombin time is prolonged (Feb DP PGMEE 2009)
b. Crigler Najjar syndrome
c. Disease may be precipitated by salicylates
c. Gilbert’s syndrome
d. Deep jaundice is present
d. Dubin Johnson syndrome Ref: Harrison’s 18/e p325-327
Ref: Harison’s 18/e p1463, 1465, 1417
121. A young male with gallbladder stones shows the following
129. Hepatorenal syndrome type 1 is characterized by:
test results; serum bilirubin 2.5, Hb 6; urine test positive for
a. Proteinuria and urinary sediment (Karnataka 2011)
urobilinogen; diagnosis is: (AI 2001)
b. Urine sodium more than 40 mmol/day
a. Hemolytic jaundice
c. Urine/plasma osmolarity ratio more than 1.5
b. Obstructive jaundice
d. Haemodialysis is very useful
c. Hepatocellular jaundice
Ref: Harrison’s 18/e p2601
d. Protoporphyria
Ref: Harrison’s 18/e 325-327, 17/e p262, 263, 264 130. Raised levels of alkaline phosphatase with absence of
increase in aminotransferases and normal bilirubin levels
122. A young pt presents with jaundice. Total bilirubin is 21, direct
is seen in: (MHPGM-CET 2010)
is 9.6, alkaline phosphatase is 84 KA units. Diagnosis is:
a. Hyperthyroidism
a. Hemolytic jaundice (AI 2001)
b. Hodgkin’s lymphoma
b. Viral hepatitis
c. Inflammatory bowel disease
c. Chronic active hepatitis
d. All of the above
d. Obstructive jaundice Ref: Harrison’s 18/e p325
Ref: Harrison’s 18/e 2528, 2529, 17/e p1925
123. True statement about unconjugated hyperbilirubinemia
131. Vitamin that should be first supplemented during treatment
(select three options): (PGI Dec 05)
of alcoholic patient? (MHPGM-CET 2009, 2010)
a. 85% of the total should be indirect
a. Cyanocobalamine
b. Seen in hemolytic anemia
b. Folic acid
c. Seen in ↑ hemoglobin destruction (↑ bilirubin production)
c. Thiamine
d. 50% of the total should be indirect
d. Vitamin C
e. It is seen in biliary atresia and neonatal hepatitis
Ref: Harrison’s 18/e 597, 17/e p442
Ref: Harrison’s 18/e 9325-27, 17/e p262
132. Not a criteria for hepatorenal syndrome: (WB PG 08)
124. Abnormal excretory function of hepatocytes may be
a. Absent Kidney disease
assessed by: (PGI June 07)
b. Does not respond to volume correction
a. Increased PT
c. Creatinine clearance > 140 ml/min
b. Increased ALT
d. Urine sodium excreaton < 10%/present in 10%
c. Increased alkaline phosphatase
population of hepatic failure.
d. Increased gamma GT Ref: Harrison’s 18/e p2529
Ref: CMDT 2008 p587; Harrison’s 18/e 2601, 17/e p1979
125. Increased B12 level is seen in all, except: (AI 2000)
MEDICINE
Ans. 119. d. Dubin Johnson... 120. a. Hemolytic Jaundice 121. a. Hemolytic jaundice 122. d. Obstructive...
123. a b, and c.
124. c and d. 125. d. Cholestatic jaundice 126. b. BSP test
127. b. Recessive trait
128. d. Deep jaundice is present 129. c. Urine/plasma... 130. d. All of the above
131. c. Thiamine
132. c. Creatinine clearance... 133. a. Danazol 134. c. Hepatic veins
Liver and Biliary Tract 1141
135. Palpable purpura is seen in: (Rajasthan 2009) a. Schistosomiasis
a. HSP b. Cirrhosis
b. Churg Strauss c. Budd-chiari syndrome
c. Wagner granulomatosis d. Portal vein thrombosis Ref: Harrison’s 18/e p2598
d. Giant cell arteritis 144. Which of the following is a feature of intravascular
Ref: Harrison’s 18/e p148, 421 hemolysis? (J & K 2012)
136. In alcoholic liver disease AST: ALT ratio is: a. Hemoglobinemia
a. > 3:1 (Rajasthan 2008) b. Increased unconjugated bilirubin
b. > 2:1 c. Increased conjugated bilirubin
c. > 1:3 d. Hemosiderinuria Ref: Harrison’s 18/e p881
d. > 1:2 145. What is the most common cause for Budd Chiari
Ref: Harrison’s 18/e p2590 syndrome:
137. All of the following are true about Warfarin therapeutic a. Right ventricular failure
usage except: (AP 2010) b. PNH (AIIMS Nov 2000)
a. The INR is maintained between 2 and 3 c. Valve in hepatic veins
b. It is very useful in the prophylaxis of thromboembolism d. Polycythemia vera
c. It’s effect is monitored by observing the clotting time Ref: Robbin’s 8/e 872-873, 6/e p883; 7/e p919
d. Used in DVT 146. In Budd Chiari syndrome, the site of venous thrombosis
Ref: Harrison’s 18/e p998-1000 is:
138. The principal causes of death in Hemochormatosis include a. Infrahepatic inferior vena cava (AI 2004)
all, except: (AP 2010) b. Infrarenal inferior vena cava
a. Hepatocellular failure or portal hypertension c. Hepatic veins
b. Congestive cardiac failure d. Portal veins Ref: Robbin’s 8/e 872-873
c. Diabetes mellitus 147. Which of the following statements about Wilson’s disease
d. Hepatocellular carcinoma is true: (AI 2010)
Ref: Harrison’s 18/e p3166, 3162 a. Low serum ceruloplasmin and low urinary copper
139. Earliest presentation of hepatic encephalopathy is: (AP 2012) b. Low serum ceruloplasmin and high urinary copper
a. Convulsions c. High serum ceruloplasmin and low urinary copper
b. Altered sleep pattern d. High serum ceruloplasmin and high urinary copper
c. Stupor Ref: Harrison’s 18/e p3189, 17/e p2450
d. Coma Ref: Harrison’s 18/e p2601 148. A 12 years old girl with tremors and emotional liability has
140. Classical human hemochromatosis actually depend on the golden brown discolouration in Descemet’s membrane.
mutations of the: The most likely diagnosis is: (AI 04)
a. HJV gene a. Fabry’s disease
b. HAMP gene b. Wilson’s disease
c. TFR2 gene c. Glycogen storage disease
d. HFE gene d. Acute rheumatic fever
Ref: Harrison’s 18/e p3162 Ref: Harrison’s 18/e p3188, 17/e p2449, 2450
141. King’s College criteria for orthotopic liver transplantation 149. All of the following statements about Wilson’s disease are
in acute liver failure (paracetamol induced) include all of true, except: (AIIMS May 04)
the following except: (Comed K 2011) a. It is an autosomal recessive disorder
a. pH < 7.30 b. Serum ceruloplasmin level is < 20 mg/dL
MEDICINE
b. PT > l00s c. Urinary copper excretion is <100 mg/dL
c. Grade three encephalopathy d. Zinc acetate is effective as maintenance therapy
d. Serum Bilurubin >300 micromol/L Ref: Harrison’s 18/e p3188, 3189 Harrison’s 17/e p2450
142. In Gilbert’s syndrome, all are true except: 150. Which of the following statements regarding liver enzymes
a. Conjugated hyperbilirubinemia (DP PGMEE 2010) is true: (DNB Dec 2012)
b. Fasting hypoglycemia a. ALT is less specific indicator of liver injury than AST
c. Normal liver histology b. Absolute levels of aminotransferase correlate with
d. Liver enzymes normal outcome
Ref: Harrison’s 18/e p2533, 17/e p1929-1930 c. Glutathione-S- transferase is used as a hepatic prognostic
143 Example of intrahepatic presinusiodal portal hypertension marker after surgery
is: (J & K 2011) d. None of the above Ref: Harrison’s 18/e p2528
Ans. 135. a. HSP 136. b. > 2:1 137. c. It’s effect is... 138. c. Diabetes mellitus
139. b. Altered sleep...
140. d. HFE gene 141. d. Serum Bilurubin... 142. b. Fasting hypoglycemia
143. d. Portal vein...
144. c. Increased conjugated... 145. d. Polycythemia vera 146. c. Hepatic veins
147. b. Low serum...
148. b. Wilson’s disease 149. c. Urinary copper... 150. c. Glutathione-S-Transferase...
1142 Jaypee’s Triple A
151. Increased LDH is an important marker for: a. Aminotransaminase
a. Bulky disease (DNB 2011) b. Alkaline phosphatase
b. Lymphoma c. Bilirubin
c. Liver metastasis d. Albumin
d. Lung metastasis Ref: Internet-pubmed Ref: Harrison’s 16/e p1815
152. Indications for Liver transplantation include: (PGI 09) 156. Treatment of choice for Echinococcus granulosus is:
a. Hemochromatosis a. Albendazole (DNB 2010)
b. Primary biliary cirrhosis b. Mebendazole
c. Sclerosing cholangitis with ulcerative colitis c. Thiobendazole
d. Sclerosing cholangitis without ulcerative colitis d. Praziquantel
e. All of the above Ref: Harrison’s 18/e p2607-2608 Ref: Harrison’s 18/e p1763
153. Quantitative assessment of liver function can be done by: 157. Bland cholestasis is caused by all of the following drugs,
a. Degree of ↑ transaminases (PGI Dec 05) except: (PGI Dec-04)
b. Degree of ↑ alkaline phosphatase a. Androgen
c. Degree of ↑ GGT b. OCP
d. Estimation of galactose elimination capacity c. Cyclosporine
Ref: Textbook of Hepatology’ by Rodes 3/e p475 d. Estrogen
154. Hepatomegaly is a feature of all of the following, except: e. Chlorpromazine Ref: Robbin’s 8/e p856
a. Von Girke’s Disease (AI 2009) 158. Wilson’s disease is best diagnosed by: (DNB June 11, 10)
b. Hurler’s Disease a. Increased serum ceruloplasmin
c. Nieman Pick Disease b. Increased urinary copper excretion
d. Hepatic porphyrias Ref: Harrison’s 18/e p3172-3179 c. Decreased liver copper
155. Which one of the following serum levels would help in d. Absence of KF rings
distinguishing an acute liver disease from chronic liver Ref: Harrison’s 18/e p3189, 17/e p2450
disease? (AI 2005)
MEDICINE
Ans. 151. a. Bulky Disease 152. e. All of the Above 153. d. Estimation... 154. d. Hepatic porphyrias
155. d. Albumin...
156. a. Albendazole 157. c. Cyclosporine 158. b. Increased urinary...
7. ENDOCRINOLOGY
ENDOCRINOLOGY (QUESTIONS)
MEDICINE
20. Which of the following is true about nephrogenic diabetes a. Desmopressin
insipidus: (DNB June 2011) b. Thiazide/Amiloride diureties and salt restriction
a. Renal tubule is unresponsive to ADH c. Desmopressin and salt restriction
b. There is central decrease in secretion of ADH d. Vasopressin and salt restriction
c. Serum sodium is low Ref: Conn’s Current Therapy (Elsevier)
d. Urine osmolality is increased after administration of ADH 2012/693; Harrison’s 18/e p2906
Ref: Harrison’s 18/e p2904, 2905 27. Mainstay of treatment of neurogenic (central) diabetes
21. A 33 year old lady presents with polydipsia and polyuria. insipidus is: (DNB)
Her symptoms started soon after a road trafic accident 6 a. Desmopressin
months ago. The blood pressure is 120/80 mm Hg with b. Vasopressin
no postural drop. The daily urinary output is 6-8 liters. c. Terlipressin
Investigation showed, Na 130 mEq/L, K. 3.5 mEq/L, urea d. Amiodarone Ref: Harrison’s 18/e p2906
Ans. 14. a. Hypo-osmolar urine 15. a. Hyponatremia ... 16. d. Demeclocycline 17. a. Multiple sclerosis
18. a. 50/300 19. c. Renal... 20. a. Renal... 21. d. Psychogenic...
22. c. Water... 23. a. High... 24. d. Metabolic Acidosis 25. a. and c.
26. b. Thiazide ... 27. a. Desmopressin...
1146 Jaypee’s Triple A
28. ACTH is produced by which of the following bronchogenic 34. Hypothyroidism may be caused by: (DNB)
carcinomas: (DNB) a. Lithium
a. Adenocarcinoma b. Hematochromatosis
b. Small cell carcinoma c. Scleroderma
c. Squamous cell carcinoma d. All of the above Ref: Harrison’s 18/e p2918 Table 341-4
d. Bronchoalveolar carcinoma Ref: Harrison’s 18/e p2945 35. Which of the following conditions is associated with
29. A 6 year old boy has been complaining of headache, Hypothyroidism: (AI 2011)
ignoring to see the objects on the sides for four months. a. Hashimoto’s thyroiditis
On examination, he is not mentally retarded, his grades at b. Grave’s disease
school are good, and visual acuity is diminished in both the c. Toxic multinodular goiter
eyes. Visual charting showed significant field defect. CT d. Struma ovarii
scan of the head showed suprasellar mass with calcification. Ref: Harrison’s 18/e p2918, 17/e p2230
Which of the following is the most probable diagnosis? 36. Hypothyroidism in sub-himalayan region is due to
a. Astrocytoma (AIIMS Nov 04) deficiency of: (DNB 2010)
b. Craniopharyngioma a. Iron
c. Pituitary adenoma b. Iodine
d. Meningioma c. Copper
Ref: Harrison’s 18/e p2883, 17/e p2201, 2607 d. Selenium Ref: Park 20th/540
30. A 30-year-old male complains of loss of erection; he has low 37. Which of the following is not associated with
testosterone and high prolactin level in blood; What is the hypothyroidism: (DNB)
likely diagnosis: (AI 2001) a. Low T3
a. Pituitary adenoma b. High TSH
b. Testicular failure c. High Triglycerides
c. Craniopharyngioma d. Low cholesterol Ref: Harrison’s 18/e p2920, 17/e p2231
d. Cushing’s syndrome
38. The best marker to diagnose thyroid related disorder is:
Ref: Harrison’s 18/e p2889, 17/e p2206
a. T3 (AI 2004)
b. T4
B. DISORDERS OF THYROID GLAND c. TSH
d. Thyroglobulin Ref: Harrison’s 18/e p2917
31. In a patient suffering from thyrotoxicosis the thyroid 39. The lab investigation of patient shows ↓T3, ↓T4, and ↓TSH.
scintigraphy reveals decreased uptake the most likely It cannot be : (AI 2000)
diagnosis is: (J and K 2012) a. Primary hypothyroidism
a. Toxic adenoma b. Pan-hypopituitarism
b. Graves’ disease c. Liver disease
c. Thyroiditis d. None of the above
d. None of the above Ref: Harrison’s 18/e p2916, 2921, 17/e p2231
Ref: Harrison’s principles of internal medicine 18/e p2938 40.
The occurrence of hyperthyroidism following
32. Which of the following antibodies is involved in the tissue administration of supplemental iodine to subjects with
destructive process associated with hypothyroidism in endemic iodine deficiency goiter is known as: (AI 2012, 04)
Hashimoto’s and atrophic thyroiditis? (Comed K 2010) a. Jod-Basedow effect
a. Thyroperoxidase antibody b. Wolff-Chaikoff effect
b. Thyroglobulin antibody c. Thyrotoxicosis factitia
c. TSH receptor antibody d. De Quervain’s thyroiditis
MEDICINE
Ans. 28. b. Small cell 29 b. Craniopharyngioma 30. a. Pituitary adenoma 31. c. Thyroiditis...
32. a. Thyroperoxidase... 33. b. Propylthiouracil 34. d. All 35. a. Hashimoto’s
36. b. Iodine 37. d. Low cholesterol 38. c. TSH 39. a. Primary
40. a. Jod-Basedow... 41. b. Hypothyroidism...
Endocrinology 1147
42. All of the following are associated with Thyroid storm, 49. The following are used in thyrotoxic crisis except:
except: (AI 2002) a. Dexamethasone
a. Surgery for thyroiditis b. Propranolol
b. Surgery for thyrotoxicosis c. Lugol’s solution
c. Stressful illness in thyrotoxicosis d. Radioactive iodine
d. I131 therapy for thyrotoxicosis Ref: Harrison’s principles of internal medicine 18/e p2911
Ref: Harrison’s 18/e p2927, 17/e p2237
43. Most common cause of Thyroiditis is: (AI 2000)
a. Reidls thyroiditis
C. DISORDERS OF CALCIUM METABOLISM
b. Subacute thyroditis
c. Hashimoto’s thyroiditis 50. Asymptomatic hypercalcemia in a 30 year old young male is
d. Viral thyroiditis Ref: Harrison’s 18/e p2929, 17/e p2239 due to: (AI 2000; AIIMS June 99)
a. Occult primary malignancy
44. A patient presents with B/L proptosis, heat intolerance and
palpitations; most unlikely diagnosis here would be: b. Primary hyperparathyroidism
a. Hashimoto’s thyroiditis (AI 2001)(AI 2000) c. Familial hypocalciuria
b. Thyroid adenoma d. Hypernephroma
c. Diffuse thyroid goitre Ref: Harrison’s 17/e p2380; Harrison 18/e p3099
d. Reidel’s thyroiditis Ref: Harrison’s 18/e p2929, 17/e p2239 51. Secondary hyperparathyroidism is seen in all of the
45. Not a feature of dequervan’s disease: (AI 2002) following, except: (AIIMS Nov 2010)
a. Autoimmune in etiology a. Rickets
b. ↑ ESR b. Osteomalacia
c. Tends to regress spontaneously c. Osteoporosis
d. Painful and associated with enlargements of thyroid d. Renal failure
Ref: Harrison’s 18/e p2928, 17/e p2238
Ref: Harrison’s 17th/287; Harrison 18th/p362
46. Decreased radio iodine uptake is/are seen in (select two
52. Secondary hyperparthyroidism is seen in all of the
options): (PGI Dec 2000)
following, except: (DNB)
a. Toxic multinodular goiter
b. Grave’s disease a. Chronic renal failure
c. Subacute thyroiditis b. Parathyroid adenoma
d. Factitious thyroiditis c. Vitamin D deficiency
Ref: Harrison’s 18/e p2917, 17/e p2229 d. Medullary carcinoma thyroid
47. Needle biopsy of solitary thyroid nodule in a young woman Ref: Chandrasoma Taylar 3/e p858-865
with palpable cervical lymph nodes on the same sides 53. A 45 year old man, known case of chronic renal failure
demonstrates amyloid in stroma of lesion. Likely diagnosis develops rugger jersy spine. The probable cause is:
is: (AI 2002) a. Aluminium intoxication (AI 2000)
a. Medullary carcinoma thyroid b. Secondary hyperparathyroidism
b. Follicular carcinoma thyroid c. Osteoporosis
c. Thyroid adenoma
d. Osteomalacia Ref: Harrison 18/e p3109
d. Multi-nodular goiter Ref: Harrison’s 18/e p2938
54. Low calcium and high phosphate is seen in: (AI 2010)
48. A 26 year old woman presents with a palpable thyroid nodule,
and needle biopsy demonstrates amyloid in the stroma of the a. Hyperparathyroidism
lesion. A cervical lymph node is palpable on the same side as b. Hypoparathyroidism
MEDICINE
the lesion. The preferred treatment should be: c. Hyperthyroidism
a. Removal of the involved node, the isthmus, and the d. Hypothyroidism Ref: Harrison’s 18/e p3113, 3119, 17/e p2391
enlarged lymph node (AI 2002) 55. All of the following statements about pseudohypoparathy-
b. Removal of the involved lobe, the isthmus, a portion of roidism are true, except: (AI 2010)
the opposite lobe, and the enlarged lymph node. a. ↓ Serum PTH
c. Total thyroidectomy and modified neck dissection on the
b. ↓ Serum calclum
side of the enlarged lymph node.
c. ↑ Serum phosphate
d. Total thyroidectomy and irradiation of the cervical
d. Albright’s hereditary osteodystrophy
lymph nodes.
Ref: Harrison’s 18/e p2938 Ref: Harrison’s 18/e p3117, 17/e p2394
Ans. 42. a. Surgery for... 43. c. Hashimoto’s Thyroiditis 44. d. Reidel’s thyroiditis 45. a. Autoimmune in etiology
46. c and d 47. a. Medullary... 48. c. Total thyroidectomy... 49. d. Radioactive Iodine
50. b. Primary... 51. c. Osteoporosis 52. b. Parathyroid adenoma 53. b. Secondary
54. b. Hypoparathyroidism 55. a. ↓ Serum PTH
1148 Jaypee’s Triple A
56. Which of the following statements about Pseudohypo- b. MEN
prathyroidism is true: (AI 2011) c. Pheochromocytoma
a. Caused by ‘gain of function’ inherited mutation in Gsa d. All of the above Ref: Harrison’s 18/e p3099
subunit 65. Increased serum calcium is seen in all except: (PGI Dec 01)
b. Decreased formation of cyclic GMP is observed a. Myxedema
c. Decreased formation of Inositol triphosphate is observed b. Multiple myeloma
d. Decreased formation of c-AMP is observed c. Sarcoidosis
Ref: Harrison’s 18/e p3117, 17/e p2394 d. Primary hyperparathyroidism
57. True about pseudohypoparathyroidism: (PGI Dec 02) e. Hyperthyroidism Ref: Harrison’s 18/e p3099
a. Heterotopic calcification 66. Hypercalciuria is seen in: (PGI June 2000)
b. ↑ed Ca2+ a. Hyperparathyroidism
c. ↓ed PO4 b. Vit. D intoxication
d. ↑ed PTH c. Sarcoidosis
e. ↑ed response of urinary CAMP on PTH d. All Ref: Harrison’s 18/e p3103
Ref: Harrison’s 18/e p3117, 17/e p2394 67. A 55 year old man, a chronic smoker, is brought to
58. A patient presents with low serum calcium, high phosphorus emergency with history of polyuria, polydipsia, nausea
and elevated PTH. Which of the following investigations is and altered sensorium for last two days. He had been
least contributory to establish a diagnosis: (AI 2011) diagnosed as having squamous cell carcinoma of lung two
a. Vitamin D levels months prior to this. On examination, he was lethargic and
b. Serum creatinine levels confused. An ECG was normal except for a narrowed QT
c. Cyclic AMP response to PTH interval. Which one of the following is the most likely
d. Urine myoglobin Ref: Harrison’s 18/e p3112, 17/e p2391 metabolic abnormality? (AIIMS Nov 03)
59. Hypercalcemia is associated with all except: (AI 2009) a. Hypernatremia
a. Hyperparathyroidism b. Hypercalcemia
b. Sarcoidosis c. Hypokalemia
c. Milk alkali syndrome d. Hyponatremia Ref: Harrison’s 18/e p3099, 17/e p2380
d. Celiac disease Ref: Harrison’s 18/e p360, 17/e p2380 68. Which of the following is not a feature of hypercalcemia:
60. Causes of hypercalcemia include all of the following a. Diarrhoea (AI 06)
except: (DNB 2010) b. Polyuria
a. Multiple myeloma c. Depression
b. Lytic skeletal metastasis d. Vomiting Ref: Harrison’s 18/e p3100, 17/e p2380, 286
c. Total parenteral nutrition 69. Treatment of hypercalcemia (select two options):
d. Acute pancreatitis Ref: Harrison’s 18/e p360 3099, 3100 a. Calcitonin (PGI Dec 04)
61. Hypercalcemia is caused by all except: (PGI Dec 03) b. Gallium nitrate
a. Thyrotoxicosis c. Orthophosphate
b. Vit. D intoxication d. Thyroxin Ref: Harrison’s 18/e p3113, 17/e p2390
c. Sarcoidosis 70. All of the following are true about hypercalcemia, except:
d. Furosemide a. Management of primary cause (PGI June 04)
e. Thiazide Ref: Harrison’s 18/e p3099 b. Malignancy may cause hypercalcemia
62. Abnormalities of bone metabolism is associated with excess c. IV fluid with furosemide is given
of which vitamins (select two best options): (PGI June 02) d. Pamidronate is not effective
a. Vitamin A Ref: Harrison’s 18/e p3111, 3112, 17/e p2370
MEDICINE
b. Thiamine 71. A 75 year old lady with fracture neck of femur presents with
c. Vitamin B12 two days history of altered sensorium and decreased urinary
d. Vitamin D output. Serum calcium is 15.5 mg/dL, Urea is 140 mg/dL,
e. Tocoferol Ref: Harrison’s 18/e p3099 Creatinine is 2 mg/dL. All of the following are useful for
63. Most common cause of hypercalcemic crisis is: immediate management of hypercalcemia, except:
a. Carcinoma breast (AIIMS May 93) a. Normal saline (AIIMS Nov 2010)
b. Parathyroid hyperplasia b. Furosemide
c. Parathyroid adenoma c. Dialysis
d. Paget’s disease Ref: Harrison’s 18/e p3106, 3107 d. Bisphosphonates
Ref: Harrison’s 18/e p3112, 17/e p2390
64. Surgical causes of hypercalcemia: (PGI Dec 06)
a. Hyperparathyroidism
Ans. 56. d. Decreased 57. a. Heterotopic... 58. a. Vitamin D levels 59. d. Celiac disease...
60. d. Acute Pancreatitis... 61. d. Furosemide 62. a and d 63. a. Carcinoma breast
64. d. All of the above 65. a. Myxedema... 66. d. All 67. b. Hypercalcemia...
68. a. Diarrhoea 69. b. Gallium nitrate 70. d. Pamidronate... 71. d. Bisphosphonates...
Endocrinology 1149
72. All of the following agents may be used in the management 81. All of the following are the known causes of osteoporosis
of chronic hypocalcemia, except: (AI 2012) except: (AI 06)
a. Etidronate a. Fluorosis
b. Thiazides b. Hypogonadism
c. Elemental calcium c. Hyperthyroidism
d. Vitamin D analogs Ref: Harrison’s 18/e p3112 d. Hyperparathyroidism
Ref: Harrison’s 18/e p3124, 17/e p449, 2400
73. Raised calcium and phosphorous seen in: (PGI June 02)
a. CRF 82. Tumor lysis syndrome causes all of the following except:
b. Vitamin D intoxication a. Hypercalcemia (DNB)
c. Hyperparathyroidism b. Hyperuricemia
c. Hypermagnesemia
d. Pseudohypoparathyroidism
d. Hyperkalemia Ref: Harrison’s 18/e p2274
Ref: Harrison’s 18/e p3108, 17/e p2371
83. Features of tumor lysis syndrome are: (PGI Dec 2000)
74. Hypocalcemia with hyperphosphatemia are seen in:
a. Hypocalcemia
a. CRF (PGI Dec 2000)
b. Hypophosphatemia
b. Pseudohypoparathyroidism c. Alkalosis
c. Vit. D deficiency d. Hypokalemia Ref: Harrison’s 18/e p2274, 17/e p1736
d. Magnesium deficiency Ref: Harrison’s 18/e p3117
84. Features of tumor lysis syndrome (select three best options):
75. Hypophophatemia is seen in: (PGI Dec 02) a. Hyperuricemia (PGI June 06)
a. Pseudohypoparathyroidism b. Hypercalcemia
b. CRF c. Hyperphosphatemia
c. Rickets d. Hypernatremia
d. Hyperparathyroidism e. Hyperkalemia
e. Respiratory acidosis Ref: Harrison’s 18/e p2274
Ref: Harrison’s 18/e p3087, 17/e p2370 85. True about tumor lysis syndrome are all except:
76. Hypophosphatemia is seen in all except: (AI 2007) a. Hyperuricemia (PGI Dec 03)
a. Acute renal failure b. Hypercalcemia
b. Resolving phases of diabetic ketocidosis c. Hyperkalemia
c. Respiratory alkalosis / COPD d. Hyperphosphatemia
d. Chronic alcoholism Ref: Harrison’s 18/e p2303 e. Hypocalcemia Ref: Harrison’s 18/e p2274, 17/e p1736
77. Which of the following is not seen in Vitamin D deficiency: 86. All of the following are seen in tumor lysis syndrome,
a. Increased alkaline phosphate (PGI Dec 01) except: (AIIMS May 09; AIIMS Nov 03)
b. Decreased phosphate in urine a. Hyperkalemia
c. Hypophosphatemia b. Hypercalcemia
d. Decreased serum calcium c. Hyperuricemia
d. Hyperphosphatemia
Ref: Harrison’s 18/e p3094, 17/e p2376
Ref: Harrison’s 18/e p2274, 17/e p2380, 286, 1736
78. True about rickets: (PGI June 01)
87. Tumor lysis syndrome is associated with all of the following
a. Hyperphosphatemia
laboratory features except: (AIIMS Nov 03)
b. Hypophosphatemia
a. Hyperkalemia
c. Hypophosphaturia b. Hypercalcemia
d. Decreased alkaline phosphatase c. Hyperuricemia
Ref: Harrison’s 18/e p3094 d. Hyperphosphatemia Ref: Harrison’s 18/e p2274
79. All of the following are seen in rickets except: 88. Which of the following is not associated with diarrhea:
MEDICINE
a. Bow legs (AIIMS May 03) a. Diabetes mellitus (DNB 2011)
b. Gunstock deformity b. Hypercalcemia
c. Pot belly c. Hyperthyroidism
d. Cratio tabes d. Carcinoids
Ref: Harrison’s 18/e p3095 Ref: Harrison’s 18/e p313, 17/e p2380
80. All are causes of osteoporosis, except: (AI 2000) 89. Metabolic bone disease associated with café au lait macules.
a. Thyrotoxicosis a. Mc Cune Albright syndrome (CALM) (AP 2012)
b. Hypothyroidism b. Osteogenesis imperfecta
c. Chronic heparin therapy c. Pyknodystosis
d. Old age Ref: Harrison’s 18/e p3124, 17/e p2400 d. Paget’s disease Ref: Harrison’s 18/e p3142
Ans. 72. a. Etidronate 73. b. Vitamin D intoxication... 74. a and b 75. c and d
76. a. Acute renal... 77. b. Decreased... 78. a. Hyperphosphatemia 79. b. Gunstock deformity
80. b. Hypothyroidism... 81. a. Fluorosis... 82. a. Hypercalcemia 83. a. Hypocalcemia
84. a. Hyperuricemia... 85. b. Hypercalcemia 86. b. Hypercalcemia 87. b. Hypercalcemia
88. b. Hypercalcemia 89. a. Mc Cune
1150 Jaypee’s Triple A
90. Which of the following is the most common cause of 97. A 40-year old diabetic patient presents with proptosis of
hypergonadotrophic hypogonadism in males: (AI 2010) one eye and black eschar over palate. The likely organism
a. Viral orchitis is: (DP PGMEE 2009)
b. Klinefelter’s syndrome a. Pseudomonas
c. Kallman’s syndrome b. Candida
d. Noonan syndrome
c. E.coli
Ref: Harrison’s 18/e p3018, 17/e p2317
d. Mucor
91. Endocrinal causes of carpal tunnel syndrome include all of
Ref: Harrison’s principles of internal medicine 18/e p1661, 17/e p1261
the following, except: (AI 2009)
a. Diabetes Mellitus 98. The DCCT (Diabetes Control and Complication Trial)
b. Hypothyroidism provided definitive proof that reduction in chronic
c. Acromegaly hyperglycemia helps to improve: (Comed K 2010)
d. Addison’s disease Ref: CMDT 2008/716 a. Microvascular complications of Type 1 DM
92. Hirsuitism is caused by all, except: (AI 2009) b. Macrovascular complications of Type 1 DM
a. Cushing’s syndrome c. Microvascular complications of Type 2 DM
b. Hyperthyroidism d. Macrovascular complications of Type 2 DM
c. Hyperprolactinemia Ref: Harrison’s 18/e p2981, 17/e p2285-86, Table 338-7.
d. Acromegaly Ref: Harrison’s 18/e p381, 17/e p301
99. Which type diabetes is HLA associated: (AI 2002)
93. The most common cause of severe hypercalcemia is: a. Type I diabetes
a. Vitamin D toxicity (Comed K 2010) b. Type II diabetes
b. Sarcoidosis c. Malnutrition related type disease
c. Chronic renal failure d. Pregnancy related type diabetes
d. Malignancy Ref: Harrison’s 18/e p2973, 17/e p2279
Ref: Harrison’s 18/e p3099 17/e p
100. A 45-year old woman visited her physician with complaints
of increased appetite and thirst with increased frequency
D. DIABETES MELLITUS of urination. She also had the symptoms of diminished or
impalpable pulses in the feet, besides gangrene of the feet.
94. In the management of diabetic ketoacidosis: (J and K 2012) Her laboratory findings on the oral glucose tolerance test
a. Intracellular water deficit is best restored using half are as follows: (AI 2004)
strength saline (0.45% saline) Parameters Fasting 1 hr 2 hr
b. Potassium should be given even before checking the Blood glucose 155 270 205
serum potassium concentration (mg/dL)
c. Bicarbonate infusion is often only necessary in severe Urine glucose –ve +++ ++
acidosis pH< 7.0 Ketone bodies –ve –ve –ve
d. 5% dextrose solution should be avoided unless Which of the following statements is not correct for the
hypoglycaemia supervenes above mentioned case:
Ref: Harrison’s principles of internal medicine 18/e p2976 a. She was suffering from insulin dependent diabetes
95. In the long term management of diabetes: (J and K 2012) mellitus
a. Retinal neovascularisation should resolve with better b. She was suffering from non-insulin dependent diabetes
glycaemic control mellitus
b. Microaneurysms are usually only visible with fluorescein c. She was treated with oral hypoglycemic drugs only
angiography when diet and exercise could not control the pathological
c. Visual symptoms correlate with the severity of retinal situation
disease d. Knowledge of family history of diabetes mellitus is useful
MEDICINE
d. The development of an autonomic neuropathy confers an in predicting the nature of the diabetes.
increased risk of sudden death Ref: Harrison’s 18/e p2974, 17/e p2281
Ref: Harrison’s principles of internal medicine 18/e p3457 101. A 29 years old person is known diabetic on oral
96. Diabetes mellitus is defined as venous plasma glucose of: hypoglycemic agents since 3 years. He has lost weight and
a. Fasting ≥ 126 mg%; 2 hrs after glucose load ≥ 200 mg% never had DKa. His grandfather is diabetic but his father is
b. Fasting ≥ 100 mg%; 2 hrs after glucose load ≥ 140 mg% nondiabetic. Which is the likely diagnosis: (AI 2009)
c. Fasting ≥ 110 mg%; 2 hrs after glucose load ≥ 180 mg% a. MODY
d. None of the above b. DM type I
Ref: Harrison’s principles of internal medicine 18/e p2969, 2970 c. DM type II
d. Pancreatic diabetes Ref: Harrison’s 18/e p2968, 2976
Ans. 90. b. Klinefelter’s... 91. d. Addison’s disease 92. b. Hyperthyroidism 93. d. Malignancy
94. c. Bicarbonate. 95. b. The development... 96. a. Fasting... 97. d. Mucor
98. a. Microvascular 99. a. Type I diabetes... 100. a. She was... 101. a and c
Endocrinology 1151
102. Laboratory values in a patient with diabetic Ketoacidosis e. 2 hrs after glucose load 140-200 mg/dL; fasting blood
includes: (AP 2011) sugar < 126 mg/dL
a. Decreased Anion gap Ref: Harrison’s 18/e p2970, 17/e p2277
b. increased magnesium levels 109. At what value for one hour glucose challenge test will you
c. Normal to increased potassium levels recommend a standard glucose tolerance test:
d. Decreased phosphate levels a. 120 mg/dL (AIIMS May 05)
Ref: Harrison’s, 18/e p2976 b. 140 mg/dL
103. All of the following statements about Type I Diabetes c. 150 mg/dL
Mellitus are true, Except (Select two options): (PGI Dec 01) d. 160 mg/dL Ref: Harrison’s 18/e p2970
a. Family history is present in 90% cases 110. HBA1C level in blood explains: (AI 2004)
b. Dependent on insulin to prevent ketoacidosis a. Acute rise of sugar
c. Time of onset is usually predictable b. Long terms status of blood sugar
d. Autoimmune destruction of beta cells occur c. Hepatorenal syndrome
e. Often occurs in children d. Chronic pancreatitis
Ref: Harrison’s 18/e p2968, 2969, 17/e p2275, 2276 Ref: Harrison’s 18/e p2992, 17/e p2296
104. In a patient with NIDDM which of the following condition 111. All are seen in DKA except: (AIIMS Nov 07)
is seen: (AIIMS May 02) a. Tachycardia
a. Ketosis commonly occurs on stopping treatment. b. Dehydration
b. Hypertriglyceridimia never occurs c. Bradycardia
c. Pancreatic beta cells stop producing insulin d. Abdominal pain/tenderness
d. Increased levels of insulin in blood, may be seen Ref: Harrison’s 18/e p2976, 17/e p2283
Ref: Harrison’s 18/e p2975, 17/e p2280, 2281 112. An obese lady aged 45 years, was brought to emergency in
105. A 40 year old male patient is suffering from type II diabetes a semi comatose condition. The laboratory investigations
mellitus and hypertension. Which of the following showed K+ (5.8 mmol/L); Na+ (136 mmol/L); blood pH
antihypertensive drugs should not be used in such (7.1), HCO3 (12 mmol/L),’ ketone bodies (350 mg/dL). The
patients: (AIIMS Nov 03) expected level of blood glucose for this lady is:
a. Lisinopril a. < 45 mg/dL (AIIMS May 03)
b. Hydrochlorthiazide b. < 120 mg/dL
c. Losartan c. >180 mg/dL
d. Trandolopril d. < 75 mg/dL Ref: Harrison’s 18/e p2976, 17/e p2283, 290
Ref: Goodman Gilman 11/e p756 113. Two most important test to be done in a comatose patient
106. Which of the following is not a test for diabetes mellitus: with blood glucose of 750 mg/dL will be: (PGI June 01)
a. Fasting blood glucose (AIIMS Nov 2010) a. Serum creatinine
b. Random blood glucose b. Serum sodium
c. D-xylose test c. CSF examination
d. Oral glucose tolerance test d. Blood pH
Ref: Harrison’s 18/e p2970, 17/e p2275, 2277 e. Blood urea Ref: Harrison’s 18/e p2978, 2979
107. Which of the following findings can establish a diagnosis of 114. If a patient with severe hyperglycemia is given IV insulin,
diabetes mellitus: (AIIMS Nov 2011) which of the following can occur? (AI 2009)
a. Fasting plasma glucose 100mg/dL and 2 hour prandial a. Hypokalemia
glucose 140 mg/dL b. Hyperkalemia
b. Fasting plasma glucose 125 mg/dL and 2 hour c. Hyponatremia
postprandial glucose 199 mg/dL d. Hypernatremia
c. Symptoms of diabetes plus random blood glucose of 190 Ref: Harrison’s 18/e p2971
MEDICINE
mg/dl 115. A diabetic patient with blood glucose of 600 mg/dL and Na
d. Glycosylated haemoglobin (HbA1C) > 6.5% 122 mEq/L was treated with insulin. After giving insulin
Ref: Harrison’s 18/e p2970 the blood glucose decreased to 100 mg/dL. What changes in
108. Impaired glucose tolerance on an oral GTT is indicated by: blood Na level is expected? (AI 2002)
(PGI Dec 02) a. Increase in Na+ level
a. Fasting plasma sugar > 126 mg/dL b. Decrease in Na+ level
b. Random blood sugar > 200mg/dL c. No change would be expected
c. Fasting blood sugar < 90 mg/dL d. Na+ would return to previous level spontaneously on
d. Fasting blood sugar < 140 mg/dL; two hours after correction of blood glucose.
glucose load > 200 mg/dL Ref: Harrison’s 18/e p2977, 2976, 17/e p2283
Ans. 102. c. Normal to... 103. a. Family history ... 104. d. Increased... 105. b. Hydrochlorthiazide...
106. c. D-Xylose test....
107. d. Glycosylated... 108. e. 2 hrs after glucose... 109. b. 140 mg/dL
110. b. Long terms...
111. c. Bradycardia 112. c. >180 mg/dL 113. a and d
114. a. Hypokalemia...
115. a. Increase in Na+ level...
1152 Jaypee’s Triple A
116. Increased insulin is charachterized by all of the following, 123. The following statements are correct in relation to diabetic
except: (DNB Dec 2009) nephropathy: (PGI 2007)
a. Increased glucagon secretion a. The presence of microalbuminuria is associated with a
b. Increased intracellular potassium reduction of cardiovascular risks in patient with diabetes
c. Hypoglycemia b. Progression of renal disease may be reduced by the use of
d. Enhanced fatty acid synthesis Ref: Harrison’s 18/e p2977 calcium channel blockers
117. Early morning hyperglycemia with increased blood glucose c. The development of diabetic nephropathy protects from
of 3.00 AM suggests: (DNB) diabetic retinopathy
a. Insufficient insulin d. False positive microalbuminuria may occur in the
b. Dawn phenomenon presence of urinary tract infections
c. Somogyi effect e. Blood pressure improves in the majority after the
d. None of the above Ref: Harrison’s 18/e p2977 development of diabetic nephropathy
Ref: Harrison’s 18/e p2982, 17/e p2287, 2288
118. Which of the following statements about Dawn
phenomenon is true: (DNB) 124. Pathognomic factors involved in foot ulcers in DM include
a. Morning hyperglycemia with midnight hypoglycemia all, except: (PGI Dec 06)
b. Morning hypoglycemia with midnight hyperglycemia a. Trophic ulcers
c. Morning hyperglycemia due to insufficient insulin b. Neuropathy
d. Morning hypoglycemia due to excess insulin c. Microangiopathic changes in blood vessels
d. Macroangiopathy Ref: Harrison’s 18/e p2987, 17/e p2292
119. Which of the following are characteristic of diabetes
mellitus: (PGI June 03) 125. Life threatening complications of diabetes mellitus are all
a. Encephalopathy except: (AIIMS May 07)
b. Myelopathy a. Malignant otitis externa
c. Neuropathy b. Rhinocerebral mucormycosis
d. Myopathy c. Emphysematous pyelonephritis
e. Retinopathy Ref: Harrison’s 18/e p2980, 17/e p2285 d. Emphysematous appendicitis Ref: Harrison’s 18/e p2988
120. Hypoglycemic unawareness that occurs in diabetic patients 126. Intensive management of diabetes is needed in all except:
when transferred from oral hypoglycemics to insulin, is due (AIIMS Nov 07)
to: (AI 2000) a. Autonomic neuropathy causing postural hypotension
a. Autonomic neuropathy b. Pregnancy
b. Insulin resistance c. Post kidney transplant in diabetic nephropathy
c. Lipodystrophy d. DM with acute MI
d. Somogi phenomenon Ref: Harrison’s 18/e p2985, 17/e p2290, 2296
Ref: Harrison’s 18/e p2984, 17/e p2289, 2307 127. Recombinant human insulin is made by: (AIIMS June 2000)
121. Which of the following statements about diabetic a. cDNA from any eukaryote cell
nephropathy is true: (PGI June 05) b. Genome of any eukaryote
a. Microalbuminiuria is not an indicator of long term c. cDNA of pancreatic cell
cardiovascular morbidity d. Genome of pancreatic cell
b. Strict glycemic control cannot prevent microalbuminuria Ref: Harrison’s 18/e p2993, 17/e p390, 2297
c. β-islet cell/pancreatic transplantation can improve the 128. If a patient with severe hyperglycemia is given IV insulin,
proteinuria in early stage which of the following can occur? (DNB)
d. Angiotensin receptor blockers have no additive a. Hypokalemia
advantage over other drugs except B.P. control b. Hyperkalemia
e. Protein restriction is not helpful c. Hyponatremia
MEDICINE
Ans. 116. a. Increased 117. a. Insufficient Insulin 118. c. Morning... 119. c. and e...
120. a. Autonomic....
121. c. β-islet cell/pancreatic... 122. b. Enalapril 123. d. False positive
124. c. Microangiopathic... 125. d. Emphysematous
126. d. DM with acute MI 127. c. CDNA of pancreatic cell
128. a. Hypokalemia
129. c. Hyperuricemia
Endocrinology 1153
130. Hypoglycemia is seen in: (AIIMS May 01) 137. Which type of diabetes is HLA associated: (AI 2002)
a. Acromegaly a. Type I diabetes
b. Cushing’s syndrome b. Tyep II diabetes
c. Hyperthyroidism c. Malnutrition related type disease
d. Hypopitutarism Ref: Harrison’s 18/e p3007, 17/e p2308 d. Pregnancy related type diabetes
Ref: Harrison’s 18/e p2973
131. Hypoglycemia is a recognized feature of all of the following
conditions, except: (AI 2002) 138. Maturity-onset diabetes of the young (MODY) associated
a. Uremia with glucokinase gene defect is: (AP 2012)
a. MODY-1
b. Acromegaly
b. MODY-2
c. Addison’s disease
c. MODY-3
d. Hepatocellular failure
d. MODY-4 Ref: Harrison’s 18/e p2969; Table 344-1
Ref: Harrison’s 18/e p3003, 17/e p2305
139. Oral anti-diabetic drug of choice in renal failure is:
132. A patient presents with symptoms of hypoglycemia. a. Tolbutamide (AP 2011)
Investigations reveal decreased blood glucose and increased b. Chlorpropamide
Insulin levels. C-peptide assay is done which shows normal c. Glipizide
levels of C- peptide. The most likely diagnosis is: (AI 2010) d. Metformin Ref: Harrison’s, 18/e p2983
a. Insulinoma
b. Accidental sulfonylurea ingestion
c. Accidental exogenous Insulin administration E. DISORDERS OF ADRENAL CORTEX
d. Accidental Metformin ingestion
Ref: Harrison’s 18/e p3003, 17/e p2309 140. The following statement about adrenal gland physiology is
133. Which of the following test is useful to distinguish between true: (J and K 2012)
insulinoma and sulfonylurea related hypoglycemia: a. ACTH normally controls the adrenal secretion of
a. Antibody to Insulin (AIIMS May 08) aldosterone
b. ACTH increases adrenal androgen and cortisol secretion
b. Plasma – C- peptide level
c. The plasma cortisol concentration normally peaks in the
c. Plasma Insulin level
evening
d. Insulin- Glucose ratio Ref: Harrison’s 18/e p3067
d. Hyperglycaemia increases the rate of cortisol secretion
134. All of the following statements about Nesidioblastosis are Ref: Harrison’s 18/e p2977
true, except: (AI 2011) 141. All of the following are clinical features of Cushing’s
a. Hypoglycemic episodes may be seen syndrome except : (DNB)
b. Occurs in adults more than children a. Insulin resistance
c. Histopathology shows hyperplasia of islet cells b. Menorrhagia
d. Diazoxide may be used for treatment c. Violaceous striae
Ref: Nelson’s 18/e p1652; ‘Endocrine Tumors’ by Clarke d. Centripetal obesity
(American Cancer-Society) 2003 /162; ‘Surgical pathology of Ref: Harrison’s 18/e p2896, 2941
Endocrine and Neuroendocrine Tumors’ by Ashraf Khan 142. All of the following are true about Cushing’s syndrome,
(Springer, 2009) 1st/145; ‘Clinical Endocrinology’ by except: (PGI 08)
Grossman (Wiley-Blackwell, 1998) /538; ‘Advanced a. Red striae
Imaging of the Abdomen’ by Skucas (Springer, 2006) b. Increased adrenaline
/556; ‘Gastrointestinal Oncology’ by Jankowski, c. Proximal muscle weakness
Kerr, Fong (Blackwell, 2008) /621 d. Edema Ref: Harrison’s 18/e p2948
135. Which of the following types of neuropathy are seen in 143. All are features of Cushing’s disease except:
MEDICINE
Diabetes: (DNB 2010) a. Central obesity (AIIMS Nov 07)
a. Distal symmetric sensory neuropathy b. Episodic hypertension
b. Autonomic neuropathy c. Easy bruising
c. Mononeuropathy d. Glucose intolerance
d. All of the above Ref: Harrison’s 18/e p3457, 3458 Ref: Harrison’s 18/e p2948
136. Which of the following drugs is used for painful diabetic 144. Cushing’s disease is associated with: (AI 2008)
neuropathy: (DNB June 2011) a. Increased ACTH and increased cortisol
a. Duloxetine b. Increased urinary Catecholamines
b. Pregabalin c. Increased ADH
c. Amitriptyline d. Decreased ACTH and increased cortisol levels
d. All of the above Ref: Harrison’s 18/e p3458 Ref: Harrison’s 18/e p2945, 17/e p2255
Ans. ..145. d. ↑ ACTH secretion 146. c. Ca lung with... 147. d. Ectopic... 148. d. Ectopic....\
149. a. Loss of...
150. b. Plasma... 151. a. Cortical adenoma 152. c. Von-Hippel...\
153. d. Edema
154. d. Metabolic acidosis 155. a. Diastolic... 156. b and c
157. c, d and e
158. d. Conn’s syndrome 159. d. Tuberculosis
Endocrinology 1155
160. Feature of addison’s disease include all of the following 168. Which of the following urinary metabolites are most
except: (DNB) sensitive for diagnosis of pheochromocytomas: (DNB)
a. Asthenia a. VMA
b. Hyperpigmentation b. Metanephrines
c. Hypertension c. 5HIAA
d. Abdominal pain d. 5HTP
Ref: Harrison’s 18/e p2963
Ref: Harrison’s 18/e p2956, 2957 Table 342-9, 17/e p2263
169. All of the following are increased in pheochromocytomas,
161. Primary adrenal insufficiency causes all of the following,
except: (DNB 2012) (DNB)
except: (DNB 2012) a. Vinyl mandelic acid (VMA)
a. Low Blood Pressure b. Metanephrines
b. Decrease in ECF c. 5HIAA
c. Decreased sodium potassium ratio d. Catecholamines
d. Increased protein breakdown Ref: Harrison’s 18/e p2963
Ref: Harrison’s 18/e p2956, 2957 170. Pheochromocytomas are known to arise from all of the
162. Addisons disease is associated with all except: following, except: (PGI 09)
a. Cardiac atrophy (AIIMS May 07) a. Adrenal gland
b. Decreased diastolic B.P. b. Mediastinum
c. Serum cortisol < 3 mcg/dL c. Neck
d. Abdomen
d. Low renin levels
e. Chest wall
Ref: Harrison’s 18/e p2957, 17/e p2263, 2264
Ref: Harrison’s 18/e p2962
163. Regarding Addisonian pigmentation, all are true except: 171. All of the following are features of pheochromocytoma
a. Involves moles and scars (AIIMS May 01) except: (AI 2002)
b. Involves palmar creases a. Hypertensive paroxysm
c. Does not involve oral mucosa b. Headache
d. Decreased fibrosis c. Orthostatic hypotension
Ref: Harrison’s 18/e p2957, 2956, 17/e p2263 d. Wheezing
164. Which one of the following clinical features is NOT seen in Ref: Harrison’s 18/e p2963 Table 343-1,
pheochromocytoma? 17/e p2270-71
a. Hypertension 172. All of the following are seen in pheochromocytoma, except:
b. Episodic palpitations a. Headache (DNB 2010)
c. Weight loss b. Sweating episodes
c. Weight loss
d. Diarrhea Ref: Davidson’s, 21/e p779, box 20.50.
d. Hypocalcemia
165. 50 yr old male presents with severe refractory hypertension, Ref: Harrison’s 18/e p2963 Table 343-1
weakness, muscle cramps and hypokalemia, the most likely 173. The predominant symptom / sign of pheochromocytoma is:
diagnosis is: a. Sweating (DNB 2012)
a. Hypoaldosteronism b. Weight loss
b. Hyperaldosteronism c. Orthostatic hypotension
c. Cushings syndrome d. Episodic hypertension Ref: Harrison’s 18/e p2962
d. Pheochromocytoma Ref: Harrison’s 18/e p2950 174. The triad originally described by Zollinger Ellison
syndrome is characterized by: (AI 2002)
a. Peptic ulceration, gastric hypersecretion, non beta cell
F. ENDOCRINE TUMORS
MEDICINE
tumour
b. Peptic ulceration, gastric hypersecretion, beta cell tumour
166. Pheochromocytoma predominantly secretes: c. Peptic ulceration, achlorhydria, non beta cell tumour
a. Epinephrine (AIIMS Nov 01) d. Peptic ulceration, achlorhydria, beta cell tumour
b. Norepinephrine Ref: Harrison’s 18/e p2455, 17/e p1868
c. Dopamine 175. Gold standard test for diagnosis of Insulinoma is: ( AI 2009)
d. DOPA Ref: Harrison’s 18/e p2963, 17/e p2270 a. ‘72 hour’ fast test
167. Extra adrenal Pheochromocytoma predominantly secretes: b. Plasma Glucose levels < 3 mmol/L
a. Epinephrine (PGI June 03) c. Plasma Insulin levels > 6µU/mL
b. Norepinephrine d. C- peptide levels < 50 p mol/e
c. Dopamine Ref: Harrison’s 18/e p3066, 3067, 17/e p2354, 2355
d. DOPA Ref: Harrison’s 18/e p2963
Ans. 160. c. Hypertension 161. d. Increased protein... 162. d. Low renin levels 163. c. Does not...
164. d. Diarrhea...
165. b. Hyperaldosteronism 166. b. Norepinephrine 167. b. Norepinephrine...
168. a and b
169. c. 5HIAA 170. e. Chest wall 171. d. Wheezing...
172. d. Hypocalcemia
173. d. Episodic Hypertension 174. a. Peptic ulceration 175. a. ‘72 hour’ fast test
1156 Jaypee’s Triple A
176. Which of the following tests is not used in the diagnosis of
insulinoma: (AI 2011) G. MISCELLANEOUS
a. Fasting blood glucose
184. The most common organ involved in MEN I is:
b. Xylose test
a. Parathyroid (Comed K 2010)
c. C- peptide levels
b. Thyroid
d. Insulin/Glucose Ratio
c. Adrenal
Ref: Harrison’s 18/e p3066, 3067, 17/e p2354
d. Testis Ref: Harrison’s 18/e p3073
177. Carcinoid tumour is most common in: (AIIMS May 04)
185. Persistent priapism is rarely seen as a consequence of:
a. Esophagus
a. Sickle cell disease. (Comed K 2010)
b. Stomach
b. Leukemia.
c. Jejunum
c. Spinal cord disease.
d. Appendix Ref: Harrison’s 18/e p3058, 17/e p2349
d. Prolonged sexual activity. Ref: Harrison’s 18/e p855
178. All of the following statements about carcinoid syndrome
186. A 20 year old male presented with chronic constipation,
are true except: (DNB)
headache and palpitations. On examination he had
a. Atypical carcinoid syndrome is usually produced by
marfanoid habitus, neuromas of tongue, medullated
foregut carcinoids
corneal nerve fibers and nodule of 2x2 cm size in the left
b. Plasma serotonin levels are normal in Atypical carcinoid
lobe of thyroid gland. This patient is a case of: (AI 2004)
syndrome
a. Sporadic medullary carcinoma of thyroid
c. Midgut carcinoids have high serotonin content
b. Familial medullary carcinoma of thyroid
d. Foregut carcinoids are usually argentaffin
c. MEN IIA
Ref: Harrison’s 18/e p3062-3063
d. MEN IIB Ref: Harrison’s 18/e p3073, 3076, 17/e p2359
179. All of the following may be raised in carcinoid syndrome,
187. In MEN I, which is seen most commonly: (AI 2007)
except: (DNB 2012)
a. Insulinoma
a. 5HIAA
b. Gastrinoma
b. 5HT
c. Glucagonoma
c. 5HTP
d. Somatostatinoma
d. VMA Ref: Harrison’s 18/e p2962, 3058, 3063, 17/e p2739
Ref: Harrison 16/e p2232; Harrison 17th/2358, 2359;
180. Diagnosis of carcinoid tumors is done by: (DNB) Harrison 18/e p3072, 3073
a. 5HIAA
188. MEN I syndrome is associated with all of the following,
b. DHEA
except: (DNB 2012)
c. VMA
a. Pancreatic Tumors
d. Metanephrines Ref: Harrison’s 18/e p3058
b. Parathyroid Tumors
181. Which of the following is most likely to be raised in patients c. Pituitary Adenoma
with Atypical carcinoids: (DNB) d. Medullary Carcinoma Thyroid
a. 5 HIAA Ref: Harrison’s 18/e p3073, 3076
b. 5 HTP
189. MEN I syndrome is most commonly associated with:
c. VMA
a. Gastrinoma (AI 2008)
d. Metanephrines Ref: Harrison’s 18/e p3063, 3064
b. Insulinoma
182. All of the following about Gastrointestinal Carcinoid c. Glucucagonoma
tumors are true, except: (AI 2010) d. Somatistatinoma
a. Small intestine and appendix account for almost 60% of Ref: Harrison’s 18/e p3073, 17/e p2358, 2359
all gastrointestinal carcinoid.
190. A 36 year old female with symptoms of hyperparathyroidism,
b. Rectum is spared
MEDICINE
Ans. 176. b. Xylose test 177. d. Appendix 178. d. Foregut... 179. d. VMA
180. a. 5HIAA
181. b. 5 HTP 182. b. Rectum is spared 183. b. Tricuspid valve
184. a. Parathyroid
185. a. Sickle cell disease 186. d. MEN IIB 187. b. Gastrinoma
188. d. Medullary...
189. a. Gastrinoma 190. a. MEN 1 191. b. Islet cell hyperplasia
Endocrinology 1157
192. All of the following are features of MEN II a, except: 197. Estimation of the following hormones is useful while
a. Pituitary tumor (AI 2001) investigating a case of gynaecomastia except: (AI 2004)
b. Pheochromocytoma a. Testosterone
c. Medullary ca thyroid b. Prolactin
d. Neuromas Ref: Harrison’s 18/e p3073, 3075-3076, 17/e p2359 c. Estradiol
193. All of the following are seen in MEN II B, except: d. Lutenizing hormone
a. Hyperparathyroidism (DNB June 2011) Ref: Harrison’s 18/e p3020, 17/e p2318
b. Neuromas 198. The karyotype of patient with androgen insensitivity
c. Pheochromocytoma syndrome is: (AI 2005)
d. Medullary Carcinoma Thyroid a. 46XX
Ref: Harrison’s 18/e 3076, 17/e p2359 b. 46XY
194. Pancreatitis, pituitary tumor and pheochromocytoma may c. 47XXY
be associated with: (AI 2004, 2005) d. 45XO Ref: Harrison’s 18/e p3019, 17/e p2344
a. Medullary carcinoma of the thyroid 199. A baby girl presents with bilateral inguinal masses, thought
b. Papillary carcinoma of the thyroid to be hernias but are found to be testes in the inguinal
c. Anaplastic carcinoma of the thyroid canals. Which karyotype would you expect to find in the
d. Follicular carcinoma of the thyroid child: (AIIMS Nov 2004)
Ref: Harrison’s 18/e p3072, 3073, 17/e p2358 a. 46, XX
195. A patient presents with intermittent headache. On b. 46, XY
c. 47, XXY
examination there is hypertension and a thyroid nodule.
d. 47, XYY Ref: Harrison’s 18/e p3051, 17/e p2344
Which of the following steps is to be taken next:
a. Urine HIAA levels (AIIMS Nov 2000) 200. A 21 year old woman presents with complaints of primary
b. Urine VMA and aspiration of the thyroid nodule amenorrhoea. Her height is 153 cms, weight is 51 kg. She
c. Ultrasound abdomen has well developed breasts. She has no pubic or axillary
hair and no hirsuitism. Which of the following is the most
d. Echocardiography
probable diagnosis? (AI 2004)
Ref: Urine VMA and aspiration of the thyroid nodule
a. Turner’s syndrome
Ref: Harrison’s 18/e p3075, 17/e p2361 b. Stein-Leventhal syndrome
196. A 25 year old young woman has recurrent episodes of c. Premature ovarian failure
headache and sweating. Her mother had renal calculi d. Complete androgen insensitivity syndrome
and died after having a neck mass. The physical Ref: Harrison’s 18/e p3051, 17/e p2344
examination reveals a thyroid nodule but no clinical sign 201. A 15 year old female presents with primary amenorrhoea.
of thyrotoxicosis. Before performing thyroid surgery, the Her breasts are Tanner 4 but she has no axillary or pubic
surgeon should order: (AIIMS Nov 02) hair. The most likely diagnosis is: (AI 06)
a. Measurement of thyroid hormones. a. Turner’s syndrome
b. Serial determinations of serum calcium, phosphorus b. Mullerian agensis
protein and alkaline phosphatase. c. Testicular feminization syndrome
c. 24-hours urine test for 5 hydroxyindoleacetic acid d. Premature ovarian failure
excretion. Ref: Harrison’s 18/e p3051, 3053, 17/e p2344
d. Serial 24 hours test for catecholamines, metanephrines 202. All of the following are associated with hypergonadotrophic
and vanillylmandelic acid excretion. hypogonadism in males, except: (AI 2010)
Ref: Harrison’s 18/e p3077, 17/e p2361 a. Viral orchitis
b. Klinefelter’s syndrome
c. Kallman’s syndrome
d. Noonan syndrome
Ref: Harrison’s 18/e p3017, 3018, 17/e p2316
MEDICINE
Ans. 192. a. Pituitary tumor 193. a. Hyperparathyroidism 194. a. Medullary... 195. b. Urine VMA
196. d. Serial 24...
197. b. Prolactin 198. b. 46XY 199. b. 46, XY
200. d. Complete
201. c. Testicular 202. c. Kallman’s syndrome
8. CONNECTIVE TISSUE DISORDERS
MEDICINE
c. Anti-Sm b. Psoriasis
d. Anti-ds DNA c. Multicentric reticulohistiocytosis
e. Anti-La d. Systemic lupus erythematosus
Ref: Harrison’s 18/e p2726 Ref: Harrison’s 18/e p2728, 17/e p2077
7. Best marker for drug induced lupus is: (AI 2007) 14. In which of the following arthritis erosions are not seen:
a. Antihistone antibodies a. Rheumatoid arthritis (AIIMS May 05)
b. Anti ds DNA b. Systemic lupus erthematosus (SLE)
c. ANA c. Psoriasis
d. Anti smith Ab d. Gout
Ref: Harrison’s 18/e p2726 Ref: Harrison’s 18/e p2728, 17/e p2077
involving both her cheeks. The lesions had never been c. Hepatomegaly
erythematous. Which of the following is the most probable d. None of the above
diagnosis: (AIIMS Nov 2000) Ref: Harrison’s 18/e p2740
a. SLE 29. NOT True about Gout: (UP 2009)
b. Chloasma a. Uncommon in vegetarians
c. Air borne contact dermatitis b. Uncommon in tallotellers
d. Photo sensitive reaction c. Not found in women after menopause
22. A female developed brown macule on the cheek, forehead d. Not common in renal disorders
and nose after exposure to light following delivery of a Ref: Harison’s 18/e 2837, 2838; 17/e p2165; Apley 18/e p69
baby, the diagnosis is: (AIIMS June 2000)
Ans. 15. e. Joint deformity 16. a. SLE 17. b. Penicillin 18. c. Renal involvement
19. c. Diffuse proliferative... 20. d. Anti LA (SSB) 21. b. Chloasma 22. b. Chloasma
23. a. Bleeding disorders 24. a. Single titre of... 25. a. Pancytopenia 26. c. Thrombocytosis
27. b. Aspirin + Low... 28. b. Splenomegaly 29. c. Not found in women...
Connective Tissue Disorders 1161
30. Which biological response modifier act through CLTA-4 in c. IgD
treatment of rheumatoid arthritis: (Rajasthan 2009) d. IgE
a. Abatacept Ref: Harrison’s 18/e p2825, 17/e p2088
b. Rituximab 38. Rheumatoid factor in Rheumatoid arthritis is: (PGI Dec 03)
c. Eqratuzumab a. IgG
d. Ocrelizumab b. IgM
Ref: Harrison’s 18/e p2749-2750 c. IgA
31. Abatacept is currently approved for use in: (AP 2011) d. IgD
a. Ankylosing spondylitis e. IgE
b. Ulcerative colitis Ref: Harrison’s 18/e p2825, 17/e p2088, 2155
c. Rheumatoid arthritis 39. Rheumatoid factor in rheumatoid arthritis is important
d. Psoriatic arthritis because: (AI 2002)
Ref: Harrison’s 18/e p2749 a. RA factor is associated with bad prognosis
32. Commonest ocular manifestation of rheumatoid arthritis b. Absent RA factor rules out the diagnosis of Rheumatoid
is: (J & K 2011) arthritis
a. Dry eyes c. It is very common in childhood Rheumatoid arthritis
b. Episcleritis d. It correlates with disease activity
c. Scleromalacia Ref: Harrison’s 18/e p2746, 17/e p2088
d. Corneal melting 40. False positive rheumatoid factor can be associated with all
Ref: Harrison’s 18/e p2738 except: (AI 2009)
33. All of the following are true about rheumatoid arthritis, a. Inflammatory bowel disease
except (Select three options): (PGI 2009) b. HbsAg
a. PIP and DIP Joints involved equally c. VDRL
b. Pathology limited to articular cartilage d. Coombs test
c. Women are affected 3 times more commonly than men Ref: Harrison’s 18/e p2746, 17/e p2088
d. Rheumatoid nodules are seen in 20% of patents 41. Which of the following is the most specific test for
e. 20 percent of patients have extra articular Rheumatoid Arthritis: (AIIMS Nov 06)
Ref: Harrison’s 18/e p2738, 2739, 17/e p2083, 2085, 2087, 2086 a. Anti- ccp antibody
34. A middle aged female presents with polyarthritis, elevated b. Anti IgM antibody
Rheumatoid factor and ANA levels. Which of the following c. Anti IgA antibody
features will help in differentiating rheumatoid arthritis d. Anti IgG antibody
from SLE: (AIIMS Nov 08) Ref: Harrison’s 18/e p2746, 17/e p2088
a. Soft tissue swelling in PIP Joint 42. Rheumatoid Arthritis is best diagnosed by:
b. Juxta-articular osteoporosis on X ray a. Anti-CCP antibodies (AIIMS Nov 2011)
c. Articular erosions on X Ray b. IgA rheumatoid factor
d. Elevated ESR c. IgG rheumatoid factor
Ref: Harrison’s 18/e p2728, 17/e p2077 d. IgM rheumatoid factor
35. Which radiological feature would help differentiate Ref: Harrison’s 18/e p2745, 17/e p2088
rheumatoid arthritis with SLE? (AIIMS Nov 2008) 43. Causative agent for rheumatoid arthritis is: (AI 2008)
a. Erosion a. Mycoplasma
b. Juxta-articular osteoporosis b. Mycobacaterium avium
c. Subluxation of MCP joint c. Yersinia
MEDICINE
d. Swelling of PIP joint d. Herpes virus
Ref: Harrison’s 18/e p2728, 17/e p2079 Ref: Harrison’s 18/e p2742, 17/e p2084
36. Rheumatoid factor is a: (AI 2008) 44. Which of the following HLA subtype is most characteristi-
a. Antibody cally associated with rheumatoid arthritis:
b. Mucopolysaccharide a. HLADR1 (PGI June 01)
c. Lipoprotein b. HLADR2
d. Glycoprotien c. HLADR3
Ref: Harrison’s 18/e p2825, 17/e p2155 d. HLADR4
37. Rheumatoid factor is: (PGI Dec 01) e. HLADR5
a. IgG Ref: Harrison’s 18/e p2741, 17/e p2083
b. IgM
Ans. 30. a. Abatacept 31. c. Rheumatoid arthritis 32. a. Dry eye 33. a, b and e
34. c. Articular erosions... 35. a. Erosion 36. a and d 37. b. IgM
38. b. IgM 39. a. RA factor is associated... 40. a. Inflammatory bowel... 41. a. Anti- ccp antibody
42. a. Anti-CCP... 43. a. Mycoplasma 44. d. HLADR4
1162 Jaypee’s Triple A
45. All of the following are true about Rheumatoid Arthritis, a. Joint aspirate reveals negative birefringent crystals
except: (PGI Dec 01) b. Allopurinol should be started immediately
a. HLA-DR determine genetic susceptibility c. Colchicine is known to provide relief
b. HLA-B27 determine genetic susceptibility d. Serum uric acid levels may be normal
c. Anemia Ref: Harrison’s 18/e p2839
d. Subcutaneous nodules 53. All of the following conditions are observed in gout
e. Joint deformity except: (AIIMS May 05)
Ref: Harrison’s 18/e p2741, 2739 a. Uric acid nephrolithiasis
46. Which of the following is not true about RA? (AI 2009) b. Deficiency of enzyme Xanthine oxidase
a. Fever c. Increase in serum urate concentration
b. Rheumatoid nodules d. Renal disease involving interstitial tissues
c. Uveitis Ref: Harrison’s 18/e p2838, 3183, 17/e p2446
d. Raynaud’s phenomenon 54. A 60-year old man with diabetes mellitus presents with
Ref: Harrison’s 18/e p2741, 17/e p2739 painless, swollen right ankle joint. Radiographs of the
47. A 35 year old male patient develops involvement of proximal ankle show destroyed joint with large number of loose
and distal interphalangeal joints and Ist carpo-metacarpal bodies. The most probable diagnosis is: (AI 2003)
joints with sparing of wrist and metacarophalangeal joint. a. Charcot’s joint
The Diagnosis is: (AI 2000) b. Clutton’s joint
a. Osteoarthritis c. Osteoarthritis
b. Psoriatic arthropathy d. Rheumatoid arthritis
c. Rheumatoid arthritis Ref: Harrison’s 18/e p2855, 17/e p2180, 2181
d. Pseudogout 55. Charcot’s joint includes all of the following except:
Ref: Harrison’s 18/e p2828 a. Syringomyelia (AIIMS Nov 06)
48. Herbeden’s arthropathy affects: (AI 2005) b. Leprosy
a. Lumbar spine c. Diabetes
b. Symmetrically large joints d. Arthrogryposis multiple congenital
c. Sacroiliac joints Ref: Harrison’s 18/e p2855-2856
d. Distal interphalangeal joints 56. Which is not true of Rheumatoid arthritis? (AP 2012)
Ref: Harrison’s 18/e p2829, 17/e p2158 a. Eosinopenia
49. Gout is a disorder of: (AI 2010) b. Neutropenia
a. Purine Metabolism c. Ineffective erythropoiesis
b. Pyrimidine Metabolism d. Thrombocytosis is present
c. Ketone Metabolism Ref: Harrison’s 18/e p2738, 481
d. Protein metabolism 57. Most common systemic association of scleritis: (WB PG 08)
Ref: Harrison’s 18/e p2838, 3181, 17/e p2444 a. Rheumatoid arthritis
50. All of the following statements about primary gouty b. Ankylosing spondylitis
arthritis are true, except: (AI 2010) c. Systemic sclerosis
a. 90% of cases are caused by over production of uric acid d. Systemic lupus erythematosus
b. Uric acid levels may be normal at the time of an acute Ref: Harrison’s 18/e p229, 17/e p185; Khurana 3/e p149
attack
c. Men are more commonly affected than women (Male >
Females) B. VASCULITIS
d. Definitive diagnosis requires aspiration of synovial fluid
MEDICINE
Ref: Harrison’s 18/e p2837, 2838, 17/e p2445, 2165, 2166 58. Which of the following is a ‘small vessel vasculitis’:
a. Polyarteritis nodosa (PAN) (AI 2012)
51. All of the following are true about gout, except:
b. Microscopic polyangitis
a. Occurs due to accumulation of urate crystals in joint
c. Giant cell vasculitis
b. Can be pptd by pyrazinamide (PGI June 02)
d. Takayasu’s disease Ref: Harrison’s 18/e p2792
c. Birefringement crystals are present in joint
d. Occurs more in females 59. All of the following are small vessels vasculitis, except:
e. Due to decreased excretion of uric acid a. Classical PAN (PGI Dec 2000)
Ref: Harrison’s 18/e p2837, 17/e p2165 b. Wegner’s granulomatosis
c. HSP
52. All of the following statements about an attack of gouty
d. Churg strauss syndrome
arthritis are true, except: (DNB June 2009)
e. Microscopic polyangitis Ref: Harrison’s 18/e p2794
Ans. 45. b. HLA-B27... 46. d. Raynaud’s phenomenon 47. a. Osteoarthritis 48. d. Distal interphalangeal joints
49. a. Purine Metabolism 50. a. 90% of cases are caused... 51. d. Occurs more in females 52. b. Allopurinol should be...
53. b. Deficiency of... 54. a. Charcot’s joint 55. d. Arthrogryposis multiple... 56. a. Eosinopenia
57. a. Rheumatoid arthritis 58. b. Microscopic polyangitis 59. a. Classical PAN
Connective Tissue Disorders 1163
60. Which of the following are small vessel vasculitides (Select 68. All of the following condition are associated with
two best options): (PGI 2009) granulomatous pathology, except: (AI 2010)
a. HUS a. Wegner’s granulomatosis (WG)
b. HSP b. Takayasu arteritis (TA)
c. Kawasaki disease c. Polyarteritis nodosa (Classic PAN)
d. Churg-Strauss syndrome d. Giant cell arteritis (GCA) Ref: Harrison’s 18/e p2794
Ref: Harrison’s 18/e p2797 69. A person with involvement of upper respiratory tract,
61. ANCA is found in all of the following except: lungs and kidney shows evidence of granulomas onhisto
a. Wegner’s granulomatosis (PGI June 02) pathology. The most likely diagnosis is: (DNB)
b. Churg-Strauss disease a. Wegener’s granulomatosis
c. Microscopic polyangitis b. Goodpasture syndrome
d. Takayasu’s arteritis c. Tuberculosis
e. SLE d. Sarcoidosis Ref: Harrison’s 18/e p2789
Ref: Harrison’s 18/e p2786 70. A 20 year old woman presents with bilateral conductive
62. ANCA positive vasculitis include all of the following deafness, palpable purpura on the legs and hemoptysis.
except: (PGI Dec 06) Radiograph of the chest shows a thinwalled cavity in left
a. Wegner’s granulomatosis lower zone. Investigations reveal total leukocyte count
b. Churg Strauss syndrome 12000/mm3, red cell casts in the urine and serum creatinine
c. Microscopic PAN 3mg/dL. What is the most probable diagnosis:
d. Goodpasture’s syndrome a. Henoch-Schonlein purpura
Ref: Harrison’s 18/e p2786 b. Polyarteritis nodosa
63. ANCA is NOT associated with which of the following c. Wegener’s granulomatosis
diseases: (AIIMS Nov 2000) d. Disseminated tuberculosis Ref: Harrison’s 18/e p2790
a. Wegener’s granulomatosis 71. Wegener’s granulomatosis does not affect: (DNB Jun 2010)
b. Henoch Schonlein purpura a. Kidney
c. Microscopic PAN b. Lungs
d. Churg Strauss syndrome c. Eye
Ref: Harrison’s 18/e p2786 d. Liver Ref: Harrison’s 18/e p2790, 2789
64. C-ANCA is associated with: (PGI June 08) 72. A 30 years old male patient presents with complaints of
a. Wegener’s granulomatosis weakness in right upper and both lower limbs of last 4
b. Microscopic polyangitis months. He developed digital infarcts involving 2nd and
c. Churg- Strauss syndrome 3rd fingers on right side and 5th finger on left side. On
d. Polyarteritis nodosa (PAN) Ref: Harrison’s 18/e p2786 examination, BP was 160/140 mm Hg, all peripheral pulses
65. c-ANCA positivity indicates, antibody formed against: were palpable and there was asymmetrical neuropathy.
a. Proteinase 3 (AI 07) Investigations showed a Hb 12 gm, TLC-12000 Cu mm,
b. Myeloperoxidase Platelets 4,30,000, ESR-49 mm. Urine examination showed
c. Cytoplasmic antinuclear antibody proteinuria and RBC – 10-15/hpf with no casts. Which of the
d. Anti-centromere antibody following is the most likely diagnosis? (AI 05)
Ref: Harrison’s 18/e p2786 a. Polyarteritis nodosa
b. Systemic lupus erythematosus
66. The presence of anti-saccharomyces cerevisae antibody is a
c. Wegener’s granulomatosis
surrogate marker of one of the following: (AI 06)
d. Microscopic polyangitis
a. Coeliac disease
MEDICINE
Ref: Harrison’s 18/e p2794
b. Crohn’s disease
c. Ulcerative colitis 73. An 18 year old boy presents with digital gangrene in 3rd
d. Tropical sprue and 4th fingers for last 2 weeks. On examination the BP is
Ref: Harrison’s 18/e p2485 170/110 mm of Hg and all peripheral pulses were palpable.
Blood and Urine examination were unremarkable.
67. Antiendomysial antibody is typically seen in: (DNB)
Antinuclear antibody, Antibody to ds DNA and DNA and
a. Celiac disease
ANCA were negative. Most likely diagnosis is: (AI 2004)
b. SLE
a. Henoch-Schonlein purpura
c. Tropical sprue
b. Polyarteritis nodosa
d. Collagenous colitis
c. Wegener’s granulomatosis
Ref: Harrison’s 18/e p2471
d. Disseminated tuberculosis
Ref: Harrison’s 18/e p2794
Ans. 60. b and d 61. d. Takayasu arteritis 62. d. Goodpasture’s.... 63. b. Henoch schonlein...
64. a. Wegener’s... 65. a. Proteinase 3 66. b. Crohn’s disease 67. a. Celiac disease
68. c. Polyarteritis... 69. a. Wegener’s... 70. c. Wegener’s... 71. d. Liver
72. a. Polyarteritis nodosa 73. b. Polyarteritis nodosa
1164 Jaypee’s Triple A
74. A 30 year old male presents with numbness of both lower 80. The treatment of choice for Kawasaki disease is:
limbs and right upper limb. Examination reveals pulse a. Cyclosporine (AIIMS May 2005)
88/minutes and BP 160/110 mm of Hg. He also has digital b. Prednisolone
gangrene involving right 2nd and 3rd finger, urine routine c. Immunoglobulins
examination is unremarkable. Microscopic examination d. Methotrexate Ref: Harrison’s 18/e p2800
shows RBC’s. Hemogram and serum biochemistry is within 81. Treatment of choice for Kawasaki disease is:
normal limits. What is the most probable diagnosis? a. IV Immunoglobulins (AIIMS Nov 2011)
a. Systemic lupus erythematosus (AIIMS May 2006) b. Steroids
b. Polyarteritis nodosa c. Dapsone
c. Malignant hypertension d. Methotrexate Ref: Harrison’s 18/e p2800
d. Churg-Strauss syndrome
82. Treatment of choice for Kawasaki disease: (AIIMS May 09)
Ref: Harrison’s 18/e p2794, 17/e p2124
a. Immunoglobulins
75. A patient presents with melaena, normal renal function, b. Corticosteroids
hypertension and mononeuritis multiplex. The most c. Azathioprine
probable diagnosis is: (AIIMS Nov 04) d. Methotrexate Ref: Harrison’s 18/e p2800, 17/e 2130
a. Classical polyarteritis nodosa
83. A 24 year old male presents with abdominal pain, rashes,
b. Microscopic polyangiitis
palpable purpura and, arthritis. The most probable
c. Henoch-Schonlein purpura
diagnosis is: (AI 08)
d. Buerger’s disease
a. Henoch Schonlein purpura (HSP)
Ref: Harrison’s 18/e p2794, Table 326-6
b. Sweet syndrome
76. Which of the following is not true about Churg Strauss c. Meningococcemia
syndrome: (DNB June 2009) d. Hemochromatosis Ref: Harrison’s 18/e p2797, 17/e 2128
a. Asthma
84. All of the following are true about HSP, except:
b. Peripheral eosinophilia
a. Palpable purpura (PGI Dec 2000)
c. Vasculitis of multiple organ systems
b. Kidney’s commonly affected
d. Intravascular granulomas
c. ANCA negative
Ref: Harrison’s 18/e p2793
d. Thrombocytopenia Ref: Harrison’s 18/e p2797
77. An elderly female presents to the emergency department
85. A 20 year old girl presents with abdominal pain, arthralgia
with history of fever, headache and double vision. Biopsy
and a palpable purpuric rash all over the body. The most
of temporal artery revealed panarteritis. The most likely
likely diagnosis is: (DNB)
diagnosis is: (AIIMS Nov 09)
a. Henoch schonlein purpura (HSP)
a. Nonspecific arteritis
b. Kawasaki disease
b. Polyarteritis nodosa
c. Hemolytic uremic syndrome (HUS)
c. Wegener’s granulomatosis
d. Idiopathic thrombocytopenic purpura (ITP)
d. Temporal arteritis
Ref: Harrison’s 18/e p2797, 17/e p2128
Ref: Harrison’s 18/e p2795, 17/e p2126, 2127
86. All are true of Henoch Scholein’s purpura, except:
78. All of the following statements about temporal arteritis are
a. Thrombocytopenia (AI 2000)
true, except: (AI 2009)
b. Abdominal pain
a. More common in females
c. Arthritis
b. Worsens on exposure to heat
d. GI bleed Ref: Harrison 17/e p2128; Harrison 18/e p2797
c. Seen in elderly women
d. Can lead to sudden bilateral blindness 87. True about Henoch Schonlein purpura: (PGI June 07)
Ref: Harrison’s 18/e p2795 a. Abdominal pain
MEDICINE
Ans. 74. b. Polyarteritis nodosa 75. a. Classical polyarteritis... 76. d. Intravascular... 77. d. Temporal arteritis
78. b. Worsens on... 79. a. Associated with... 80. c. Immunoglobulins 81. a. IV Immunoglobulins
82. a. Immunoglobin 83. a. Henoch Schonlein... 84. d. Thrombocytopenia 85. a. Henoch Schonlein...
86. a. Thrombocytopenia 87. e. All of the above 88. a. Blood in stool
Connective Tissue Disorders 1165
89. A 8 year old male had non blanching rashes over the shin 97. A person presents with unilateral headache and blurring
and swelling of knee joint with haematuria +++ and protein of vision of 2 days/examination reveals a thickened cord-
+. Microscopic analysis of his renal biopsy specimen is like structure on the involved side with elevated ESR,
most likely to show: (AIIMS Nov 07) Diagnosis: (WB PG 08)
a. Tubular necrosis a. Migraine
b. Visceral podocyte fusion b. Temporal arteritis
c. Mesangial deposits of IgA c. Cluster headache
d. Basement membrane thickening d. Tension Headache
Ref: Harrison’s 18/e p2797, 17/e p2128 Ref: Harrison’s 18/e p2795
90. Henoch-Schonlein purpura is characterized by the 98. Which among the following is not a feature of PAN:
deposition of the following immunoglobulin around the a. Mononeuritis multiplex (Kerala PG 10)
vessels: (AIIMS Nov 05) b. Abdominal pain
a. IgM c. Heart block
b. IgG d. Hypertension
c. IgA Ref: Harrison’s 18/e 2794, 17/e p2125 table 319-5
d. IgE Ref: Harrison’s 18/e p2797 99. All are true about temporal arteritis except:
91. Henoch Schonlein Purpura presents with deposition of: a. Average age of onset is 70 years (Karnatka 2011)
a. IgG (DNB) b. More common in men
b. IgM c. About half of patients with untreated temporal arteritis
c. IgA develop blindness
d. IgE Ref: Harrison’s 18/e p2797 d. Steroids are the drugs of choice
92. Behcet’s syndrome is characterized by all, except: Ref: Harrison’s 18/e p2795, 17/e p2126; Kanski, 6/e p876
a. Myocarditis (PGI Dec 2000) 100. The most common leukocytoclastic vasculitis affecting
b. Erythema nodosum children is: (Feb DP PGMEE 2009)
c. Oral and genital ulcers a. Takayasu disease
d. Thrombophlebitis Ref: Harrison’s 18/e p2801 b. Mucocutaneous lymph node syndrome (Kawasaki
93. Recurrent bilateral hypopyon formation associated with disease)
thrombophlebitis is most consistent with which of the c. Henoch Schonlein purpura
following: (AI 2009) d. Polyarteritis nodosa Ref: Harison’s 18/e p2797
a. HLA B 27 associated uveitis
b. Behcet’s syndrome
c. Syphilis
C. MISCELLANEOUS
d. Herpes Zoster Ref: Harrison’s 18/e p2801
101. All of the following are associated with secondary Sjogren’s
94. Which of the following is most commonly involved in syndrome except: (Karnatka 2011)
hypersensitivity vasculitis: (AIIMS May 09) a. Rheumatoid arthritis
a. Capillaries b. Primary biliary cirrhosis
b. Arterioles c. Pheochromocytoma
c. Post-capillary venules d. Systemic lupus erythematosus
d. Medium sized arteries Ref: Harrison’s 18/e p2770; Table 324-1
Ref: Harrison’s 18/e p2798, 17/e p2128
102. Type 1 cryoglobulinemia is associated with all the following
95. Which of the following disease is/are mediated through except: (Karnataka 2011)
complement activation: (PGI Dec 03) a. Hyperviscosity
MEDICINE
a. Atopic dermatitis b. Monoclonal IgM paraprotein .
b. Graft versus host disease c. Normal complement
c. Photoallergy d. Strongly positive rheumatoid factor
d. Necrotizing vasculitis Ref: Davidson’s, 21/e p87, table 4.14
e. Urticaria Ref: Harrison’s 18/e p2784
103. What is the most common extra-articular manifestation of
96. Wegener’s granulomatous disease almost always affects: ankylosing spondylitis? (Karnataka 2010)
a. Nasal septum (MP PG 2010) a. Anterior uveitis
b. Lungs b. Aortic regurgitation
c. Kidneys c. Cataract
d. Ear d. Inflammatory bowel disease
Ref: Harrison’s 18/e p2790 Ref: Harrison’s 18/e 2775, 17/e p2111
Ans. 89. c. Mesangial deposits... 90. c. IgA 91. a. IgG 92. a. Myocarditis
93. b. Behcet’s syndrome 94. c. Post-capillary... 95. d. Necrotizing... 96. a. Nasal septum
97. b. Temporal arteritis 98. c. Heart Block 99. b. More common... 100. c. Henoch schonlein...
101. c. Pheochromocytoma 102. d. Strongly positive...
103. a. Anterior uveitis
1166 Jaypee’s Triple A
104. The most specific autoantibody associated with lupus 113. Adenoma sebaceum is seen in: (UP 2011)
erythematosus is: (Karnatka 2010) a. Scleroderma
a. dsDNA b. Tuberous sclerosis
b. PCNA c. SLE
c. Cardiolipin d. Systemic sclerosis
d. Clq Ref: Harrison’s 18/e 2726, 17/e p2076, Table 313-1, 2088 Ref: Harrison’s 18/e p410, 417
105. Leucopenia in SLE is almost always: (MHPGM-CET 2010) 114. All of the following features about generalized (diffuse)
a. Lymphopenia systemic sclerosis are true, except: (PGI June 07)
b. Neutropenia a. Raynaud’s phenomenon seen years before skin changes
c. Eosinopenia b. Trunk involvement
d. Any of tha above c. Anti centromere antibodies are characteristic
Ref: Harrison’s 18/e p2730, 17/e p2077; Table 313-3 d. Frequent systemic symptoms
106. Arthritis mutilans is due to: (Kerela PG 2008) e. All statements are true
a. Psoriatic arthritis Ref: Harrison’s 18/e p2758, 17/e p2097
b. Osteoarthritis 115. Vasanti, 28-year-old, presents with complaints of tightness
c. Rheumatoid arthritis of fingers. There is also history of dysphagia. Which of the
d. Rheumatic arthritis following is the probable diagnosis: (AIIMS Nov 2000)
Ref: Harrison’s 18/e p2780-2781, 16/e p1998; 17/e p2115 a. Dermatomyositis
107. Bilateral acoustic neuroma is seen in: (Kerela PG 2008) b. Scleroderma
a. Type 1 neurofibromatosis c. Rheumatoid arthritis
b. Type 2 neurofibromatosis d. Polyarteritis nodosa
c. Tuberous sclerosis Ref: Harrison’s 18/e p2763, 17/e p2101, 2102
d. Von Hippel Lindau syndrome 116. Features of systemic sclerosis is include all of the following,
Ref: Harrison’s 16/e p1998; 18/e p3389, 17/e p2607 except: (PGI June 03)
108. In pseudogout, material deposited is: (Kerala PG 09) a. Calcinosis
a. Sodium oxalate b. Sclerodactyly
b. Calcium apatite c. Hyperpigmentation (melanin deposition)
c. Monosodium urate d. More common in women
d. Calcium pyrophosphate e. More common in young patients
Ref: Harrison’s 18/e p2839, 17/e p2167 Ref: Harrison’s 18/e p2758
109. What is not found in carcinoid syndrome: (WB PG 08) 117. A 14 year old girl on exposure to cold has pallor of
a. Increased VMA extremities followed by pain and cyanosis. In later stages of
b. Flushing life she is most prone to develop: (AIIMS Nov 08)
c. Wheeze a. SLE
d. Bronchospasm b. Scleroderma
Ref: Harrison’s 18/e p3061, 3062, 17/e p2351-2352 c. Rheumatoid arthritis
d. Dermatomyositis
110. Migratory necrolytic erythema is seen in: (MP PG 2010)
Ref: Harrison’s 18/e p2763, 17/e p2096, 2101
a. Glucagonoma syndrome
b. Peutz Jeghers syndrome 118. Following cranial nerve is most commonly involved in
c. Sarcoidosis patients with sarcoidosis: (AIIMS Nov 02)
d. Amyloidosis Ref: Harrison’s 18/e p3073 a. II cranial nerve
b. III cranial nerve
111. Tuberculides are seen in:
c. VII cranial nerve
MEDICINE
a. Lupus vulgaris
d. IX cranial nerve
b. Scrofuloderma
Ref: Harrison’s 18/e p2809, 17/e p2139
c. Lichen scrofulosorum
d. Erythema nodosum 119. Bacillary angiomatosis not true: (AP 2011)
Ref: Behl 10/e p202; Neena Khanna 3/e p219; Dorland 218/e p1756 a. Caused by henslae
b. Can cause Peliosis hepatis
112. Tuberous sclerosis is characterized by all except:
c. Aminoglycosisdes useful
a. Multiple renal cyst
d. Brain is involved in AIDS patient/associated with
b. Arachnoid cyst
cerebral involvement in HIV
c. Renal angiolipoma
Ref: Harrison’s 18/e p1318, 1319
d. Renal cell carcinoma
Ref: CMDT-10-903; Harison’s 18/e p2360, 3390, 17/e p1800, 2607
Ans. 104. a. dsDNA 105. a. Lymphopenia 106. a. Psoriatic arthritis 107. b. Type 2...
108. d. Calcium...
109. a. Increased VMA 110. a. Glucagonoma... 111. c. Lichen scrofulosorum
112. b. Arachnoid cyst
113. b. Tuberous sclerosis 114. c. Anti centromere... 115. b. Scleroderma
116. e. More common in... 117. b. Scleroderma
118. c. VII Cranial nerve 119. c. Aminoglycosisdes useful
Connective Tissue Disorders 1167
120. All are seronegative (spondyloepiphyseal) arthritis with 128. Keratoderma blenorrhagica is typically seen in:
ocular manifestations, except: (AIIMS Nov 01) a. Rheumatoid arthritis (DNB June 2010)
a. Ankylosing spondylitis b. Psoriatic arthritis
b. Reiter’s disease c. Reactive arthritis
c. Rheumatoid arthritis d. Ankylosing spondylitis Ref: Harrison’s 18/e p2779
d. Psoriatic arthritis 129. En-coup-de sabre is a form of: (Comed K 2010)
Ref: Harrison’s 18/e p2746, 17/e p22109 a. Scleroderma
121. HLA-B27 is typically associated with: (AIIMS Nov 09) b. Syphilis
a. Rheumatoid Arthritis c. Lupus erythematosis
b. Ankylosing spondylitis d. Alopecia
c. Sjogren’s syndrome 130. Which of the following statements about fibromyalgia is
d. Scleroderma true: (AI 2012)
Ref: Harrison’s 18/e p2774 a. More common in males
122. Features of seronegative spondyloarthropathy include all of b. Sleep EEG studies have shown disruption in REM sleep
the following, except: (PGI June 01) c. Increased cortisol response to stress
a. Strong association with HLA B27 d. SPECT studies have reduced blood flow to the thalamus
b. Negative rheumatoid factor Ref: Harrison’s 18/e p2849-2851, 17/e p2180
c. Symmetrical polyarthritis 131. A young, tall, thin, male with archnodactyly has ectopia
d. Enthesitis lentis in both eyes. The most likely diagnosis is:
e. Extraarticular feature including uveitis a. Marfan’s syndrome (AIIMS Nov 05)
Ref: Harrison’s 18/e p2738 b. Marchesani’s syndrome
123. All are seronegative (spondyloepiphyseal) arthritis with c. Homocystinuria
ocular manifestations, except: (AIIMS Nov 01) d. Ehler’s danlos syndrome
a. Ankylospondilitis Ref: Harrison’s 18/e p3212, 17/e p2468-2469
b. Reiter’s disease 132. Most common cause of death in primary amyloidosis is
c. Rheumatoid arthritis a. Respiratory failure (DNB)
d. Psoriatic arthritis Ref: Harrison’s 18/e p2746, 17/e p2088 b. Renal failure
124. All the following diseases are associated with HLA B-27 c. Cardiac failure
and Uveitis, except: (AI 2000) d. Septicemia Ref: Harrison’s 18/e p948
a. Behcet’s syndrome 133. True about Raynaud’s disease are all except: (AI 2007)
b. Psoriasis a. More common in females
c. Ankylosing spondylitis b. Good prognosis
d. Reiter’s syndrome c. Positive antinuclear phenomenon
Ref: Harrison’s 18/e p2801, 17/e p2132 d. Most common cause of Raynaud’s phenomenon
125. A patient presents with foreign body sensation in eye and Ref: Harrison’s 18/e p2071, 17/e p1572
swollen knee joint after a leisure trip. The most probable 134. A 10-year old boy presents to the pediatric emergency
diagnosis is: (AI 2009) unit with seizures. Blood pressure in the upper extremity
a. Sarcoidosis measured as 200/140 mm Hg. Femoral pulses were not
b. Reiter’s disease palpable. The most likely diagnosis amongst the following
c. Behcet’s disease is: (AI 2010)
d. SLE Ref: Harrison’s 18/e p2779, 17/e p2113, 2114 a. Takayasu’s aortoarteritis
126. What is not seen in Reiters syndrome: (AIIMS Nov 08) b. Renal parenchymal disease
MEDICINE
a. Subcutaneous nodules c. Grand mal seizures
b. Keratoderma blennorrhagicum d. Coarctation of aorta Ref: Harrison’s 18/e p1925
c. Circinate balanitis 135. Renal artery stenosis may occur in all of the following,
d. Oral ulcers Ref: Harrison’s 18/e p2779, 17/e p2114 except: (AI 06)
127. Which of the organisms most commonly causes reactive a. Atherosclerosis
arthritis? (AIIMS Nov 08) b. Fibromuscular dysplasia
a. Ureaplasma urealyticum c. Takayasu’s arteritis
b. Group A beta hemolytic streptococci d. Polyarteritis nodosa
c. Borrelia burggorferi Ref: Harrison’s 18/e p2794
d. Chlamydia Ref: Harrison’s 18/e p2778, 17/e p2113
Ans. 120. c. Rheumatoid arthritis 121. b. Ankylosing spondylitis 122. c. Symmetrical... 123. c. Rheumatoid arthritis
124. a. Behcet’s syndrome 125. b. Reiter’s disease
126. a. Subcutaneous nodules 127. d. Chlamydia
128. c. Reactive Arthritis
129. a. Scleroderma 130. d. SPECT studies have... 131. a. Marfan’s syndrome
132. c. Cardiac Failure
133. c. Positive antinuclear... 134. d. Coarctation of Aorta 135. d. Polyarteritis nodosa
1168 Jaypee’s Triple A
136. What is the best method for confirming amyloidosis: 140. Immunoglobulins infusion is indicated in all except:
a. Colonoscopy (AI 2007) a. Wiskott aldrich syndrome (PGI June 07)
b. Sigmoidoscopy b. X-linked aggamaglobulinemia
c. Rectal biopsy c. Kawasaki disease
d. Tongue biopsy d. PAN Ref: Harrison’s 18/e p2795
Ref: Harrison’s 18/e p945, 17/e p2145 141. IV Immunoglobulins are indicated in all except:
137. Select two medical conditions for IV immunoglobulin a. Kawasaki disease (PGI June 03)
infusions: (PGI June 02) b. PAN
a. Myesthenia gravis c. GBS
b. Idiopathic thrombocytopenic purpura d. Bruton’s hypogammaglobulinemia
c. Takayasu arteritis Ref: Harrison’s 18/e p2795
d. Hemolytic uremic syndrome 142. Plasmapharesis is used in all of the following except:
e. Multiple myeloma a. Myaesthenic crisis (AI 2010)
Ref: Harrison’s 18/e p3485 b. Cholinergic crisis
138. Intravenous immunoglobulin is given in: (select three c. Gullian Barre syndrome
options): (PGI June 04) d. Polymyositis
a. Kawasaki disease Ref: Harrison’s 18/e p3485, 3477, 3517
b. GB syndrome 143. A woman is admitted with complain of low-grade fever
c. Heart block of 6 weeks duration. Chest radiograph reveals bihilar
d. Atrial fibrillation adenopathy with clear lung fields. All of the following
e. Myasthenia gravis investigations will be useful in differential diagnosis
Ref: Harrison’s 18/e p2684, 17/e p2044, 2059 except: (AI 2004)
139. Immunoglobulins used in Rx of: a. CD4/CD8 counts in the blood
a. Myasthenia gravis (PGI Dec 06) b. Serum ACE levels
b. I.T.P c. CECT of chest
c. PAN d. Gallium scan Ref: Harrison’s 18/e p2801, 2811
d. Wegener’s granulomatosis
e. Guillian Barre syndrome
Ref: Harrison’s 18/e p887, 888, 17/e p663
MEDICINE
Ans. 136. c. Rectal biopsy 137. a and b 138. a, b and e 139. a and b
140. d. PAN
141. b. PAN 142. b. Cholinergic crisis 143. a. CD4/CD8 counts...
9. NERVOUS SYSTEM
A. SEIZURES AND EPILEPSY 7. A 64 year old lady Kamla complains of severe unilateral
headache on the right side and blindness for 2 days. On
1. Cluster headache is characterized by all, except: (AI 2005) examination there is a thick cord like structure on the lateral
a. Affects predominantly females side of the head. The ESR is 80 mm/hr in the first hour. The
b. Unilateral headache most likely diagnosis is: (AIIMS May 01)
c. Onset typically in 20-50 years of life a. Temporal arteritis
d. Associated with conjunctival congestion b. Migraine
Ref: Harrison’s 18/e p122, 17/e p101 c. Cluster headache
d. Sinusitis Ref: Harrison’s 18/e p114, 17/e p96
2. A female has episodic, recurrent headache in left
hemicranium with nausea and parasthesia on right upper 8. A young girl presents with repeated episodes of throbbing
and lower limbs is most probably suffering from: occipital headache associated with ataxia and vertigo. The
a. Migraine (AIIMS June 2000) family history is positive for similar headaches in her
b. Glossopharyngeal neuralgia mother. Most likely diagnosis is: (AI 2011)
c. Herpes zoster infection of trigeminal Nerve a. Vestibular neuronitis
d. Brain tumour b. Basilar migraine
Ref: Harrison’s 18/e p114,115, 17/e p96, 97 c. Cluster headache
d. Tension headache
3. A female aged 30, presents with episodic throbbing
Ref: ‘Current Diagnosis and Treatment in Neurology’ (LANGE) 1st/66;
headache for past 4 yrs. It usually involves one half of the
Pediatric ENT (Springer, 2008) /473; ‘Vertigo and Disequilibrium:
face and is associated with nausea and vomiting. There is
A practical guide to diagnosis and management’ by Weber (Thieme);
no aura. Most likely diagnosis is: (AI 2001)
‘Handbook of Headache (Lippincott- Williams)’ 2nd/214
a. Migraine
b. Cluster headache 9. All of the following statements about treatment of migraine
c. Angle closure glaucoma are true, except: (DNB 2012)
d. Temporal arteritis a. Narotriptan has slower onset and longer t1/2 than
Ref: Harrison’s 18/e p114,115, 17/e p96,97 sumatriptan
b. Rizatriptan is more efficacious than sumatriptan
4. A woman complains of headache associated with
c. Sumatriptan is a selective 5-HT 1B/1D agonist
paresthesias of the right upper and lower limb, likely
d. Sumatriptan is used for chronic migraine
diagnosis is: (AI 2001)
Ref: Harrison 18/e p118, 119
a. Trigeminal neuralgia
b. Glossopharyngeal neuralgia 10. All of the following agents are used for prophylaxis of
c. Migraine migraine, except: (AI 2010)
d. Cluster headache Ref: Harrison’s 18/e p114, 17/e p96, 97 a. Propanalol
b. Valproate
5. Ophthalmoplegic migraine means: (AIIMS May 03)
c. Topiramate
a. When headache is followed by complete paralysis of the
d. Ethosuxamide Ref: Harrison’s 18/e p121, 17/e p102
IIIrd and VI nerve on the same side as the hemicrania.
b. When the headache is followed by partial paralysis of the 11. Absence seizures are characterized on EEG by: (AI 2003)
IIIrd nerve on the same side as the hemicrania with out a. 3 Hz spike and wave
any scotoma. b. 1-2 Hz spike and wave
c. Headache associated with IIIrd, IVth and VIth nerve c. Generalized poly spikes
d. Hypsarrythmia Ref: Harrison’s 18/e p3252
MEDICINE
paralysis.
d. Headache associated with optic neuritis 12. A child presents with short episodes of vacant stare several
Ref: Parson 19/e p147, 18/e p269 times a day. The vacant episode begins abruptly and the
6. A woman has bilateral headache that worsens with child remains unresponsive during the episode. There is
emotional stress; she has two children, both doing badly in no associated history of aura or postictal confusion and the
school; diagnosis is: (AI 2001) child is otherwise normal: (AI 2010)
a. Migraine The likely diagnosis is
b. Cluster headache a. Grandmal seizures
c. Tension headache b. Absence seizures
d. Trigeminal neuralgia c. Complex partial seizures
Ref: Harrison’s 18/e p121,122, 17/e p100,101 d. Day dreaming Ref: Harrison’s 18/e p3252
MEDICINE
than: (AI 2012)
b. Early age of onset
a. 10 ml/100g/minute
c. Developmental abnormalities
b. 20 ml/100g/minute
d. Positive family history of epilepsy
Ref: Harrison’s 18/e p3256 c. 40 ml/100g/minute
d. 50 ml/100g/minute Ref: Harrison’s 18/e p3271
20. All of the following drugs are used for managing status
epilepticus except: (AIIMS May 06), (AIIMS Nov 04) 27. The most common location of hypertensive intracranial
a. Phenytoin hemorrhage is: (AI 2006)
b. Diazepam a. Subarachnoid space
c. Thiopentone sodium b. Basal ganglia
d. Carbamazepine c. Cerebellum
Ref: Harrison’s 18/e p3262, 17/e p2511 d. Brainstem
Ref: Harrison’s 18/e p3270, 3294, 17/e p2531, 2532
Ans. 13. c. Postictal confusion 14. c. Automatism 15. a and c 16. c. Carbamezapine
17. c. Within ... 18. c. Subtle 19. b. Early age of onset... 20. d. Carbamazepine
21. c. Absence seizures... 22. a. Valproate 23. d. Lorazepam 24. d. Lymphadenopathy
25. c. Ophthalmoplegia 26. b. 20 ml/100g/minute 27. b. Basal ganglia
1172 Jaypee’s Triple A
28. Which of the following is the most common location of 36. A 45 years old hypertensive male presented with sudden
hypertensive hemorrhage? (AI 1994, 2003) onset severe headache, vomiting and neck stiffness. On
a. Pons examination he didn’t have any focal neurological deficit.
b. Thalamus His CT scan showed blood in the Sylvain fissure. The
c. Putamen/external capsule probable diagnosis is: (AIIMS May 03)
d. Subcortical white matter a. Meningitis
Ref: Harrison’s 18/e p3294, 17e/p2531,2532 b. Ruptured aneurysm
29. The most common site for hypertensive bleed is? c. Hypertensive bleed.
a. Pons (DNB Dec 2010) d. Stroke Ref: Harrison’s 18/e p3296, 17/e p1727
b. Putamen 37. All of the following are true about anterior choroidal artery
c. Frontal lobe syndrome except: (AI 2011)
d. Thalamus Ref: Harrison’s 18/e p3294, 17e/p2531,2532 a. Hemiparesis
30. Which of the following is the most common location of b. Hemisensory loss
hypertensive hemorrhage? (AIIMS Nov 02) c. Homonymous Hemianopia
a. Pons d. Involvement of anterior limb of internal capsule
b. Thalamus Ref: Harrison’s 18/e p3285, 17/e p2524
c. Putamen/external capsule 38. Which of the following is not a usual feature of right middle
d. Cerebellum Ref: Harrison’s 18/e p3294, 17/e p2531,2532 cerebral artery territory infarct: (AIIMS Nov 02)
31. The most common location of hypertensive intracranial a. Aphasia. (AIIMS May 03)
hemorrhage is: (AI 2006) b. Hemiparesis.
a. Subarachnoid space c. Facial weakness.
b. Basal ganglia d. Dysarthria. Ref: Harrison’s 18/e p3284, 17/e p2523, 2524
c. Cerebellum
39. Which of the following sites is not involved in a posterior
d. Brainstem Ref: Repeat; 17e/p2531, 2532; 18/e/p3294
cerebral artery infarct: (AI 2011)
32. The most common intracranial site of hypertensive a. Midbrain
haemorrhage is: (AIIMS May 07) b. Thalamus
a. Basal ganglia
c. Temporal lobe
b. Brainstem
d. Anterior Cortex
c. Cerebellum
Ref: Harrison’s 18/e p3286, 3287, 17/e p2525
d. Hippocampus
40. A hypertensive individual had a sudden headache and
33. Berry aneurysm is caused by: (AIIMS May 2011)
became unconscious within a few minutes. On regaining
a. Degeneration of internal elastic lamina
b. Degeneration of tunica media consciousness, there was complete flaccid hemiplegia with
c. Degeneration of muscular layer no involvement of upper face, absence of tendon reflexes
d. Degeneration of external elastic lamina and a positive Babinski sign. Which one of the following
Ref: Harrison’s 18/e p2262, 17/e p1727 arteries could have ruptured: (AIIMS Nov 03)
a. Lateral striate branch of middle cerebral
34. A patient known to have mitral stenosis and atrial
fibrillation presents with acute onset of weakness in the left b. Medial striate branch of anterior cerebral
upper limb which recovered completely in two weeks. The c. Posterolateral branch from posterior cerebral
most likely diagnosis is: (AI 2010) d. Posterior choroidal branch of posterior cerebral
a. Transient ischemic attack Ref: Harrison’s 18/e p3294, 17/e p2532
b. Ischemic stroke 41. Lacunar infarcts are caused by: (AI 2001)
c. Hemorrhagic stroke a. Lipohyalinosis of penetrating arteries
MEDICINE
d. Vasculitis Ref: Harrison’s 18/e p3274, 17/e p2516 b. Middle carotid artery involvement
35. ‘Duret Hemorrhages’ are seen in: (AIIMS May 08) c. Emboli to anterior circulation
a. Brain d. None of the above
b. Kidney Ref: Harrison’s 18/e p3276, 17/e p2519
c. Heart 42. Which is true about carotid stenosis: (PGI June 2000)
d. Lung a. Ipsilateral hemiplegia by embolism of MCA
Ref:‘Handbook of Clinical Neurology’ by Young (Elsevier) /85; b. Bruit indicates severity of stenosis
’Fundamentals of Diagnostic Radiology’ by Brant and Helms 3rd/73; c. Common in external carotid artery
‘Vascular Neurology: Questions and Answers’ (2008) /Q 479 p-266 d. Aspirin reduces risk of TIA
Ref: Harrison’s 18/e p3283, 17/e p2517, 2520, 2523
Ans. 28. c. Putamen/external... 29. b. Putamen 30. c. Putamen/external... 31. b. Basal ganglia
32. a. Basal ganglia 33. a. Degeneration of... 34. b. Ischemic stroke 35. a. Brain
36. b. Ruptured... 37. d. Involvement... 38. a. Aphasia 39. d. Anterior Cortex...
40. a. Lateral striate... 41. a. Lipohyalinosis... 42. d. Aspirin reduces ...
Nervous System 1173
43. Investigation of choice for screening of proximal internal 51. Pontine Stroke is associated with all except: (AI 2007)
carotid artery stenosis is: (AIIMS June 2000) a. Bilateral pin point pupil
a. Doppler flow USG b. Pyrexia
b. CT substraction angiography c. Vagal palsy
c. MRI d. Quadriparesis
d. Angiography (DSA) Ref: Harrison’s 18/e p3276 Ref: Harrison 16/e 1630, 239; Harrison’s 18/e
44. Lesion of posterior inferior cerebellar artery at brain p3290, 3291, 3292, 17/e p2528, 2529, 2530;
involves/affects: (PGI Dec 03) 52. The only thrombolytic agent approved for the treatment of
a. Spinal tract of trigeminal nerve acute ischemic stroke is: (AIIMS May 04, Nov 03)
b. Tractus Solitarus a. Tissue plasminogen activator
c. Spinothalamic tract b. Streptokinase
d. Corticospinal tract Ref: Harrison’s 18/e p3288, 17/e p2383 c. Urokinase
45. Abdul Khan presents with pain, numbness and impaired d. Pro-urokinase Ref: Harrison’s 18/e p3272, 17/e p2515
sensation over half of the face along with ataxia, nystagmus, 53. Which one of the following agents has been associated with
dysphagia and hoarseness of voice. His pain and thermal hemorrhagic stroke? (AI 2006)
sensations over opposite half are impaired). Horner’s a. Phenylpropanolamine
syndrome is present. Likely cause of the disease is b. Terfenadine
thrombosis of which vessel? (AIIMS Nov 01) c. Quinidine
a. AICA (Anterior inferior cerebellar artery) d. Fenfluramine Ref: KDT 5th/115
b. PICA (Posterior inferior cerebellar artery)
54. Which of the following is not involved in Wernicke’s
c. Basilar
d. Pontine vessels Ref: Harrison’s 18/e p3288, 17/e p2526 Korsakoff psychosis: (AI 2012)
a. Mamillary body
46. Clinical features of medial medullary syndrome (select two
b. Thalamus
options):
c. Periventricular Grey matter
a. Ipsilateral numbness of arm, trunk (PGI June 05)
d. Hippocampus Ref: Harrison’s 18/e p2260
b. Horner’s syndrome
c. Ipsilateral 12th nerve palsy 55. Which of the following sites is responsible for the amnestic
d. Contralateral pyramidal tract sign defect in Wernicke’s Korsakoff syndrome: (AI 2012)
Ref: Harrison’s 18/e p3288, 17/e p2383 a. Mamillary body
b. Thalamus
47. Which of the following is a features of Medial Medullary
c. Periventricular Grey matter
Syndrome, except: (PGI June 07)
a. Ipsilateral numbness of face and trunk d. Hippocampus Ref: Harrison’s 18/e p2260
b. Horner’s syndrome
c. Ipsilateral ataxia
d. Contralateral paralysis
C. DISORDERS OF SPINAL CORD
Ref: Harrison’s 18/e p3288, 17/e p2383
56. Fasciculation is seen in: (PGI Dec 03)
48. All of the following are true about Weber’s syndrome.
a. UMN type of lesion
except: (DNB 09, 12)
b. LMN type of lesions
a. Ipsilateral occulomotor nerve palsy
c. Myoneural junction
b. Diplopia
d. Peripheral neuropathy
c. Contralateral hemiplagia
Ref: Harrison’s 18/e p182, 17/e p148
d. Ipsilateral facial nerve palsy Ref: Harrison’s 18/e p3287
57. Bulbar paralysis refers to: (PGI Dec 03)
MEDICINE
49. Benedict’s syndrome, all are true except: (AI 2007)
a. LMN lesion
a. Contralateral tremor
b. UMN lesion
b. 3rd nerve palsy
c. Paralysis of cranial nerve IX to XII
c. Involvement of the penetrating branch of the basilar artery
d. Paralysis of cranial nerve III to XII
d. Lesion at the level of the pons
Ref: Harrison’s 18/e p182
Ref: Harrison’s 18/e p238, 17/e p193
58. Which of the following represents the site of lesion in
50. Millard Gubler syndrome includes the following except:
motor neuron disease: (AI 2010)
a. 5th nerve palsy (AI 2007)
a. Anterior horn cells
b. 6th nerve palsy b. Peripheral nerve
c. 7th nerve palsy c. Spinothalamic tract
d. Contralateral hemiparesis d. Spinocerebellar tract
Ref: Harrison’s 18/e p239, 17/e p193 Ref: Harrison’s 18/e p182, 17/e p148, 2572, 2573
Ans. 43. a. Doppler flow USG 44. a, b and c 45. b. PICA... 46. b and c
47. d. Contralateral... 48. d. Ipsilateral facial... 49. d. Lesion at ... 50. a. 5th nerve palsy
51. c. Vagal palsy 52. a. Tissue... 53. a. Phenylpropanolamine 54. d. Hippocampus
55. b. Thalamus 56. b. LMN type ... 57. a. LMN lesion ... 58. a. Anterior Horn...
1174 Jaypee’s Triple A
59. Amyotrophic lateral sclerosis involve: (PGI June 03) a. Ipsilateral loss of temperature
a. Anterior horn cell b. Contralateral loss of pain
b. Posterior horn cell c. Contralateral loss of vibration
c. Dorsal root ganglia d. Bilateral segmental signs
d. Ventral root ganglia Ref: Harrison’s 18/e p3367, 17/e p2589
e. Myoneural junction 67. Which of the following is not a feature of extramedullary
Ref: Harrison’s 18/e p3345, 17/e p2572 tumour: (AI 2008)
60. A middle aged man presents with progressive atrophy a. Early corticospinal signs and paralysis
and weakness of hands and forearms. On examination he b. Root pain or midine back pain
is found to have slight spasticity of the legs, generalized c. Abnormal CSF
hyper-reflexia and increased signal in the cortico-spinal d. Sacral sparing
tracts on T2 weighted MRI. The most likely diagnosis is: Ref: Harrison’s 18/e p3367, 17/e p2589, 2590
a. Multiple sclerosis (AIIMS Nov 04) 68. Clinical features of conus medullaris syndrome include all
b. Amyotrophic lateral sclerosis of the following except: (AI 2008)
c. Subacute combined degeneration a. Plantar extensor response
d. Progressive spinal muscular atrophy b. Absent knee and ankle jerks
Ref: Harrison’s 18/e p3345 c. Sacral anesthesia
61. Which of the following sensations are transmitted by the d. Lower sacral and coccygeal involvement
Dorsal Tract/Posterior column: (AI 2008) Ref: Harrison’s 18/e p3367
a. Fine touch 69. Clinical Features of conus medullaris syndrome include all
b. Pain of the following except (DNB 2012)
c. Temperature a. Extensor plantar Reflex
d. All of the above Ref: Harrison’s 18/e p3238 b. Saddle anesthesia
62. Which of the following is not affected by a lesion in c. Late bladder involvement
posterior column of spinal cord: (PGI Jun 05, 07) d. Bilateral and usually symmetric sensory loss
a. Romberg’s sign Ref: Harrison’s 18/e p3367
b. Temperature sense 70. Early loss of bladder control is seen in (DNB 2012)
c. Vibration sense a. Conus medullaris syndrome
d. Ataxia Ref: Harrison’s 18/e p3238 b. Cauda equina syndrome
63. A ventrolateral cordotomy is performed to produce relief of c. Gullain barre syndrome
pain from the right leg. It is effective because it interrupts d. Amyotrophic Lateral Sclerosis
the: (AI 2012) Ref: Harrison’s 18/e p3367
a. Left Dorsal Column 71. Hypotension in acute spinal cord injury is due to:
b. Left Lateral Spinothalamic tract a. Loss of sympathetic tone (AIIMS Nov 06)
c. Right Lateral Spinothalamic tract b. Loss of parasympathetic tone
d. Right Corticospinal tract c. Vasovagal attack
Ref: Ganong 23/e p179 (Q1); ‘Bonica’s Management of Pain’ 4/e pChapter d. Orthostatic hypotension
‘Neurosurgical Operations on the Spinal Cord’ Ref: Harrison’s 18/e p3366, 17/e p2588, 2589
64. All of the following are true about Brown Sequard 72. Spinal shock is characterized by (select three best options):
Syndrome, except: (AIIMS Nov 2011) a. Spasticity (PGI June 03)
a. Ipsilateral Pyramidal Tract Features b. Wasting
b. Contralateral Spinothalamic Tract Features c. Sensory loss
c. Contralateral Posterior Column Features d. Urinary retention
MEDICINE
Ans. 59. a. Anterior Horn... 60. b. Amyotrophic lateral... 61. a. Fine touch| 62. b. Temperature sense
63. b. Left Lateral ... 64. c. Contralateral. 65. d. Contralateral... 66. b. Contralateral...
67. d. Sacral sparing 68. b. Absent knee... 69. c. Late Bladder... 70. a. Conus...
71. a. Loss of... 72. c, d and e 73. a. Spastic paralysis
Nervous System 1175
74. Which of the following signs is not suggestive of a cervical 82. A 12-year-old boy presents to the outpatient department
spinal cord injury: (AIIMS Nov. 05) with history of progressively increasing difficulty in
a. Flaccidity walking and frequent falls. Physical examination reveals
b. Increased rectal sphincter tone an ataxic gait and nystagmus. All deep tendon reflexes
c. Diaphragmatic breathing were observed to be absent while the plantar response was
d. Priapism Ref: Harrison’s 18/e p3367 ‘extensor’. What is the most likely diagnosis? (AI 2012)
75. Spastic paraplegia is caused by all, except: (AI 2009) a. Friedreich’s ataxia
a. Vitamin B12 deficiency b. Subacute combined degeneration of cord (SACD)
b. Cervical spondylosis c. Becker’s muscular dystrophy
c. Lead poisoning d. Tabes dorsalis
d. Motor neuron disease Ref: Neurology by Anish Bahra/68; Ref: Harrison’s 18/e p3338, 17/e p2510, 2570
Oxford Handbook of tropical Medicine 2nd/436 83. Pyramidal tract involvement with absent ankle jerk is seen
76. Causes of Acute Flaccid Paralysis include: (PGI June 08) in: (AIIMS May 01)
a. Poliomyelitis a. Friedrich’s
b. Tick paralysis b. Subacute combined degeneration of the spinal cord
c. AIDP (Acute inflammatory demyelinating polyneuropathy) c. Lathyrism
d. ADEM (Acute disseminated encephalamyelitis) d. Tabes dorsalis
e. All of the above Ref: Harrison’s 18/e p3343, 17/e p2570
77. A patient involved in a road traffic accident presents with 84. Vitamin B12 deficiency can give rise to all of the following,
quadriparesis, sphincter disturbance, sensory level up to except: (AI 2005)
the upper border of sternum and a respiratory rate of 35/ a. Myelopathy.
minute. The likely level of lesion is: (AI 2010) b. Optic atrophy.
a. C1-C2 c. Peripheral neuropathy.
b. C4-C5 d. Myopathy. Ref: Harrison’s 18/e p3374, 17/e p2595
c. T1-T2 85. Subacute combined degeneration of cord is caused due to
d. T3-T4 Ref: Harrison’s 18/e p3367, 17/e p2588 deficiency of? (DNB June 2012)
78. A patient with traumatic paraplegia due to injury of the a. Vitamin B1
thoracic cord of ‘T3 level’ is observed to have a blood b. Vitamin B5
pressure of 210/. What should be the initial management? c. Vitamin B6
a. Subcutaneous LMWH (AI 2012) d. Vitamin B12
b. Steroids Ref: Harrison’s 18/e p3374
c. Nifedipine 86. Subacute combined degeneration due to Vit B12 deficiency
d. Normal saline/dextrose mainly involves: (PGI Dec 03)
Ref: Disease of kidney and Urinary Tract/1416; Medical Management of a. Peripheral nerve
Adults with Neurological Diabetes/11; Spine and Spinal Cord Trauma: b. Corticospinal tract
Evidence Based Management Chapter 46 c. Posterior column
79. Beevor’s sign is seen in: (AIIMS May 09) d. Spinocerebellar tract
a. Abdominal muscle e. Spinothalamic tract
b. Facial muscle Ref: Harrison’s 18/e p3374, 17/e p2595
c. Respiratory muscle 87. Features of syringomyelia include all of the following,
d. Hand muscle Ref: Harrison’s 18/e p3367, 17/e p2589 except: (PGI 09)
80. Beevor’s sign indicates a lesion in the: (PGI 05) a. Dissociative sensory loss
MEDICINE
a. Thoracic cord b. Bilateral involvement
b. Cervical cord c. Segmental sensory loss
c. Cauda Equina d. Wasting of small muscles of hand
d. Conus Medullaris Ref: Harrison’s 18/e p3367 e. Ascending weakness
Ref: Harrison’s 18/e p3374
81. Anterior spinal artery thrombosis is characterized by all,
except: (PGI Dec 2000) 88. Lesions of the lateral cerebellum cause all of the following,
a. Loss of pain and touch except: (AI 2010)
b. Loss of vibration sense a. Incoordination
c. Loss of power in lower limb b. Intention tremor
d. Sphincter dysfunction Ref: Harrison’s 18/e p3367 c. Resting tremor
d. Ataxia Ref: Harrison’s 18/e p193
Ans. 74. b. Increased... 75. c. Lead poisoning 76. e. All of the above 77. b. C4-C5
78. c. Nifedipine 79. a. Abdominal muscle... 80. a. Thoracic cord 81. b. Loss of vibration...
82. a. Friedreich’s Ataxia 83. a. Friedreich’s Ataxia 84. d. Myopathy 85. d. Vitamin B12
86. b and c 87. e. Ascending... 88. c. Resting tremor
1176 Jaypee’s Triple A
89. Dysmetria is due to lesion of: (DNB June 2010) c. NFTs appear extracellularly before intracellular
a. Midbrain appearance
b. Pons d. Number of NFTs correlates with dementia
c. Medulla Ref: Harrison’s 18/e p3306, 17/e p2541
d. Cerebellum 97. Which of the following areas of brain is most resistant to
Ref: Dejong’s Neurological Examination 7/e p621; Guyton 10/e p655, 656 Neurofibrillary tangles in Alzheimer’s disease: (AI 2012)
90. Lesion in which of the following structure leads to Kluver- a. Entorhinal cortex
Bucy syndrome: (AIIMS May 04) b. Hippocampus/temporal lobe
a. Amygdala c. Lateral geniculate body
b. Hippocampus d. Visual association area Ref: Harrison’s 18/e p3306
c. Hypothalamus 98. The following is not a feature of Alzheimer’s disease:
d. Temporal lobe Ref: Guyton 10/e p687 a. Neurofibrillary tangles (AI 2004)
b. Senile (neuritic) plaques
c. Amyloid angiopathy
D. DEMENTIAS AND EXTRAPYRAMIDAL d. Lewy bodies
DISORDERS Ref: Harrison’s 18/e p3306, 17/e p2540, 2541,2542
99. An 80 year old female presents with progressive loss of
91. All of the following cause subcortical dementia, except: memory, difficulty in recalling names, difficulty in speech
a. Alzheimer’s disease (AIIMS May 09) and inability to perform desired tasks. Which of the
b. Parkinsonism following pathological features is likely to be found along
c. HIV encephalopathy with neurofibrillary tangles: (AIIMS May 09)
d. Progressive supranuclear palsy a. Beta amyloid
Ref: Harrison’s 18/e p3305 b. Lewy bodies
92. Which of the following is predominantly involved in c. Ceramidase
Alzheimer’s dementia: (AIIMS May 08) d. Pick’s bodies
a. Frontal cortex Ref: Harrison’s 18/e p3305,3306, 17/e p2540,2541
b. Temporo-parietal cortex 100. Which of the following metal ions is associated with
c. Fronto-parietal cortex secondary Parkinsonisms: (DNB June 2012)
d. Franto-temporal cortex Ref: Harrison’s 18/e p3305 a. Manganese (Mn)
93. A chromosomal anomaly associated with Alzheimer’s b. Magnesium (Mg)
dementia is: (AI 2001) c. Selenium (Se)
a. Trisomy 18 d. Molybdenum (Mo) Ref: Harrison’s 18/e p3318; Table 372-2
b. Patau syndrome 101. Dinesh, a 56 yr aged man presents with complaints of
c. Trisomy 21 slowness of movements, postural instability, tremors,
d. Turners syndrome rigidity and memory loss. Most likely diagnosis is: (AI 2001)
Ref: Harrison’s 18/e p3307, 17/e p2541, 2542 a. Multi-infarct dementia
94. Which of the following is the most important b. Alzheimer’s disease
neurotransmitter deficient in the cortex of patients with c. Parkinsonism
Alzheimer’s Disease: (DNB 2012) d. None of the above Ref: Harrison’s 18/e p3317, 17/e p2550
a. Acetylcholine 102. Transcranial Magnetic Stimulation of which part of the
b. Serotonin brain has been shown to reduce frequency of symptoms in
c. Dopamine Parkinsonism: (AI 2012)
d. Noradrenaline Ref: Harrison’s 18/e p3306 a. Striatus
MEDICINE
95. Which of the following is not seen in early onset Alzheimer’s b. Globus pallidus externus
Disease: (AIIMS Nov 2011) c. Subthalamic nucleus
a. Aphasia d. Putamen Ref: Harrison’s 18/e p3325
b. Apraxia 103. Which of the following is the most commonly used site for
c. Acalculia Transcranial Magnetic Stimulation to reduce frequency of
d. Agnosia Ref: Harrison’s 18/e p3305 Parkinsonism symptoms: (AI 2012)
96. Which of the following statements about the pathology in a. Striatus
Alzheimer’s disease is not true: (AIIMS Nov 2011) b. Globus pallidus
a. Neuritic plaques are formed of amyloid protein c. Subthalamic nucleus
b. Neurofibrillary tangles (NFT) are made of tau protein d. Putamen Ref: Harrison’s 18/e p3325
Ans. 89. d. Cerebellum... 90. a. Amygdala... 91. a. Alzheimer’s disease 92. b. Temporo...
93. c. Trisomy 21... 94. a. Acetylcholine 95. d. Agnosia 96. c. NFTs...
97. c. Lateral... 98. d. Lewy bodies 99. a. Beta amyloid 100. a. Manganese (Mn)
101. c. Parkinsonism
102. c. Subthalamic 103. c. Subthalamic nucleus
Nervous System 1177
104. Huntington’s disease is due to the loss of: (AIIMS May 04) c. Trisomy 21
a. Nigrostriatal dopaminergic neurons d. Turners syndrome
b. Intrastriatal cholinergic neurons Ref: Harrison’s 18/e p3307, 17/e p2542
c. Intrastriatal GABAergic neurons
d. Intrastriatal cholinergic and GABAergic neurons
Ref: Harrison’s 18/e p3330, 17/e p2561, 2562 E. HEAD INJURIES AND CONCUSSION
105. All are true about Huntington’s disease except: (AI 2001)
112. According to the Glasgow Coma Scale (GCS), a verbal score
a. Chorea
of 1 indicates: (AI 2005)
b. Behavioral disturbance
a. No response
c. Early onset of memory loss
b. Inappropriate words
d. Cog-wheel rigidity.
c. Incomprehensible sounds
Ref: Harrison’s 18/e p3330, 17/e p2561, 2562
d. Disoriented response Ref: Harrison’s 18/e p3381
106. A patient presents with ataxia, urinary incontinence and
113. Which of the following is not a component of Glasgow
dementia. The likely diagnosis is: (AI 2010)
Coma Scale? (AI 2006)
a. Alzheimer’s disease
a. Eye opening
b. Parkinson’s disease
b. Motor response
c. Steel richardson syndrome
c. Pupil size
d. Normal pressure hydrocephalus
d. Verbal response Ref: Harrison’s 18/e p3381
Ref: Harrison’s 18/e p3313, 17/e p25462
114. A head injured patient, who opens eyes to painful stimulus,
107. An elderly man presents with features of dementia, ataxia,
is confused and localizes to pain. What is the Glassgow
difficulty in downward gaze and a history of frequent falls.
coma score: (AIIMS Nov 05)
Likely diagnosis is: (AI 2001) (AIIMS May 01)
a. 7
a. Parkinsons disease
b. 9
b. Progressive supranuclear gaze palsy
c. 11
c. Alzheimers disease
d. 13
d. None of the above.
Ref: Harrison’s 18/e p3381 Table:378.1, 17/e p2601 Table:378.1
Ref: Harrison’s 18/e p3311, 17/e p2559
115. Total score in Glasgow Coma Scale of a conscious person
108. A 76 year old male comes with a history of frequent falls
is: (AI 2006)
and difficulty in looking downwards and laterally. The
a. 8
diagnosis is: (AIIMS May 01)
b. 3
a. Alzheimer’s disease
c. 15
b. Supranuclear palsy
d. 10 Ref: Harrison’s 18/e p3381
c. Amyotropic lateral sclerosis
d. Oculomotor nerve palsy 116. A person with ‘Inappropriate speech’ evaluated by the
Ref: Harrison’s 18/e p3311, 17/e p2561, 2562 ‘Glasgow Coma Scale’ will have a verbal score of: (AI 2012)
a. 4
109. A 45-year-old man presents with history of frequent falls.
b. 3
He has difficulty in looking down also. What is the most
c. 2
probable diagnosis: (AIIMS Nov 2000)
d. 1 Ref: Harrison’s 18/e p3381
a. Normal pressure hydrocephalus
b. Parkinson’s disease 117. The cause of systemic secondary insult to injured brain
c. Alzheimer’s disease include all of the following except: (AIIMS May’ 06)
d. Progressive supranuclear palsy a. Hypercapnia
b. Hypoxaemia
MEDICINE
Ref: Harrison’s 18/e p3311, 17/e p2529
c. Hypotension
110. Which of the following is a cause of reversible dementia?
d. Hypothermia
a. Subacute combined degeneration. (AI 2005)
Ref: Harrison 18/e 2257; Harrison 17th/1722
b. Picks disease.
c. Creutzfeldt – Jakob disease. 118. In a patient with head injury damage in the brain is
d. Alzheimer’s disease. aggravated by: (AI 2010)
Ref: Harrison’s 18/e p3302 (Table: 371.3), 17/e p2538 (Table 365-3) a. Hyperglycemia
b. Hypothermia
111. A chromosomal anomaly associated with Alzheimer’s
c. Hypocapnia
dementia is: (AI 2001)
d. Serum osmolality
a. Trisomy 18
Ref: Harrison’s 18/e p2257, 17/e p1722, 1723
b. Patau syndrome
Ans. 104. d. Intrastriatal... 105. c. Early onset .... 106. d. Normal... 107. b. Progressive...
108. b. Supranuclear
109. d. Progressive... 110. a. Subacute 111. c. Trisomy 21
112. a. No response
113. c. Pupil size 114. c. 11 115. c. 15
116. b. 3
117. d. Hypothermia 118. a. Hyperglycemia
1178 Jaypee’s Triple A
119. APACHE II score includes: (PGI June 08) a. Ipsilateral dilated pupils (AIIMS May 01)
a. Age b. Ipsilateral hemiplegia
b. PaO2 c. Cheyne stokes respiration
c. Chronic medical condition d. Decorticate rigidity
d. Respiratory rate Ref: Harrison’s 18/e p2248, 17/e p1714, 1715
e. BP 127. A patient presents with unilateral painful ophthalmoplegia.
f. All of the above Ref: Harrison’s 18/e p2197, 17/e p1674 Imaging revealed an enlargement of cavernous sinus on the
affected side. The likely diagnosis is: (AIIMS May 08)
120. Best prognostic factor for head injury is: (AI 2007)
a. Gradenigo syndrome
a. Glasgow coma scale
b. Covernous sinus thrombosis
b. Age c. Tolosa-Hunt Syndrome
c. Mode of injury d. Orbital Pseudotumor
d. CT Ref: Harrison’s 18/e p3381, 17/e p2601 Ref:‘Imaging of the Globe and Orbit: A Guide to Differential Diagnosis’ by
121. Subdural haematoma most commonly results from: Norbert Hosten (Thieme) 1998/128
a. Rupture of intracranial aneurysm (AIIMS May 04) 128. Non-noxious stimuli perceived as pain is termed as:
b. Rupture of cerebral AVM a. Allodynia (AIIMS May 08)
c. Injury to cortical bridging veins b. Hyperalgesia
d. Hemophilia Ref: Harrison’s 18/e p3379 c. Hyperesthesia
122. A 62-year old diabetic female patient presented with history d. HyperpathiaRef: Harrison’s 18/e p186-187; 17/e p154,155
of progressive right-sided weakness of one month duration. 129. All of the following statements about Diffuse Axonal
The patient was also having speech difficulty. Fundus Injury (DAI) are true except: (AI 2008)
examination showed papilledema. Two months ago, she a. Caused by shearing force
also had a fall in her bathroom and struck her head against b. Predominant white matter haemorrhages, in basal
a wall. The most likely clinical diagnosis is: ganglion and corpus callosum
c. Increased intracranial tension is seen in all cases
a. Alzheimer’s disease (AIIMS Nov 04)
d. Most common at junction of grey and white Matter
b. Left parietal glioma
Ref: Harrison’s 18/e p3377, 17/e p2597
c. Left MCA territory stroke
130. The most important area involved in planning and
d. Left chronic subdural haematoma
organizing complex sequential skilled movements is:
Ref: Harrison’s 18/e p3379
a. Primary motor area (DNB 2012)
123. A 24-year-old man falls on the ground when he is struck b. Premotor area
in the right temple by a baseball. While being driven to c. Supplementary motor area
the hospital, he lapses into coma. He is unresponsive with d. Primary sensory area
the dilated right pupil when he reaches the emergency Ref: Noback’s Human Nervous system 7th/436
department. The most important step in initial management 131. Hippocampus lesion affects: (DNB June 2010)
is: (AI 2002) a. Implicit memory
a. Craniotomy b. Procedural memory
b. CT scan of the head c. Non-declarative memory
c. X-ray of the skull and cervical spine d. Explicit memory
d. Doppler ultrasound examination of the neck Ref: Basic Neurochemistry (Academic Press) 2010/965; Psychology:
Ref: Harrison’s 18/e p3379 Concepts and Applications (Congage Learning) 2008 (3rd)/234; Practical
Neurology (Lippincott Williams) 2012 /37
124. A patient is brought to the emergency as a case of head
injury, following a head on collision road traffic accident. 132. All of the following statements about Argyll Robertson
His BP is 90/60 mmHg. Tachycardia is present. Most likely Pupil are correct, except: (AI 2011)
diagnosis is: (AI 2001) a. Near reflex normal
MEDICINE
Ans. 119. f. All of the above 120. a. Glasgow coma scale 121. c. Injury to... 122. d. Left chronic ...
123. a. Craniotomy...
124. d. Intra-abdominal bleed 125. c. Altered mental... 126. d. Decorticate...
127. c. Tolosa-Hunt
128. a. Allodynia 129. c. Increased 130. c. Supplementary...
131. d. Explicit memory
132. c. Consensual Light... 133. d. Apparent...
Nervous System 1179
134. EEG is usually abnormal in all of the following, except:
a. Subacute sclerosing panencephalitis. (AI 2005) F. DISORDERS OF MUSCLES/MYOPATHIES
b. Locked – in state.
141. Myasthenia gravis is associated with: (PGI Dec 01)
c. Creutzfeldt – Jackob disease
a. Decreased acetylcholine at nerve endings
d. Hepatic encephalopathy
b. Decreased myosin
Ref: Harrisons 18/e p2251-2252, 16/e p1629; Harrison 17/e p1718
c. Absent troponin C
135. A symmetric high-voltage, triphasic slow wave pattern is d. Decreased myoneural junction transmission
seen on EEG in the following: (AIIMS May 06) Ref: Harrison’s 18/e p3480, 17/e p2672, 2673
a. Hepatic encephalopathy 142. A 45-year old woman, presenting with the history of
b. Uremic encephalopathy diplopia and dysphagia worsening as the day progresses,
c. Hypoxic encephalopathy can be diagnosed to have: (AIIMS Nov 05)
d. Hypercarbic encephalopathy a. Thyrotoxciosis
Ref: Harrison’s 18/e p2601, 17/e p1979 b. Myasthenia gravis
136. Burst supression pattern on EEG is typically seen in: c. Muscular dystrophy
a. Anoxic encephalopathy (DNB 2012) d. Brain tumor
b. Absence seizures Ref: Harrison’s 18/e p3481, 17/e p2673
c. SSPE 143. The most sensitive test for the diagnosis of myasthenia
d. Herpes simplex encephalitis gravis is: (AI 2005)
Ref: Ultimate Review of Neurology board a. Elevated ACH receptor antibodies (ACHR Antibodies)
(Demos Medical publishing) 2011/143; b. Repetitive nerve stimulation test (RNS)
Clinical Neurophysiology (Oxford) 2009/170; c. Positive edrophonium test (Tensilon test)
Practical Guide for Clinical Neurophysiologycal d. Single fiber electromyography (SFEMG)
Testing. EEG’ (Lippincott) 2012/232 Ref: Harrison’s 18/e p3481, 17/e p2674
137. Burst – suppression EEG pattern is seen in all of the 144. A thirty five year old female has proximal weakness of
following, except? (DNB 2012) muscles, ptosis and easy fatigability. The most sensitive test
a. Hypoxic ischemic encephalopathy to suggest the diagnosis is: (AI 2011)
b. Phenobarbital administration to lower ICP in traumatic a. Muscle Biopsy
b. CPK levels
brain injury
c. Edrophonium test
c. Severe hypothermia
d. EMG
d. Creutzfeldt Jakob disease
Ref: Harrison’s 18/e p3481, 17/e p2354
Ref: Ultimate Review of Neorology Boards (Demos medical publishing)
Chapter Neurophysiology Q1 / 123, 143 (Answer); Practical Neurology by 145. Which of the following statements about Lambert Eaton
Biller / 416; Critical care study Guide (Springer) 2010/162 myaesthenic syndrome is true: (AI 2009)
a. Tensilon test is positive
138. Criteria for brainstem death includes: (AI 2012)
b. Extraocular muscles are most commonly Involved
a. Positive doll’s eye reflex
c. Incremental response to repeated electrical stimulation
b. Absent pupillary light reflex and delated pupils
d. Associated with adenocarcinoma of lung
c. Pinpoint pupils
Ref: Harrison’s 18/e p3482, 17/e p2674
d. Positive vestibulo-ocular reflex
Ref: Harrison’s 18/e p2252 146. Which one of the following is correct regarding Lambert-
eaton syndrome: (AIIMS Nov 04)
139. Which test is not useful in a patient with history of syncopal
a. It commonly affects the ocular muscle
attack? (AIIMS Nov 06)
b. Neostigmine is the drug of choice for this syndrome
a. Electrophysiological testing
c. Repeated electrical stimulation enhances muscle power
MEDICINE
b. Tilt table testing
in it.
c. PET Scan
d. It is commonly associated with adenocarcinoma of lung
d. Holter monitoring
Ref: Harrison’s 18/e p3482, 17/e p2674
Ref: Harrison’s 18/e p177
147. All of the following may be associated with Thymoma,
140. Which of the following cranial structures are insensitive to
except: (AI 2010)
pain: (AI 2009)
a. SIADH
a. Dural sheath surrounding vascular sinuses
b. Myasthenia Gravis
b. Choroid plexus
c. Hypogammaglobulinemia
c. Falx cerebri
d. Cushing’s syndrome
d. Middle meningeal artery
Ref: Harrison’s 18/e p3480, 3481, 17/e p2060
Ref: Harrison’s 18/e p113; 17/e p95, Neurological Emergencies 2nd/161
Ans. 134. b. Locked... 135. a. Hepatic encephalopathy 136. a. Anoxic... 137. d. Creutzfeldt
138. b. Absent pupillary... 139. c. PET Scan
140. b. Choroid Plexus 141. a. Decreased...
142. b. Myasthenia gravis 143. d. Single Fiber
144. c. Edrophonium test 145. c. Incremental ...
146. c. Repeated electrical... 147. a. SIADH
1180 Jaypee’s Triple A
148. Thymoma is associated with: (AI 2000) 156. All are congenital myopathies except:
a. Myasthenia gravis a. Centralcore myopathy (AIIMS Nov 01, May 2011)
b. Scleroderma b. Nemaline myopathy
c. Oesophageal atrasia c. Z band myopathy
d. Hyper-gammaglobulinemia d. Centronuclear myopathy
Ref: Harrison’s 18/e p3480, 3481, 17/e p2673, 2675 Ref: Harrison’s 18/e p3499, 17/e p2688
149. Which of the following is not associated with Thymomas: 157. Which of the following antibodies is specific for myositis:
a. Red cell aplasia (AI 2001) a. Anti – Jo-1 (AIIMS Nov 08)
b. Myasthenia gravis b. Anti – Scl-70
c. Hypergammaglobulinemia c. Anti - Sm
d. Compression of the superior mediastinum d. Anti- ku Ref: Harrison’s 18/e p3512
Ref: Harrison’s 17th/2060, 626,627,668; Harrison’s 16th/1746 158. Which one of the following clinical findings excludes the
150. Thymoma is associated with: (AI 2008) diagnosis of polymyositis? (AI 06)
a. Myasthenia gravis a. Neck muscle involvement
b. Hypergammaglobulinemia b. Extraocular muscle involvement
c. SLE c. Dysphagia
d. Multiple sclerosis d. Abdominal muscle involvement
Ref: Harrison’s 18/e p3480, 17/e p2673, 2675 Ref: Harrison’s 18/e p3509, 17/e p2696
151. In myasthenia gravis, correct statement regarding 159. All of the following are feature of dermatomyositis, except:
thymectomy is: (AI 2001) a. Salmon patch (AIIMS Nov 09)
a. Should be done in all cases b. Gottron’s patch
b. Should be done in cases with ocular involvement only c. Mechanic finger
c. Not required if controlled by medical management d. Periungual telengiectasias
d. Should be done only in cases that are associated with Ref: Harrison’s 18/e p3510, 17/e p2696
thymoma
Ref: Harrison’s 18/e p3483, 17/e p2675
152. Dystrophic gene mutation leads to: (AIIMS May 03)
G. EPISODIC WEAKNESS AND
a. Myasthenia gravis CHANNELLOPATHIES
b. Motor neuron disease
c. Poliomyelitis 160. Episodic generalized weakness can occur due to all of the
d. Duchenne muscular dystrophy following acute electrolyte disturbances, except:
Ref: Harrison’s 18/e p3492, 17/e p2683 a. Hypokalemia (AIIMS May 06)
153. A young adult presents with proximal weakness of upper b. Hypocalcemia
limbs, features of facial palsy and winging of scapula. The c. Hyponatremia
most likely diagnosis is: (AI 2012) d. Hypophosphatemia
Ref: Harrison’s 18/e p185 Table 22-2, 17/e p150 Table 23-2
a. Facio-scapulo-humeral dystrophy
b. Limb-girdle dystrophy 161. Episodic weakness is seen in all of the following, except:
c. Scapuloperoneal dystrophy a. Lambert-eaton syndrome (DNB Dec 2011)
d. Duchene muscular dystrophy b. Hypokalemia
Ref: Harrison’s 18/e p3492 Table 387.6 c. Hypercalcemia
d. Hyperglycemia Ref: Harrison’s 18/e p185, 17/e p150
154. Duchenne muscular dystrophy is a disease of: (AI 2004)
162. All of the following are associated with ‘Episodic’ weakness,
a. Neuromuscular junction
except: (AI 2012)
b. Sarcolemmal proteins
MEDICINE
a. Channelopathy
c. Muscle contractile proteins
b. Lambert-eaton syndrome
d. Disuse atrophy due to muscle weakness. c. Hyperphosphatemia
Ref: Harrison 15/e p2530; Nelson 17/e p2061; Harrison 16/e p2527; Harrison d. Hyperkalemia
17/e p2683 (2684-fig 382-6); Harrison 18/e p3491-3494 (Table: 387.6) Ref: Harrison’s 18/e p185, 17/e p150 Table 23-2
155. Which of the following is not a limb girdle dystrophy: 163. Episodic generalized weakness is associated with all
a. Sarcoglycan dystrophy (AIIMS Nov 06) except: (PGI June 03)
b. Dystrophin dystrophy a. ↓K+
c. Dysferlin dystrophy b. Lambert Eaton syndrome
d. Calpain dystrophy c. Myasthenia gravis
Ref: Harrison’s 18/e p3492, 17/e p2682, 2683 d. Tuberculosis
e. Multiple sclerosis Ref: Harrison’s 18/e p185, 17/e p150
Ans. 148. a. Myasthenia gravis 149. c. Hyper... 150. a. Myasthenia gravis 151. d. Should be done...
152. d. Duchenne ...
153. a. Facio-Scapulo 154. b. Sarcolemmal. 155. b. Dystrophin...
156. c. Z band myopathy 157. a. Anti – Jo-1
158. b. Extraocular... 159. a. Salmon Patch...
160. b. Hypocalcemia
161. d. Hyperkalemia 162. c. Hyperphosphatemia 163. d. Tuberculosis
Nervous System 1181
164. All of the following are neurologic channelopathies, c. Aneurysmal subarachnoid haemorrhage
except: (AIIMS May 04) d. Arteriovenous malformation haemorrhage
a. Hypokalemic periodic paralysis Ref: Harrison’s 18/e p2262, 2263, 17/e p1727
b. Episodic ataxias 171. A young female presents with severe headache and neck
c. Familial hemiplegic migraine stiffness of abrupt onset. She says, she has never had such
d. Huntington’s disease severe headache before. She also complains of associated
Ref: Harrison’s 18/e p3225 nausea and photophobia. Likely diagnosis is:
165. All of the following are neurologic channelopathies except: a. Subarachnoid hemorrhage (SAH) (AIIMS May 09)
a. Hypokalemic periodic paralysis (AI 2005) b. Migraine
b. Episodic ataxia type 1 c. Viral Encephalitis
c. Familial hemiplegic migraine d. Hydrocephalus
d. Spinocerebellar ataxia 1 Ref: Harrison’s 18/e p2263, 17/e p1727
Ref: Harrison’s 18/e p3225, 17/e p2478 172. Sudden excruciating headache is most characteristic of:
a. SAH (PGI Dec 01)
b. Aneurysmal bleeding
H. SUBARACHNOID HAEMORRHAGE c. Epilepsy
d. Intracerebral hemorrhage
166. The common cause of subarachnoid hemorrhage is: e. Hysteria
a. Arterio-venous malformation (AI 2006) Ref: Harrison’s 18/e p2263, 17/e p1726-1728
b. Cavernous angioma 173. A patient presented with thunder clap headache followed
c. Aneurysm by unconsciousness and progressive III cranial nerve palsy.
d. Hypertension Which of the following is the most likely diagnosis:
Ref: Harrison’s 18/e p3294, 3296 a. Extradural hemorrhage (AIIMS Nov 2010)
167. A patient with suspected subarachnoid haemorrhage b. Aneurysmal subarachnoid hemorrhage
presents with blood isolated in the fourth ventricle on a CT c. Basilar migraine
scan. Aneurysmal rupture is likely to have resulted from: d. Cluster headache
a. Posterior inferior cerebellar artery aneurysm
b. Anterior communicating artery aneurysm (AI 2012)
c. Posterior communicating artery aneurysm I. MENINGITIS
d. Basilar artery tip aneurysm
174. Which of the following is not seen in Tubercular meningitis
168. Which of the following is the most common cause of late
a. Low sugar (DNB June 2011)
neurological deterioration in a case of cerebrovascular
b. High protein
accident: (AIIMS Nov 2000)
c. Low opening pressure
a. Rebleeding
d. Lymphocytic ploeocytosis
b. Vasospasm
Ref: Chandrasoma Taylor 3rd/ 915
c. Embolism
d. Hydrocephalus Ref: Harrison’s 18/e p2262, 17/e p1727 175. Which of the following is the classical CSF finding seen in
169. A 45 years old hypertensive male presented with sudden TBM? (AI 2007)
onset severe headache, vomiting and neck stiffness. On a. Increased protein, decreased sugar, increased lymphocytes
examination he didn’t have any focal neurological deficit. b. Increased protein, sugar and lymphocytes
His CT scan showed blood in the Sylvain fissure. The c. Decreased protein, increased sugar and lymphocytes
probable diagnosis is: (AIIMS May 03) d. Increased sugar, protein and neutrophils
a. Meningitis Ref: Harrison’s 18/e p3436, 17/e p2642]
b. Ruptured aneurysm
MEDICINE
176. A 25 years old lady with history of fever for 1 month
c. Hypertensive bleed presents with headache and ataxia. Brain imaging shows
d. Stroke dilated ventricles and significant basal exudates. Which of
Ref: Harrison’s 18/e p2261, 2262 17/e p1726, 1727, 1728, 96 the following will be the most likely CSF finding:
170. A 45 year old male patient presented in the casualty with two (AIIMS Nov 2011)
hours history of sudden onset of severe headache associated a. Lymphocytosis, low glucose, high protein
with nausea and vomiting on clinical examination the b. Lymphocytosis, normal glucose, high protein
patient had necks stiffness and right sided ptosis. Rest c. Lymphocytosis, low glucose, normal protein
of the neurological examination was normal. What is the d. Neutrophillia, low glucose, low protein
clinical diagnosis: (AIIMS Nov 03) Ref: Harrison’s 18/e p3414
a. Hypertensive brain haemorrhage
b. Migraine
Ans. 164. d. Huntington’s... 165. d. Spinocerebellar... 166. c. Aneurysm 167. a. Posterior Inferior...
168. b. Vasospasm...
169. b. Ruptured aneurysm 170. c. Aneurysmal... 171. a. Subarachnoid...
172. a and b
173. b. Aneurysmal... 174. d. Lymphocytic... 175. a. Increased...
176. a. Lymphocytosis...
1182 Jaypee’s Triple A
177. Characteristic finding in CT in a TB case is: (AI 2001) c. Human immunodeficiency virus
a. Exudate seen in basal cistern d. Rubeola virus Ref: Harrison’s 18/e p3421, 17/e p2630
b. Hydrocephalus is non communicating 186. A young male develops fever, followed by headache,
c. Calcification commonly seen in umbellium confusional state, focal seizures and right hemiparesis.
d. Ventriculitis is a common finding The MRI performed shows bilateral frontotemporal hyper
Ref: Harrison’s 18/e p3436 intense lesion. The most likely diagnosis is: (AI 2004)
178. Basal exudates, infarcts and hydrocephalus are findings a. Acute pyogenic meningitis
observed in brain imaging studies. The most likely b. Herpes simplex encephalitis
diagnosis is: (AI 2012) c. Neurocysticercosis
a. Tubercular meningitis d. Carcinomatous meningitis
b. Viral meningitis Ref: Harrison’s 18/e p3422, 17/e p2631
c. Herpes encephalitis 187. Which of the following is the most common central nervous
d. Cerebral malaria Ref: Harrison’s 18/e p3436 system parasitic infection? (AIIMS May 03)
179. Pneumococcal meningitis is associated with the following a. Echinococcosis.
CSF findings: (AI 2012) b. Sparganosis.
a. Pleocytosis with low protein and low sugar c. Paragonimiasis.
b. Pleocytosis with high protein and low sugar d. Neurocysticercosis. Ref: Harrison’s 18/e p3437
c. Lymphocytosis with low protein and low sugar 188. Commonest presentation of neurocysticercosis is: (AI 2003)
d. Lymphocytosis with high protein and low sugar a. Seizures
Ref: Harrison’s 18/e p3414 b. Focal neurological deficits
180. Neurological complications of meningitis include all of the c. Dementia
d. Radiculopathy Ref: Harrison’s 18/e p3431, 17/e p2637
following, except: (AIIMS Nov 02)
a. Seizures 189. Which of the following is the most common location of
b. Increased intracranial pressure. intracranial neurocysticercosis: (AIIMS Nov 05)
c. Cerebral hamartoma. a. Brain parenchyma
d. Subdural effusions. b. Subarachnoid space
Ref: Harrison 15/e p2464, 2481; O.P. Ghai Pediatrics 5/e p393; c. Spinal cord
d. Orbit
181. Which of the following agents is most commonly associated
Ref: Harrison’s 18/e p3431, 17/e p2638
with recurrent meningitis due to CSF leaks: (AI 2010)
a. Meningococci 190. True statement about neurocysticercosis is: (AI 2001)
b. Pneumococci a. Seizures due to neurocysticercosis are resistant to anti
c. Hemophilus influenza epileptic drugs
d. E. coli Ref: Harrison’s 18/e p3410, 3435, 3436 b. Albendazole is superior to praziquantel in the treatment
of above condition
182. Subdural empyema is most commonly caused by: (AI 2000)
c. Common presentation is 6th CN palsy and hemiparesis.
a. H influenza
d. Steroids are used in the management of hydrocephalus.
b. Staphylococcus aureus
Ref: Harrison’s 18/e p3431
c. Streptococcus pneumoniae
d. E. Coli Ref: Harrison’s 18/e p3432, 17/e p2638 191. All of the following statements are true regarding central
nervous system infections, except: (AIIMS Nov 04)
183. Which of the following viruses is not a common cause of
a. Measles virus is the causative agent for subacute
viral encephalitis? (AI 2004)
a. Herpes simplex virus type 2 sclerosing pan encephalitis (SSPE)
b. Japanese encephalitis virus b. Cytomegalovirus causes bilateral temporal lobe
c. Nipah virus hemorrhagic infarction
MEDICINE
d. Cytomegalovirus Ref: Harrison’s 18/e p3419, 17/e p2628 c. Prions infection causes spongiform encephalopathy
d. JC virus is the causative agent for progressive multifocal
184. Which of the following is the most common cause of
leucoencephalopathy
meningoencephalitis in children: (AI 2010)
Ref: Harrison’s 18/e p3427, 17/e p2631, 2635
a. Mumps
b. Arbovirus 192. Which of the following statements about Prions is true:
c. HSV a. They are infections proteins (AI 2008)
d. Enterovirus Ref: Nelson’s 18/e p2521 b. They are made upof bacteria and virus
c. They have rich nuclear material
185. Commonest cause of sporadic encephalitis is:
a. Japanese B virus (AIIMS May 04) (AI 2003) d. They can be cultured in cell free media
b. Herpes simplex virus Ref: Harrison’s 18/e p3441
Ans. 177. a. Exudate... 178. a. Tubercular meningitis 179. b. Pleocytosis... 180. c. Cerebral...
181. b. Pneumococci
182. c. Streptococcus... 183. a. Herpes... 184. d. Enterovirus
185. b. Herpes Simplex... 186. b. Herpes simplex ...
187. d. Neurocysticercosis 188. a. Seizures
189. a. Brain parenchyma 190. b. Albendazole...
191. c. Prions... 192. a. They are infections
Nervous System 1183
193. Prions include: (AIIMS Nov 07) 200. Most common site of subependymal astrocytoma (giant
a. DNA and RNA cell): (AIIMS Nov 07)
b. Only RNA a. Trigone of lateral ventricle
c. Proteins b. Foramen of Munro
d. Only DNA Ref: Harrison’s 18/e p3441 c. Temporal horn of lateral ventricle
194. Which one of the following is not a prion associated d. 4th ventricle
disease: (AI 2005) Ref: Neurology in clinical practice 4th/1428;|
a. Scrapie Neuro-ancology by Bernstein (2000)
b. Kuru 201. A young female patient with long history of sinusitis
c. Creutzfeldt-Jakob disease presented with frequent fever along with personality
d. Alzheimer’s disease changes and headache of recent origin. The fundus
Ref: Harrison’s 18/e p3442, 17/e p2647 Table: 383.2 examination revealed papilledema. The most likely
195. A 60 year old man with progressive dementia of recent diagnosis is: (AIIMS Nov 04)
onset presents with intermittent irregular jerky movements. a. Frontal lobe abscess
EEG shows periodic sharp biphasic waves. The most likely b. Meningitis
diagnosis is: (AI 2011, AIIMS Nov 2010) c. Encephalitis
a. Alzheimer’s disease d. Frontal bone osteomyelitis Ref: Harrison’s 18/e p246
b. Creutzfeldt jakob disease
c. Lewy body dementia
d. Herpes simplex encephalitis J. MISCELLANEOUS
Ref: Harrison’s 18/e p3444, 17/e p2649
196. All of the following statements about Creutzfeldt-Jakob 202. Which of the following speech indicates damage to the
disease are true, except: (AIIMS May 04) categorical hemisphere: (AIIMS May 08)
a. It is a neurodegenerative disease a. Normal speech
b. It is caused by infectious proteins b. Increased speech
c. Myoclonus is rarely seen c. Decreased speech
d. Brain biopsy is specific for diagnosis d. Senseless, fluent speech
Ref: Harrison’s 18/e p3444, 3445 17/e p2649, 2650 Ref: Harrison’s 18/e p204, 17/e p164
197. Seizures may be the presenting feature in all of the 203. A 45 years male presents with hypertension. He has sudden
following, except: (DNB 2012) abnormal flinging movements in right upper and lower
a. Cryptococcus meningitis limbs. Most likely site of haemorrhage is: (AI 2001)
b. Toxoplasmosis a. Substantia nigra
c. CMV b. Caudate nuclei
d. Entamoeba histolytica c. Pons
d. Subthalamic nuclei
Ref: Child Neurology 7/e p517, Oxford Textbook of Medicine ‘Infection’ 5/e
Ref: Harrison’s 18/e p3332, 17/e p2563
p2012/661 ‘Surgical Diseases in Tropical countries by Sood (Jaypee) 1996/8
204. All of the following neurophysiological defects are likely to
198. Which of the following is the most common type of Glial
result from right lobe involvement, except:
tumors? (AI 06)
a. Visuospatial defects (AIIMS May 08)
a. Astrocytomas
b. Anosognosia
b. Medulloblastomas
c. Dyscalculi
c. Neurofibromas
d. Spatial dysgraphia
d. Ependymomas
MEDICINE
Ref: ‘High yield Neuroanatomy’ by Fix 4th/144, 146; ’Lecture Notes in
Ref: Harrison’s 18/e p3384 Neurology’ by Ginsberg 9th/14, 15, 16; Davidson’s 21st/1136;
199. Which of the following statements about cerebellar 205. Which of the following structure is most likely to be
astrocytomas in paediatric age group is false: (AI 2008) compressed by an aneurysm of the posterior communicating
a. These are usually low grade tumors artery: (AIIMS May 08)
b. These tumors have a good prognosis a. Occulomotor nerve
c. These are more commonly seen in the 1st and 2nd decades b. Optic nerve
d. These tumours are more common in females c. Trochlear nerve
Ref: Nelsons 18/e p2130, 2131; WHO classification of Tumours of the d. Hypophysis cerebri
Central Nervous System (2007) /14 Ref: Harrison’s 18/e p2262, 17/e p1727
Ref: Merrst’s Neurology 12th/373; Essential Neurology c. Supratentorial lesions are common
(John Wiley and Sons) 2009/44 d. Tumors of Schwann cells are common
213. All of the following statements about neurofibromatosis are Ref: Harrison’s 18/e p2360
true, except: (AI 2009) 221. A child presents to the clinic with history of seizures and
a. Autosomal recessive inheritance mental retardation. Clinical examination reveals multiple
b. Cutaneous neurofibromas hypopigmented macules. What is the likely diagnosis:
c. Cataract a. Tuberous sclerosis (AI 2010)
d. Scoliosis Ref: Harrison’s 18/e p3389 b. Neurofibromatosis
214. All of the following are seen in neurofibromtosis, except: c. Sturge weber syndrome
a. Lisch nodules (DNB 2011) d. Linear epidermal nevus syndrome
Ref: Harrison’s 18/e p3390
Ans. 206. a. Tau protein 207. b. Frontotemporal... 208. b. 2/3 plasma glucose 209. d. Craniopharyngoma
210. c. Lung metastasis
211. a. Lymphoma 212. d. Diplopia 213. a. Autosomal...
214. c. Shagreen Patch
215. a, c and e 216. b. Optic pathway... 217. b. Gastric carcinoma
218. a. Cerebellum
219. c. Cerebellum 220. b. Craniospinal... 221. a. Tuberous sclerosis
Nervous System 1185
222. Triad of tuberous sclerosis includes all, except: (AI 2009) 230. Predominantly sensory neuropathy is/are caused by:
a. Epilepsy a. Cisplatin (PGI June 01)
b. Adenoma sebaceum b. Pyridoxine excess
c. Low intelligence c. Suramin
d. Hydrocephalus Ref: Harrison’s 18/e p3390, 17/e p2607 d. Diptheria
223. A middle aged female presents with prolonged history e. Guillain-Barre syndrome Ref: Harrison’s 18/e p3463, 3465
of back pain followed by slowly progressive weakness 231. All of the following can cause neuropathies with
of both lower limbs, spasticity and recent onset difficulty predominant motor involvement except: (AI 2004)
in micturation. On neurological examination there is a. Acute inflammatory demyelinating polyneuropathy
evidence of dorsal myelopathy. MRI scan of spine shows b. Acute intermittent porphyria
a well-defined mid-dorsal intradural homogenous contrast c. Lead intoxication
enhancing mass lesion. The likely diagnosis is: d. Arsenic intoxication
a. Intradural Lipoma (AIIMS Nov 2011) Ref: Harrison’s 18/e p3466, 17/e p2653 379-3
b. Dermoid cyst 232. All of the following are predominant motor neuropathy
c. Epidermoid cyst except: (AIIMS May’ 06)
d. Spinal meningioma a. Acute inflammatory demylinating poly radiculoneu-
Ref:‘Meningiomas: Diagnosis, Treatment and Outcome’ by Lee ropathy
(Springer) 2009/530, 531, 532;‘Spine Radiosurgery (Thieme) b. Porphyric neuropathy
2008/113, 114; ‘Spinal Cord and Spinal Column Tumors (Thieme) 2006 c. Lead intoxication
224. All of the following are classified as ‘Small fiber’ d. Arsenic intoxication
neuropathies’ except: (AI 2012) Ref: Harrison’s 18/e p3466, 17/e p2653
a. Vitamin B12 deficiency 233. Most common cause of mononeuritis multiplex in India is:
b. HIV Induced a. Hansen’s disease (AIIMS Nov 08)
c. Hansen’s disease b. Rheumatoid arthritis
d. Amyloidosis Ref: Harrison’s 18/e p3448-3449, 17/e p2653 c. Tuberculosis
225. Small fiber neuropathy presents as: (PGI June 08) d. PAN
a. Burning pain Ref: Neurological Practice: an Indian Perspective by Walia (2005)/591
b. Tingling and Numbness ‘Clinical Neurophysiology’ 2nd (2006)
c. Foot drop 234. Pure motor paralysis is seen in: (AI 2000)
d. Twitching Ref: Harrison’s 18/e p3448-3449, 17/e p2652 a. Polio
226. All of the following are associated with Autonomic b. Guillain Barre syndrome
Neuropathies, except: (DNB 2009) c. Diabetes mellitus
a. Diabetes d. Sub-Acute Combined Degeneration
b. Amyloid Ref: Harrison’s 18/e p3347, 17/e p2541, 2542
c. Hyperthyroidism 235. All cause ascending motor paralysis except: (AI 2000)
d. Botulinism Ref: Harrison’s 18/e p3353, 17/e p1764 a. Diabetes mellitus
227. All of the following are feature of autonomic neuropathy, b. Diphtheria
except: (AIIMS May 08) c. Guillain Barre syndrome
a. Resting tachycardia d. Porphyria Ref: Harrison’s 18/e p3458, 3460, 3473, 3173
b. Silent myocardial infarction 236. Trophic ulcers are seen in all, except: (PGI Dec 2000)
c. Orthostatic hypotension a. Leprosy
d. Bradycardia Ref: Harrison’s 18/e p3353 b. Syringomyelia
MEDICINE
228. Neuropathy is not seen in: (PGI June 03) c. Polio
a. Tuberculosis d. All of the above (seen in all)
b. SLE Ref: Harrison’s 18/e p1363, 3374
c. Diabetes mellitus 237. All of the following can lead to trophic ulcers in the fingers
d. Polyarteritis nodosa except: (AIIMS Nov 2000)
e. Sarcoidosis Ref: Harrison’s 18/e p3353 a. Cervical disc prolapse
229. Sensorimotor neuropathy may be caused by all, except: b. Subacute combined degeneration of spinal cord
a. DM (PGI Dec 02) c. Leprosy
b. Lead poisoning d. Syringomyelia
c. Arsenic Ref: Harrison’s 18/e p3367, 3374, 1363
d. Isoniazid
e. Cryoglobulinemia Ref: Harrison’s 18/e p3466
Ans. 222. d. Hydrocephalus 223. d. Spinal meningioma 224. a. Vitamin B12 ... 225. a. Burning pain
226. c. Hyperthyroidism
227. d. Bradycardia 228. a. Tuberculosis 229. b. Lead poisoning
230. b. Pyridoxine excess 231. d. Arsenic intoxication
232. d. Arsenic intoxication 233. a. Hansen’s disease
234. a. Polio
235. b. Diphtheria 236. c. Polio 237. b. Subacute...
1186 Jaypee’s Triple A
238. Dying back neuropathy is seen in all except: (AI 2008) recovery. Which of the following is the most likely
a. Diabetic neutropathy suspected diagnosis: (AI 2012)
b. Arsenic neuropathy a. Post polio syndrome
c. Porphyria b. Polymyositis
d. Guillain Barre syndrome (GBS) c. Muscular dystrophy
Ref: Diagnostic Neuropathology by Vinters, Farrell and Mischel d. Neuropathy Ref: Harrison’s 18/e p1595
(1998) / 12; Acute Rheumatic and Immunological diseases by Mandell 246. Charcot’s Joint includes all of the following except:
(1994) / 486, 487; Harrison 16/e p2504, 2505. a. Syringomyelia (AIIMS Nov 06)
239. All of the following are true about Guillain Barre syndrome b. Leprosy
(GBS), except: (AI 2010) c. Diabetes
a. Ascending paralysis d. Arthrogryposis multiplex congenita
b. Flaccid paralysis Ref: Harrison’s 18/e p2856, 17/e p2180 (Table 330.2)
c. Sensory level 247. Impotence is a feature of which of the following:
d. Albumino-Cytological Dissociation a. Multiple sclerosis (AIIMS Nov 2000)
Ref: Harrison’s 18/e p3473, 17/e p2667, 2668, 2670 b. Poliomyelitis
240. All of the following statements about Guillain – Barre c. Amyotropic lateral sclerosis
Syndrome are true, except: (DNB Dec 2011) d. Meningitis Ref: Harrison’s 18/e p3398, 17/e p2613
a. Inflammatory 248. Which of the following drugs is not recommended for the
b. Demyelinating treatment of Multiple Sclerosis: (AIIMS Nov 2011)
c. Descending a. Interferon β-1a
d. Cranial nerve involvement Ref: Harrison’s 18/e p3473 b. Interferon β-1b
241. Conduction velocity of nerve is reduced in all of the c. Glatiramer acetate
following conditions, except: (AI 2012) d. Mycophenolate Ref: Harrison’s 18/e p3402
a. Acute motor axonal neuropathy (AMAN) 249. Horner’s syndrome may be caused by all of the following,
b. Acute inflammatory demyelinating neuropathy (AIDP) except: (AIIMS Nov 2010)
c. Hereditary sensory motor neuropathy (HSMN) a. Carotid artery aneurysm
d. Multifocal motor neuropathy b. Medial medullary syndrome
Ref: Harrison’s 18/e p3473; Table 385-1 c. Iatrogenic
242. A child presents with ascending flaccid paralysis. There d. Multiple sclerosis
is subsequent respiratory muscle involvement. CSF Ref: Harrison’s 18/e p3398, 17/e p3402, 3403, 2404
examination shows albuminocytological dissociation. 250. Multiple sclerosis is associated with all of the following,
Treatment of choice is: (AIIMS May 02) except: (DNB 2012)
a. Cycloserine a. Hydrocephalus
b. Oral prednisolone b. Optic neuritis
c. I.V. methyl prednisolone c. Spasticity
d. I.V. immunoglobins Ref: Harrison’s 18/e p3477, 17/e p2670 d. Spinal cord involvement Ref: Harrison’s 18/e p3398
243. An 18-year-old male presented with acute onset descending 251. Which of the following is used in the treatment of Multiple
paralysis of 3 days duration. There is also a history of Sclerosis: (AIIMS Nov 2006)
blurring of vision for the same duration. On examination, a. Interferon alpha
the patient has quadriparesis with areflexia. Both the pupils b. Interferon beta
are non-reactive. The most probable diagnosis is: c. Infliximab
a. Poliomyelitis (AIIMS May 06) d. Interferon gamma Ref: Harrison’s 18/e p3403,3404
b. Botulism
MEDICINE
Ans. 238. d. Guillain 239. c. Sensory level 240. c. Descending 241. a. Acute motor...
242. d. I.V. immunoglobins 243. b. Botulism
244. c. Fever 245. a. Post polio syndrome
246. d. Arthrogryposis ...
247. a. Multiple sclerosis 248. b. Interferon β-1b 249. d. Multiple sclerosis
250. a. Hydrocephalus
251. b. Interferon Beta 252. d. Autonomic dysfunction 253. d. Multiple sclerosis
Nervous System 1187
254. After a minor head injury a young patient was unable to 256. Shirmer’s test is used to assess: (DNB 2012)
close his left eye and had drooling of saliva from left angle a. Greater superficial petrosal nerve
of mouth. He is suffering from: (AIIMS May 03) b. Stapedial branch
a. VIIth nerve injury c. Chorda tympani nerve
b. Vth nerve injury d. Stylohyoid branch
c. IIIrd nerve injury Ref: Yanoff and Duker Ophthalmology 3/e p327
d. Combined VIIth and IIIrd nerve injury
257. Herpes zoster in geniculate ganglion causes: (DNB 2010)
Ref: Harrison’s 18/e p3362
a. Bell’s palsy
255. Shirmer’s test is done for: (DNB Dec 2011) b. Ramsay Hunt syndrome
a. Occulomotor nerve c. Merkelson Rosenthal syndrome
b. Optic nerve d. Frey’s syndrome Ref: Harrison’s 18/e p3362
c. Facial nerve
d. Hypoglossal nerve
Ref: Yanoff and Duker Ophthalmology 3/e p327
MEDICINE
Ans. 254. a. VIIth nerve injury 255. c. Facial nerve 256. a. Greater... 257. b. Ramsay Hunt...
10. ACID-BASE DISORDERS
Acid-Base Disorders 1189
1. A 2 year old child is being evaluated for persistent metabolic a. Paracetamol poisoning
acidosis. Blood tests show Na+ 140mEq/l. K+ 3mEq/l, Ca2+ b. Diethyl glycol poisoning
8mg/l, Mg+2 mEq/l. phosphate 3 mEq/l, pH 7.22, bicarbonate c. Severe malaria
16mEq/l and chloride 112mEq/l. The plasma anion gap is: d. Hanta virus infection Ref: Harrison’s 18/e p367, 17/e p291
a. 9 (AIIMS Nov 2004) 8. Urinary anion gap, an indication of excretion of: (AI 2002)
b. 15 a. Ketoacids
c. 22 b. NH4+ ion
d. 25 Ref: Harrison’s 18/e p365, 17/e p289,290 c. H+ ion
2. A normal-anion-gap metabolic acidosis occurs in patients d. Na+ ion Ref: Harrison’s 18/e p367, 17/e p291, 292
with: (AIIMS Nov 2003) 9. Acute metabolic acidosis: (AI 2002)
a. Diarrhoea a. Has biphasic effect on K+ excretion
b. Diabetic ketoacidosis b. Does not effect K + excretion
c. Methyl alcohol poisoning c. Decreases urinary K+ excretion
d. Acute renal failure d. Increase urinary K+ excretion
Ref: Harrison’s 18/e p368, 17/e p292 Ref: Harrison’s 18/e p366
3. Normal anion gap metabolic is caused by: (AI 2003) 10. pH 7.24, PaO2 55 mm Hg, PaCO250 mm Hg, HCO3-30
a. Cholera mEq/L consistent with: (DP PGMEE 2009)
b. Starvation a. Respiratory acidosis
c. Ethylene glycol poisoning b. Respiratory alkalosis
d. Lactic acidosis Ref: Harrison’s 18/e p368, 17/e p292 c. Metabolic acidosis
4. All of the following cause high anion gap metabolic acidosis d. Metabolic alkalosis
except: (AIIMS Nov 2002) Ref: Harrison’s 18/e p363, 17/e p384, 2061
a. Lactic acidosis 11. Causes of metabolic alkalosis include all the following
b. Salicylate poisoning except: (AI 2003)
c. Ethylyene glycol poisoning a. Mineralocorticoid deficiency
d. Ureterosigmoidostomy Ref: Harrison’s 18/e p368,369 b. Bartter’s syndrome
5. The following condition is not associated with an increased c. Thiazide diuretic therapy
anion-gap type of metabolic acidosis: (AI 2002) d. Recurrent vomiting Ref: Harrison’s 18/e p370, 17/e p293
a. Shock 12. A patient with salicylic acid poisoning has the following
b. Ingestion of ante-freeze arterial blood gas analysis report: pH = 7.12 : pCO2 =
c. Diabetic ketoacidosis 18mmHg; HCO3 = 12mmol/L. The resulting acid base
d. COPD Ref: Harrison’s 18/e p369, 17/e p292 abnormality can be best labeled as: (AIIMS Nov 03)
6. A 7 years girl, parents gave history of fever for which a. Metabolic acidosis with compensatory respiratory
she was treated with Paracetamol following which the alkalosis
fever subsided. Later she developed seizures and altered b. Metabolic acidosis with compensatory respiratory
sensorium. The urine examination revealed oxalate crystals alkalosis
on microscopy, Blood anion gap and osmolality were c. Respiratory acidosis with metabolic alkalosis
increased. The diagnosis is: (AIIMS May 2002) d. Metabolic acidosis
MEDICINE
a. Lactic acidosis Ref: Harrison’s 18/e p363 Table 47-1
b. Ethylene glycol poisoning 13. In a patient PO2 is 85 mmHg, PCO2 – 50mmHg, Ph is 7.2
c. Renal tubular acidosis and HCO3 is 32 meq/l is suffering from:
d. Paracetamol poisoning a. Respiratory acidosis with compensatory metabolic
Ref: Harrison’s 18/e p367, 17/e p291 alkalosis (AIIMS June 2000)
7. A 2 year old boy presents with fever for 3 days which re- b. Respiratory acidosis with compensatory metabolic
spond to administration of paracetamol. Three days later he acidosis
developed acute renal failure, marked acidosis and enceph- c. Metabolic acidosis
alopathy. His urine showed plenty of oxalate crystals. The d. Metabolic alkalosis
blood anion gapand osmolal gapwere increased. Which of Ref: Harrison’s 18/e p364, 17/e p1590, 288, 289
the following is the most likely diagnosis: (AIIMS Nov 05)
Ans. 14. c. Respalkalosis 15. c. Respiratory and metabolic... 16. d. Siggard-Andersen... 17. b. Respiratory acidosis
18. a. Ureterosigmoid... 19. d. Less ammonia 20. a. Hyper ventilation 21. b. Hepatic failure
22. a. Normal saline 23. b. Methanol poisoning 24. d. Bicarbonate level 24. d. Respiratory...
25. b. Metabolic alkalosis
11. GENETIC DISORDERS
1192 Jaypee’s Triple A
MEDICINE
19. Males who are sexually under developled with rudimentary b. Decreased level of α-fetoprotein
testes and prostate glands, sparse pubic and facial hair, long c. Chromosomal breakage
arms and legs and large hands & feet are likely to have the d. IgA deficiency Ref: Harrison’s 18/e p3344]
chromosome complement of: (AI 2004) 27. Which of the following tests on maternal serum is most
a. 45, XYY useful in distinguishing between open neural tube defects
b. 46, XY and ventral wall defects in a fetus? (AI 2004)
c. 46, XXY a. Carcinoembryogenic antigen
d. 46, X b. Sphingomyelin
Ref: Harrison’s 18/e p3048 c. Alpha-feto protein
20. A patient of 47XXY karyotype with features of d. Pseudocholinesterase
hypogonadism; likely diagnosis is: (AI 2001) Ref: Dutta obstetrics 6/e p107
Ans. 13. a. Phosphate rickets 14. d. Mental retardation 15. a. Occurrence of Turner... 16. b. Turner’s syndrome
17. d. 45 X 18. c. Infertility in females 19. c. 46, XXY 20. b. Kilenfelters syndrome
21. c. Bilateral shortness... 22. c. Vaginal atresia 23. a. Karyotyping 24. a. 21 hydroxylase deficiency
25. c. Talipes equinovarus 26. b. Decreased level of... 27. d. Pseudocholinesterase
1194 Jaypee’s Triple A
28. Which of following is the feature of Y chromosome? c. 46 XY
a. Acrocentric (AI 2004) d. 47 XXY Ref: Harrison’s 18/e p3048, 515
b. Telocentric 36. Sexual infantilism is associated with:
c. Submetacentric a. Pituitary tumors
d. Metacentric b. Gonadal aplasia
Ref: Molecular genetics by Precid and Strachan /49, 153 c. Dwarfism
29. Which of the following procedures as routine technique for d. All of the above
karyotyping using light microscopy: (AI 2003) Ref: Harrison’s 18/e p3046, 3047
a. C-banding 37. Anticentromere antibody is seen in: (Rajasthan 2008)
b. G-banding a. Diffuse scleroderma
c. Q-banding d. Dermatomyositis
d. Brd V-staining c. CREST
Ref: Robbins 6/e p165, Harshmohan 5/e p22 d. Sojgren’s syndrome
30. Differential expression of same gene depending on parent Ref: Harrison’s 18/e p409, 431, 2737
of origin is referred to as: (AI 2005) 38. In gene therapy which is used as vehicle? (Rajasthan 2009)
a. Genomic imprinting a. E.coli
b. Mosaicism b. Corynebacterium
c. Anticipation c. Adenovirus
d. Nonpenetrance d. Pneumococcus
Ref: Harrison’s 16/e p375 Ref: Harrison’s 18/e p548, 1502
31.
Hereditary retinoblastomas developthe following 39. HCO3 is predominantly increased in: (Rajasthan 2009)
chromosomal deletion: (AI 2003) a. Respiratory alkalosis
a. 13 q 14 b. Metabolic alkalosis
b. 13 p14 c. Respiratory acidosis
c. 14 p13 d. Metabolic acidosis
d. 14 q 13 Ref: Harrison’s 18/e p369, 370
Ref: Harrison’s 18/e p502
40. A patient with normal parents while his maternal uncle
32. All the following diseases are associated with HLA B-27 & suffers from disease. The mode of inheritance is: (UP 2011)
Uveitis, except: (AI 2000) a. Autosomal dominant
a. Behcet’s syndrome b. Autosomal recessive
b. Psoriasis c. X-linked dominant
c. Ankylosing Spondylitis d. X-linked recessive
d. Reiter’s syndrome Ref: Harrison’s 18/e p500-501
Ref: Harrison’s 18/e p2801, 17/e p2132
41. Which of the following laboratory studies is helpful to
33. Gluten sensitive enteropathy is most strongly associated diagnose cystic fibrosis? (MP PG 2010)
with: (AI 2003) a. Serum IgE level
a. HLA-DQ2 b. Chromosomal study
b. HLA-DR4 c. Serum amylase
c. HLA-DQ3 d. Sweat chloride test
d. Blood group‘B’ Ref: Harrison’s 18/e p2149, 2150, 17/e p1632
Ref: Harrison’s 18/e p2470, 17/e p118
42. Hereditary Ataxia may be a feature of abnormality in
34. A patient of cystic fibrosis (homozygous) married a carrier interaction of vitamin E with: (MP PG 2009)
(heterozygous). The chances of developing cystic fibrosis in
MEDICINE
a. LDL
the off springs is: (DP PGMEE 2010) b. Chylomicron
a. 50% carriers, 50% affected c. VLDL
b. 25% carriers, 75% affected d. Lipoprotein (a)
c. 75% carriers, 25% affected Ref: Harrison’s 18/e p3334, 17/e p2571
d. All are carries
43. The gene located for cystic fibrosis is on: (MP PG 2008)
Ref: Harrison’s 18/e p507, 2147, 17/e p1632
a. Chromosome 7
35. The chromosomal complement is persons with Klinefelter’s b. Chromosome 14
syndrome is: (Rajasthan 2008) c. Chromosome 21
a. 45 XO d. X chromosome
b. 47 XXX Ref: Harrison’s 18/e p2147, 17/e p1632
MEDICINE
Ans. 44. d. Sterility is common... 45. c. Diabetes mellitus 46. b. XXY 47. c. MEN 2
48. d. Heriditary sensory... 49. a. Absent tonsils and no... 50. d. All of the above 51. a. Choloroquine
12. MISCELLANEOUS
Miscellaneous 1197
MISCELLANEOUS (QUESTIONS)
MEDICINE
7. The following feature is common to both cytotoxic T cells c. Digeorge syndrome
and NK cells: (AIIMS 02) d. C8 deficiency Ref: Harrison’s 15/e p1849, 2059
a. Synthesize antibody 15. Humoral immunodeficiency is suspected in a patient and
b. Require antibodies to be present for action he is under investigation. Which of the following infections
c. Effective against virus infected cells would not be consistent with the diagnosis:
d. Recognize antigen in association with HLA class II a. Giardiasis (AIIMS Nov 04)
markers b. Pneumocystis carinii pneumonia
Ref: Harrison’s 18/e p2688, 2669 c. Recurrent sinusitis
8. The marker for B lymphocyte is: (AIIMS 2000) d. Recurrent subcutaneous abscesses
a. CD19 Ref: Harrison’s 18/e p2699
b. CD68
Ans. 1. a. IgG1 2. a. C2, C4 and C3 decreased... 3. a. Astrocytes 4. d. Class III MHC does...
5. b. Decreased size of... 6. d. There is antibody induced... 7. c. Effective against virus... 8. a. CD19
9. a. T lymphocyte... 10. a. T cells 11. b. IgE mediated reaction 12. b. Severe combined...
13. a. Severe combined... 14. b. Common variable... 15. b. Pneumocystis carinii...
1198 Jaypee’s Triple A
16. Ataxaia telengiectasia is associated with all of the following c. Rheumatoid arthritis
except: (AI 2004) d. Uremia Ref: Harrison’s 18/e p2180, 17/e p1763
a. Recurrent sinopulmonary infections 24. The following are characteristics of central fever except that
b. Lymphatic reticular malignancies there is: (DP PGMEE 2010)
c. Increased fraction of IgA immunoglobulins a. No diurnal variation
d. Insulin resistance b. No sweating
Ref: Harrison’s 18/e p3344, 17/e p2057, 2058 c. Decreased response to external cooling
17. Which one of the following is not used as a tumor marker in d. Resistant to antipyretics Ref: Harrison’s 17/e p1716
testicular tumors? (AI 2005) 25. An indian adult who has never travelled abroad comes with
a. AFP a history of high fever, headache, jaundice, marked oliguria
b. LDH and shock with TLC of 16, 000/cumm. The most likely
c. HCG diagnosis is: (DP PGMEE 2010)
d. CEA Ref: Harrison’s 18/e p652, Table 81-6 a. Viral hepatitis
18. A 60 year old male presented to the emergency with b. Leptospirosis (Weil’s disease)
breathlessness, facial swelling and dilated veins on the c. Yellow fever
chest wall. The most common cause is: (AI 2003) d. Haemolytic uraemic syndrome
a. Thymoma Ref: Harrison’s 18/e p1393-1395, 17/e p1050
b. Lung cancer 26. In human body, which of the following trace elements is
c. Hodgkin’s lymphoma next to iron: (DP PGMEE 2010)
d. Superior vena caval obstruction a. Ca2+
Ref: Harrison’s 18/e p2266, 17/e p1730 b. Zn2+
19. A man presents with mass at duodenojejunal flexurs c. Cu2+
invading renal papillae. Histopathology reports it as d. Selenium Ref: OP Ghai 6/e p123
lymphoma; true statement is: (AI 2001) 27. Drug of choice in intractable hiccoughs is:
a. II E stage a. Metoclopramaide (DP PGMEE 2010)
b. III E Stage b. Haloperidol
c. IV E stage c. Thioridazine
d. Staging cannot be done until bone marrow examination d. Chlorpromazine Ref: KDT 5/e p399, Internet
is performed Ref: Meningot 10/e p1181
28. Sequestration lung is best diagnosed by: (DP PGMEE 2010)
20. Which of the following viruses does not produce viral a. C.T Scan
esophagitis? (DP PGMEE 2009) b. M. R. I
a. Herpes simplex c. Barium swallow
b. Adenovirus d. Angiography Ref: Sumer sethi 4/e p35
c. Varicella
29. The most strongly implicated premalignant condition of
d. Cytomegalovirus
the oral cavity is: (Comed K 2010)
Ref: Harrison’s 18/e p2436, 17/e p1852-1853
a. Fordyce spots
21. Which of the following of drugs is not recommended in b. Erythroplakia
septic shock? (DP PGMEE 2009) c. Median rhomboid glossitis
a. Normal saline d. Erythema mültiforrme
b. Activated protein C Ref: Harrison’s 17/e p217, Table 32.1-32.4
c. Steroids
30. Kyasanur Forest Disease (KFD) is transmitted by:
d. Rituximab Ref: Harrison’s 18/e p2223, 17/e p1699-1701
a. Fleas (Comed K 2010)
22. Which of the following manifestations is not a clinical
MEDICINE
b. Mosquitoes
manifestation of parvovirus infection? (DP PGMEE 2009) c. Ticks
a. Erythema infectiosum d. Mites Ref: Harrison’s 17/e p1236, Table 189-4
b. Polyarthropathy
31. In primary immune deficiency the following plasma protein
c. Pure red cell aplasia
fraction is reduced: (Comed K 2010)
d. Tropical spure
a. a-1-globulin
Ref: Harrison’s 18/e p1435, 1478, 17/e p1116-1117
b. a-2-globulin
23. Which of the following conditions can produce c. b-globulin
hemothorax? (DP PGMEE 2009) d. g-globulin
a. Myxoma Ref: Harrison’s 18/e p2696 Textbook of microbiology,
b. Congestive heart failure CP Baveja, 3/e p147
Ans. 16. c. Increased fraction... 17. d. CEA 18. d. Superior vena caval... 19. c. IV E stage
20. b. Adenovirus 21. c. Steroids 22. d. Tropical spure 23. d. Uremia
24. c. Decreased... 25. b. Leptospirosis... 26. b. Zn2+ 27. d. Chlorpromazine
28. d. Angiography 29. b. Erythroplakia 30. c. Ticks 31. d. g-globulin
Miscellaneous 1199
32. Which of the following is TRUE regarding classical c. Infectious mononucleosis
spontaneous bacterial peritonitis? (Comed K 2010) d. Chronic myeloid leukemia
a. Ascitic fluid neutrophil count is > 250/cu mm Ref: Davidson’s 20/e p307; Harrison’s 17/e p1108, 18/e p1467-1468
b. Bowel perforation should be present 40. Mental retardation is NOT a feature of one of the
c. Multiple organisms are isolated from ascetic fluid Mucopolysaccharidosis: (Comed K 2011)
d. Board like rigidity is present in abdomen a. Hurler MPS-I
Ref: Harrison’s 17/e p1979 b. Hunter MPS -II
33. Fresh water swimming can lead to: (Comed K 2010) c. Sanfillipo MPS -III
a. Brucellosis d. Morquio MPS —IV
b. Leptospirosis Ref: Harrison’s 17/e p2452, table 355-1
c. Babesiosis 41. Wernicke’s hemianopic pupillary response is seen in lesion
d. Lassa fever of: (Comed K 2011)
Ref: Harrison’s 18/e p1393 a. Optic tract
34. Which of the following components is not considered in the b. Optic chiasm
definition of ‘Metabolic syndrome’? (DP PGMEE 2009) c. Optic radiation
a. High LDL d. Lateral geniculate body
b. Low HDL Ref: Parson 19/e p528, 120
c. Abdominal obesity 42. Dissociated sensory loss is seen in: (Comed K 2011)
d. Hypertension a. Syringomyelia
Ref: Harrison’s 18/e p1992, 17/e p1509-1510 b. Vitamin B12 deficiency
35. Heyde’s syndrome is: (Comed K 2010) c. Transverse myelitis
a. Mitral stenosis, arthritis and biliary cirrhosis d. Pellagra
b. Mitral regurgitation, hiatus hernia and cirrhosis Ref: Davidson’s 21/e p1157
Aortic stenosis, gastrointestinal bleeding and 43. All of the following immunosuppressives cause profound
angiodysplasia of colon myelosuppression except: (Comed K 2011)
d. Pulmonary arterial hypertension, tricuspid regurgitation a. Sirolimus
and cirrhosis b. Cyclosporine
Ref: http://en.wikipedia.org/wiki/Heyde’s_syndrome c. Azathioprine
36. All the following statements are TRUE regarding methyl d. Mercaptopurine
alcohol poisoning except: (Comed K 2010) Ref: KD Tripathi, 6/e p839
a. It is characterized by high anion gapacidosis 44. Localization in insulinoma is best with: (Comed K 2011)
b. It causes ocular toxicity a. Contrast Computerised Tomography
c. It causes high osmolal gap b. Magnetic Resonance Imaging
d. Concurrent ethanol ingestion worsens the clinical c. Somatostatin Receptor Scintigraphy
features d. Selective arteriography
Ref: Harrison’s 18/e p366, KD Tripathi, 6/e p387; Davidson’s 20/e p219
45. Hallervorden-Spatz disease: (AP 2011)
37. Which of the following is NOT used in post exposure a. Is an iron storage disorder in brain
prophylaxis for HIV? (Comed K 2010) b. Most commonly occurs in adults
a. Zidovudine + Laniivudine c. Relentlessly non progressive disease
b. Nelfinavir d. Inherited as Autosomal dominant
c. Stavudine + Didanosine
46. HIV …………………… classification 4 doesn’t include?
d. Acyclovir
a. Oral candidiasis (AP 2011)
MEDICINE
Ref: KD Tripathi, 6/e p776
b. Pneumocystis carini pneumonia
38. Hepatitis B virus is NOT present in: (Comed K 2010) c. Wasting syndrome
a. Milk d. Invasive cervical carcinoma
b. Sweat Ref: Harrison’s 18/e p1506, 1507
c. Stool
47. Menkes syndrome is a disorder of: (AP 2011)
d. Lymph
a. Copper transport
39. A 20-year-male presents with fever, fatigue, posterior cervical b. Iron Transport
lymphadenopathy, palatal petechiae and splenomegaly. c. Sodium transport
Peripheral smear shows atypical lymphocytes and d. Potassium Transport
heterophile antibody is positive. The likely diagnosis is: Ref: Harrison’s 18/e p604
a. Acute leukemia (Comed K 2010)
b. Lymphocytic leukemia
Ans. 32. a. Ascitic fluid... 33. b. Leptospirosis 34. a. High LDL 35. c. Aortic stenosis...
36. d. Concurrent ethanol... 37. d. Acyclovir 38. d. Lymph 39. c. Infectious mononucleosis
40. d. Morquio MPS —IV 41. a. Optic tract 42. a. Syringomyelia 43. b. Cyclosporine
44. d. Selective... 45. a. Is an iron storage disorder... 46. a. Oral candidiasis 47. a. Copper transport
1200 Jaypee’s Triple A
48. Not caused by echo virus? (AP 2011) 56. Management of febrile neutropenic patients on anti cancer
a. Herpagina therapy include all except: (AP 2010)
b. Pleurodynia a. Colony stimulating factors
c. Hemorrhagic fever b. Transfusion of granulocytes
d. Myocarditis c. Antibiotic Prophylaxis
Ref: Harrison’s 18/e p1028 d. Repeated hand washing
49. Patient on risperidone case: (AP 2011) 57. Corpulence index means: (Rajasthan 2008)
a. Weight loss a. Measurement of copper level in the serum
b. Neuroleptic malignant syndrome b. Measurement of obesity
c. Hypopigmentation c. Measurement of iron loss in faeces
d. Prolactin deficiency d. Pressure difference between various chambers of the
Ref: Harrison’s 18/e p144, 3333 heart
50. All of the following are known to worsen hyperkalemia: Ref: Park 20/e p348
a. Spironolactone (AP 2010) 58. Which of the following statement is false? (Rajasthan 2008)
b. ACE inhibitors a. Sleepapneas can be central (CSA) or obstructive (OSA)
c. Furosemide b. In CSA the neural drive to all respitory muscles is
d. Amiloride transiently abolished
51. All are true statements about “Beheet’s syndrome” except: c. In OSA airflow ceases despite contuning respitatory
a. Recurrent aphthous ulceration are a sine qua non for the drive because of occlusion of oropharyngeal airway
diagnosis (AP 2010) d. None of the above
b. In endemic areas the syndrome is associated with Ref: Harrison’s 18/e p220, 2189, 2190
HLA-B5 59. Pariarteriole space is formed in spleen by: (Rajasthan 2008)
c. Neurological involvement are seen in 5-10% a. T lymphocyte
d. Superficial or deepperipheral vein thrombosis b. B lymphocyte
Ref: Harrison’s 18/e p2801 c. Rete
52. Which one of the following drugs is the most common d. Macrophage
cause of progressive multifocal leukoencephalopathy? Ref: Gray’s Anatomy 40/e p1194
a. Natalizumab (AP 2010) 60. On the basis of BMI obesity is labelled at: (Rajasthan 2008)
b. Cituximab a. 20
c. Rituximab b. 25
d. Bevacizumab c. 30
Ref: Harrison’s 18/e p3427 d. 18
53. Antidote of paracetamol is: (AP 2010) Ref: Harrison’s 18/e p608, 622, 630
a. N-acetylcysterine 61. What is the eosinophil count for hypereosinophilic
b. Methylene blue syndrome? (Rajasthan 2008)
c. EDTA a. > 1500/ml
d. No effective antidote known b. > 500
Ref: Goodman Gilman 11/e p694 c. > 1000
54. Pseudoporphyria can be induced by all except: (AP 2010) d. > 2000 Ref: Harrison’s 18/e p481, 3459
a. Methotrexate 62. Which is false about a1-antitrypsin? (Rajasthan 2008)
b. Infliximab a. A1-antitryprin is mainly produced in Liver
c. Naproxen b. Muttion that causes severe a1-antitrypsin deficiency
MEDICINE
Ans. 48. c. Hemorrhagic fever 49. b. Neuroleptic malignant... 50. c. Furosemide 51. Is none
52. a. Natalizumab 53. a. N-acetylcysterine 54. a. Methotrexate 55. a. SLE – APL
56. b. Transfusion of... 57. b. Measurement of obesity 58. d. None of the above 59. a. T lymphocyte
60. b. 25 61. a. > 1500/ml 62. d. None of the above 63. a. Staphylococcus
Miscellaneous 1201
64. Human mast cells originate from: (Rajasthan 2008) b. Amyotrophic lateral sclerosis
a. CD34+ c. Cerebellar degeneration
b. CD 4+ d. Opsoclonus – myoclonus
c. CD 8+ Ref: Harrison’s 18/e p826, 830
d. All of the above 73. Commonest site of metastases in: (Rajasthan 2009)
65. Hamman’s CRUNCH is seen in: (Rajasthan 2009) a. Lung
a. Caries tooth b. Brain
b. Fracture mandible c. Liver
c. Mediastinal emphysema d. Kidney
d. Nieman pick-diseases Ref: Harrison’s 18/e p773, 785
66. Which of the following does not cause obesity? 74. Arsenic is associated with increased risk of:
a. Prader willi (Rajasthan 2009) a. Mesothelioma (Rajasthan 2009)
b. Laurence moon biedil b. Melanoma
c. Carpenter c. Basal cell carcinoma
d. Denys Drash d. Squamous cell carcinoma
67. Which is integrase inhibitor in HIV therapy: Ref: Harrison’s 18/e p2123, 2124
a. Maraviroc (Rajasthan 2009) 75. The most common ophthalmic affection of diphtheria is:
b. Enfuviritide a. Plosis (Rajasthan 2009)
c. Raltegravir b. Total ophthalmoplegia
d. Tegaserode c. Isolated ocular palsies
Ref: Harrison’s 18/e p158, 1850 d. Ophthalmoplegia externa
68. A person came from visiting Thailand which is Ref: Harrison’s 18/e p1190
chemoprophylaxis for malaria: 76. Which of the following is the commonest infection which
a. Primaquine causes blindness in adult men: (Rajasthan 2009)
b. Chloroquine a. Toxocara
c. Artemether b. Toxoplasma gondii
d. Mafloquine Ref: Harrison’s 18/e p1703-1705 c. Tenia solium
69. Which of the following carcinoma is associated with d. Plasmodium falciparum
Nickel: (Rajasthan 2009) Ref: Harrison’s 18/e p229, 1724, 1726
a. Lung 77. The transmission of AIDS transplacentally:
b. Larynx a. 10-20% (Rajasthan 2009)
c. Ethmoid b. 20-30%
d. Nasopharynx Ref: Harrison’s 18/e p656, 733, 737 c. 30-40%
70. Indication for changing retroviral drug are all except: d. 40-50%
a. <1 log dropin plasma HIV RNA by 4 week following the Ref: Harrison’s 18/e p956, 1585
initiation of therapy (Rajasthan 2009) 78. True about HIV in the neonate includes all the following
b. Side effect except: (Rajasthan 2009)
c. Clinical deterioration a. Cannot be diagnosed accurate by current methods
d. CD4 count <200 b. Failure to thrive may be presentation
Ref: Harrison’s 18/e p1582; Table 189-24 c. Transmission rate during pregnancy exceeds 90%
71. Which complication will occur in neurosyphilis after 25-30 d. Transmission vertically from mother
MEDICINE
years? (Rajasthan 2009) Ref: Harrison’s 18/e p976-977, 1519, 1585
a. Meningeal syphilis 79. Infant of HI V +ve mother, which should be done:
b. General paresis a. BCG vaccine should not be give (Rajasthan 2009)
c. Meningovascular syphilis b. Azt therapy
d. Tabes dorsalis c. Separation from mother
Ref: Harrison’s 18/e p1384, 3375 d. Other than BCG.all over vaccines are given
72. Para neoplastic syndrome are all except: (Rajasthan 2009) Ref: Harrison’s 18/e p61, 53
a. Progressive multi focal leukoencephalopathy
Ans. 64. a. CD34+ 65. c. Mediastinal emphysema 66. d. Denys Drash 67. c. Raltegravir
68. b. Chloroquine 69. c. Ethmoid 70. d. CD4 count <200 71. d. Tabes dorsalis
72. a. Progressive multi... 73. c. Liver 74. a. Mesothelioma 75. c. Isolated ocular palsies
76. b. Toxoplasma gondii 77. c. 30-40% 78. a and b 79. b. Azt therapy
1202 Jaypee’s Triple A
80. MC complication of chicken pox in children: 88. Medical disorder that mimic anxiety disorders are all
a. Encephalitis (Rajasthan 2009) except: (UP 2009)
b. Sec.bacterial infection a. Hyperthyroidism
c. Pneumonia b. Pheochromocytoma
d. Otitis media c. Alcohol withdrawl
Ref: Harrison’s 18/e p1463, 1464 d. Cornonary artery disease
81. True about koilocytosis: (Rajasthan 2009) Ref: Neeraj Ahuja 6/e p98
a. Glycogen containing cells 89. All are true about Asteatotic eczema except: (UP 2009)
b. Cytopathic effect of virus a. Also known as Xerotic eczema
c. HPV b. Pruritus
d. HSV c. Responds well to topical moisturizers
Ref: Harrison’s 18/e p1482 d. Leads to premalignant condition
82. True about chicken pox: (Rajasthan 2009) Ref: Harrison’s 17/e p314; Roxburg 17/e p118-19
a. IP.2-3 days 90. Magnesium deficiency can cause all, except: (AP 2012)
b. Scabs are infective a. Impaired synthesis of 1, 25 (OH)2D
c. Centrifugal rash b. Cellular resistance to PTH
d. Rash appears of first day c. Hypokalemia
Ref: Harrison’s 18/e p415, 1463 d. Hyperkalemia
83. Which positive test done not necessarily indicate HIV Ref: Harrison’s 18/e p370, 3105
infection in a newborn? (Rajasthan 2009) 91. Pemphigus vulgaris, antibodies against:
a. ELISA for HIV lgG antibody a. Basement membrane (UP 2006 and 2009)
b. p24 antibody b. Intercellular substance
c. Virus culture c. Laminin membrane
d. ELISA for HIV I Ga antibody d. Sub epidermal
Ref: Harrison’s 18/e p1538 Ref: Harrison’s 18/e p425
84. Rosela infantum can be caused by: (Rajasthan 2009) 92. Treatment of Tinea versicolor are all except: (UP 2009)
a. Herpes virus 6 a. Selenium sulfide
b. Parvovirus B 19 b. Ciclopirox cream
c. Echovirus 19 c. Ketoconazole
d. All d. Corticosteroid
Ref: Harrison’s 18/e p148, 149 Ref: Harrison’s 17/e p1263
85. Which of the following test is prognostic value in syphilis 93. Folic acid supplemention is to reduce: (UP 2009)
disease: (UP 2011) a. Neural tube defects
a. TPI b. Cleft palate and harelip
b. FTA-ABS c. omocystinuria
c. VDRL d. All of the above
d. TPHA test Ref: Harrison’s 17/e p645 – 651
Ref: Harrison’s 18/e p1384-1385 94. All of the following are major features to diagnose atopic
86. Most common presentation of obsessive compulsive Dermatitis except: (MP PG 2010)
disorder is: (UP 2011) a. Pruritus
a. Symmetry b. Family history
c. History of asthma
MEDICINE
b. Pathological behaviors
c. Intrusive thought d. Pityriasis alba
d. Contamination Ref: Harrison’s 17/e p218
Ref: Harrison’s 18/e p3535; 17/e p344 95. The most common from of drug rash is: (MP PG 2010)
87. All are true about Nosocomial infections except: (UP 2009) a. Exanthematic
a. Reduces host immunity of the Diabetes mellitus b. Fixed drug eruption (FDE)
b. Tobacco - smoking c. Steven Johnson Syndrome (SJS)
c. Aspiration of GI material d. Urticarial
d. Endoscopy – aspiration cystoscopic Ref: Harrison’s 17/e p345
Ref: Harrison’s 17/e p835
Ans. 80. b. Sec. bacterial... 81. c. HPV 82. d. Rash appears of first day 83. a. ELISA for HIV lgG antibody
84. d. All 85. c. VDRL 86. d. Contamination 87. b. Tobacco - smoking
88. c. Alcohol withdrawl 89. d. Leads to premalignant... 90. d. Hyperkalemia 91. b. Intercellular substance
92. d. Corticosteroid 93. d. All of the above 94. d. Pityriasis alba 95. a. Exanthematic
Miscellaneous 1203
96. Patch test is best read at: (MP PG 2010) 104. “Anhedonia” means: (MP PG 2008)
a. 1 days a. Lack of interest
b. 2 days b. Panic attacks
c. 3 days c. Phobic attack
d. 4 days Ref: Harrison’s 17/e p312 d. Mood swings
97. A sexually active male comes with multiple painful ulcers, Ref: N. Ahuja 5/e p56, 58
what is the most likely diagnosis: (MP PG 2010) 105. Among the following, earliest test to be positive in
a. Chancre leptospirosis is: (MP PG 2008)
b. Chancroid a. Microagglutination test (MAT)
c. Llymphogranuloma b. Leptospira culture of blood
d. Aphthosis c. IgM ELISA of serum
Ref: Harrison’s 17/e p215, 832, 18/e p1108, Table 130-7 d. Macroagglutination test
98. Penicillamine is an antimetabolite of: (MP PG 2009) Ref: Harrison’s 17/e p1050
a. Vitamin B1 106. All the following third generation cephalosporin’s have
b. Vitamin B6 significant activity against Pseudomonas Species except:
c. Folic Acid a. Ceforperazone
d. Cyancocobalamine b. Ceftazidime
Ref: Harrison’s 18/e p599; Goodman Gilman’s c. Cefotaxime
pharmacology 11/e p1771 d. Cefepime
99. Which one of the following statements regarding Ref: Harrison’s 17/e p952, 18/e p1269, Table 152-2
hyperuricemia in malignancy if false”: (MP PG 2009) 107. 3 % hypertonic saline contains which of the following
a. It poses a major problem in myeloproliferative disorder sodium concertration in mEq per litre?
b. It is commonly encountered in solid toumours following a. 510
systematic chemotherapy b. 512
c. Vigourous hydration and diuresis is the corerstone in c. 514
treatment d. 516
d. Alkalinisation of urine should be maintained Ref: A.K. Jain’s physio 3/e p26, 30
Ref: Harrison’s 17/e p1736 108. 14 years old girl with fever diarrhea, pneumonia, confusion,
100. The most commonly associated tumor maker in embryonal organism responsible: (WB PG 08)
carcinomas and yalk sac tumors is: (MP PG 2009) a. Meningococcus
a. Carcino Embryonic Antigen (CEA) b. Streptococcus
b. Placental alkaline phosphatase (PAP) c. Pneumococcus
c. Alpha Fetoprotein (AFP) d. Legionella
d. Cancer Antigen 15-3 (CA 15-3) Ref: Harrison’s 17/e p943, 18/e p1238
Ref: Robbins Basis 7/e p338 109. A person got needle prick with HIV positive blood;
101. “Pellagra” like skin lesons occurs with: (MP PG 2008) prophylaxisis: (WB PG 08)
a. Systemic Mastocytosis a. Zidovudine + Lamivudine + Indinavir
b. Bronchial carcinoid b. Zidovudine + Stavudine
c. Colorectal villous adenoma c. Stavudine + Didanosin
d. Medullary thyroid carcinoma d. Lamivudin + Didanosine
Ref: Harper’s 27/e p458 Ref: KDT 6/e p776; Harrison’s 17/e p1201, 18/e p1583
102. Which structure involvement is uncommon in” churgstmuss 110. Most Common Nosocomial infection: (WB PG 08)
MEDICINE
syndrome? (MP PG 2008) a. UTI
a. Skin b. Pneumonia
b. Lungs c. Wound infection
c. Kidney d. Bed sore
d. Periphereal nerve Ref: Harrison’s 17/e p2124 Ref: Harrison’s 17/e p837-838
103. Which channel is defective in “Malignant Hyper pyrexia”? 111. Diagnostic criteria for SIRS include all except:
a. Sodium (MP PG 2008) a. Hypotension (Kerala PG 09)
b. Potassium b. Tachycardia
c. Calcium c. Tachypnoea
d. Magnesium d. Leucocytosis
Ref: Harrison’s 17/e p118, 18/e p144 Ref: Harrison’s 18/e p2223, Table 271-1
Ans. 96. b. 2 days 97. b. Chancroid 98. b. Vitamin B6 99. b. It is commonly encountered...
100. c. Alpha Fetoprotein... 101. b. Bronchial carcinoid
102. a. Skin 103. c. Calcium
104. a. Lack of interest 105. c. IgM ELISA of serum 106. c. Cefotaxime 107. d. 516
108. d. Legionella
109. a. Zidovudine + Lamivudine... 110. a. UTI 111. a. Hypotension
1204 Jaypee’s Triple A
112. Bioterrorism groupA organism are all except: a. IV dextrose
a. Ebola (Kerala PG 09) b. IV normal saline
b. Botulism c. Insulin
c. Rickettsia d. IV potassium
d. Yersinia Ref: Harrison’s 17/e p2284; Table 338-6
Ref: Harrison’s 17/e p1343, 18/e p1769, Table 221-2 121. 50 year old male bought in coma, BP-220/140mHg, Pulse-100/
113. Which of the following is a vector for Dengue fever: mt, No FNd) taken-normal study; Treatment should include:
a. Aedes aegypti (Kerala PG 09) a. S/c nifidipine (Kerala PG 10)
b. Culex b. IV Na nitroprusside
c. Anophelous c. Oral Amlodipine
d. Mansonia d. IV Mannitol
Ref: Harrison’s 17/e p1230
122. 25 year old male presented with high grade fever &
114. Disease due to deficiency of B Complex vitamins is: hypotension Hb-5g% Tc-9000/mm3 DC-P2 L96 E2 treatment
a. Pellagra (Kerala PG 09) should include all except? (Kerala PG 10)
b. Night blindness
a. Oral ciprofloxacin
c. Osteomalacia
b. Packed cell transfusion
d. Spinocerebellar ataxia
c. IV ciprofloxacin
Ref: Harrison’s 17/e p442
d. Colony stimulating factor
115. All the drugs are used in the treatment of Giardiasis except: Ref: Harrison’s 17/e p539
a. Nitazoxanide (Kerala PG 09)
123. Photosensitive reaction is caused by all except: (UP 2009)
b. Metronidazole
a. Chlorpromazine
c. Tinidazole
b. Thazide quinolones
d. Diloxanide furoate
c. Pio-glitazone
Ref: Harrison’s 17/e p1313, 18/e p1731
d. Tetracycline
116. False about lymphoma in AIDS patients: (Kerala PG 09) Ref: Goodman Gilman 11/e p481, 1741, 756
a. Intranodal
b. Extranodal 124. Conus lesion characterised by? (Kerala PG 10)
c. Highly aggressive a. Early loss of ankle jerk
d. Less responsive to treatment b. Rectal Pain
Ref: Harrison’s 17/e p1188, 1189 c. Loss of anal reflex
d. Loss of Potency
117. Most common test for HIV is: (Kerala PG 09)
a. ELISA Ref: Harrison’s 17/e p2589, 18,/e, p3367
b. Western blot 125. Which one of the following is NOT a 5HT receptor antagonist?
c. Northern blot a. Ketanserin (AP 2012)
d. Virus isolation b. Methysergide
Ref: Harrison’s 17/e p1165 c. Tropisetron
118. In septicaemia all are bad prognostic signs except: d. Lanreotide Ref: Goodman Gilman 11/e p312, 313, 1001
a. Hypoxia (Kerala PG 09) 126. Which is true about Diaphragmatic Paralysis?
b. Lactate levels (Rajasthan 2008)
c. Leucocytosis a. The most common cause of “Unilateral paralysis of
d. Hypoglycemia diaphragm” is nerve invasion from malignancy
Ref: Harrison’s 17/e ped/ p.1696, 18/e p2228 b. “Sniff test” is used for the confirmation of unilateral
paralysis of diaphragm
MEDICINE
119. All of the following are true regarding drug induced lupus
except: (Kerala PG 10) c. Most patients presented with hypercapnic respitatory
a. Anti histone antiobodies present failure, atelectasis & pneumonia
b. Rarely anti ds DNA positive d. All of the above
c. Renal failure 127. In significant bacteriuria, count will be: (Kerala PG 10)
Ref: Harrison’s 17/e p2284 table 338-6 a. > 105/ml.
120. 20 year old male brought to casualty in state of Coma. RBS- b. > 103
493, Na-140 K-4meq/1 op2 90 pco2 10. Treatment should c. > 102
include all except? (Kerala PG 10) d. > 104
Ref: Harrison’s 18/e p2393
Ans. 112. c. Rickettsia 113. a. Aedes aegypti 114. a. Pellagra 115. d. Diloxanide furoate
116. d. Less responsive... 117. a. ELISA
118. c. Leucocytosis 119. c. Renal failure
120. a. IV dextrose 121. b. IV Na nitroprusside 122. a. Oral ciprofloxacin 123. c. Pio-glitazone
124. c. Loss of anal reflex 125. d. Lanreotide
126. d. All of the above 127. a. > 105/ml
Miscellaneous 1205
128. All are viral causes of PUO except: (Kerela PG 2008) 136. Erythema multiforme can be caused by following except?
a. EBV a. Sulfonamides (MHPGM-CET 2010)
b. CMV b. Mycoplasma
c. HIV c. Herpes
d. Leptospirosis d. Leprosy
Ref: Harrison’s 18/e p161; Table 18-3 Ref: Harrison’s 17/e p328
129. Which of the following is the drug of choice for 137. Neurogenic tumor occurs most commonly in:
oropharyngeal and esophageal candidiasis: a. Anterior mediastinum (MHPGM-CET 2010)
a. Fluconazole (Kerela PG 2008) b. Middle mediastinum
b. Nystatin c. Posterior mediastinum
c. Amphotercin d. Superior mediastinum
d. Griseofulvin Ref: Harrison’s 17/e p1660
Ref: Harrison’s 16/e p1187, 17/e p1255, 18/e p1653 Table 203-1 138. Calymmatobacter granulomatics causes:
130. All of the following conditions will have hypermetabolic a. Cat scratch disease (MHPGM-CET 2010)
state except: (Kerela PG 2008) b. Condyloma lata
a. Accident c. Donovaniasis
b. Burns d. Chancre
c. PEM Ref: Harrison’s 17/e p991
d. Anorexia nervosa 139. Which of the following is not a Non-nucleoside reverse
Ref: Harrison’s 16/e p412, 17/e p477 transcriptase inhibitor? (MHPGM-CET 2010)
131. Deficiency of vit D will cause: (Kerela PG 2008) a. Zalcitabine
a. Scurvy b. Lamivudine
b. Rickets c. Nevirapine
c. Phrynoderma d. Didanosine
d. Beri beri Ref: Harrison’s 17/e p1191; 18/e p1571 Table 182-18, Table 189-20
Ref: Harrison’s 17/e p2375 140. Which antimalarial drug is known to cause neuropsychiatric
132. Kaposi’s sarcoma is associated with: (Kerela PG 2008) reaction as its adverse effect? (MHPGM-CET 2007, 2010)
a. Bronchial asthma a. Artesunate
b. HIV b. Artimisnin
c. Hepatitis B c. Quinine
d. Hairy cell leukaemia d. Mefloquine
Ref: Harrison’s 16/e p1120; 1121; 17/e p1113; 18/e p1564 Ref: Harrison’s 17/e p1288; 18/e p Table 210-7
133. Hospital acquired pneumonia is infection occurring after 141. Which of the following anti-malarial drugs kills all stages
how many days of hospital admission? (Kerela PG 2008) of gametocytes? (MHPGM-CET 2010)
a. 1 days a. Quinine
b. 2 days b. Mefloquine
c. 4 days c. Artesunate
d. 1 week d. Primaquine
Ref: Harrison’s 16/e p1538, 17/e p1619 Ref: Harrison’s 17/e p1289; Table 203-7
134. Acute gonococcemiais also called purpura fulminans 142. Hypokalemia with hypertension is associated with
because: (Kerela PG 2008) following except: (MHPGM-CET 2010)
a. DIC is present a. Liddle’s syndrome
MEDICINE
b. Leads to the formation of purpura or petechiae b. Conn’s syndrome
c. Most important cause of septic shock c. Bartter’s syndrome
d. The purpuria is d. Cushing’s syndrome
Ref: Harrison’s 16/e p710, 851; 17/e p910; 18/e p155 Table 17-1 Ref: Harrison’s 18/e p2357, 2946, 2949; Table 284-2
135. Neuroleptic malignant syndrome can be treated by: 143. First line drugs used for treating neuropathic pain are
a. Dantrolene (MHPGM-CET 2010) following except: (MHPGM-CET 2010)
b. Bromocryptine a. Lidocaine
c. Nifedipine b. Gabapentin
d. All of the above c. Duloxetine
Ref: Harrison’s 17/e p121; 18/e p3333 d. Opioids
Ref: Harrison’s 18/e p101
Ans. 128. d. Leptospirosis 129. a. Fluconazole 130. d. Anorexia nervosa 131. b. Rickets
132. b. HIV
133. b. 2 days 134. a. DIC is present 135. d. All of the above
136. d. Leprosy 137. c. Posterior mediastinum 138. c. Donovaniasis 139. c. Nevirapine
140. d. Mefloquine
141. d. Primaquine 142. c. Bartter’s syndrome 143. d. Opioids
1206 Jaypee’s Triple A
144.
Drug effective pharmacologic interventions for d. Uncontrolled increase in body temperature despite a
management of smoking cessation: (MHPGM-CET 2010) normal hypothalamic temperature setting
a. Bupropion Ref: Harrison’s 17/e p201, 18/e p143
b. Varenicline 152. The other name for dengue fever is:
c. Clonidine a. Break-bone fever (Karnataka 2010)
d. All of the above b. Chikungunya
Ref: Harrison’s 18/e p738 c. Yellow fever
145. Which of the following can be given as a single weight- d. Kyasanur-forest fever
based intravenous bolus over 10 seconds? Ref: Harrison’s 17/e p1230, 18/e p1622
a. Tissue plasminogen activator (tPA) (Maharashtra 2011) 153 Mild steatorrhea is caused by: (Karnataka 2010)
b. Tenecteplase (TNK) a. Trichomonas vaginalis
c. Reteplase (rPA) b. Giardia lamblia
d. Any of the above c. E. Histolytica
Ref: Harrison’s 17/e p1537 d. Entamoeba coli Ref: Harrison’s 18/e p1730
146. Best drug for chemoprophylaxis in a pregnant woman who 154 Acute infectious purpura fulminans is caused by:
is traveling to chloroquine resistant falciparum malaria a. Neisseria meningitidis and Varicélla
endemic zone? (Maharashtra 2011) b. Gonococci (Karnataka 2010)
a. Mefloquine c. E. coli
b. Doxycycline d. Proteus Ref: Harrison’s 18/e p1214, 155
c. Ataquinone/Proguanil
155. Complement deficiency predisposes to infection with:
d. Primaquine
a. Pseudomonas aeruginosa (Karnataka 2011)
Ref: Harrison’s 18/e p1703
b. Cytomegalovirus
147. Prazosin is effective in management of bite by? c. Neisseria meningitidis
a. Viper snake (Maharashtra 2011) d. Giardia lamblia
b. Naja Naja Ref: Harrison’s 17/e p2043, and table 308-15: davidson’s, 21/e p77
c. Mesobuthus tamulus
156. All the following are associated with obesity except:
d. All of the above
a. Hypothyroidism (Karnataka 2011)
Ref: Harrison’s 17/e p2752, 18/e p3581
b. Cushing’s syndrome
148. AFP is tumor marker for? (Maharashtra 2011) c. Insulinoma
a. Hepatoma d. Addison’s disease
b. Seminoma Ref: Harrison’s 18/e p2919, 2946; Davidson’s 21/e p117, box 5.25
c. Neural tube defects
157. Hutchinson’s triad consists of Hutchinson’s teeth, 8th nerve
d. All of the above
deafness and: (Karnataka 2011)
Ref: Harrison’s 17/e p483
a. Interstitial keratitis
149. Henoch Schnolein Purpura is characterized by following b. Saddle nose
except: (Maharashtra 2011) c. Clutton’s joints
a. Palpable purpura d. Sabre tibiae Ref: Harrison’s 18/e p1384
b. Renal failure
158. Perifollicular & petechial haemorrhages are characteristic
c. GIT hemorrhage
of: (Karnataka 2011)
d. Thrombocytopenia
a. Acrodermatitis enteropathica
Ref: Harrison’s 17/e p2128
b. Pellagra
150. Normal body mass index is: (Karnataka 2010) c. Scurvy
a. 18.5 to 24.99
MEDICINE
d. Phrynoderma
b. 15 to18 Ref: Harrison’s 17/e p445, 18/e p599
c. 25.00 to 29.99
159. All are indications for splenectomy, except:
d. Greater than 30
a. Herediatary spherocytosis (Feb DP PGMEE 2009)
Ref: Davidson’s 20/e p113, Table 5.23
b. Hairy cell leukemia
151. Hyperthermia is defined as: (Karnataka 2010) c. ITP
a. Core temperature > 40.0°C d. Chediak higashi syndrome
b. Core temperature > 41.50°C Ref: Harrison’s 18/e p599, 478
c. Elevated temperature that normalizes with anti-pretic
dugs
Ans. 144. d. All of the above 145. b. Tenecteplase (TNK) 146. a. Mefloquine 147. c. Mesobuthus tamulus
148. d. All of the above
149. d. Thrombocytopenia 150. a. 18.5 to 24.99 151. d. Uncontrolled increase...
152. a. Break-bone fever 153. b. Giardia lamblia 154. a. Neisseria meningitidis... 155. c. Neisseria meningitidis
156. d. Addison’s disease 157. a. Interstitial keratitis
158. c. Scurvy 159. d. Chediak higashi syndrome
Miscellaneous 1207
160. A 23 years old woman has experienced episodes of myalgias, 168. Most sensitive marker of myocardial cell damage is:
pleural effusion, pericarditis and arthralgias without joint a. Troponin (J & K 2011)
deformity over course of several years. The best laboratory b. CK-MB
screening test to diagnose her disease would be: c. AST
a. CD lymphocyte count (Feb DP PGMEE 2009) d. Erythema marginatum
b. Erythrocyte sedimentation rate Ref: Harrison’s 18/e p2023-2024, 2016
c. Antinuclear antibody 169. Anaphylactoid reaction (non-IgE) mediated mast cell
d. Assay for thyroid hormones Ref: Harrison’s 18/e p2728 degranulation can occur on exposure to: (J & K 2011)
161. The following are characteristic of tumor lysis syndrome, a. Opiates
except: (J & K 2010) b. Bee venom
a. Hyperkalemia c. Waspvenom
b. Hypocalcamia d. Penicillin
c. Lactic acidosis Ref: Harrison’s 18/e p3439
d. Hyperuricemia 170. Which of the following is not a feature of heat stroke?
Ref: Harrison’s 18/e p362 a. Multi organ failure (J & K 2011)
162. Which one of these is not the side effect of steroids? b. Shock
a. Osteoporosis (J & K 2010) c. Profuse sweating
b. Avascular necrosis of femoral head d. Confusion
c. Proximal myopathy Ref: Harrison’s 18/e p114
d. Lytic lesions in the bones 171. Antidote for Paracetamol poisoning is: (J & K 2011)
Ref: Goodman Gilman 11/e p1604 a. Desferrioxamine
163. Vitamin B12 is absorbed from: (J & K 2010) b. Nalaxone
a. Proximal small intestine c. N-acetylcysteine
b. Mid small intestine d. Calcium gluconate
c. Distal small intestine Ref: Goodman Gilman 11/e p694
d. Stomach Ref: Harrison’s 18/e p862 172. Toxic shock syndrome is caused by: (J & K 2011)
164. A primary tumor of which of these organs is least likely to a. Staphylococcus aureus only
metastasize to bone? (J & K 2010) b. Staphylococcus aureus and GroupA Streptococci
a. Breast c. Klebsiella
b. Colon d. Pseudomonas
c. Kidney Ref: Harrison’s 18/e p1028
d. Lung 173. Oral drug effective in treatment of Kala azar is:
165. Specific therapy for paracetamol poisoning is: (J & K 2010) a. Paramomycin (J & K 2011)
a. Oral lactulose b. Pentamidine
b. N-acetylcysteine c. Miltefosine
c. Methylpredisolone d. Amphotericin
d. Pantoprazole Ref: Goodman Gilman 11/e p694 Ref: Harrison’s 18/e p1713
166. Mycotic Aneurysm is caused by following organisms: 174. Which of the following is an AIDS defining illness?
a. Fungus (J & K 2010) a. Oral hairy leukoplakia (J & K 2011)
b. Bacteria b. Recurrent Oropharyngeal candidiasis
c. Virus c. HIV associated dementia
d. Protozoa d. Herpes zoster
MEDICINE
Ref: Harrison’s 18/e p2061, 2065 Ref: Harrison’s 18/e p1538
167. Therapy of Toxoplasmosis include: (J & K 2010) 175. The dosing of tablet acyclovir for chicken pox in an average
a. Artesunate adult is: (J & K 2011)
b. Ciprofloxacin a. 200 mg five times a day
c. Pyrimethamine b. 400 mg five times a day
d. Thiacetazone c. 800 mg five times a day
Ref: Harrison’s 18/e p1725 d. 400 mg five times a day
Ref: Harrison’s 18/e p1465
Ans. 160. c. Antinuclear... 161. b. Hypocalcamia 162. d. Lytic lesions in the... 163. c. Distal small intestine
164. b. Colon
165. b. N Acetyl cysteine 166. b. Bacteria 167. c. Pyremathamine
168. a. Troponin T and I 169. a. Opiates 170. c. Profuse sweating 171. c. N-acetylcysteine (NAC)
172. b. Staph and...
173. c. Miltefosine 174. b. Recurrent... 175. c. 800 mg five times a day
1208 Jaypee’s Triple A
176. The class of HIV commonest in India is: (J & K 2011) 179. Drug used to prevent niacin induced flushing is: (AP 2012)
a. A a. Paracetamol
b. B b. Cetrizine
c. C c. Aspirin
d. D d. Dexamathasone
Ref: Harrison’s 18/e p1510 Ref: Harrison’s 18/e p598
177. All are true regarding hyper IgE syndrome except: 180. Septicaemic shock with multi organ failure is treated by:
a. Inheritance is as a single locus autosomal dominant trait a. Adrenaline (AP 2012)
with variable expression (DP PGMEE 2009) b. Ephedrine
b. Coarse facial features c. Phenylephrine
c. Recurrent staphylococcal abscesses involving skin, lungs d. Nor-epinephrine
d. High serum lgE with low igG, lgA and lgM Ref: Harrison’s 18/e p2223, 2228
Ref: Harrison’s 18/e p482, 17/e p384, 2061
178. All are true about wiskott-aldrich syndrome except:
a. Bloody diarrhea during infancy (DP PGMEE 2009)
b. Low IgM and elevated lgA and lgE
c. Large size platelets
d. Atopic dermatitis
Ref: Harrison’s 18/e p2702, 17/e p384, 2060
MEDICINE
Ans. 176. c. C 177. d. High serum lgE with low... 178. c. Large size platelets 179. c. Aspirin
180. d. Nor-epinephrine
Section B
PRACTICE QUESTIONS
(Comprising of Questions from Recent Exams and
NEET Pattern Questions)
MEDICINE
Practice Questions
1. Least commonly affected site in arterial thromboembolism 9. Hypokalemia with hypertension is seen in: (DNB 2013)
is: (DNB 2013) a. Bartter syndrome
a. Liver b. Primary hyperalodosteronism
b. Kidney c. Primary hyperparathyroidism
c. Heart d. Diuretic therapy
d. Brain Ref: Harrison’s 18/e p2049
Ref: Internet 10. Antihypertensive used in angina pectoris is: (DNB 2013)
2. Most common lymph nodes involved in hodgkin’s a. Alpha blocker
lymphomas is: (DNB 2013) b. Beta blocker
a. Inguinal c. Calcium channel blockers
b. Cervical d. ACE Inhibitors
c. Axillary Ref: Harrison’s 18/e p2010
d. Subclavian 11. Herpes virus involves which lobe: (DNB 2013)
Ref: Harrison’s 18/e p934 a. Frontal
3. Drug of choice for pneumocystis carinii is: (DNB 2013) b. Parietal
a. Doxycycline c. Temporal
b. Cotrimoxazole d. Occipital
c. Tetracyclines Ref: Harrison’s 18/e p1457
d. Dapson 12. FEV1/FVC is decreased in: (DNB 2013)
Ref: Harrison’s 18/e p1672 a. Asthma
4. Takyasu arteritis mainly affects: (DNB 2013) b. Kyphosis
a. Pulmonary artery c. Scoliosis
b. Celiac artery d. Fibrosis
c. Subclavian artery Ref: Harrison’s 18/e p2092
d. SMA Ref: Harrison’s 18/e p796 13. Most common complication of cardiac catheterization:
5. Bilateral upper limb pulse less disease is: (DNB 2013) a. Arrhythmia (DNB 2013)
a. Giant Cell Arteritis b. Hypertension
b. Polyarteritis Nodosa c. Vascular bleeding
c. Aortoarteritis d. Contrast reaction
d. HSP Ref: Harrison’s 18/e p1853-1854
Ref: Harrison’s 18/e p2065 14. Macrophage activation syndrome is seen in association
6. Which among the following is true about iron deficiency with:
anemia: (DNB 2013) a. Systemic sclerosis
a. Increased Ferritin b. Sweet’s syndrome
b. Increased TIBC c. Polyarteritis nodosa
c. Increased saturation d. SLE
d. Macrocytic Hypochromic Anemia Ref: Internet
Ref: Harrison’s 18/e p848 15. True about Sipple syndrome (MEN type 2):
7. Drug of choice for madura mycosis is: (DNB 2013) a. MCT, Pheochromomcytoma, Mucocutaneous neuromas
a. Imipenem b. MCT, Pheochromocytoma, Parathyroid adenomas
b. Dapsone c. MCT, Pheochromocytoma, Panceratic tumours
c. Itraconazole d. MCT, Pheochromocytoma, Diabetes
d. Amikacin Ref: Internet Ref: Harrison’s 18/e p3075
8. Farmer’s lung is caused due to exposure to: (DNB 2013) 16. Specific antibody for SLE is:
a. Bacillus subtilis a. Anti Sm antibodies
b. Thermoactinomyces sacchari b. Anti-ds DNA antibodies
c. Aspergillus fumigatus c. Anti-Histone antibodies
d. Penicillium nalgiovense d. Anti Ro (SS-A) antibodies
Ref: Harrison’s 18/e p2117 Ref: Harrison’s 18/e p2726
PRACTICE Qs – Medicine
18. Pulsus bigeminy is seen in: (DNB 2013) d. Resistance to all
a. Digitalis Ref: Harrison’s 18/e p1341
b. Beta Blockers 27. Which ion channel is affected in hypokalemic periodic
c. ACE Inhibitors paralysis: (DNB 2013)
d. Calcium channel blockers a. K+
19. Antiepileptic of choice in Post diabetic peripheral b. Na+
neuropathy: (DNB 2013) c. Cl–
a. Pregabalin d. Ca2+
b. Amitryptylene Ref: Harrison’s 18/e p3504
c. Carbamazepine 28. HIV subtype seen in Africa is: (DNB 2013)
d. Duloxetine a. Subtype A
Ref: Harrison’s 18/e p2984 b. Subtype B
20. Which of the following is associated with hypothyroidism c. Subtype D
in sub Himalayan region: (DNB 2013) d. Subtype C
a. Cu Ref: Harrison’s 18/e p1510
b. Fe 29. Gaisbock syndrome is better known as: (DNB 2013)
c. Zn a. Primary familial polycythemia
d. Se b. High-altitude erythrocytosis
Ref: Harrison’s 18/e p604 c. Spurious polycythemia
21. Judkins technique used for: (DNB 2013) d. Polycythemia vera
a. Central venous line placement Ref: Harrison’s 18/e p456
b. Coronary arteriography 30. Neurotransmitter in striatal pathway is: (DNB 2013)
c. Renal angiography a. Glutamine
d. Chest tube insertion b. Glycine
Ref: Harrison’s 18/e p1854, 2005 c. Serotonin
22. Carcinoid of heart presents as: (DNB 2013) d. Dopamine
a. Aortic stenosis Ref: Harrison’s 18/e p3227
b. Tricuspid regurgitation 31. Loss of striatal fibres in caudate nucleus is associated with:
c. Mitral stenosis a. Huntington’s disease (DNB 2013)
d. Aortic regurgitation b. Parkinsons’ disease
Ref: Harrison’s 18/e p3062 c. Charcotmarie tooth disease
23. All are true about Diabetes mellitus except: (DNB 2013) d. Hemiballismus
a. DKA is common is type II Ref: Harrison’s 18/e p3319
b. HHS is primarily seen in individuals with type 2 DM 32. Dose of clonidine in suppression test done in
c. Serum sodium in DKA is 125 – 135 mmol/L pheochromocytoma is: (DNB 2013)
d. Serum bicarbonate in DKA is < 15 meq/L a. 0.3 mg
Ref: Harrison’s 18/e p2976 b. 10 mg
24. Increased uric acid production in tumour: (DNB 2013) c. 100 mg
a. Increased purine degradation d. 200 mg Ref: Harrison’s 18/e p2963
b. Increased pyrimidine degradation 33. Most common cause of Dilated cardiomyopathy is:
c. Denovo purine synthesis a. Alcohol (DNB 2013)
d. Increased pyrimidine synthesis b. Viral infection
Ref: Harrison’s 18/e p3182 c. Pregnancy
25. Cause of thick pancreatic secretions in cystic fibrosis: d. Metabolic disease
a. Overproduction of mucin (DNB 2013) Ref: Harrison’s 18/e p1961
Ans. 17. a. Libman Sachs… 18. b. Beta Blockers 19. a and d 20. d. Selenium
21. b. Coronary… 22. b. Tricuspid regurgitation 23. a. DKA is commoner… 24. a. Increased purine…
25. b. Failure to clean… 26. b. INH & Rifampicine 27. d. Ca2+ 28. d. Subtype C
29. c. Spurious 30. d. Dopamine 31. b. Parkinsons’ disease 32. a. 0.3 mg
33. a. Alcohol
1212 Jaypee’s Triple A
34. Esophageal motility disorder is best diagnosed by: c. Iron deficiency anemia
a. Barium studies (DNB 2013) d. Fanconi’s anemia
b. Esophagoscopy Ref: Harrison’s 18/e p887-890
c. 24 hour pH monitoring 42. Transportation of Iron in forms of: (FMGE March 2013)
d. Manometry a. Serum transferrin
Ref: Harrison’s 18/e p2428 b. Serum ferritin
35. Which of the following is true about pneumoconiosis: c. Serum bilirubin
(DNB 2013) d. Serum alkaline
PRACTICE Qs – Medicine
a. Pleural plaques of asbestosis is always symptomatic Ref: Harrison’s 18/e p844-845; 17/e p628-629
b. Silicosis is never associated with tuberculosis 43. Splenectomy is most useful in: (FMGE March 2013)
c. Asbestosis and smoking act synergistically to cause lung a. Thrombocytopenia
cancer b. Hereditary spherocytosis
d. Anthracosis causes lung cancer c. H.S. pupura
Ref: Harrison’s 18/e p2123 d. Sickle cell anemia
36. Osborne waves in ECG in seen in: (DNB 2013) Ref: Harrison’s 18/e p875, 876; 17/e p655
a. Hypothyrodism 44. Which is NOT seen in chronic case of sickle cell anemia:
b. Hypothermia a. Hepatomegaly (FMGE March 2013)
c. Hypokalemia b. Pulmonary hypertension
d. Hypercalcemia c. Splenomegaly
Ref: Harrison’s 18/e p1838 d. Cardiomegaly
37. Most common LMN cause of Facial nerve palsy is: Ref: Harrison’s 18/e p855; 17/e p638
a. Trauma (DNB 2013) 45. All are features of ARDS except: (FMGE March 2013)
b. Bell’s palsy a. Hypoxia
c. Infections b. Hypercapnia
d. Vascular causes c. Pulmonary edema
Ref: Harrison’s 18/e p3362 d. Stiff lungs
38. Aphasia which affects the arcuate fibres is called: Ref: Harrison’s 18/e p2205-2207; 17/e p1680, 1681
a. Global aphasia (DNB 2013) 46. All are type I respiratory failure except:
b. Anomic aphasia a. Interstial lung disease (FMGE March 2013)
c. Conduction aphasia b. Asthma
Ref: Harrison’s 18/e p204, 205 c. COPD
39. Koenen’s tumor is seen in: (DNB 2013) d. ARDS
a. Neurofibromatosis Ref: Harrison’s 18/e p2084-2086; 17/e p1583, 1584
b. Tuberous sclerosis 47. ECG pattern seen in pulmonary embolism is:
c. Struge Weber syndrome a. S3 Q3 T1 (FMGE March 2013)
d. Tuberculosis b. S1 Q1 T3
Ref: Harrison’s 18/e p2360, 3384 c. S1 Q3 T3
40. Patient present with fasting sugar as 167mg/dL, skin d. S3 Q3 T3 Ref: Harrison’s 18/e p2172; 17/e p1653
pigmentation and hypogonadism. His liver enzymes 48. All are true about normal Parkinson’s disease except:
showed SGOT as 678 and SGPT as 692. Most probable a. Spastic rigidity (FMGE March 2013)
diagnosis is: (DNB 2013) b. Tremor
a. Alpha 1 antitrypsin deficieny c. Acute lesion with drugs
b. Wilson’s disease d. Preserved postural reflexes
c. Hemochromatosis Ref: Harrison’s 18/e p3317-3321; 17/e p2550, 2551
d. Glycogen storage disease
49. In hepatic encephalopathy the EEG shows:
Ref: Harrison’s 18/e p3165
a. a-activity waves (FMGE March 2013)
41. All of the following cause Microcytic Hypochromic anemia b. d (delta) – waves
except: (FMGE March 2013) c. Rapid a – waves
a. Lead poisoning d. None of the above
b. Thalassemia Ref: Harrison’s 17/e p1868
Ans. 34. d. Manometry 35. c. Asbestosis and… 36. b. Hypothermia 37. b. Bell’s palsy
38. c. Conduction aphasia 39. b. Tuberous sclerosis 40. c. Hemochromatosis 41. d. Fanconi’s anemia
42. a. Serum transferrin 43. b. Hereditary spherocytosis 44. c. Splenomegaly 45. b. Hypercapnia
46. c. COPD 47. c. S1 Q3 T3 48. d. Preserved postural… 49. b. d (delta) – waves
Practice Questions 1213
50. Giant A wave in JVP is seen in: c. Rheumatoid pleuritis
a. Tachycardia (FMGE March 2013) d. Dressler’s syndrome
b. Atrial ectopic Ref: Harrison’s 17/e p1658
c. Ist degree A-V block 59. All are true of Nephrotic syndrome except:
d. Complete heart block a. RBC casts in urine (FMGE March 2013)
Ref: Harrison’s 18/e p1823; 17/e p1384 b. Hypoproteinemia
51. S4 is NOT seen in: (FMGE March 2013) c. Oedema
a. Ventricular aneurysm d. Hyperlipidemia
PRACTICE Qs – Medicine
b. Mitral regurgitation Ref: Harrison’s 18/e p2345; 17/e p1785
c. Hypertrophic cardiomyopathy 60. Barret’s esophagous is diagnosed by:
d. Hypertension a. Squamous metaplasia (FMGE March 2013)
Ref: Harrison’s 18/e p1827; 17/e p1386 b. Intestinal metaplasia
52. Earliest valvular lesion in a case of acute rheumatic fever is: c. Squamous dysplasia
a. MR (FMGE March 2013) d. Intestinal dysplasia
b. AR Ref: Harrison’s 18/e p2434; 17/e p1852
c. MS 61. Hyperchloraemic metabolic acidosis is caused by:
d. AS a. Cholera (FMGE March 2013)
Ref: Harrison’s 18/e p2753; 17/e p2094 b. Starvation
53. Rheumatoid factor is mainly: (FMGE March 2013) c. Ethylene glycol poisoning
a. Ig M d. Lactic acidosis
b. Ig G Ref: Harrison’s 17/e p292
c. Ig D 62. The HLA important in IDDM is: (FMGE March 2013)
d. Ig A a. HLA-A3
Ref: Harrison’s 18/e p2746; 17/e p2088, 2155 b. HLA-B27
54. Pulsus paradoxus is seen in all except: c. HLA-DR3/DR4
a. IPPV (FMGE March 2013) d. HLA-W1
b. COPD Ref: Harrison’s 18/e p2694; 17/e p2279
c. Cardiac tamponade 63. Increased Reid’s Index is used to characterized:
d. Constrictive pericarditis a. Chronic Bronchitis (FMGE March 2013)
Ref: Harrison’s 18/e p1824; 17/e p1384 b. Bronchiectasis
55. All are used in bronchial asthma except: c. Bronchial Asthma
a. Salbutamol (FMGE March 2013) d. Emphysema
b. Morphine Ref: Respiratory Therapy Secrets 2/e p 45
c. Aminophylline 64. Rheumatoid Arthritis is best diagnosed by:
d. Steroid a. Anti CCP antibodies (FMGE March 2013)
Ref: Harrison’s 18/e p2110-2111; 17/e p1605 b. IgA Rheumatoid Factor
56. Most common cause of empyema is: (FMGE March 2013) c. IgG Rheumatoid Factor
a. Bronchopleural fistula d. IgM Rheumatoid Factor
b. Tubercular pneumonia Ref: Harrison’s 18/e p2746; 17/e p2088
c. Bacterial pneumonia 65. Most common cause of Addison’s Disease in India:
d. Pleurisy a. Autoimmune (FMGE March 2013)
Ref: Harrison’s 18/e p1156; 17/e p1658, 1659 b. Postpartum
57. Rasmussen’s aneurysm arises from: (FMGE March 2013) c. HIV
a. Bronchial artery d. Tuberculosis
b. Pulmonary artery Ref: Harrison’s 18/e p2955; 17/e p2141
c. Vertebral artery 66. All of the following are true about Brown Sequard
d. Posterior intercostals artery Syndrome, except: (FMGE March 2013)
Ref: Churchill Livingstone Medical Dictionary by Booker 16/e p410; a. Ipsilateral Pyramidal Tract Features
Harrison’s 18/e p1345 b. Contralateral Spinothalamic Tract Features
58. All of the following show low glucose in pleural fluid c. Contralateral Posterior Column Features
except: (FMGE March 2013) d. Ipsilateral Plantar Extensor
a. Empyema Ref: Harrison’s 18/e p184; 17/e p157, 2589
b. Malignant pleural effusion
a. Diplopia in left lateral gaze distinguishing an acute liver disease from chronic liver
b. Inability to accommodate on left gaze diseases: (FMGE March 2013)
c. Impaired adduction in left eye a. Amino transaminase
d. Ptosis of left eye b. Alkaline phosphatase
Ref: Harrison’s 18/e p239; 17/e p2504, 2505 c. Bilirubin
69. Which neo-adjuvant chemotherapy is used in Esophageal d. Albumin Ref: Harrison’s 18/e p2529; 17/e p1815
carcinoma: (FMGE March 2013) 77. A 25 years old female presents normal PT but high PTT and
a. Cisplatin high BT, diagnosis is: (FMGE March 2013)
b. Cyclophosphamide a. DIC
c. Doxorubicin b. Haemophilia
d. Methotrexate c. VWD
Ref: Harrison’s 18/e p765; 17/e p571 d. TTP Ref: Harrison’s 18/e p971-972; 17/e p725-726
70. True about Anorexia nervosa: (FMGE March 2013) 78. A 26 year old female presents with severe headache to
a. Is more common than bulimia emergency department. She is having similar episodes for
b. Can have a low T4 in the presence of a normal T3 the past 6 months. All the features favour a diagnosis of
c. May have a hypochloremic hypokalemic alkalosis cluster headache in this patient except:
d. Is associated with constipation more commonly than a. Periodic pain
diarrhea b. Conjunctival injection
Ref: Harrison’s 18/e p637-638; Rudolf’s Pediatrics 21/e p1972 c. Bilateral photophobia
71. Alpha-1 antitrypsin deficiency occurs is: d. No response to oral sumatriptan
a. Emphysema (FMGE March 2013) Ref: Harrison’s 18/e 2012; Chapter 14; p112, 122
b. Bronchiectasis 79. Clearance of Theophylline levels are increased by all except
c. Empyema a. Rifampicin
d. Bronchogenic carcinoma b. Smoking
Ref: Harrison’s 18/e p2152; Robbin’s 5/e p685 c. Phenobarbitone
72. Reed-Sternberg cells are seen in: (FMGE March 2013) d. Cimetidine
a. Infectious mononucleosis Ref: Harrison’s 18/e 2012; Chapter 254; Table 254-4
b. Hodgkin’s lymphoma 80. The class of lupus nephritis with worst prognosis is:
c. Mycosis fungoid a. Class 2
d. All of the above b. Class 3
Ref: Harrison’s 18/e p934; 17/e p699 c. Class 4
73. The most sensitive test for the diagnosis of myasthenia d. Class 5
gravis is: (FMGE March 2013) Ref: Harrison’s 18/e 2012; Chapter 283
a. Elevated serum Ach-receptor binding antibodies 81. HIV-associated thrombocytopenia is primarily due to:
b. Repetitive nerve stimulation test a. Infection of megakaryocytes
c. Positive edrophonium test b. Antibodies against platelets
d. Measurement of jitter by single fibre electromyography. c. Both of the above
Ref: Harrison’s 18/e p3481; 17/e p2354 d. None of the above
74. Amyloid deposit in Alzheimer’s disease is: Ref: Harrison’s 18/e 2012; Chapter 189, 1557
a. AA (FMGE March 2013) 82. The minimum possible score in “Glasgow coma scale” is:
b. Ab a. 0
c. Ab2 b. 1
d. AL c. 3
Ref: Harrison’s 18/e p3305; 17/e p2540-2541 d. 5 Ref: Harrison’s 18/e 2012; Chapter 267 Table 267-1
Ans. 67. d. All of the above 68. a. Diplopia in left… 69. a. Cisplatin 70. d. Is associated with…
71. a. Emphysema 72. d. All of the above 73. c. Positive… 74. b. Ab
75. a. Flaccid paralysis 76. d. Albumin 77. c. VWD 78. c. Bilateral photophobia
79. d. Cimetidine 80. c. Class 4 81. c. Both of the above 82. c. 3
Practice Questions 1215
83. A 57 year old chronic smoker and weight loss for 4 months c. Asthma
presented to emergency department with respiratory d. Eosinophilia
distress. His EKG showed electrical alternans. The most Ref: Harrisons textbook of internal medicine 18/e p1658;
likely diagnosis is: 17/e p97 chapter
a. Pneumothorax 91. All the following drugs are used in the treatment of
b. Pleural effusion Multidrug resistant tuberculosis except:
c. Cardiac tamponade a. Kanamycin
d. Constrictive pericarditis b. Ofloxacin
PRACTICE Qs – Medicine
Ref: Harrison’s 18/e 2012. Chapter 239. ECG features c. Ethionamide
84. All the following are supplemented in cystic fibrosis with d. Thiacetazone
Gastrointestinal disease except: Ref: RNTCP; Harrison’s 18/e p1352-1354
a. Vitamin E
92. All the following causes micronodular cirrhosis except:
b. Vitamin K
a. Laennec’s cirrhosis
c. Vitamin B12
b. Primary biliary cirrhosis
d. Pancreatic lipase
c. Indian childhood cirrhosis
Ref: Harrison’s 18/e 2012. Chapter 259
d. Wilson’s disease
85. Which of the following is not seen in Parkinsonism: Ref: Harrison’s textbook of internal medicine 18/e p3188-318
a. Preserved postural reflexes
b. Hypokinesia 93. Prophylaxis of febrile seizures: (NEET Pattern Question)
c. Rigidity a. Valproic
d. Static tremors b. Diazepam
Ref: Harrison’s 18/e 2012. Chapter 372. Table 372-3 c. Carbamazepine
d. Phenytoin
86. Endocrine causes for hypertension are all the following
except: 94. Most common diseases that affects anterior horn cells:
a. Cushing’s syndrome a. Polio (NEET Pattern Question)
b. Hypopitutarism b. SACD
c. Hyperaldosteronism c. SSPE
d. Gigantism d. Japanese encephalitis
Ref: Harrison’s 18/e p2048; 17/e p2008. Chapter 241. p1554. Table 241-3 Ref: Harrison’s 18/e p1594
87. All are true in Kawasaki disease except: 95. Non DM, non HTN, afebrile with complaints of diplopia,
a. Exudative conjuctivits after sensorium, headache, nuchal rigidity indicates:
b. Pedal edema a. SAH (NEET Pattern Question)
c. Strawberry tongue b. Encephalitis
d. Thrombocytosis c. Meningitis
Ref: Harrison’s 18/e p416; CMDT 2013, p1417-1418 d. Intracerebral hmg
88. Which of the following is characteristic of Cushing’s disease: 96. Which is not seen in tuberous sclerosis:
a. Incresed ACTH, increased cortisol a. Cherry haemangioma (NEET Pattern Question)
b. Decreased ACTH, increased cortisol b. Shagreen patch
c. Increased ACTH, decreased cortisol c. Subependymal nodule
d. Decreased ACTH, decreased cortisol d. Rhabdomyomas
Ref: Harrison’s 18/e p2940-2947; Davidson 21/e p771 Ref: Harrison’s 18/e p2360
89. All of the following about Grave’s disease is true except: 97. Shagreen patch: (NEET Pattern Question)
a. Cardiac failure is common a. Tuberous sclerosis
b. The Hypertrophy and Hyperplasia of thyroid gland is b. NF
due to TSH Ab
c. VLH
c. Remissions and exacerbations are not infrequent
d. Struge Weber Syndrome
d. It is highly vascular with audible bruit
e. Uniformly enlarged thyroid gland 98. Anomic aphasia is due to injury to:
Ref: Harrison’s 18/e p2923-2924; CMDT 2013, p1110 a. Wernicke’s area (NEET Pattern Question)
b. Broca’s area
90. All the following are true about bronchopulmonary
c. Angular gyrus
aspergillosis except:
d. Arcuate fasciculus
a. Central bronchiectasis
b. Pleural effusion Ref: Harrison’s 18/e p204
Ans. 83. c. Cardiac tamponade 84. b. Vitamin B12 85. a. Preserved postural… 86. b. Hypopitutarism
87. a. Exudative… 88. b. Decreased ACTH… 89. b. The Hypertrophy… 90. b. Pleural effusion
91. d. Thiacetazone 92. d. Wilson’s disease 93. b. Diazepam 94. a. Polio
95. a. SAH 96. a. Cherry haemangioma 97. a. Tuberous sclerosis 98. b. Broca’s area
1216 Jaypee’s Triple A
99. Striate vertebra is seen in: (NEET Pattern Question) c. Twitching and fasciculation
a. Hemanigoma d. Bowel bladder not affected
b. Multiple myeloma Ref: Harrison’s 18/e p3345
c. Fibrous dysplasia 108. Pure motor type of neuropathy occurs in:
d. Chordoma a. GB syndrome (NEET Pattern Question)
Ref: Internet b. Polio
100. All of the following are seen in Wernicke’s encephalopathy c. Multiple sclerosis
except: (NEET Pattern Question) d. HIV
PRACTICE Qs – Medicine
Ans. 99. a. Hemanigoma 100. c. Epilepsy 101. b. Heaadache is not 102. a. Area 4
103. c. Epdrophonium 104. c. Rigidity 105. c. Tension 106. b. Etiology is hemorrage...
107. b. Muscular… 108. b. Polio 109. b. Hypocalcemia 110. a. TSH
111. b. Atrial fibrillation 112. a. Adenoma 113. a. HbA1C 114. c. Glucagon
115. d. Pheochromocytoma
Practice Questions 1217
116. Wermer’s syndrome is: (NEET Pattern Question) c. Treatment with medication is satisfactory
a. MEN 1 d. Renal vasoconstriction is present
b. MEN 2 Ref: Harrison’s 18/e p2601
c. MEN 2A 125. Hydatid cyst of liver true is: (NEET Pattern Question)
d. VHL syndrome a. PAIR technique is most commonly done in superficial
Ref: Harrison’s 18/e p3072 cysts
117. Most common tumour of MEN type 2 among these are: b. Diagnostic aspiration is most commonly recommended
a. Gastrinoma (NEET Pattern Question) c. Can metastasise in brain
PRACTICE Qs – Medicine
b. Somatinoma d. Surgical resection remain the treatment of choice
c. Insulinoma Ref: Harrison’s 18/e p1763
d. Carcinoid tumours
126. Celiac disease false is: (NEET Pattern Question)
Ref: Harrison’s 18/e p3076
a. Immunological factor is present in many cases
118. Not associated with weight gain: (NEET Pattern Question) b. Does not predispose to malignancy
a. Pheochromocytoma c. Anti endomysial antibody most sensitive
b. Cushing syndrome
d. Intestinal biopsy is to be done to confirm diagnosis
c. Myxedema
Ref: Harrison’s 18/e p2471
d. None
Ref: Harrison’s 18/e p2963 127. Treatment of retroperitoneal fibrosis:
a. Corticosteroids (NEET Pattern Question)
119. Pellagra occurs due to deficiency of:
b. NSAIDs
a. B12 (NEET Pattern Question)
c. Cyclosporine
b. B2
c. B3 d. Radiotherapy
d. B5 128. Best investigation in Takayasu’s arteritis is:
Ref: Harrison’s 18/e p598 a. ESR (NEET Pattern Question)
120. The signs and symptoms of CRF become prominent when b. pANCA
GFR is below: (NEET Pattern Question) c. cANCA
a. 50% d. Angiography
b. 60% Ref: Harrison’s 18/e p2797
c. 70% 129. Low Fe and Low TIBC are found in which:
d. 80% a. Sickle cell anemia (NEET Pattern Question)
Ref: Harrison’s 18/e p2319 b. Iron deficiency anemia
121. Erythropoietin secreted by: (NEET Pattern Question) c. Anemia of chr. disease
a. Macula densa d. Thalassemia
b. Juxtaglomerular apparatus Ref: Harrison’s 18/e p455
c. Peritubular capillary bed 130. Normal RBC with decrease amount is usually seen in:
d. Pericytes a. Thalassemia (NEET Pattern Question)
Ref: Harrison’s 18/e p448 b. Sickle cell anemia
122. Which causes renal papillary necrosis: c. Hemolytic anemia
a. Analgesic (NEET Pattern Question) d. Anemia of chronic disease
b. Contrast media Ref: Harrison’s 18/e p850
c. Dehydration 131. Non proliferative anemia with normal iron and iron stores:
d. Overhydration
a. CRF (NEET Pattern Question)
Ref: Harrison’s 18/e p2372
b. Iron deficiency anemia
123. Urinary incontinence due to blocking lesion is called: c. Anemia of chronic disease
a. Overflow incontinence (NEET Pattern Question) d. Thalassemia
b. Stress incontinence Ref: Harrison’s 18/e p850
c. Genuine stress incontinence
132. Anemia microcytic hypochromic is found in all except:
d. Urge incontinence
a. Iron deficiency (NEET Pattern Question)
Ref: Harrison’s 18/e p581
b. Sideroblastic
124. Hepatorenal syndrome type 2, true is:
c. Thalassemia
(NEET Pattern Question)
d. Fanconi anemia
a. Prognosis better than type 1
Ref: Harrison’s 18/e p496
b. GFR remain normal
Ans. 133. a. Penicillamine 134. d. None 135. d. VIII 136. c. Activated partial…
137. c. Primary systemic 138. d. Reiter’s disease 139. d. All 140. b. Blood culture
141. d. Renal tubular… 142. c. Resp. acidosis 143. a. High HCO3 144. c. All decreased
145. b. ↑HCO3 146. b. Pri. Hyperparathyroidism 147. a. Alkaline urine 148. c. Rales
149. b. Urea decreased
Practice Questions 1219
150. Characteristic arrhythmia in digoxin toxicity is: 159. Absent a-wave is seen in: (NEET Pattern Question)
a. SA nodal block (NEET Pattern Question) a. Severe tricuspid stenosis
b. Mobitz type II block b. Mitral stenosis
c. Atrial flutter c. Atrial fibrillation
d. Atrial fibrillation d. Sick sinus syndrome
Ref: Harrison’s 18/e p1868 Ref: Harrison’s 18/e p1881-1882
151. Earliest change on ECG in hyperkalemia is: 160. Most common manifestation of rheumatoid arthritis on
a. Tall T wave (NEET Pattern Question) cardiovascular system is: (NEET Pattern Question)
PRACTICE Qs – Medicine
b. Wide QRS complex
a. Endocarditis
c. AV conduction defect
b. Fibrous pericarditis
d. Inversion of T wave
c. Cardiac tamponade
Ref: Harrison’s 18/e p357
d. None Ref: Harrison’s 18/e p2739
152. In Hypocalcemia which wave is seen in ECG:
a. QT prolonged (NEET Pattern Question) 161. To calculate pressure, ECHO uses: (NEET Pattern Question)
b. QT shortened a. Bernoulli principle
c. Tall peaked t wave b. Laplace rule
d. No change in QT interval c. Homan law
Ref: Harrison’s 18/e p3114 d. None
153. Hypomagnesemia, which is seen: (NEET Pattern Question) Ref: Harrison’s 18/e p1842
a. Prolongation of QT wave 162. All are true about Buerger’s disease except: (PGI May 2013)
b. ST straightening a. Mesangial proliferation
c. T wave flattening b. Increased polyclonal IgA
d. All c. IgA, C3 and IgG deposites in the measangium
Ref: Harrison’s 18/e p3091 d. Hematuria may be gross or microscopic
154. Prominent U wave is found in: (NEET Pattern Question) e. Absence of proteinuria is pathognomic
a. Hyperkalemia Ref: Harrison’s 18/e p2069, 2070
b. Hypomagnesemia 163. True about acute pancreatitis: (PGI May 2013)
c. Hypokalemia a. Amylase has prognostic value
d. Digoxin toxicity b. CT is the best imaging study for initial evaluation
155. Sterile vegetation is not seen in: (NEET Pattern Question) c. Nasojejunal feeding is better than total parenteral
a. Infective endocarditis nutrition
b. Non bacterial thrombotic endocarditis d. ERCP is better than CT
c. Rheumatic fever e. Ultrasound confirms the diagnosis in most cases
d. All Ref: Harrison’s 18/e p2640, 2637
Ref: Harrison’s 18/e p1052-1053
164. True about enteric nutrition: (PGI May 2013)
156. CKMB seen in: (NEET Pattern Question)
a. Given early in Post-op period to prevent mucosal
a. Pulmonary infarction
atrophy
b. Myocardial infarction
b. More chance of spread of infection than total parenteral
c. Myocardial infarction and myocarditis
d. All nutrition
c. It must be given in all post-operative patients
157. Lesion seen on both surface of valve:
d. Enterocutaneous fistula is indication of parenteral
a. Leibmann-sach’s endocarditis (NEET Pattern Question)
nutrition
b. Infective endocarditis
c. Rheumatoid heart disease e. Stimulates the production of immunoglobulins in
d. Non bacterial thrombotic endocarditis the gut Ref: Harrison’s 18/e p612-619
Ref: Internet 165. True about esophageal adenocarcinoma: (PGI May 2013)
158. Cardiac enzyme found on 5th day of MI: a. Majority of cases arise in Barrett’s oesophagus
a. CK MB (NEET Pattern Question) b. Common in upper part of esophagus
b. CK MB + Trop-T c. Commonly arise in the distal esophagus
c. Trop-T d. Tobacco exposure and obesity are risk factors
d. CK mm + LDH e. Incidence is increasing
Ref: Harrison’s 18/e p2023-2024 Ref: Harrison’s 18/e p764-765
Ans. 150. a. SA nodal block 151. a. Tall T wave 152. a. QT prolonged 153. d. All
154. c. Hypokalemia 155. a. Infective endocarditis 156. c. Myocardial infarction… 157. a. Leibmann-sach’s...
158. c. Trop-T 159. c. Atrial fibrillation 160. b. Fibrous pericarditis 161. a. Bernoulli principle
162. e. Absence of…
163. b and c 164. a, d and e 165. a, c, d and e
1220 Jaypee’s Triple A
166. All are true about systemic sclerosis except: 173. Orally used direct thrombin inhibitors is/are:
a. Skin involvement occurs (PGI May 2013) a. Argatroban (PGI May 2013)
b. Generalized blood vessel involvement b. Bivalirudin
c. Pulmonary arterial hypertension c. Lepirudin
d. Bleomycin may cause systemic sclerosis like illness d. Dabigatran etexilate
e. Raynaud’s phenomenon precedes skin involvement e. Rivaroxaban
diffuse cutaneous form Ref: Harrison’s 18/e p997, 1000
Ref: Harrison’s 18/e p2762-2766
PRACTICE Qs – Medicine
180. Eosinophilia is/are seen in: (PGI May 2013) 186. Routine examination of a 17 years old asymptomatic boy
a. Visceral leishmania reveals short PR interval, and delta wave on ECG. All of the
b. Churg-Strauss syndrome following can be done except: (AIIMS May 2013)
c. Visceral larva migrans a. Holter monitoring
d. Drug reaction b. Treadmill test
e. Parasitic infestations c. Beta blocker
Ref: Harrison’s 18/e p481 d. Reassurance
181. Clubbing is/are seen in: (PGI May 2013) 187. Which of the following features helps in distinguishing
PRACTICE Qs – Medicine
a. Lung carcinoma seizures from synocope: (AIIMS May 2013)
b. Idiopathic pulmonary fibrosis a. Loss of consciousness
c. COPD b. Injury due to fall
d. Tuberculosis c. Urinary incontinence
e. Lung abscess d. Weakness with preserved sensorium
Ref: Harrison’s 18/e p290 Ref: Harrison’s 18/e p3261
182. Transesophageal echocardiogram (TEE) is superior to 188. Which of the following is not involved in the armamentarium
transthoracic echocardiogram (TTE) because of: for investigation of malabsorption: (AIIMS May 2013)
a. More convenient (AIIMS May 2013) a. 14 C triolein breath test
b. Is more sensitive in picking up left ventricular apical waves b. 13 C trictanoin breath test
c. Is more sensitive in picking up atheromatous plaques in c. d-xylose test
ascending thoracic aorta d. 13 C Triclosan breath test
d. TEE is more sensitive picking up atrial appendage 189. In medically intractable seizures, which of the following
thrombus Ref: Harrison’s 18/e p1843 modalities of treatment has shown the best seizure-free
183. An elderly female presented with nasal blockade, nasal period: (AIIMS May 2013)
discharge, diplopia and facial swelling. On examination, a. Epileptic surgery
there is blackish discharge from the nasal cavity with b. Deep brain stimulation
necrosis of nasal mucosa, facial skin and plate. There is c. Vagal stimulation
fixation of the right globe. There is elevated blood sugar d. Ketogenic diet
and urinary ketones are positive. Which of the following Ref: Harrison’s 18/e p3267
would be the best medication to be used in this patient: 190. Bilateral Babinski sign is positive in: (AIIMS May 2013)
a. Amphotericin B (AIIMS May 2013) a. Pontine hemorrhage
b. Itraconazole b. Basal ganglia and thalamic hemorrhage
c. Ketoconazole c. Cerebellar hemorrhage
d. Broad spectrum antibiotics d. Subarachnoid hemorrhage
184. A 16-year old girl was given sulfa drugs. She developed 191. LBBB on ECG is seen in all of the following except:
acute abdominal pain and was admitted for seizures. Which a. Acute MI (AIIMS May 2013)
of the following is the most likely possibility: b. Ashmann syndrome
a. Acute intermittent porphyria (AIIMS May 2013) c. Hypokalemia
b. Congenital erythropoetic porphyria d. Hyperkalemia
c. Adenosine deaminase deficiency Ref: Harrison’s 18/e p1835
d. HGPRTase deficiency 192. The genetic mutation seen in the most common type of
Ref: Harrison’s 18/e p3173 maturity onset diabetes of young (MODY1) is:
185. A child presented with a history of fever from 3 days and a. Hepatocyte nuclear factor-4 alfa (AIIMS May 2013)
neck rigidity. There was two episodes of GTCS. Patient b. Hepatocyte nuclear factor-1 alfa
was unconscious, CSF examination reveals a cell count of c. Glucokinase
300 polymorphonuclear cells/mm3, proteins 70 mg/dL, CSF d. Insulin promotor factor-1
glucose 50 mg/dL (corresponding blood sugar 95 mg/dL). Ref: Harrison’s 18/e p2976
CT scan of the patient was shown to be normal. Which of 193. In arterial blood gas analysis of a patient with pCO2-30,
the following is the most likely diagnosis: pO2-115 and Ph-7.45, patient has compensated:
a. Pyogenic meningitis (AIIMS May 2013) a. Respiratory alkalosis (AIIMS May 2013)
b. Tubercular meningitis b. Metabolic alkalosis
c. Herpes encephalitis c. Respiratory acidosis
d. Cerebral malaria d. Metabolic acidosis
Ref: Harrison’s 18/e p1693 Ref: Harrison’s 18/e p363
Ans. 180. d and e 181. a, b, d and e 182. d. TEE is more… 183. a. Amphotericin B
184. a. Acute intermittent... 185. d. Cerebral malaria 186. c. Beta blocker 187. c. Urinary incontinence
188. d. 13 C Triclosan…
189. a. Epileptic surgery 190. a. Pontine hemorrhage 191. b. Ashman syndrome
192. a. Hepatocyte… 193. a. Respiratory alkalosis
1222 Jaypee’s Triple A
194. A patient presents with renal failure and bone pain. On a. Behaviour changes
X-ray, evidence of skeletal destruction and on laboratory b. Frequent blood transfusion
examination, hypercalcemia is seen. Serum electrophoresis c. Thin limbs and obese trunks
shown “M” spike. 35% plasma cell with expression of d. Gynecomastia with full body hairs
abnormal antigen is seen. Diagnosis: 200. A 55-year old diabetic patient presents with transient
a. Multiple myeloma (AIIMS May 2013) obscuration in vision for 2-3 days followed by sudden loss
b. Monoclonal gammopathy with unknown signifcance of vision. Which of the following would be the best test to
c. Smouldering myeloma
PRACTICE Qs – Medicine
Ans. 194. a. Multiple myeloma 195. d. Acute infective… 196. b. Generalized… 197. a. Treatment often curative
198. a. Behcet’s disease 199. c. Thin limbs and… 200. d. Serum creatinine levels 201. a. Levetiracetam
202. a. Polyarthralgia
203. a. Subarachnoid… 204. d. Contraindicated… 205. a. Temporoparietal
Practice Questions 1223
206. A 62 years old woman presented with acute onset confusion c. Anton syndrome
and bumping into things. On examination, she was alert, d. Korsakoff psychosis
oriented with fluent speech and normal comprehension. Ref: Harrison’s 18/e p206
Further examination revealed impaired writing (acalculia), 207. Features of iron deficiency anemia are all except:
right-left confusion, impaired arithematic abilities, (AIIMS May 2013)
difficulty in finger identification. MRI demonstrated foci A. Increases red cell distribution width (RDW)
of cortical and sub cortical increase T2 signals and areas of B. Decreased serum iron
leptomeningeal enhancement. Most likely diagnosis: C. Decreased TIBC
PRACTICE Qs – Medicine
a. Gerstman syndrome (AIIMS May 2013) D. Decreased serum ferritin
b. Millard-Gubler syndrome Ref: Harrison’s 18/e p846-847